SóProvas



Questões de Filosofia do Direito


ID
183208
Banca
FCC
Órgão
DPE-SP
Ano
2010
Provas
Disciplina
Filosofia do Direito
Assuntos

Ao comentar a doutrina aristotélica da justiça, Tercio Sampaio Ferraz Júnior, em sua obra Estudos de Filosofia do Direito, indica aquele que seria o "preceito básico do direito justo, pois só por meio dele a justiça se revelaria em sua atualidade plena". Este preceito, que também pode ser definido como "uma feliz retificação do justo estritamente legal" ou ainda "o justo na concretude", é denominado

Alternativas
Comentários
  • Equidade consiste na adaptação da regra existente à situação concreta, observando-se os critérios de justiça e igualdade. Pode-se dizer, então, que a equidade adapta a regra a um caso específico, a fim de deixá-la mais justa. Ela é uma forma de se aplicar o Direito, mas sendo o mais próximo possível do justo para as duas partes.
    Essa adaptação, contudo, não pode ser de livre-arbítrio e nem pode ser contrária ao conteúdo expresso da norma. Ela deve levar em conta a moral social vigente, o regime político Estatal e os princípios gerais do Direito. Além disso, a mesma "não corrige o que é justo na lei, mas completa o que a justiça não alcança".
    Sem a presença da eqüidade no ordenamento jurídico, a aplicação das leis criadas pelos legisladores e outorgadas pelo chefe do Executivo acabariam por se tornar muito rígidas, o que beneficiaria grande parte da população; mas ao mesmo tempo, prejudicaria alguns casos específicos aos quais a lei não teria como alcançar. Esta afirmação pode ser verificada na seguinte fala contida na obra "Estudios sobre el processo civil" de Piero Calamandrei:
    [...] o legislador permite ao juiz aplicar a norma com eqüidade, ou seja, temperar seu rigor naqueles casos em que a aplicação da mesma (no caso, "a mesma" seria "a lei") levaria ao sacrifício de interesses individuais que o legislador não pôde explicitamente proteger em sua norma.
    É, portanto, uma aptidão presumida do magistrado.

  • Aristóteles foi o primeiro filósofo a trazer o conceito de equidade. Para ele a idéia de justiça somente está completa com a idéia de equidade. O Estado estabelece as Leis e estas cumprem sua função de promover o bem comum. A Lei tem uma característica de ser abstrata, ou seja, é pensada para a generalidade dos casos. Lei é uma regra geral e abstrata. Existem determinadas situações concretas, contudo, que fogem à generalidade da Lei. A previsão legal, portanto, não seria suficiente. Em situações tais é preciso lançar mão do juízo de equidade. É preciso pensar como o legislador resolveria naquele momento. A equidade, portanto, é a explicação da justiça no caso concreto. A verdadeira justiça só se realizará no caso concreto. Equidade, portanto, é a igualdade no caso concreto.
  • ESQUIDADE = JUSTIÇA

  • Penso que São Tomás de Aquino é o primeiro a trabalhar com o conceito de dignidade e o Kant eleva esse conceito, que, posteriormente, toma centralidade no Direito contemporâneo.

  • A questão demanda interpretação e leitura da obra de Tércio Sampaio Ferraz Jr.

    Inobstante tenha linguagem rebuscada e difícil, a clareza metodológica e a profundidade de tão autor são notórias.

    O que o autor quis explicar é que, em Aristóteles, o justo vai além da abstração da lei, que, quando necessário, deve ser reinterpretada, ou até afastada, para que, no caso concreto, observadas as particularidades da casuística, exista a real Justiça.

    Justiça ao caso concreto enquanto categoria supletiva da dimensão legal do Direito representa a EQUIDADE.

    Cabe comentar as alternativas da questão.

    LETRA  A- INCORRETA. Não representa a resposta correta, qual seja, a equidade.

    LETRA  B- INCORRETA. Não representa a resposta correta, qual seja, a equidade.

    LETRA C- CORRETA. De fato, equidade é o padrão de resposta da questão.

    LETRA  D- INCORRETA. Não representa a resposta correta, qual seja, a equidade.

    LETRA  E- INCORRETA. Não representa a resposta correta, qual seja, a equidade.

    GABARITO DO PROFESSOR: LETRA C


ID
253825
Banca
TJ-PR
Órgão
TJ-PR
Ano
2010
Provas
Disciplina
Filosofia do Direito
Assuntos

O pensador inglês Herbert Hart, tido como um dos principais representantes da escola do Positivismo Jurídico, que teve lugar na segunda metade do século XX, manteve a defesa da tese kelseniana da separação entre o Direito e a Moral, sendo que, a partir dessa premissa metodológica, propôs um conceito analítico de Direito. Os críticos do pensamento de Herbert Hart normalmente lhe atribuem a aceitação de cinco teses que seriam consequências lógicas deduzidas da ideia de separação entre Direito e Moral, entre as quais apenas alguma(s) foi/foram verdadeiramente defendida(s) por Herbert Hart e, de resto, pelos principais autores positivistas do século XX, sob o argumento de que as tais cinco teses são logicamente independentes e que, nessa condição, pode-se aceitar a validade de alguma(s) e, ao mesmo tempo, rechaçar outras sem que se caia em contradição. Das cinco teses abaixo que os críticos de Herbert Hart associam ao seu pensamento, marque como falsa(s) (F) aquela(s) que ele não defendeu e como verdadeira(s) (V) aquela(s) que ele de fato sustentou. Em seguida, assinale a opção CORRETA.

( ) A tese da Lei, segundo a qual o conceito de Direito deve ser definido mediante o conceito de Lei.

( ) A tese da Neutralidade, segundo a qual o conceito de Direito tem que ser definido prescindindo-se de seu conteúdo.

( ) A tese da Subsunção, segundo a qual a aplicação do Direito pode ser levada a cabo em todos os casos mediante uma subsunção livre de valorações.

( ) A tese do Subjetivismo, segundo a qual os critérios do Direito "reto" são de natureza subjetiva.

( ) A tese do Legalismo, segundo a qual as normas do Direito devem ser obedecidas em todas as circunstâncias.

Alternativas
Comentários
  • Segundo o livro de Herbert Hart, "O conceito de direito", há uma definição do direito como uma textura aberta, vejamos: "Em qualquer grande grupo, as regras gerais, os padrões e os princípios devem ser o principal instrumento de controle social, e não as diretivas particulares dadas separadamente a cada indivíduo. Se não fosse possível comunicar padrões gerais de conduta, que multidões de indivíduos pudessem perceber, sem ulteriores diretivas, padrões esses exigindo deles certa conduta conforme ocasiões, nada daquilo que agora reconhecemos como direito poderia existir. Daí resulta que o direito deva predominantemente, mas não de forma alguma exclusivamente, referir-se a categorias de pessoas e a categorias de atos, coisas e circuntancias, e seu funcionamento com exito sobre vastas áreas da vida social depende de uma capacidade largamente difundida de reconhecer atos, coisas e circunstancias particulares como casos das classificações gerais que o direiro faz."

    De tal explanação, fica mais fácil perceber que o conceito de direito deve ser feito levando-se em conta as características da população ao qual será submetida a tais regramentos. Portanto, nada mais correta do que a letra "d" da presente questão.

  • Kelsen, Bobbio e Hart são positivistas no que diz respeito ao conceito de direito. Por positivismo legal se deve compreender tão somente que esses autores entendem que o direito e a moral são, e devem ser entendidos, como fenômenos distintos. Existe na obra de Hart o reconhecimento de que a moralidade pode orientar decisões em muitas circunstâncias diferentes, e até mesmo que o direito e a moral andam juntos em alguns assuntos. Mas há, sobretudo, a percepção teórica de que a lei não pode e não deve naturalizar preconceitos morais, dentre outras razões, porque moralidade e direito são fenômenos distintos Existe no positivismo legal uma batalha cética contra as leis fundadas em preconceitos morais, um dos feitos, obtidos pelos argumentos de Hart, foi a descriminalização da homossexualidade na Inglaterra. Cabe dizer que a separação entre direito e moral serve para tirar do âmbito da jurisprudência qualquer tipo de função moralizadora.

    Hart, pois bem, é um filósofo analítico, que teve como interlocutor privilegiado o filósofo da linguagem John L. Austin (1911-1960), seu contemporâneo, e com ele compartilhou a pesquisa do fenômeno linguístico da lei – que, por vezes, pode ser entendida como um ato performativo da linguagem, isto é, algo que não é apenas descritivo, mas representa uma ação, uma realização, e tem uma conseqüência. Com base nesse entendimento, Hart diferenciou os conceitos de simples hábito e de regra. Todas as sociedades, explicou, são formadas por uma série de jogos linguísticos particulares, e nestes encontramos alguns que se expressam como hábitos (os quais não têm uma sanção associada) e outros que se revelam como regras sociais (marcadas pela presença de sanções ou da pressão social).

    A grande revolução empreendida por Hart aconteceu com o livro O conceito de direito, no qual define a lei como a união de regras primárias e regras secundárias – as primeiras dizendo respeito ao mandamento de obrigações e as segundas aos modos de se compreender a aplicação das obrigações. Ele afasta a noção de lei apenas como comando, afirmando que, para além do mal da punição, do desprazer, que o comando traz, o que é fundamental para o conceito de direito é a noção de reconhecimento. Isso significa dizer que a comunidade daqueles que participam da esfera pública sempre encontra um modo de identificar qual é a regra jurídica e qual o discurso legitimador para a necessidade social, ou seja, reconhecem o lugar da lei. Com efeito, no caso do direito, o reconhecimento se legitima não por simples medo do mal punitivo, mas pelo fato de que, quando queremos saber o que é o direito, sabemos exatamente para onde olhar. Hart faz do positivismo legal uma teoria da sociedade.
  • Assim, a lei depende de uma regra de reconhecimento, que é social, o que faz descrever seu projeto como uma sociologia analítica. Nessa ótica, o fundamento da separação entre direito e moral não é jurídico, mas social. Isso significa dizer que os participantes do jogo de linguagem da lei devem reconhecer que esta tem o poder de regularizar, ao realizar a organização social. Entre os atores falantes da temática das regras, todos nós, os juízes e os advogados possuem privilégio na criação de modos de reconhecimento, mas apenas porque pretensamente estão mais envolvidos. O comando é importante para a ordem pública, pois necessitamos de aparelhos que monopolizem a sanção, mas não é fundamental para a compreensão do fenômeno do direito. Assim, para Hart, o principal para se entender o direito é a noção de que as leis existem porque reconhecemos nelas autoridade para a regulação do mundo da vida. O direito não impõe sua necessidade, mas a retira da vida social.

    Originalmente publicado no Nº 276 da Revista Ciência Hoje.
    Cesar Kiraly

  • Em sua obra "O Conceito de Direito" Herbert Hart busca dissociar o conceito de Direito da moral, expondo, sobretudo, que as leis devem se afastar dos preconceitos sociais, caracterizando-se como um comando a ser imposto por uma autoridade a ser efetivamente exercida,sendo somente essa autoridade legitimada para a aplicação da regra jurídica. Para Hart o ponto nodal de sua teoria está no legitimado para a aplicação da lei.  (TEORIA SUBJETIVISTA). O modo de obediência era centralizador e não havia pluralidade, de acordo com seu entendimento.

    Em discussão com Jeremy Benthan, Herbert Hart concorda com o preceito estabelecido por Austin no sentido de que a lei é verdade em sua forma, desconsiderando o seu conteúdo social.( TESE DA NEUTRALIDADE) Logo, o simples fato de existir já rechaça qualquer influência de ordem social ou moral. A LEI, REGRA JURÍDICA, É IMPOSTA PELA AUTORIDADE QUE A CRIA. EXISTE UMA CENTRALIZAÇÃO DO PODER LEGAL.
    RESSALTE-SE QUE O SISTEMA JURÍDICO INGLES É JURISPRUDENCIAL, LOGO, AS REGRAS SÃO CONSTRUÍDAS A PARTIR DE UMA LÓGICA SOCIAL, SENDO POSTERIORMENTE IMPOSTAS.
    O comando é importante para a ordem pública, pois necessitamos de aparelhos que monopolizem a sanção, mas não é fundamental para a compreensão do fenômeno do direito. Assim, para Hart, o principal para se entender o direito é a noção de que as leis existem porque reconhecemos nelas autoridade para a regulação do mundo da vida. O direito não impõe sua necessidade, mas a retira da vida social.



     

  • Prezados colegas,
    A tese da lei, tese da subsunção e a tese da legalidade são teses identificadas por Norberst Hoerst. A tese da lei pode ser entendida como aquela que defende que o conceito de Direito se define segundo conceito da lei. A tese da subsunção entende que aplicação do direito pode ser em qualquer caso por um metódo lógico simples, e, a tesa da legalidade exprime a ideia de obediencia absoluta às leis. 

    Hoerst aponta que a crítica ao positivismo jurídico, em sintese, refere-se a ideia do Direito que se constrói  sem indagações valorativas, essa crítica pode ser relacionada a duas ideias basica:  a tese da neutralidade e a tesa do subjetivismo. A tese da neutralidade aponta que o conceito de direito deve ser estabelicido dispensando o seu conteudo, assim qualquer norma criada regularmente pelos poderes legalmente instuídos, é norma válida e imperativa, independete de seu conteudo, e a tese do subjetivismo, segundo a qual  qual os critérios do Direito "reto" são de natureza subjetiva.
    fonte:  Das Criticas e dos Preconceitos  ao Positivismo Juridico, de  Roberto Denis Machado.
  • Não concordo com o enunciado da questao. Me recuso a fazê-la.

    Hart é a favor de uma Regra de Reconhecimento para validação das normas e do ordenamento jurídico, isto é, os funcionários públicos devem efetivamente aceitar e aplicar as normas, daí há um componente moral no direito SIM.

     

    "Em qualquer comunidade há uma sobreposição parcial de conteúdo entre a obrigação jurídica e a moral; embora as exigências das regras jurídicas sejam mais específicas e estejam rodeadas por exceções mais detalhadas do que as correspondentes regras morais (H.L.A. Hart CD, p. 185)".

    "Hart defende uma conexão entre direito e moral, em que pese sua autonomia."

    https://jus.com.br/artigos/17484/a-moral-em-hart

    Deve ter sido algum desembargador idoso e frustrado que elaborou essa questão que aproveitou a oportunidade para querer ficar famoso com uma questão polêmica sobre o único livro de filosofia que leu na vida. Banca local é sempre assim. Tem esses doutores amadores.

     

    #Olavotemrazão

     

     

     

     

  • John Austin constitui-se como patrono do moderno positivismo jurídico ao apartar o Direito do “contexto social”78, o que foi seguido por Hart e Hans Kelsen ao escrever a Teoria pura do direito. Jhon Austin é o pai de Kelsen e Hart (brincadeira, mas bom para lembrar!) Não é John Rawl é John Austin; parecido com Austin Powers!!!

    Abraços


ID
298594
Banca
CESPE / CEBRASPE
Órgão
DPU
Ano
2007
Provas
Disciplina
Filosofia do Direito
Assuntos

Conhecemos pouco dos sofistas. Em primeiro lugar,
porque, com exceção de um sofista tardio, Isócrates, de quem
temos as obras, não possuímos senão fragmentos dos dois
principais sofistas: Protágoras de Abdera e Górgias de Leontini.
Em segundo, porque os testemunhos recolhidos pela doxografia
foram escritos por seus inimigos — Tucídides, Aristófanes,
Xenofonte, Platão e Aristóteles —, que nos deixaram relatos
altamente desfavoráveis nos quais o sofista aparece como
impostor, mentiroso e demagogo. Esses qualificativos
acompanharam os sofistas durante séculos e a palavra sofista era
empregada sempre com sentido pejorativo.

Marilena Chaui. Introdução à história da filosofia – dos pré-socráticos
a Aristóteles
. São Paulo: Cia. das Letras, 2002 (com adaptações).


Tendo o texto acima como referência inicial, julgue os itens que
se seguem.

Desde o final do século XIX, tem-se observado uma reabilitação da sofística. Historiadores da filosofia, a partir de então, consideram os sofistas fundadores da pedagogia democrática mestres da arte da educação do cidadão.

Alternativas
Comentários
  • ITEM CONSIDERADO CORRETO NOS GABARITOS (PRÓVISÓRIO E DEFINITIVO).
  • "Sofistas, palavra que significa sábio em grego. 
    Esses homens, portadores de uma eloquência incomum, propunham ensinar qualquer coisa aos cidadãos que almejassem os cargos públicos ou simplesmente que se defenderiam em um caso litigioso. No entanto, suas técnicas nada mais eram do que ensinar a persuadir convencendo seu interlocutor em um debate, seja pela emoção, seja pela passividade deste. Ardilosos oradores, os sofistas fascinavam àqueles que ouviam suas palestras, ensinando como transformar um argumento fraco em um argumento forte e vice-versa. Para eles, fácil era convencer conforme seus interesses, por isso conseguiam provar que uma coisa ora era branca, ora preta. O importante era convencer a qualquer custo. Mediante salários (ou seja, cobravam pelo ensino), eles ensinavam a quem pudesse pagar, sobre qualquer coisa, dizendo serem portadores de um saber universal." 

    Fonte: www.brasilescola.com

  • Em uma sociedade que mede o sucesso pelo impacto imediato de que se reveste o quotidiano e não escapa a uma certa superficialidade e conotação de sensacionalismo efémero, não é de espantar a ressurreição do sofismo como pedagogia educacional.

    Mas o que o texto aborda é que os Sofistas abriram a possibilidade de aprendizado e acesso a todos os interessados em apreender, até então privilégio das elites. Desde o final do século XIX, tem-se observado uma reabilitação da sofística. Historiadores da filosofia, a partir de então, consideram os sofistas fundadores da pedagogia democrática mestres da arte da educação do cidadão.

    Por este viés, correta a alternativa, pese parecer que o examinador possui um conhecimento raso de filosofia, limitado a citações de almanaque.

    Veja passagem da querida professora Maria Vaz em seu texto A ESCOLA, A LINGUAGEM E O PODER:

     

     

    Não se via com bons olhos que recebessem um pagamento em dinheiro pelos seus préstimos de docentes, assim como representava um motivo difuso de mal estar que qualquer um pudesse, através da nova educação de que eram agentes esses mestres, conseguir uma posição social que fora até então o privilégio das elites, tradicionalmente adstritas a determinados estratos sociais. Na debatida questão acerca da possibilidade do ensino da aretê, os sofistas alinharam obviamente com aqueles que defendiam a viabilidade de conseguir levar a cabo essa tarefa com o desejado bom êxito! (A ESCOLA, A LINGUAGEM E O PODER - Profª Drª Maria José Vaz Pinto ).

  • Os sofistas não foram vistos com bons olhos, principalmente por Sócrates, Platão e Aristóteles, exatamente porque foram reconhecidos como os "vendilhões da palavra". Essa questão do CESPE pode gerar desacordo. Não acredito que, embora fossem considerados os fundadores da pedagogia democrática, fossem os "mestres da arte da educação do cidadão". No entanto, não podemos negar que os sofistas foram um marco na evolução do pensamento filosófico. Desta forma, a questão está correta!
  • Análise da questão:

    Existe uma ligação entre a reabilitação da sofística com a “arte" de educação do cidadão, de tal forma que podemos falar em verdadeiras contribuições dos sofistas à educação.

    Conforme SANTOS E SILVA (2015) “o termo sofista provém das palavras gregas sophos (sábio), sophia (sabedoria), de tal modo que “sofista" quer dizer sábio ou “professor de sabedoria" (ARANHA, 1992, p. 219), cuja denominação inicialmente está vinculada a quem possui uma capacidade específica e que mais tarde o sentido do termo denota conhecimento não só de parte, mas do todo. Embora a palavra sofista tenha surgido com significado positivo, devido às críticas filosóficas da época, acabou ganhando uma conotação pejorativa que duraria por muito tempo e que somente a partir do século XIX restabeleceria sua significação originária.

    As contradições sobre os sofistas representaram um preconceito histórico. O movimento da sofística chegou a ser considerado um período de declínio do pensamento grego, contudo, “[...] essa  imagem, de certa forma caricatural, da sofística, tem sido reelaborada no sentido de procurar resgatar a verdadeira importância do seu pensamento" (ARANHA, 1992). Assim, “[...] os sofistas foram reabilitados por Hegel no século XIX" (ARANHA, 1992, p. 219). Posteriormente, no século XX, “[...] foi possível uma revisão sistemática desses juízos" (REALE, 2011), revelando tamanha importância dos sofistas para o conhecimento; desse modo, “[...] a conclusão a que se chegou é que os sofistas constituem um elo essencial na história do pensamento antigo" (REALE, 2011, p. 73).

    Dessa maneira, a sofística, mediante pesquisas realizadas mais recentemente, desvencilha-se da visão inadequada e ganha destaquecomo importante movimento intelectual".

    A assertiva, portanto, está correta.

    Fontes:
    ARANHA, M. L. de A.; MARTINS, M. H. P. Introdução à filosofia. São Paulo: Moderna, 1992.
    REALE, G. História da filosofia Antiga: léxico, índices, bibliografia. 5. ed. São Paulo: Loyola, 1995.
    SANTOS, Valmir Francisco; SILVA, Paulo Rogério. Algumas contribuições dos sofistas à educação Educação. Batatais, v. 5, n. 1, p. 95-108, 2015


    Gabarito: CERTO
  • GABARITO: CERTO

    Diferente da lógica aristotélica, por exemplo, em que para se chegar a uma conclusão verdadeira, a premissa deve ser verdadeira, o sofismo está preocupado em vencer as discussões pela defesa do argumento, ainda que estes não fossem válidos. Ou seja, a busca pela verdade não era o objetivo, até porque, os sofistas a consideravam relativa, mas o convencimento da verdade.

    Fonte: https://www.educamaisbrasil.com.br/enem/filosofia/sofistas


ID
298597
Banca
CESPE / CEBRASPE
Órgão
DPU
Ano
2007
Provas
Disciplina
Filosofia do Direito
Assuntos

Conhecemos pouco dos sofistas. Em primeiro lugar,
porque, com exceção de um sofista tardio, Isócrates, de quem
temos as obras, não possuímos senão fragmentos dos dois
principais sofistas: Protágoras de Abdera e Górgias de Leontini.
Em segundo, porque os testemunhos recolhidos pela doxografia
foram escritos por seus inimigos — Tucídides, Aristófanes,
Xenofonte, Platão e Aristóteles —, que nos deixaram relatos
altamente desfavoráveis nos quais o sofista aparece como
impostor, mentiroso e demagogo. Esses qualificativos
acompanharam os sofistas durante séculos e a palavra sofista era
empregada sempre com sentido pejorativo.

Marilena Chaui. Introdução à história da filosofia – dos pré-socráticos
a Aristóteles
. São Paulo: Cia. das Letras, 2002 (com adaptações).


Tendo o texto acima como referência inicial, julgue os itens que
se seguem.

A sofística é uma arte e uma ciência. Além de um modo de ensinar, ela designa uma doutrina, tal qual a dos filósofos diferindo da destes apenas por seus desdobramentos práticos e por suas implicações políticas.

Alternativas
Comentários
  • Na visão de Aristóteles: "A sofística é uma sabedoria aparente e sofista é quem visa essa sabedoria aparente". Apesar de seus discursos conterem questões filosóficas, é-lhes negado o caráter de filósofos, visto serem considerados pregadores de uma Filosofia falsa.

    Os maiores sofistas foram: Górgias (cético), Protágoras (relativista, para alguns; subjetivista para outros), Hípias, Prodico e Eutidemo.

    Sócrates foi grande opositor dos sofistas (era da classe dominante, aristocrático). (p. 2899, vol. letras R, S; PAPE - Programa Auxiliar de Pesquisa Estudantil.

    Dicionário Aurélio: 

    para o vocábulo sofista:  "Os sofistas desenvolvem especialmente a retórica, a eloquência e a gramática."

    Górgias: que atribuiu grande importância à linguagem.

    Dicionário Aurélio: 

    para o vocábulo sofística: 1. "o conjunto das doutrinas dos sofistas gregos. 2.Tendência intelectual que lhes é comum. 3. Arte de sofismar.

    O ITEM ESTÁ ERRADO, porque a sofística é o conjunto das doutrinas dos sofistas gregos. E pertence ao campo da filosofia na atualidade, embora na época em que nasceu o fenômeno não fosse reconhecido como tal. O item menciona que é uma doutrina isolada e paralela à filosofia, o que está equivocado.

    os senhores concordam? Alguém tem o conceito de sofística pelo dicionário Houaiss?
  • É uma arte enquanto retórica e uma ciência enquanto linha filosófica. Forte inspiradora de NIetszche, afastou a moral e a religião como freios a realização do homem. Designa sim uma doutrina, não tal qual a dos filósofos, pois ela mesma é uma linha filosófica. Se a comparação é em relação a escola de Sócrates, posteriormente evoluída na linha platônica e aristotélica, os desdobramentos vão além de questões práticas e implicações políticas, discernindo na própria essência da construção teórica. 

    Os sofistas são os primeiros a romperem com a busca pré-socrática por uma unidade originária (a physis) iniciada com Tales de Mileto e finalizada em Demócrito de Abdera (que embora tenha falecido pouco tempo depois de Sócrates, tem seu pensamento inserido dentro da filosofia pré-socrática).

    A principal doutrina sofística consiste, em uma visão relativa de mundo (o que os contrapõe a Sócrates que, sem negar a existência de coisas relativas, buscava verdades universais e necessárias). A principal doutrina sofística pode ser expressa pela máxima de Protágoras: "O homem é a medida de todas as coisas".

    Tal máxima expressa o sentido de que não é o ser humano quem tem de se moldar a padrões externos a si, que sejam impostos por qualquer coisa que não seja o próprio ser humano, e sim o próprio ser humano deve moldar-se segundo a sua liberdade.

    Outro sofista famoso foi Górgias de Leontini, que afirmava que o 'ser' não existia. Segundo Górgias, mesmo que se admitisse que o 'ser' exista, é impossível captá-lo. Mesmo que isso fosse possível, não seria possível enunciá-lo de modo verdadeiro e, portanto, seria sempre impossível qualquer conhecimento sobre o 'ser'.

    Estas visões contrastantes com a de Sócrates (que foram adotadas também por Platão e Aristóteles, bem como sua "luta" anti-sofista) somada ao fato de serem estrangeiros - o que lhes conferia um menor grau de credibilidade entre os atenienses - contribuiu para que seu pensamento fosse subvalorizado até tempos recentes.

  • Tb não é considerado ciência até onde eu saiba... arte até pode ser, por causa da retórica, acredito...

  • Analisando a questão:

    “Sofística" (do lat. sophisticus, do gr. sophistike) é uma denominação genérica do conjunto de doutrinas de filósofos contemporâneos de Sócrates e Platão, conhecidos como sofistas. A sofística se caracteriza pela preocupação com questões práticas e concretas da vida da cidade, pelo relativismo em relação à moral e ao conhecimento, pelo antropocentrismo, pela valorização da retórica e da oratória como instrumentos da persuasão que caracterizava a função do sofista, e, em consequência, pelo conhecimento da linguagem e domínio do discurso, essenciais para o desenvolvimento da argumentação sofística. A sofistica não chegou a constituir propriamente uma escola, porém o termo é utilizado, frequentemente com sentido negativo, sobretudo para designar o contraste entre o racionalismo teórico e especulativo da filosofia de Sócrates, Platão e Aristóteles, com a atitude pragmática e antimetafisica dos sofistas".

    Acredito que a sofística não pode ser considerada uma ciência, afirmação que torna a assertiva falsa.

    Fonte: Dicionário de Filosofia. Hilton Japiassú e Danilo Marcondes. Rio de Janeiro. Jorge Zahar Editor, 2001.

    Gabarito: ERRADO
  • "Os sofistas representaram a ruptura com o pensamento mitológico dos pré-socráticos, voltando o pensamento dos deuses para os homens. Famosa frase: “o homem é a medida de todas as coisas”.

    Os sofistas romperam com a herança cultural pré-socrática ao voltar a filosofia para o estudo do homem, como ser individual e social, colocando-se como radicais opositores da tradição, sobre definições absolutas, conceitos fixos e eternos, sobre tradições inabaláveis, com isso relativizando o conceito de justiça que é igualado ao conceito de lei, de legalidade.Os sofistas, tais como Protágoras e Górgias, relativizaram a possibilidade de conhecimento, afirmando, no que se refere ao direito, que o senso de justiça não advém de deuses, mas sim do fruto das convenções humanas, variando com o tempo e com o imperativo das circunstâncias. Ademais, eles apontavam a identidade entre a legalidade e a justiça, de modo a favorecer o desenvolvimento de ideias que associavam à inconstância da lei a inconstância do justo.Por passarem o foco ao homem, os sofistas foram tidos como os precursores da sociologia. Por intermédio do relativismo, eles inauguraram a crítica social, já que lançaram dúvidas sobre a capacidade de justiça da polis grega". (RESUMO DE HUMANÍSTICA).

  • "A assertiva é falsa. Falsa vez que seus críticos, denominados filósofos pelo examinador, criticavam sua essência de educar, e não por razões políticas. A sofística, portanto, não pode ser tratada como uma doutrina, mas como um parâmetro de educação, essencialmente criticado por ser um sistema de formação de pessoas e pensadores que não se preocupava com o que estava formando, mas apenas com o lucro na formação de estadistas.

    A diferença residia nos ideais. Enquanto Sócrates pregava a justiça e a solidariedade por meio da educação, os sofistas pregavam o individualismo, ao saber em si considerado, ainda que o conhecimento geral de tudo, quando Sócrates revelava que nada sabia."

    http://ww3.lfg.com.br/public_html/article.php?story=20100610152444680  

  • Acho que o termo ciência nesse contexto é utilizado como técnica (ou mesmo como episteme em oposição a doxa). Até porque a própria filosofia não é científica, em termos atuais. Achei confusa a questão, talvez o erro seja dizer que a sofistica seja uma doutrina, tendo em vista a diversidade de teses dos sofistas. Acho que atualmente ninguém discorda que os sofistas foram filósofos e que utilizavam não só da erística como também da reflexão sobre a "verdade" e sua possibilidade. Minha opinião...

  • GABARITO: ERRADO

    Diferente da lógica aristotélica, por exemplo, em que para se chegar a uma conclusão verdadeira, a premissa deve ser verdadeira, o sofismo está preocupado em vencer as discussões pela defesa do argumento, ainda que estes não fossem válidos. Ou seja, a busca pela verdade não era o objetivo, até porque, os sofistas a consideravam relativa, mas o convencimento da verdade.

    Fonte: https://www.educamaisbrasil.com.br/enem/filosofia/sofistas


ID
298600
Banca
CESPE / CEBRASPE
Órgão
DPU
Ano
2007
Provas
Disciplina
Filosofia do Direito
Assuntos

Muitas têm sido as explicações das causas históricas para a origem da filosofia na Jônia. Alguns consideram que as navegações e as transformações técnicas tiveram o poder de desencantar o mundo e forçar o surgimento de explicações racionais sobre a realidade. Outros enfatizam a invenção do calendário (tempo abstrato), da moeda (signo abstrato para a ação de troca) e da escrita alfabética (transcrição abstrata da palavra e do pensamento), que teriam propiciado o desenvolvimento da capacidade de abstração dos gregos, abrindo caminho para a filosofia.

Idem, ibidem

Tendo como referência inicial o texto acima, julgue o item a seguir.

A formação da pólis, a cidade-Estado, é a principal determinação histórica para o nascimento da filosofia.

Alternativas
Comentários
  • ITEM CONSIDERADO CORRETO NO GABARITO PROVISÓRIO E NO DEFINITIVO.
  • A Pólis foi o berço para que a civilização grega criassem elementos como o calendário, a moeda e a escrita, o que trouxe a compreensão do tempo enquanto elemento abstrato e dimensão do ser, a matemática como linguagem exata por excelência e a escrita como firme fundamento para consolidar e repassar o conhecimento adquirido. A Pólis também permitiu uma estrutração produtiva  que deu lugar a uma maior disposição para o ócio, elemento igualmente essencial para a atividade de reflexão e construção doconhecimento;
  • Análise da questão:

    Para Jean Pierre Vernant, em As origens do pensamento grego, o aparecimento da polis constitui, na história do pensamento grego, um acontecimento decisivo. A palavra adquire uma preeminência sobre os demais instrumentos de poder e do caráter público das manifestações da vida social. É nesse contexto da pólis, portanto, que os saberes e os valores vão se tornando acessíveis a uma esfera cada vez mais ampla da população. Assim, sem muito esforço ,é correto concordar que a formação da pólis, a cidade-Estado, é a principal determinação histórica para o nascimento da filosofia.

    A assertiva, portanto, está correta.

    Fonte: VERNANT, Jean Pierre. As Origens do Pensamento Grego. Rio de Janeiro, Difel, 2002.


    Gabarito: CERTO
  • O que mais contribuiu para a busca de um princípio explicativo da natureza e do ser, foi o desenvolvimento da política enquanto prática social humana, com o desenvolvimento da Polis grega.

    CERTO


ID
298618
Banca
CESPE / CEBRASPE
Órgão
DPU
Ano
2007
Provas
Disciplina
Filosofia do Direito
Assuntos

A respeito do peso das Ciências Sociais e da Sociologia em suas
relações com as demais áreas do conhecimento humano, julgue
os itens que se seguem.

O conceito de relações de poder confere mobilidade ao conceito tradicional de poder, relacionando-o à idéia de exercício e saber.

Alternativas
Comentários
  • ITEM CONSIDERADO CERTO NO GABARITO PROVISÓRIO E DEFINITIVO.
  • Focault, membro da Escola Objetiva Francesa e do grupo de Émile Durkheim, ensina que o poder está vinculado ao exercício do saber, que tem a função política e de controle da sociedade. 

  • Análise da questão:

    Michel Foucault, nas obras A ordem do discurso e Microfísica do poder, é um dos autores que expõe a relação entre o saber e o poder nas relações sociais, bem como teoriza acerca das amarras sociais desenvolvidas nessa interação. Conforme o autor, “O exercício do poder cria perpetuamente saber e, inversamente, o saber acarreta efeitos de poder." (FOUCAULT, 2007, p. 80).

    A assertiva está correta.

    Fontes:
    FOUCAULT, Michel. A ordem do discurso. Aula inaugural no College de France, pronunciada em 2 de Dezembro de 1970. Tradução de Laura Fraga de Almeida Sampaio. Edições Loyola, São Paulo, 2004.
    FOUCAULT, Michel. Microfísica do poder. Tradução Roberto Machado. 24. ed. Rio de Janeiro: Edições Graal, 2007.


    Gabarito: CERTO
  • GABARITO CERTO

     

    Para quem gosta de sociologia, aí vai!

     

    Poder é um termo que  derivou do latim'' possum,'' que significa “ser capaz de”, 

    Segundo a sociologia, poder é a habilidade de impor a sua vontade sobre os outros, não somente de maneira persuasiva e ativa, mas de maneira insconsciente e passiva. No entanto existem mais de um tipo de poder: o poder social, o poder econômico, o poder militar, o poder político, ''O PODER AMOROSO'' ,CUIDADO, ESSE MATA!(...)

    Um sociólogo muito importante para a ciência social, é max weber, este escreveu sobre diversos assuntos peculiares na sociedade, INCLUSIVE SOBRE ÉTICA PROTESTANTE E O ESPÍRITO DO CAPITALISMO , que com certeza tem a vê com poder, porém não cheguei a terminar nenhum de seus livros físicos, no entanto, eu tive contato com ele (em diversos livros didáticos de sociologia, creio que li uns 3, 4) , no entanto numa visão simples e resumida de sua estrutura sociológica, a visão de Weber é de uma Teia de Relações, semelhante a uma teia de aranha . Os pontos se encontram formando um todo, que representa o todo da teia, ou, transferindo para a realidade, a sociedade em seu ''íntegro''. Cada individuo, nessa configuração, se relaciona com o outro num processo positivo que influi o “bom funcionamento da sociedade”. Porém, para que haja esse bom funcionamento da sociedade, na ideia de ''teias gigantes'', é necessário um elemento que force os indivíduos a agirem de tal forma.Para Weber, esse elemento  é inerente ao ser humano e chamado por ele de espíritus. E por chegarmos ao assunto espírito, logo, perceberá que o que descrevo está se derivando para o assunto ''ética'', no entanto, uma das célebres obras de webber, é A ética protestante e o espírito do capitalismo. E sendo a partir da ética e moral, que o indivíduo , tendo em si o espírito, haverá de escolher como atuar através destas teias, mesmo notando que se um não se move nas teias, este um fica bloqueado, e preso a mesma classe social. Terminando,a ideia de poder em webber, significa a probabilidade de impor a própria vontade dentro de uma relação social, mesmo que contra toda a resistência e qualquer que seja o fundamento dessa probabilidade.

    Para pierre bordieu, é mais interessante ainda.  Recomendo ,para você que leu até aqui, que pesquise sobre. (Bordieu é um seguidor de webber, de uma forma intelectual, não falo que ele é fã...)

     

    Bons estudos!

     


ID
726667
Banca
FCC
Órgão
DPE-SP
Ano
2012
Provas
Disciplina
Filosofia do Direito
Assuntos

A Ciência do Direito (...), se de um lado quebra o elo entre jurisprudência e procedimento dogmático fundado na autoridade dos textos romanos, não rompe, de outro, com o caráter dogmático, que tentou aperfeiçoar, ao dar-lhe a qualidade de sistema, que se constrói a partir de premissas cuja validade repousa na sua generalidade racional. A teoria jurídica passa a ser um construído sistemático da razão e, em nome da própria razão, um instrumento de crítica da realidade”.

Esta caracterização, realizada por Tercio Sampaio Ferraz Júnior, em sua obra A Ciência do Direito, evoca elementos essenciais do

Alternativas
Comentários
  • Trecho do livro: A Ciência do Direito, nos quadros do jusnaturalismo, se de um lado quebra o elo entre jurisprudência e procedimento dogmático fundado na autoridade dos textos romanos, não rompe, de outro, com o caráter dogmático, que tentou aperfeiçoar, ao dar-lhe a qualidade de sistema, que se constrói a partir de premissas cuja validade repousa na sua generalidade racional.  A teoria jurídica passa a ser um construído sistemático da razão e, em nome da própria razão, um instrumento de crítica da realidade.
  • O Jusnaturalismo é subdividido em:
    - Jusnaturalismo Sofistico;
    - Jusnaturalismo Estoico;
    - Jusnaturalismo Romano;
    - Jusnaturalismo Medieval;
    - Jusnaturalismo Moderno;
    - Jusnaturalismo Ilustrado;
    O trecho colocado na prova referia-se ao moderno, conforme explicitou a colega acima.
  • A inclusão dessas matérias nos editais de concurso vai tirar muita gente do páreo...eu, modéstia parte, vou dançar!! Usando apenas meu bom senso eu não acerto nada....estou pensando em um suicídio lento.
  • Jusnaturalismo Moderno

    A esfera política da era moderna foi marcada pelo surgimento do Estado Moderno, tendo como principal característica a centralização do poder. Nesse periodo a ideia de direito natural foi absorvida e adaptada, prevalescendo a idéia de que o direito natural tinha origem na razão. Nessa época foi muito importante a doutrina de Grócio que excluiu a figura de Deus da idéia do direito natural, difundindo essa idéia de direito natural e da necessidade de que o direito positivo e as Constituições dos Estados se adequarem a esse direito.

    A principal diferença é que enquanto no jusnaturalismo antigo e medieval o direito natural consistia numa norma objetiva, no moderno trata-se de uma doutrina exclusivamente de direito subjetivos.

    Com o surgimento das teorias contratualistas surgem novas idéias que dão uma “nova cara” ao conceito de direito natural, revitalizando o jusnaturalismo, ressaltando o seu aspecto subjetivo. Esse jusnaturalismo moderno tem grande influência nas doutrinas políticas de tendência liberal, ressaltando a importância de que a as autoridades políticas respeitem os “direitos inatos do individuo”.

    Ao definir o direito natural como subjetivo diminui-se um pouco a sua força, pois o exercício dos direitos fica, em muitos casos, sujeito ao exercício voluntário do individuo. Isto ocorre em virtude do surgimento de um Estado que define a lei objetiva. O Estado passa a ser considerado, portanto, uma obra voluntária dos individuos que tem a obrigação de proteger os direitos naturais.

  • Resumo do panorama histórico tratado por Tércio:

    1) Jurisprudência Romana: - evitou a expressão ciência; - muito mais ligada à prática; Tópica; ligada ao empirismo (método indutivo – abstração a partir do caso concreto, de tal sorte a formular uma regra geral); o jurista coloca um problema e trata de encontrar argumentos para soluciona-lo; - a jurisprudência romana é um exemplo de ciência prática: problemática da chamada ciência prática que não apenas contempla e descreve, mas também age e prescreve. 2) Glosadores:

    - “Ciência” europeia do Direito – Bolonha, sec. XI;

    - objeto: textos de Justiniano;

    - método: trivium (gramática, dialética e retórica);

    - caráter exegético (interpretativo) de seus propósitos – a margem ampla de interpretação prejudicaria a cientificidade;

    3) Jusnaturalismo Moderno:

    - sec XVI – surge como uma crítica aos glosadores;

    - séc XVII e XVIII – ligação entre ciência e pensamento sistemático;

    - identifica, pela primeira vez, o direito como SISTEMA (entendido como organismo, mecanismo e ordenação);

    - caráter lógico-demonstrativo de um sistema fechado;

    - natureza decaída do homem (Pufendorf);

    - método: dedução racional + observação empírica; a partir do dualismo cartesiano (método analítico* e sistemático).



  • Antes, era decoreba de leis. Daí eles queriam aprofundar e melhorar isso... Só que virou decoreba de Teorias. E os burros são os candidatos... Cadê a disposição do Examinador ao raciocínio?

  • GABARITO LETRA A

    “A Ciência do Direito, nos quadros do jusnaturalismo, se de um lado quebra o elo entre jurisprudência e procedimento dogmático fundado na autoridade dos textos romanos, não rompe, de outro, com o caráter dogmático, que tentou aperfeiçoar, ao dar-lhe a qualidade de sistema, que se constrói a partir de premissas cuja validade repousa na sua generalidade racional. A teoria jurídica passa a ser um [construto - ?] sistemático da razão e, em nome da própria razão, um instrumento de crítica da realidade. Duas contribuições importantes, portanto: a) o método sistemático conforme o rigor lógico da dedução; b) o sentido crítico-avaliativo do direito posto em nome de padrões éticos contidos nos princípios reconhecidos pela razão”.

    P.S.: excelente o comentário de karine pereira!!

  • A questão exige conhecimento de conceito contido na obra A Ciência do Direito, Conforme, de Tércio Sampaio Ferraz Júnior. Segundo Tércio, “A Ciência do Direito, nos quadros do jusnaturalismo, se de um lado quebra o elo entre jurisprudência e procedimento dogmático fundado na autoridade dos textos romanos, não. Tompe, de outro, com o caráter dogmático, que tentou aperfeiçoar, ao dar-lhe a qualidade de sistema, que se constrói a partir de premissas cuja validade repousa na sua generalidade racional. A teoria jurídica passa a ser um construído sistemático da razão e, em nome da própria razão, um instrumento de crítica da realidade. Duas contribuições importantes, portanto: a) o método sistemático conforme o rigor lógico da dedução; b) o sentido crítico-avaliativo do direito posto em nome de padrões éticos contidos nos princípios reconhecidos pela razão.

    Gabarito do professor: letra a.

    Fonte:

    FERRAZ JÚNIOR, Tércio Sampaio. A Ciência do Direito. São Paulo: Atlas, 1977.


  • Jusnaturalismo Moderno: Direito é sistema:

                                              Lógico;

                                              Demonstrativo;

                                               De estrutura fechada com regras próprias;

                                               Princípios comuns.


ID
761656
Banca
FCC
Órgão
DPE-PR
Ano
2012
Provas
Disciplina
Filosofia do Direito
Assuntos

A contribuição da Filosofia para o exercício do ser Defensor Público que somente se realiza sendo Defensor Público, é:

Alternativas
Comentários
  • Vamos chutar um comentário sobre essa questão:
    A) Não tenho a mínima ideia
    B) Correta
    C) A filosofia não é pensar descompromissado, tem método, é racional.
    D) A filosofia não é visão histórica, quem estuda isso é história.
    E) Filosofia não é erudição. erudito é quem tem sabedoria, instrução, todo filósofo é erudito, mas nem todo erudito é filósofo.
    Chute básico. 
  • Questão absurda, e não sou eu quem diz, é Lenio Streck, vejamos o que o Pós-Doutor disse em recente artigo no Conjur: 
    Mas, se de um lado existem esses exageros dogmático-manualísticos, já começam a aparecer “adaptações darwinianas”, tentando “superar” o modelo “em vigor”. No Concurso para a Defensoria do Paraná, exageraram na dose “antitética”, ao perguntarem se a filosofia torna livre no Defensor o seu “Ser”... Em outra questão indagaram sobre Baumann e Bourdieu (não sei qual é a relação entre eles, mas, enfim...). OK. Tudo bem que tenhamos que mudar os concursos. Mas há que se ter cuidado para não “espantar o freguês”. Filosofia do Direito não é “capa de sentido”. E nem Sociologia é isso. Não basta “colar” conceitos. Essa questão sobre o “Ser” do Defensor (sic) foi demais. Aliás, se a questão queria ter tido algo a ver com o “Ser” (ou a questão do “Ser”) de que fala, por exemplo, Heidegger, é bom lembrar que ele dizia que “o ser não é um ente”; “o ser não é uma capa de sentido” (o ser não pode ser visto; ele só serve para dar sentido aos entes). Complicado, não? De todo modo, “uma no cravo, outra na ferradura”. Vejamos: o mesmo concurso pergunta sobre a hipótese de um presídio estar sem segurança e se, mesmo assim, as presas tinham direito ao sexo. Mesmo sem segurança, a resposta pedida era “sim”. Bom, direito todos tem a uma porção de coisas... mas isso a questão não definia. Outra questão era sobre um executivo do mercado financeiro que, demitido, estressado, atropelou pessoas, agrediu um motoqueiro, além de subtrair a sua motocicleta. Os meios de comunicação noticiaram o fato com “fervor”. O que foi perguntando? Que o candidato identificasse a teoria que melhor explica o caso: o labbeling approach, a teoria crítica, a Escola de Chicago, a associação diferencial ou a teoria da anomia... Uau. Não joguemos fora a água suja com a criança dentro. Sair de um modelo fast food para um modelo de perguntas desse tipo é não inovar em nada. Lamento informar, mas, com esse tipo de prova corremos o risco de retroceder, além de reforçar o lado tradicional. É mais ou menos como o professor de Filosofia do Direito que chega na aula e, como é doutor em Hegel, manda os alunos de primeiro ano lerem a obra de Hegel. Ele passa todo o semestre falando em Hegel, sem liga-lo à especificidade jurídica. Resultado: os alunos não só odiarão Hegel, como também o professor e a Filosofia do Direito. Ou seja, espanta-se a clientela.
    http://www.conjur.com.br/2012-ago-16/senso-incomum-vinhas-ira-ou-quando-reu-nao-ajuda
  • Indiquei para comentários. Kkk

  • Análise da questão:

    A banca, infelizmente, elaborou uma questão com forte conteúdo poético, mas desprovida de embasamento teórico. Não existem – ao meu ver - parâmetros objetivos que possam levar a uma resposta correta e sem margens para dúvidas.

    Resta, somente, apontar o que a banca considerou como correto. Talvez, com base no que ela considera a assertiva mais compatível com seus princípios institucionais: “A Filosofia torna livre no Defensor o seu Ser, a necessidade interna de resgate de sua essência mais própria, de modo a conferir a essa essência a sua dignidade de ser Defensor Público".

    A assertiva considerada correta é a letra “b".

    Gabarito: Alternativa B
  • acompanhando os chutes da GABRIELA eliminei a "A" porque o Defensor não pode ser distante..... como disse.. chute..

  • Questão marcada pelo critério RA.FO (a mais razoável e fofinha).

  • Conceitos puramente Marxistas, cujo objetivo é conduzir ao erro!

  • Misericórdia

  • Só Jesus na causa...


ID
761662
Banca
FCC
Órgão
DPE-PR
Ano
2012
Provas
Disciplina
Filosofia do Direito
Assuntos

A concepção de justiça que mais se aproxima de um dos objetivos, positivado, das Defensorias Públicas no Brasil é:

Alternativas
Comentários
  • Tão logo promulgada a CF/88, já entendíamos que os privilégios processuais (não 
    confundir com  jus imperii) da Fazenda Pública haviam desaparecido, especialmente agora 
    com o advento da Lei Complementar, bipartindo o Ministério Público com a Advocacia Geral 
    da União (CF/88, art. 131 e parágrafo único)  e em breve o surgimento também da Lei 
    Complementar (art. 134 da Cf/88) da Defensoria Pública.
  • LC 80

    Art. 3º-A.  São objetivos da Defensoria Pública: (Incluído pela Lei Complementar nº 132, de 2009).

    I – a primazia da dignidade da pessoa humana e a redução das desigualdades sociais; (Incluído pela Lei Complementar nº 132, de 2009).

    II – a afirmação do Estado Democrático de Direito; (Incluído pela Lei Complementar nº 132, de 2009).

    III – a prevalência e efetividade dos direitos humanos; e (Incluído pela Lei Complementar nº 132, de 2009).

    IV – a garantia dos princípios constitucionais da ampla defesa e do contraditório. (Incluído pela Lei Complementar nº 132, de 2009).

    Pela lógica, trata-se de promover o princípio da igualdade, e a alternativa que mais se aproxima disso, com o conceito de igualdade material é a letra C
  • Conceito clássico aristotélico do livro "A Política": Trate com igualdade os iguais e com desigualdade aos desiguais.

  • A questão exige conhecimento relacionado às perspectivas distintas do princípio constitucional da isonomia/igualdade, quais sejam: igualdade formal e igualdade material.

    A concepção de justiça que mais se aproxima de um dos objetivos, positivado, das Defensorias Públicas no Brasil é: Justiça que manda dar aos iguais coisas iguais e aos desiguais coisas desiguais. Essa concepção está relacionada à igualdade em sua faceta material.

    Historicamente, a concepção puramente formalista de igualdade demonstrou sua insuficiência em equacionar verdadeiramente a igualdade entre os indivíduos, já que os marginalizados seguiam sem acesso às mesmas oportunidades, bens e "condições de partida” que os socialmente favorecidos. Vedava-se um tratamento discriminatório pela lei, mas nada se fazia para mudar a situação fática e evitar a perpetuação das profundas desigualdades concretas que marcavam a vida social. Iniciou-se, então, um processo de questionamento dessa leitura oitocentista do princípio da isonomia, criando o cenário adequado para o robustecimento da perspectiva material (substancial), que considerasse as desigualdades reais existentes na vida fática, permitindo que situações desiguais fossem destinatárias de soluções distintas. Recuperava-se, com isso, a lógica aristotélica de que os desiguais devem ser tratados desigualmente, na medida da sua desigualdade.

     

    Gabarito do professor: letra c.



ID
811255
Banca
CESPE / CEBRASPE
Órgão
DPE-RO
Ano
2012
Provas
Disciplina
Filosofia do Direito
Assuntos

Assinale a opção correta à luz da filosofia do direito.

Alternativas
Comentários
  • Análise da questão:

    Conforme as lições de Paulo Dourado Gusmão (Introdução ao Estudo do Direito, 1997, Editora Forense):

    Piaget, do ponto de vista psicogenético, preocupou-se em demonstrar a heteronomia tanto da norma moral como da norma jurídica, bem como o fato de ambas, em suas origens, pressuporem uma autoridade, passando a primeira, gradualmente, da heteronomia para uma “autonomia'' relativa.

    Segundo Gusmão (1997, p. 67), “a criança, diz Piaget, inicialmente como deveres só conhece as instruções de seus pais. Dessas instruções ela tira novas normas por generalização e aplicação a outras pessoas, "até alcançar uma interiorização espiritualizada e autônoma deste conjunto que será incessantemente trabalhado''. Na origem de ambas as normas encontra­-se uma autoridade: na moral, a autoridade dos pais ou do educador, enquanto no direito a dos mais velhos. Assim, a coercibilidade não seria específica ao direito, pois, em sua origem, também estaria presente na moral".

    A assertiva correta, portanto, é a contida na letra "b".


    Gabarito: Alternativa B.
  • Quem acertou pelo menos uma de filosofia nessa prova, pode se sentir feliz na vida. Pois nem no chute está dando..


ID
811258
Banca
CESPE / CEBRASPE
Órgão
DPE-RO
Ano
2012
Provas
Disciplina
Filosofia do Direito
Assuntos

Considerando a relação entre direito e controle social, assinale a opção correta.

Alternativas
Comentários
  • Letra D chega a ser poética!!!
  • A questão exige conhecimento específico sobre a relação entre direito e controle social,  desenvolvida por Pedro Scuro Neto na obra “Sociologia Geral e Jurídica”. Conforme o autor, “O controle social assume portanto a fisionomia de um subsistema de normas (o Direito) sustentado pela autoridade de outro (o Estado), cujos agentes e equipamentos aplicam a lei, e, ao fazê-lo, criam Direito – as funções de investigar e acusar, por exemplo, ficam a cargo de pessoas especializadas, funcionários públicos dotados de recursos e autoridade para tanto (SCURO NETO, 2004, p. 202)”.

    Gabarito do professor: letra d.

    Referência:

    SCURO NETO, Pedro. Sociologia Geral e Jurídica: manual dos

    cursos de Direito. 5.ed. São Paulo: Saraiva, 2004.


  • Sobre a letra C:

    Insulamento burocrático é caracterizado como um fenômeno no qual a burocracia possui um alto grau de independência em relação aos controles político ou social. O insulamento burocrático seria uma estratégia das elites para superar a arena controlada pelos partidos. 

    positivo: Evitar pressões de interesse dos grupos poderosos

    negativo:  Deixar de ouvir a sociedade, reduzindo assim, a participação desta.

     

    fonte:  Bresser-Pereira (1997)


ID
811261
Banca
CESPE / CEBRASPE
Órgão
DPE-RO
Ano
2012
Provas
Disciplina
Filosofia do Direito
Assuntos

Com relação ao conceito de direito e de equidade, assinale a opção correta.

Alternativas
Comentários
  • Livro Segundo

    No segundo livro, Dante vai demonstrar que o Império Romano dominou o universo "sem qualquer resistência", portanto, não pela força, mas pelo Direito, ou seja, por obra da divina providência.

    “Após prolongada reflexão, porém, reconheci por sinais evidentes que a grandeza romana era obra da divina Providência.”[100]

    Neste livro não se percebe aspectos importantes para o presente estudo; o poeta o dedica a argumentar com passagens bíblicas e relatos de historiadores ilustres que Deus revelou a sua vontade para que o povo romano governasse o universo; com a conclusão de que se o povo romano é o legítimo governante do universo, e o Sacro Império Romano Germânico é o seu sucessor (como apregoavam naquela época), era o Sacro Império legítimo para governar o universo. Há, no entanto, uma passagem que se destaca. No capítulo V deste livro, Dante apresenta o conceito de Direito.

    “O direito é uma proporção real e pessoal de homem para homem que, servida, serve a sociedade, e, corrompida, a corrompe.”[101]

  • Sobre a "E"

    Me parece que o erro está no trecho "O direito pode ser definido como a justa aplicação da norma jurídica geral ao caso concreto", já que, de acordo com o Summum ius, summa iniuria, o direito não pode ser interpretado de forma literal rígida.

    Caso contrário, estar-se-á promovendo a injustiça e não a justiça.

    Defende-se que, na realidade, o apego à norma na sua pura e literal expressão gera a injustiça, pois Summum ius, summa iniuria = "o máximo do direito, o máximo da injustiça".

  • A questão em comento exige uma leitura refinada de Filosofia do Direito e de noções de equidade e da natureza do Direito.

    Vejamos a seguinte menção de Dante Aligheri, em sua obra “ Da Monarquia":

    (...)“o direito é uma porção real e pessoal, de homem para homem que, conservada, conserva a sociedade, corrompida, corrompe-a."

    Dante é um autor que cria uma concepção interessante de Direito ainda na Idade Média, de tal forma que o Direito, conservado, preservado, representa o bem, o justo, e garante a coesão e unidade social, ao passo que, uma vez corrompido, violado, gera a desagregação social.

    Feitas tais ponderações, nos cabe comentar as alternativas:

    LETRA A- INCORRETA. A equidade não é um condicionante das leis, sendo, via de regra, invocada nos casos de lacuna da lei

    LETRA B- INCORRETA. A equidade não nasce do Direito Objetivo, mas sim de uma perspectiva de Justiça ao caso concreto.

    LETRA C- INCORRETA. É uma assertiva com acrasias. Se o Direito tem princípios imutáveis, o Direito não se adapta à mudanças e contextos diferenciados conforme a realidade histórica e geográfica.

    LETRA D- CORRETO. Representa, de fato, pensar de Dante no livro “Da Monarquia".

    LETRA E- INCORRETO. Trata-se de uma construção genérica e que reduz o Direito à equidade, quando, em verdade, a equidade é só uma das dimensões do Direito.

    GABARITO DO PROFESSOR: LETRA D


ID
859471
Banca
MPE-PR
Órgão
MPE-PR
Ano
2012
Provas
Disciplina
Filosofia do Direito
Assuntos

Assinale a alternativa incorreta:

Alternativas
Comentários
  • Análise da questão:

    A concepção criminológica do labelling approach - que emergiu na década de 60 - desloca o objeto da investigação científica do crime, do criminoso e da pena para o sistema penal, buscando atribuir a este a produção das condições da criminalidade, ao contrário do paradigma etiológico, que concebia estas categorias como ontológicas e pré-constituídas" (BISSOLI FILHO, p. VI).

    “O labelling approach ou teoria do “etiquetamento social" demonstra que as condutas tuteladas pela lei penal não são lógicas, tal constatação está muito distante do saber dogmático e mais próximo do entendimento crítico da sociologia.

    Verificamos isso no saber dogmático, assim como nas instâncias oficiais de investigação e aplicação da Lei. Podemos afirmar que furtar é crime, porém, se o furto for praticado por uma pessoa rica que poderia facilmente comprar o produto furtado, seria considerado distração e etc.

    Esse fato traduz que o criminoso é selecionado pelas características do meio o qual está inserido, e não pela conduta criminosa, portanto, o sistema punitivo não combate a criminalidade, mas atribui rótulos através de uma convenção discursiva".

    A conceituação da labelling approach não condiz com a assertiva de letra “a". O anacronismo da legislação se dá por outros fatores que não é o da não adoção das ideias centrais da teoria supracitada.

    A alternativa incorreta, portanto, é a letra “a".

    Fontes:
    BISSOLI FILHO, Francisco. O ESTIGMA DA CRIMINALIZAÇÃO NO SISTEMA PENAL BRASILEIRO: DOS ANTECEDENTES À REINCIDÊNCIA CRIMINAL. 1997. 360 f. Dissertação (Mestrado) - Curso de Direito, Centro de Ciências Jurídicas, Universidade Federal de Santa Catarina, Florianópolis, 1997.
    CARDOSO, Fabio Fettuccia. O criminoso segundo a teoria do "labelling approach". 2015. Disponível em: <http://fabiofettuccia.jusbrasil.com.br/artigos/175...>. Acesso em: 14 mar. 2016.

    Gabarito: Alternativa A
  • "Em 1984, as leis 7.209/84 e 7.210/84 operaram modificações sensíveis na parte geral do código penal, reformulando-a. No tocante à introdução de institutos com fundamentos na principiologia do labelling, a medida mais importante foi a instituição do regime progressivo de cumprimento da pena privativa de liberdade previsto no § 2º do art. 33 do código penal. A lei 7.210/8437, com redação modificada pela lei 10.792/03, no que se refere aos requisitos para a progressão de regime, determinou: Art. 112. A pena privativa de liberdade será executada em forma progressiva com a transferência para regime menos rigoroso, a ser determinada pelo juiz, quando o preso tiver cumprido ao menos um sexto da pena no regime anterior e ostentar bom comportamento carcerário, comprovado pelo diretor do estabelecimento, respeitadas as normas que vedam a progressão. A leitura desses dispositivos permite observar que o legislador de 1984 incorporou as idéias do labelling, e concebeu o regime progressivo como uma forma de reinserir gradativamente o condenado no meio social, mitigando o inevitável processo de estigmatização por este sofrido."

    http://www.publicadireito.com.br/conpedi/manaus/arquivos/anais/salvador/sergio_reis_coelho.pdf

     

  • progressao de regime

    Podemos citar ainda a grande contribuição das penitenciárias para a implementação da lógica segregacionista e estigmatizante exposta no tópico anterior. Aqui se trata basicamente das implicações psicológicas que a condenação à pena privativa de liberdade pode ter na auto aceitação do individuo como “criminoso”.

    O processo tem início com a própria chegada à penitenciária, com a despersonalização do indivíduo, que é despojado de sua identidade. O condenado é privado de seus pertences, suas vestimentas, tem seu cabelo cortado, recebe um uniforme, igual ao de todos os outros internos, e um número, pelo qual passará a ser tratado. A partir desse momento, o condenado passa a ser apenas mais um “criminoso”, despojado de características pessoais que o diferenciem dos demais que ali estão.

    Durante o cumprimento da pena, o condenado passa por uma série de humilhações, não possui garantia alguma de integridade física e é vigiado (ou pelo menos possui esta impressão) em todos os momentos do dia. Esses aspectos acabam por despertar dentro desse indivíduo um senso de sobrevivência que o faz assumir uma postura de obediência, portando-se como os carcereiros esperam que ele se porte, visando a recuperar a sua . Dentro dessa ambiente hostil, passa-se a conviver e a interagir com indivíduos que também são consideradas desviantes. Esse afastamento completo da sociedade e aproximação com o grupo de indivíduos desviantes faz com que o condenado, em vez de se ressocializar, seja integrado à vida dentro do cárcere.

    Dentro da , o condenado deve viver de acordo com as regras que lá são impostas, inclusive pelos próprios internos, que se organizam em facções. Portanto, longe de estar sendo incluído na sociedade, o indivíduo passa a assumir uma conduta condizente com a vida na penitenciária, assumindo comportamentos que a sociedade reputa como desviantes.

    As consequências negativas dessa vida prisional são evidentes. O indivíduo que sai do cárcere após um considerável tempo cumprindo uma pena carrega uma nova identidade, que lhe foi atribuída no momento da condenação e aprimorada durante o tempo na prisão. Essa nova identidade é estigmatizada tanto pela sociedade, através do preconceito com aquele que passou pela experiência prisional, quanto pelo próprio indivíduo, que internaliza a etiqueta de “desviante”, assumindo-se como tal. Isso porque o comportamento humano ajusta-se ao ambiente e às expectativas que nos rodeiam. Os atos praticados são resultado das expectativas que nos cercam e suas consequências dependem da reação social apresentada a determinado comportamento.


ID
904819
Banca
CESPE / CEBRASPE
Órgão
DPE-TO
Ano
2013
Provas
Disciplina
Filosofia do Direito
Assuntos

Com relação ao conceito de justiça, assinale a opção correta.

Alternativas
Comentários
  • Para Aristóteles: 

    "Justiça é aquilo em virtude do qual se diz que o homem justo pratica, por escolha própria, o que é justo, e que distribui, seja entre si mesmo e um outro, seja entre dois outros, não de maneira a dar mais do que convém a si mesmo e menos do que convém a si mesmo e menos ao seu próximo (e inversamente no relativo ao que não convém), mas de maneira a dar o que é igual de acordo com a proporção; e da mesma forma quando se trata de distribuir entre duas outras pessoas."
  • A justiça, em sua acepção subjetiva, apresenta três significações de extensão diferente, a saber: sentido latíssimo; sentido lato; sentido próprio ou estrito. Em sentido latíssimo, justiça significa a virtude em geral: o justo é o virtuoso. Justiça significa, nesse caso, santidade.
    É esta a acepção do vocábulo em diversas passagens da Bíblia, em que o justo é equiparado ao santo. É o caso da expressão citada: “A justiça do simples dirige o seu caminho”.

    Na filosofia estóica predominou, também, esse sentido amplo da justiça exercendo grande influência sobre o Direito Romano, nos textos do Digesto vamos encontrar o mesmo conceito: “Direito é a arte do bem e do eqüitativo” (Jus est ars boniet aequi).  E entre os precepta juris, de Ulpiano, vem, em primeiro lugar, o “viver honestamente” (honeste vivere).
     Ora, é esse um preceito de moral geral. Justiça é identificada aqui como a virtude em S. João Crisóstomo,  o qual concebeu-a como o cumprimento dos mandamentos ou das obrigações em geral.
    Mas, em sentido estrito e próprio, a justiça designa uma virtude com objeto especial. Nesse sentido, “a essência da justiça consiste em dar a outrem o que lhe é devido, segundo uma igualdade” (simples ou proporcional), conforme a definição lapidar de São Tomás de Aquino.
    Só é justiça propriamente dita, a relação que tem por objeto dar a outrem; o que lhe é devido; segundo uma igualdade. A essas três notas correspondem as características essenciais da justiça, em sentido estrito: a alteridade ou pluralidade de pessoas (alteritas, de alter); o devido (debitum); igualdade (aequalitas).
    https://www.metodista.br/revistas/revistas-metodista/index.php/RFD/article/viewFile/526/524
  • A alteridade, o débito e a igualdade estão associados ao conceito estrito de justiça.
  • ACERTIVA CORRETA LETRA "D"
  • Estudando o tema para a prova da DPE-SP, descobri que justiça comutativa e recíproca são sinônimos. Trata-se, em suma, daquela justiça que preside os contratos em geral, tal como as trocas comerciais (compra e venda). Se o adquirente pagou pelo produto, o alienante deve dar ao outro o que lhe é devido de direito.
  • LETRA E - INCORRETA Na antiguidade clássica, Platão definiu justiça como a vontade constante e perpétua de dar a cada um o que lhe pertence.

    Este conceito de justiça pertence a Polemarco e é refutado por Sócrates em obra de autoria de Platão (Livro I República) : " Polemarco, personagem da obra, sintetiza o que seria o justo utilizando-se da definição, já tradicional na época, dada pelo poeta Simônides 'É justo restituir a cada um o que se lhe deve'. Polemarco interpreta esta sentença do seguinte modo: dar a cada um o que é seu consiste em fazer bem aos amigos e mal aos inimigos, pois deve se restituir o bem àqueles que nos fazem o bem e o mal àqueles que nos fazem o mal. Sócrates refuta Polemarco introduzindo o conceito de virtude (areté) relacionado à justiça: 'Quando se faz mal a cavalos, eles se tornam melhores ou piores? Piores. Em relação à excelência ou virtude dos cães ou à dos cavalos? À dos cavalos. Assim, também quando se faz mal aos cães eles se tornam piores em relação à virtude dos cães. Com os homens, diz Sócrates, ocorre o mesmo, pois se lhes fazemos mal, eles se tornam piores em relação à virtude humana. Se a justiça é a virtude específica do homem e fazendo mal aos homens os tornamos mais injustos, ao fazer isso o justo estaria se tornando outrem injusto por meio da justiça, o que é incompatível com a natureza da justiça como virtude. Logo, conclui Sócrates, fazer mal, mesmo que seja a um inimigo, não é ato do homem justo, mas sim do injusto. Aí está a raiz da concepção, ainda vigente entre nós de que é moralmente superior sofrer que praticar o mal" OLIVEIRA, André Gualteri de. Saberes do Direito: Filosofia do Direito. Saraiva: 2012,  

  • Análise da questão:

    No bojo da justiça comutativa primitivamente, as trocas só podiam ser feitas na exata proporção das necessidades de cada um, consta ortodoxamente no pensamento aristotélico como sendo uma máxima intransponível elevada como sendo uma lei universal eminentemente deontológica. Na justiça comutativa os escolásticos tipificam pela igualdade das coisas permutadas. Aristóteles encara como "corretiva", pois equipara todas as vantagens e desvantagens de troca entre os homens, tanto voluntária quanto involuntariamente feito (KRASSUSKI, Prof. Dr. Jair Antonio; MORAES, Simone Becher Araujo; SOARES, Rafael Luiz, p. 9).

    Nesse sentido, "Se a justiça distributiva ordenava as relações entre a sociedade e seus membros, a justiça corretiva ou sinalagmática ordena as dos membros entre si. Contudo, quando intervém nela como elemento principal a vontade dos interessados, se chama justiça comutativa; e se chama justiça judicial quando se impõem inclusive contra a vontade de um deles, por decisão do juiz, qual ocorre no castigo de um delito." (TRUYOL Y SERRA, Antonio. Historia de la filosofia del derecho y estado. Madrid: Revista de Occidente S.A., 1970, p. 178).

    Tendo em vista as lições sobre justiça aristotélica, portanto, a assertiva correta é a contida na letra “d".

    Fontes:
    KRASSUSKI, Prof. Dr. Jair Antonio; MORAES, Simone Becher Araujo; SOARES, Rafael Luiz. ENSINO DE FILOSOFIA: ENSAIO DE METODOLOGIA E CONTEÚDOS. Santa Maria: Universidade Federal de Santa Maria Centro de Ciências Sociais e Humanas, 2008. 19 p.
    TRUYOL Y SERRA, Antonio. Historia de la filosofia del derecho y estado. Madrid: Revista de Occidente S.A., 1970.


    Gabarito: Alternativa D
  • D:" Para Aristóteles, justiça é o agir com cooperação interpessoal (homem é um ser político). Não se trata de algo individual, mas algo essencialmente social, que se manifesta nas relações entre os homens. Como se concretiza a justiça? Pelo alcance da igualdade. Ele via a justiça em duas acepções, justiça particular (justiça na relação entre as partes) e justiça universal (justiça que envolve o todo, ou seja, a legislação e toda comunidade por ela protegida). A justiça particular podia ser:

    Justiça particular comutativa ou corretiva: trata-se da justiça entre particulares, entre pessoas que atuam com coordenação, sem diferenciações hierárquicas, a qual deve ser concretizada de forma simples ou aritmética. Os ganhos e perdas das partes devem ser iguais, não importando o mérito individual. Esse justo conduz à noção de reciprocidade proporcional das forças dentro da malha social.

    Justiça particular distributiva: trata-se da justiça entre sociedade e particulares, não devendo ser implementada de forma direta, e sim proporcional. Nela se insere a importância do mérito (avaliação subjetiva do merecimento ou não de benefícios) para se fixar a justiça na distribuição dos bens. Aristóteles reconhecia que o mérito era um valor variável, conforme o sistema político adotado.

    Para se completar a teoria da justiça em Aristóteles, ele agregou o elemento da equidade em sua concepção. Equidade significaria avaliar o justo no caso concreto, visto que a lei possui um caráter geral e abstrato. Assim, equidade é a correção dos rigores da lei'. (RESUMO DE HUMANÍSTICA).

  • 18. Aristóteles (384 - 321 a. C.). 18.3. DIREITO E JUSTIÇA. Para Aristóteles, a justiça é um meio para que os homens alcancem o bem, sendo o bem aquilo que todas as coisas visam;...

    (BIBLIOGRAFIA: RACHID, Alysson. Filosofia do Direito. Concursos e OAB. Editora Revista dos Tribunais, São Paulo, p. 99).

  • A) O vocábulo justiça é empregado, em sentido lato, como equivalente a organização judiciária. ERRADO

    Organização judiciária tem relação com o Poder Judiciário, e o vocábulo justiça não está necessariamente limitado ao Poder Judiciário.

    B) O sentido estrito de justiça está associado ao conjunto das virtudes que regulam as relações entre os homens. ERRADO

    O sentido estrito de justiça é uma virtude particular.

    C) De acordo com a doutrina majoritária, caracterizam o sentido lato de justiça a alteridade, o débito e a igualdade. ERRADO

    Esse é o sentido ESTRITO de justiça.

    Sentido lato de justiça é aquele que envolve o justo legal e a conjunção de todas as virtudes.

    D) Consoante a doutrina aristotélica, a justiça comutativa caracteriza-se como aquela em que o particular dá a outro o bem que lhe é devido. CORRETO

    E) Na antiguidade clássica, Platão definiu justiça como a vontade constante e perpétua de dar a cada um o que lhe pertence. ERRADO

    Essa é a noção ARISTOTÉLICA de justiça, que sistematiza: justiça universal x justiça particular. Dentro da ideia de justiça particular existem as noções de justiça corretiva ou comutativa e justiça distributiva, que se relacionam à assertiva.

    A noção platônica de justiça se relaciona à harmonia ("FAZER cada um o seu"). É Platão quem analisa a ideia de uma "Cidade justa", mediante a divisão harmônica entre as funções de cada uma das classes na cidade: filósofos, guerreiros e produtores.

    Fonte: anotações da Aula 2 do curso CEI de Filosofia do Direito


ID
936895
Banca
FGV
Órgão
OAB
Ano
2013
Provas
Disciplina
Filosofia do Direito
Assuntos

“Manter os próprios compromissos não constitui dever de virtude, mas dever de direito, a cujo cumprimento pode-se ser forçado. Mas prossegue sendo uma ação virtuosa (uma demonstração de virtude) fazê-lo mesmo quando nenhuma coerção possa ser aplicada. A doutrina do direito e a doutrina da virtude não são, consequentemente, distinguidas tanto por seus diferentes deveres, como pela diferença em sua legislação, a qual relaciona um motivo ou outro com a lei”.

Pelo trecho acima podemos inferir que Kant estabelece uma relação entre o direito e a moral. A esse respeito, assinale a afirmativa correta.

Alternativas
Comentários
  • Falta a parte final da questão, trazida agora: 

    Pelo trecho acima podemos inferir que Kant estabelece uma relação entre o direito e a moral. A esse respeito, assinale a  afirmativa correta.
    gab(B)  É PRECISO  LER COM CUIDADO E ATENÇÃO!
    VEJA-SE A LIÇÃO DO PROFESSOR TÉRCIO FERRAZ , EM SEU  BEST  SELLER: INTRODUÇÃO AO ESTUDO DO DIREITO NO CAPITULO 7, REFERENTE A MORALIDADE DO DIREITO:
    " A distinção mais famosa proposta pelo pensamento ocidental corresponde à tese segundo a qual  normas jurídicas dizem respeito à conduta EXTERNA do indivíduo, sendo referente aos motivos e às intenções, ao paso que os preceitos morais referem-se ao aspecto INTERNO do comportamento. A distinção é vaga e ambigua, e não resiste a um exame acurado." (fragmentou-se) .

    Repare-se que se tem vinculo de interpretação ao pensamento do autor da citação: I. Kant.
    "Manter os próprios compromissos não constitui dever de virtude, mas dever de direito, a cujo cumprimento pode-se ser forçado." (fragmentou-se)

    Próprios compromissos (são os compromisso com o juízo interior), dever de virtude, (relaciona-se com o campo da moral.) 
    ...  quando nenhuma coerção possa ser aplicada .( A norma moral não imputa sanções concretas, quando desobedecidas, pois são unilateirais! A qui há COAÇÃO. Já as normas jurídicas, estudadas miudentemente pelo direito, são COERCITIVAS, e bilaterias!)
    Esse pensamentos são divergentes para a doutrina, quer seja sobre lateralidade, quer seja sobre coação/coerção !!!
    Sem embargo, a questão faz referência ao pensamento Kanteniano...
  • tema abordado é sobre a relação do Direito com a Moral a partir da ótica do filósofo: Imannuel Kant.
    Lendo o fragmento acima é possível chegar a alternativa “B” como a correta, porque:
    1) A alternativa “A” e “D” não são corretas, pois estabelecem uma relação idêntica ou totalmente distinta entre Direito e Moral, ou seja, por demais exageradas;
    2) A alternativa “C” poderia se cogitar, porém a coerção é um elemento que justamente distingue o Direito da moral e não é um ponto em comum - como o comentário tenta induzir o leitor. Além disso, a moral depende da vontade (outro elemento falso do raciocínio);
    3) Por fim, a alternativa “B” é a melhor, já que a moral depende da vontade (raciocínio contrário da proposta “C”) e o Direito pode se concretizar independente da moral. É o gabarito da FGV.


    By Adroaldo Junior Vidal Rodrigues


  • A alternativa que traduz o pensamento de Immanuel Kant de forma correta, sendo, portanto, o gabarito da questão é a “B”, a qual afirma que a conduta moral refere-se à vontade interna do sujeito, enquanto o direito é imposto por uma ação exterior e se concretiza no seu cumprimento.

                    Kant, empenhado na construção de uma metafísica dos costumes (ou seja, uma construção racional, a priori, sintética, sobre a conduta humana), confronta-se com o antigo problema das relações entre Moral e Direito e resolve-o no sentido de uma separação (ou mesmo antítese) entre as duas instâncias. Se para a moral o que interessa é a determinação interior da ação do homem, aquilo que o leva a agir, para o Direito é só o aspecto físico, o componente externo, que é relevante.

                    Assim, se o direito se conforma com a mera legalidade, ou seja, a simples concordância do ato com o comando, a lei moral tem em conta o respectivo móbil e exige seu cumprimento por dever ético (NOVAIS, 2006).

                    Se para a Moral a lei suprema é “age apenas segundo uma máxima tal que possas ao mesmo tempo querer que ela se torne lei universal”, para o direito é “age exteriormente de tal sorte que o livre uso do teu arbítrio possa coexistir com a liberdade de cada um segundo uma lei universal” (KANT, 2007).

    Fontes:

    NOVAIS, Jorge Reis. Contributo para uma teoria do estado de direito. Coimbra: Almedina, 2006.

    KANT, Immanuel. Fundamentação da Metafísica dos Costumes. Lisboa: Edições 70 Ltda, 2007. 120 p. (70 textos filosóficos). Traduzida do Alemão por Paulo Quintela.


ID
950290
Banca
FGV
Órgão
Senado Federal
Ano
2012
Provas
Disciplina
Filosofia do Direito
Assuntos

O controle judicial da discricionariedade administrativa evoluiu ao longo do tempo, alterando a noção de imunidade judicial da discricionariedade. Várias teorias procuraram explicar e legitimar o controle judicial da atuação estatal discricionária. Nessa linha, as principais teses são

Alternativas
Comentários
  • No decorrer das décadas seguintes, pouco a pouco, consolidou-se a teoria do 
    desvio de poder pelo Superior Tribunal Administrativo Francês78, superando, segundo 
    Maurice Hauriou, a idéia de que o desvio de poder estaria circunscrito ao exame intrínseco da 
    legalidade, pois, como expôe: “A noção de excesso de poder é muito mais extensa que a de 
    legalidade; corresponde à moral que é mais extensa que o direito.”79. Moralidade essa que 
    guardaria em si os valores do que é ‘bom’, enquanto a lei descreveria a realidade da ‘justiça’, 
    vez que: “o espírito da lei é o limite a impor aos direitos no interesse do bem, havendo assim 
    uma nítida distinção entre o que é justo e o que é bom”
  • Uma delas é a teoria dos motivos determinantes, que nas palavras da Professora Maria Sylvia Zanella Di Pietro (03) é conceituada da seguinte forma:

    “(...)quando a administração indica os motivos que levaram a praticar o ato, este somente será válido se os motivos forem verdadeiros. Para apreciar este aspecto, o Judiciário terá que examinar os motivos, ou seja, os pressupostos de fato e as provas de sua ocorrência.(...)”.
  • Excelentes comentários fez o colega acima, contudo não vamos esquecer de colocar qual letra indica a assertiva correta, sendo neste caso a letra "a".

    Bons Estudos!!!
  • Gabarito A

    O termo “desvio de poder” ou “détournement de pouvoir” tem origem no Conselho de Estado francês. A teoria do desvio de poder (ou de finalidade) é fruto de construção da jurisprudência do Conseil d´Etat francês (contencioso administrativo) como um limite à ação estatal, um freio ao transbordamento da competência legal. Buscou-se, pois, reprimir ou fazer cessar os abusos inerentes à natureza humana e egoísmo dos agentes públicos.


    A teoria dos motivos determinantes sustenta a validade do ato administrativo se vincula aos motivos indicados como seu fundamento. Essa teoria sustenta que quando a administração motiva o ato – mesmo que a lei não indicar isto como pressuposto inexorável – a validade do mesmo depende da verdade dos motivos alegados.

    Motivo: é o fundamento do ato administrativo, é o pressuposto de direito e de fato que serve para sua prática. É o conjunto de circunstâncias, situações e acontecimentos que levam a administração a praticar o ato. Como pressuposto de direito podemos remeter ao dispositivo legal que se deve levar em consideração, como base do ato.

    Motivação: é a exposição de motivos, ou seja, a exteriorização, a materialização dos motivos.


  • PARA A BANCA CESPE, O CONCEITO DE DESLEGALIZAÇÃO FOI DADO ASSIM:

    No tocante aos poderes administrativos, julgue o seguinte item.

    O fenômeno da deslegalização, também chamada de delegificação, significa a retirada, pelo próprio legislador, de certas matérias do domínio da lei, passando-as para o domínio de regulamentos de hierarquia inferior. QUESTÃO CORRETA.


    Aplicada em: 2015

    Banca: CESPE

    Órgão: STJ

    Prova: Analista Judiciário - Administrativa

  • Análise da questão:

    As principais teorias utilizadas para explicar e legitimar o controle judicial da atuação estatal discricionária são: a Teoria do Desvio de Poder, a Teoria dos Motivos Determinantes e a Teoria dos Princípios.

    Em relação à Teoria do Desvio de Poder, conforme Di Pietro (2007, p. 203), é possível dizer que “o desvio de poder ocorre quando a autoridade usa do poder discricionário para atingir fim diferente daquele que a lei fixou. Quando isso ocorre, fica o Poder Judiciário autorizado a decretar a nulidade do ato, já que a Administração fez uso indevido da discricionariedade, ao desviar-se dos fins de interesse público definidos na lei".

    No que diz respeito à Teoria dos Motivos Determinantes, Segundo Bandeira de Mello (p. 386) “de acordo com essa teoria, os motivos que determinaram a vontade do agente, isto é, os fatos que serviram de suporte à sua decisão, integram a validade do ato. Sendo assim, a invocação de “motivos de fato" falsos, inexistentes ou incorretamente qualificados vicia o ato mesmo quando (...) a lei não haja estabelecido, antecipadamente, os motivos que ensejariam a prática do ato. Uma vez enunciados pelo agente os motivos em que se calçou, ainda quando a lei não haja expressamente imposto a obrigação de enunciá-los, o ato só será válido se estes realmente ocorreram e o justificavam.

    Em relação à Teoria dos Princípios aplicada à administração, segundo Freitas (2008), podemos dizer que “toda discricionariedade, exercida legitimamente, encontra-se, sob determinados aspectos, vinculada aos princípios constitucionais, acima das regras concretizadoras. Nessa ordem de ideias, quando o administrador público age de modo inteiramente livre, já deixou de sê-lo. Tornou-se arbitrário. Quer dizer, a liberdade apenas se legitima ao fazer aquilo que os princípios constitucionais, entrelaçadamente, determinam".

    A alternativa correta, portanto, é a letra “a".

    Fontes:
    BANDEIRA DE MELLO, Celso Antônio. Curso de direito administrativo. 22. ed. rev. e atual. São Paulo: Malheiros, 2007.
    DI PIETRO, Maria Sylvia Zanella. Direito administrativo. 20. ed. São Paulo: Atlas, 2007.
    FREITAS, Juarez. Discricionariedade administrativa e o direito fundamental à boa administração pública. São Paulo: Malheiros, 2007.

    Gabarito: Alternativa A
  • Alguém saberia explicar por que a assertiva "e" não está correta?

    Tudo o que consta alí não se presta para o controle da discricionariedade dos atos adminstrativos por parte do Judiciário?

  • Correta A letra - A-

    Teoria do Desvio de Poder, Teoria dos Motivos Determinantes e Teoria dos Princípios.


ID
1049143
Banca
FGV
Órgão
OAB
Ano
2013
Provas
Disciplina
Filosofia do Direito
Assuntos

Considere a seguinte afirmação de Herbert L. A. Hart:

“Seja qual for o processo escolhido, precedente ou legislação, para a comunicação de padrões de comportamentos, estes, não obstante a facilidade com que atuam sobre a grande massa de casos correntes, revelar-se-ão como indeterminados em certo ponto em que a sua aplicação esteja em questão.”
(HART, Herbert. O Conceito de Direito. Lisboa: Calouste Gulbenkian, 1986, p. 141)

Hart admite um grau de indeterminação nos padrões de comportamento previstos na legislação e nos precedentes judiciais. A respeito, assinale a afirmativa correta.

Alternativas
Comentários
  • Questão de pura doutrina:

    Em alguns casos, o conteúdo das normas se esforça para ser bem amplo com a finalidade de abarcar o maior número de ações humanas possíveis, porém, existem casos, em que a linguagem demonstra uma falha que faz com que o aplicador tenha dificuldade em aplicar as normas.

    Um exemplo bem interessante que Hart utiliza para explicar a questão da textura aberta é a placa de proibida a entrada de veículos no parque.

    A primeira questão que nos vem à cabeça é: porque será que é proibida a entrada de veículos no parque? Que fato aconteceu para que não fosse mais permitida a entrada de veículos no parque?

    A segunda questão é: o que a Administração do parque entende por veículo? Um patins é um veículo? Uma bicicleta é um veículo? Um carrinho de brinquedo é um veículo?

    Neste caso a textura aberta se dá pela indeterminação da regra: o que a Administração do parque entende por veículo? Mesmo sendo uma regra simples (É proibida a entrada de veículos no parque), o poder discricionário que foi deixado pela linguagem pode ser muito amplo, de qualquer forma que, a aplicação da regra pode, na verdade, constituir uma escolha, ainda que possa não ser arbitrária ou irracional. 

    Ou seja, a Administração do parque pode, de acordo com a sua escolha entender que é veículo apenas aqueles que dotados de motor, com placa e que são utilizados como meio de transporte.

    Para Dworkin, constata-se a existência da textura aberta do direito quando em um texto da Lei são observados: ambiguidade, obscuridade e termos abstratos.

    Fonte: http://pt.wikipedia.org/wiki/Textura_aberta_do_direito

  • Lembrar as aulas do Noel (Teoria do Direito). Caso da placa "proibida a permanência de animais".

  • A resposta correta é a letra “b”. Hart refere-se à textura aberta do direito. Segundo Hart, a textura aberta é terminologia utilizada para demonstrar a possibilidade de imprecisão no conteúdo das Leis. Na verdade, não apenas nas leis, mas em qualquer tipo de linguagem. Trata-se de um problema inerente à linguagem, conforme Hart:

    “Em todos os campos da experiência, e não só no das regras, há um limite à natureza da linguagem, quanto à orientação que a linguagem geral pode oferecer. Haverá na verdade casos simples que estão sempre a ocorrer em contextos semelhantes, aos quais as expressões gerais são claramente aplicáveis (...), mas haverá também casos em que não é claro se se aplicam ou não (...) (HART, p. 124)

    O exemplo dado por Hart para retratar a questão diz respeito à regra que proíbe o ingresso de “veículos” em determinado local, como um parque (HART, p. 124), sem que se possa saber, de fato, a qual tipo de automóvel a regra se refere. Nesse sentido, mesmo que se possa afirmar categoricamente que não se deve ingressar com um carro no parque, não é tão simples o caso de se a regra está proibindo o uso de bicicletas ou de patins no parque (SGARBI, p. 132). Ou seja, no primeiro caso, pouca discussão há, mas, no segundo, está presente alguma dificuldade e necessidade de atuar discricionariamente.

    Nesse sentido, “a textura aberta do direito significa que há, na verdade, áreas de conduta em que muitas coisas devem ser deixadas para serem desenvolvidas pelos tribunais ou funcionários, os quais determinam o equilíbrio, à luz das circunstâncias, entre interesses conflitantes que variam em peso, de caso para caso” (p.131-132).

    FONTES:

    HART, H.L.A. O conceito de direito. 3. ed. Lisboa: Fundação Calouste Gulbenkian, 1994. 348 p.

    SGARBI, Adrian. Clássicos de Teoria do Direito. 2. ed. Rio de Janeiro: Lumen Juris, 2009. 238 p.

  • Incompletude do Ordenamento Jurídico: é a impossiblidade de legislador prever todas as hipóteses possíveis, de modo que; a partir de que essas surjam, busque-se o preenchimento de lacunas; seja por princípios gerais do direito, seja por analogia, seja por equidade.

    Textura Aberta do Direito: existe previsão legal, o que acontece  é a possibilidade de ampliação ou de constrição no sentido das palavras expressas no texto legal.


ID
1085431
Banca
CESPE / CEBRASPE
Órgão
MPE-AC
Ano
2014
Provas
Disciplina
Filosofia do Direito
Assuntos

Considerando os aspectos filosóficos relacionados ao conceito de justiça, assinale a opção correta.

Alternativas
Comentários
  • A Incorreto e B CorretoPitágoras foi o primeiro filósofo a criar uma definição que quantificava o objetivo final do Direito: a Justiça. Ele definiu que um ato justo seria a chamada "justiça aritmética", na qual cada indivíduo deveria receber uma punição ou ganho quantitativamente igual ao ato cometido.

    C - Incorreto - Tal argumento de Pitágoras foi refutado por Aristóteles, pois ele acreditava em uma justiça geométrica, na qual cada indivíduo receberia uma punição ou ganho qualitativamente, ou proporcionalmente, ao ato cometido; ou seja, ser desigual para com os desiguais a fim de que estes sejam igualados com o resto da sociedade.

  • d) Incorreta, Segundo Anaximandro afirma, portanto, é que há uma justiça imanente ao próprio acontecer, no qual se realiza para cada caso a compensação das desigualdades. diké: Na mitologia gregaDiké (ou Dice; em grego Δίκη), é a filha de Zeus com Têmis, é a deusa grega dos julgamentos e da justiça (deusa correspondente, na mitologia romana, é a Iustitia), vingadora das violações da lei.

    Anaximandro é considerado, pela tradição, um filósofo pré-socrático

    que viveu no século VI antes de Cristo e foi discípulo do primeiro filósofo,

    Tales de Mileto. De Tales não restou nenhum fragmento escrito, mas o

    mesmo não ocorreu com seu sucessor, de cujo pensamento restou um único

    fragmento de texto. Assim, esse fragmento de Anaximandro é o primeiro

    texto escrito conhecido na história da filosofia, o que faz que a chamada

    sentença de Anaximandro seja reconhecida como a mais antiga sentença

    do pensamento ocidental. Nessa primeira frase escrita que foi transmitida

    pela filosofia ocidental, encontra-se a palavra justiça (díke).1 Segundo

    Peters “o primeiro uso da dike num contexto filosófico ocorre no único

    fragmento de Anaximandro” (PETERS, 1974, p. 54).

    Anaximandro (em grego: Ἀναξίμανδρος; 610 — 547 a.C.) foi um geógrafomatemáticoastrônomopolítico e filósofopré-Socrático; discípulo de Tales, seguiu a escola jônica1 . Os relatos doxográficos nos dão conta de que escreveu um livro intitulado "Sobre a Natureza"; contudo, essa obra se perdeu.

  • E - Incorreto - Demócrito, considerado o último pré-socrático. Para ele, 

    Justiça quer dizer: fazer o que necessário. Injustiça que dizer: não fazer o que é necessário, esquivando-se.

    Com Empédocles de Agrigento, Anaxágoras de Clazômena e Leucipo de Abdera, Demócrito de Abdera (considerado o pai do atomismo grego) fez parte da Escola da Pluralidade, a Escola dos Atomistas, sucessora das Escolas Jônica (Tales de Mileto, Anaximenes de Mileto, Anaximandro de Mileto e Heráclito de Êfeso), Itálica (Pitágoras de Samos, Filolau de Crotona e Árquitas de Tarento) e Eleata (Xenófanes de Cólofon, Parmênides de Eléia, Zenão de Eléia e Melisso de Samos). A característica fundamental da Escola da Pluralidade era a admissibilidade de que a matéria era constituída por átomos e vácuo (uma espécie de não-ente). Demócrito (cerca de 460 a.C. – 370 a.C.), discípulo e depois sucessor de Leucipo de Mileto, não deve ser considerado propriamente um filósofo pré-socrático, já que foi contemporâneo de Sócrates.

  • A importância da teoria matemática de Pitágoras é fundamental para alcançar a resposta correta da assertiva. Os filósofos (jusfilósofos) que se inspiram em Pitágoras adotam proporção de igualdade geométrica como critério norteador da justiça distributiva. “De acordo com a doutrina de Pitágoras, a justiça era entendida como igualdade. Sua teoria numérica revelou a tríade e a tetraktys  como os símbolos da justiça, especialmente esta última, representação da omónoia, harmonia dos contrários existentes entre os lados do quadrado” (BITTAR, p. 97).

    Pitágoras foi o primeiro filósofo a definir quantitativamente o objetivo final do Direito como sendo a Justiça. Para Pitágoras, um ato justo se enquadra na denominada "justiça aritmética", na qual cada indivíduo receberia uma punição ou um ganho quantitativamente igual ao ato cometido.

    A Escola Pitagórica também defendia conceitos variados, dentre os quais se destacavam, a lealdade entre os membros e a distribuição comunitária dos bens materiais e a obediência à hierarquia da Escola (MAFRA FILHO, 2014).

    A alternativa correta, portanto, é a “b”.

    Fontes:

    BITTAR, Eduardo. C.B. A justiça em Aristóteles.

    MAFRA FILHO, Francisco de Salles Almeida. Pitágoras e o Direito. 2014. Disponível em:<http://conteudojuridico.com.br/?colunas&colunista=36_Francisco_Filho&ver=180.Acesso em: 02 dez. 2015.

    PESSOA, Flávia Moreira Guimarães. JUSTIÇA EM ARISTÓTELES. 2006. Disponível em: <http://www.evocati.com.br/evocati/interna.wsp?


  • essa questão é muito difícil ou eu que sou muito burra?


ID
1085443
Banca
CESPE / CEBRASPE
Órgão
MPE-AC
Ano
2014
Provas
Disciplina
Filosofia do Direito
Assuntos

Considerando as relações entre política e direito bem como os princípios gerais do direito, assinale a opção correta.

Alternativas
Comentários
  • Em que pese o Direito, em um dado tempo e espaço, possa, de fato, estar reduzido à condição de "mero instrumento do poder político", parece-me problemática na assertiva "d" a afirmação de que o seu estudo, por isso, estará limitado às relações de poder.

  • A questão exige conhecimento relacionado à conexão entre política e direito. Conforme Willis Santiago Guerra Filho, na obra “Teoria política do direito: a expansão política do direito”, “o direito, inevitavelmente, possui relação com o poder tão estreita que, muitas vezes, encontra-se quem o reduza às relações de poder, com consequente politização absoluta – tendencialmente absolutista, autoritária e, quando não, totolitária – do direito, acabando por se transformado à condição de uma espécie de disfarce da política, um mero e degradador instrumento do poder”  (GUERRA FILHO, 2013, p. 111).

    Portanto, considerando as lições de Guerra Filho, está correta a assertiva que alega que o direito tem relação tão estreita com o poder que seu estudo, muitas vezes, reduz-se às relações de poder, o que acarreta uma politização absoluta, autoritária e, em certos casos, totalitária do direito, que passa à condição de uma espécie de disfarce da política e mero instrumento do poder político.

    Gabarito do professor: letra d.

     

    Referência: GUERRA FILHO, Willis Santiago. Teoria política do direito: a expansão política do direito. 2.ed. São Paulo: Revista dos Tribunais, 2013.


  • Que o Senhor nos salve do ativismo judicial!

    Abraços.

  • @Marconi

    Tenho o livro que o examinador utilizou para fazer a questão nele o autor explica que essa forma de pensar o Direito dá-se por pensadores marxistas. Escreve que "Este modo reducionista de tratar o direito [...] pode ser encontrado entre defensores de um marxismo vulgar." Ou seja, você pode concordar ou não com o que está aqui, no entanto isso é doutrina, eu também não concordo, acontece que o examinador abriu o livro em alguma página aleatória, achou interessante esse ponto, e colocou na questão.

  • Para Miguel Reale os princípios podem ser discriminados em três grandes categorias

    a) princípios omnivalentes, quando são válidos para todas as formas de saber, como é o caso dos princípios de identidade e de razão suficiente;

    b) princípios plurivalentes, quando aplicáveis a vários campos de conhecimento, como se dá com o princípio de causalidade, essencial as ciências naturais, mas não extensivo a todos os campos do conhecimento;

    c) princípios monovalentes, que só valem no âmbito de determinada ciência, como é o caso dos princípios gerais de direito.”

    REALE, Miguel. Lições preliminares de direito. 27 ed. São Paulo: Saraiva 2002, p. 303-304.

  • a) Errado - no Brasil, o fenômeno da "judicialização da política" se deu especialmente após a CF/88:

    " O judiciário brasileiro, sobretudo após 1988, passou a interagir com o sistema político, num processo complexo, do qual participam: (a) os tribunais judiciais, especialmente o STF; (b) governo e partidos políticos; (c) associações profissionais relevantes, especialmente a Associação dos Magistrados Brasileiros e a Associação Juízes para a Democracia, que têm orientações, valores e concepções distintas acerca do papel institucional do Poder judiciário; e (d) a opinião pública. 

     

    (...)

    Esses fatos indicam que o judiciário e especialmente o STF, têm causado algum impacto sobre o Legislativo e sobre o governo, freqüentemente através de concessão de liminares e de ações não jurisdicionaisPor outro lado, a atuação do STF mediante a produção jurisprudencial resultante do uso de garantias constitucionais, como o mandado de injunção (MI) e a ação direta de inconstucionalidade (ADIn), não se coaduna com uma clara disposição para a interação institucional estável.  Minha pesquisa sobre as ADIns dos partidos políticos entre 1988 e 1992 mostrou, por exemplo, que uma parcela diminuta dessas ações resultaram em decisões substantivas de mérito"

    Fonte:

    b) Errado ver letra A. Na verdade, a judicialização da política atinge o Poder Executivo (ór exemplo, deicsões judiciais que afetam políticas públicas, que adentram em exame de mérito político-administrativo ou execução de políticas públicas) e o pdoer legislativo.

  • c) Errado - os princípios gerais de direito somente são admitidos e usados pelo intérprete, conforme o ordenamento jurídico brasileiro, de forma secundária, ou seja, caso haja omissão legal, conforme artigo 4º da Lei de Introdução às Normas do Direito Brasileiro. Ao contrário dos princípios fundamentais que estão positivados na CF/88 e que são entendidos, modernamente, como normas que devem ter eficácia concreta. Ainda assim, não permitem eles "livre" criação jurisprudencial, ao contrário, delimitam e informam a atuação dos magistrados:

    LINDB - Art. 4o  Quando a lei for omissa, o juiz decidirá o caso de acordo com a analogia, os costumes e os princípios gerais de direito.

    Os princípios gerais são as regras que, embora não estejam escritas, servem como mandamentos que informam e dão apoio ao direito, utilizados como base para a criação e integração das normas jurídicas, respaldados pelo ideal de justiça.

     

    (...)os princípios gerais de direito acima enunciados como a última alternativa à colmatação de lacunas legais não se confundem com os princípios constitucionais. Os princípios constitucionais espelham categoria diversa e, repitase, não podem ser confundidos com os princípios que se prestam ao suprimento de omissões do legislador como derradeira fórmula. É que os princípios constitucionais possuem força vinculante e são na verdade o início, o ponto de partida de qualquer atividade judicante, seja de interpretação, integração ou de aplicação da lei. São de observância necessária e obrigatória em qualquer situação, sob pena de invalidade por vício de inconstitucionalidade. E, antes de mais nada, devem informar a própria atividade legislativa, bem como a atuação de todos os entes estatais."

     

    Fonte:

     

  • d) CERTO:

    "A questão exige conhecimento relacionado à conexão entre política e direito. Conforme Willis Santiago Guerra Filho, na obra “Teoria política do direito: a expansão política do direito”, “o direito, inevitavelmente, possui relação com o poder tão estreita que, muitas vezes, encontra-se quem o reduza às relações de poder, com consequente politização absoluta – tendencialmente absolutista, autoritária e, quando não, totolitária – do direito, acabando por se transformado à condição de uma espécie de disfarce da política, um mero e degradador instrumento do poder” (GUERRA FILHO, 2013, p. 111).

    Portanto, considerando as lições de Guerra Filho, está correta a assertiva que alega que o direito tem relação tão estreita com o poder que seu estudo, muitas vezes, reduz-se às relações de poder, o que acarreta uma politização absoluta, autoritária e, em certos casos, totalitária do direito, que passa à condição de uma espécie de disfarce da política e mero instrumento do poder político.

     

    Referência: GUERRA FILHO, Willis Santiago. Teoria política do direito: a expansão política do direito. 2.ed. São Paulo: Revista dos Tribunais, 2013."

    Fonte: site QC - Bruno Farage

    e) Errado:

    a) princípios omnivalentes, quando são válidos para todas as formas de saber, como é o caso dos princípios de identidade e de razão suficiente;

    b) princípios plurivalentes, quando aplicáveis a vários campos de conhecimento, como se dá com o princípio de causalidade, essencial as ciências naturais, mas não extensivo a todos os campos do conhecimento;

    c) princípios monovalentes, que só valem no âmbito de determinada ciência, como é o caso dos princípios gerais de direito.”

    Fonte: REALE, Miguel. Lições preliminares de direito. 27 ed. São Paulo: Saraiva 2002.

  • Para entender melhor a assertiva correta:

    Hans Kelsen, teórico da 'Teoria Pura do Direito' teve sua obra bastante difundida durante o III Reich alemão.

    A suposta neutralidade, cientificidade pura, ausência de valoração do intérprete, permitiram, dentro de um Estado dito 'de Direito' que fossem cometidas barbaridades contra a própria população, e também deram ares de verdade ao Ius Ad Bellum (Direito à Guerra) - não confundir com Ius In Bellum (Direito dos Conflitos Armados) - como se houvesse, por parte de alguma nação, autorização jurídica para invadir e dominar outros países e escravizar povos.

    Por isso, a relação entre Direito e Política não pode se tratar de mera reprodução de normas positivas, ou de procedimentalização pura do poder.


ID
1108879
Banca
FGV
Órgão
OAB
Ano
2014
Provas
Disciplina
Filosofia do Direito
Assuntos

Em seu livro Levando os Direitos a Sério, Ronald Dworkin cita o caso Riggs contra Palmer, em que um jovem matou o próprio avô para ficar com a herança. O Tribunal de Nova Iorque (em 1889) julga o caso considerando que a legislação do local e da época não previa o homicídio como causa de exclusão da sucessão. Para solucionar o caso, o Tribunal aplica o princípio, não legislado, do direito que diz que ninguém pode se beneficiar de sua própria iniquidade ou ilicitude. Assim, o assassino não recebeu sua herança.

Com esse exemplo podemos concluir que a jusfilosofia de Ronald Dworkin, dentre outras coisas, pretende

Alternativas
Comentários
  • Ronald Dworkin foi o responsável para chamar atenção para o fato de que as normas jurídicas se classificam em regras e princípios e que existem diferenças fundamentais entre tais espécies normativas. Tratando de questões como a história institucional e o romance em cadeia, o filósofo estadunidense promoveu o ingresso dos princípios na interpretação jurídica, de forma a que as decisões pudessem ser tomadas com base em valores, buscando um julgamento mais justo e adequado.

     

    Ou seja, alternativa D correta.

  • O trabalho de Dworkin passou por diferentes mudanças ao longo de sua carreira.No final dos anos de 1970 o autor americano produziu a teoria, destacando as falhas do positivismo em reconhecer os argumentos de princípio.Tal teoria está contida no livro Levando os direitos a sério.Dworkin recebeu críticas devido ao aparente pragmatismo e ativismo de sua teoria.Ele reformulou sua teoria.

    Desta reformulação surgiu o livro "uma questão de princípio". No livro Dworkin começa a estudar o sistema sob um prisma interpretativo.As proposições jurídicas ,na visão do autor, seriam interpretativas.Segue-se que elas não seriam proposições factuais,mas proposições em que intérpretes dão o melhor sentido para as práticas postas em vigor pela comunidade.Mas daí não se pode concluir que não haja objetividade e verdade em direito.Dworkin defende a objetividade da interpretação.

    O livro seguinte de Dworkin foi O império do Direito .Esse foi o livro mais impactante.Nele Dworkin defende a teoria do direito como integridade.Segundo essa concepção, as proposições jurídicas seriam verdadeiras apenas quando decorressem dos princípios de equidade, justiça e devido processo legal que uma dada sociedade colocou em vigor .A teoria do direito como integridade opõe-se a outras formulações como o pragmatismo e o convencionalismo jurídico.

    Dworkin não deixou de debater questões polêmicas.Num conhecido livro ele argumentou a respeito de temas polêmicos como aborto, eutanásia.


  • A resposta correta reside na alternativa “d”. Como bem apontado pela banca, o caso Rigg vs. Palmer resume-se na pretensão de um neto - demandante da ação – o qual era beneficiário do testamento de seu avô. A questão problemática do caso gira em torno do fato de o neto ter matado o avô justamente com a intenção de herdar a herança. Quando analisada a pretensão arguida em juízo - em 1889 – e tendo em vista a lacuna legislativa em relação ao testamento em situações como esta, a corte de apelação do Estado de Nova Iorque decretou que o demandante, Elmer Palmer, não tinha o direito de herdar, tendo em vista o princípio de que ninguém pode beneficiar-se da própria torpeza, favorecendo, assim, os demais herdeiros.

    A peculiaridade reside no fato de que, para Dworkin, nesse caso, assim como em Henningsen vs. Blomfield, os tribunais utilizaram mais do que as regras jurídicas para resolvê-los. Com base nesses dois casos paradigmáticos, principalmente, Dworkin defende que a teoria hartiana ignorou a importância dos princípios como “fontes do direito”. Isso é bem visível em “o modelo de regras I” reproduzido em Levando os direitos a sério. Dworkin assinalou que a teoria tradicional positivista, mesmo a mais sofisticada de Hart, simplifica o direito a ponto de descrevê-lo como um conjunto de regras que são válidas ou inválidas somente quando respeitam ao critério formal do pedigree, a partir do qual essa validade ou invalidade é mensurada. Mas, para Dworkin, o direito não se restringe à essa descrição limitada, pois existem normas diferentes das regras tradicionais, tais como os princípios e as políticas, que extrapolam a seara da formalidade das regras. Nesse sentido, Dworkin sustentou que o direito é composto: por um conjunto de regras jurídicas, mas também por princípios “morais”, os quais não são remissíveis a um critério de validade como o da regra de reconhecimento de Hart. Dworkin afirmou que os princípios jurídicos não podem ser identificados por critérios derivados de uma norma de reconhecimento manifestada na prática dos tribunais e que, uma vez que os princípios constituem elementos essenciais do direito, deve-se abandonar a doutrina que postula a existência de uma norma de reconhecimento. Para Hart, o remédio para a incerteza do regime das regras primárias é a introdução de regras de reconhecimento. Cabe à regra de reconhecimento “especificar alguns aspectos, ou aspectos, que identificam uma regra como pertencente ao grupo das que deve ser apoiada pela pressão social que ela exerce” (HART, 1994, p. 104).

    FONTES:

    DWORKIN, Ronald. Levando os direitos a sério. São Paulo: Martins Fontes, 2002.

    FARAGE, Bruno da Costa Felipe. O pragmatismo antiteórico de Richard A. Posner e as respostas da teoria moral para a decisão judicial. Dissertação de Mestrado no Programa de Pós-Graduação em Direito da UERJ, 2015.

    HART, H.L.A. O conceito de direito. 3. ed. Lisboa: Fundação Calouste Gulbenkian, 1994.

  • Ninguém pode se valer da própria torpeza.

  • Inhaiii, amores!

    A resposta para tal questão encontra-se em uma obra que eu não li. :(

    De fato, ninguém pode se valer da própria torpeza.

    Lembrem do Dworkin como o carinha que chegou para dizer que regras e princípios

    NÃO SÃO A MESMA COISA.

    Veja como isso é ataca o positivismo jurídico de forma enfática.

    Dica PRIMORDIAL: leiam, nem que seja um resumindo, de todos esses filósofos que podem vir a cair em sua provinha.

    Gabarito: D.

    Brian Gentil

    Insta: @briangentil

    Fone: (79) 9 9640-2846


ID
1138039
Banca
FCC
Órgão
DPE-SP
Ano
2013
Provas
Disciplina
Filosofia do Direito
Assuntos

Na obra O que é justiça?, ao discorrer sobre a justiça como um problema de justificação do comportamento humano, Hans Kelsen afirma: “(...) não é de admirar que as inúmeras teorias de justiça apresentadas desde tempos imemoriais até os dias de hoje se deixem reduzir facilmente a dois tipos básicos”. Estes dois tipos básicos são denominados pelo autor:

Alternativas
Comentários
  • Letra C, pois Hans Kelsen quando se refere ao tema da Justiça costuma relacioná-lo ao jusnaturalismo.
    Para o professor austríaco, a Ciência do Direito, que não é o mesmo que Direito, segundo ele, não tem como objeto de estudo o tema da justiça, mas, sim, a norma jurídica. Logo, a alternativa C é a correta, pois se refere a dois tipos de jusnaturalismo, o medieval e o moderno. O medieval entende a justiça como um problema metafísico-religioso, ao passo que o jusnaturalismo moderno entende a questão da justiça como uma questão racional. Porém, Kelsen considera a abordagem do jusnaturalismo moderno pseudo (falso ou aparente) racional.

    Fonte: https://pt-br.facebook.com/bernardo.montalvao.14/posts/192053440978258 

  • Acertou essa, virou Defensor. 

  • A frase completa, que pode ser extraída da obra “o que é justiça?” do teórico do direito juspositivista, Hans Kelsen, é: “Por esse motivo não é de admirar que as inúmeras teorias de justiça apresentadas desde tempos imemoriais até os dias de hoje se deixem reduzir facilmente a dois tipos básicos: um metafísico-religioso e outro racionalista ou, melhor dizendo, pseudo-racionalista” (KELSEN, p.11). A alternativa correta, portanto, é a letra “c”.

    Importante destacar que, conforme bem salienta André Franco Montoro, uma teoria positivista do direito, segundo o próprio Kelsen, não pretende sustentar que a justiça não exista ou que qualquer ordem jurídica positiva não possa ser julgada justa ou injusta. Mas o positivismo jurídico pretende ocupar-se, apenas, com o sistema de normas do direito positivo; e, permanecendo rigorosamente dentro desse sistema, examinar a hierarquia e a correlação entre as diversas normas que o integram (MONTORO, p. 281).

    Entretanto, o próprio Kelsen admite, ao lado de “teoria pura do direito”, uma teoria de justiça e uma investigação sociológica do direito.

    Acontece, contudo, que são várias as teorias que tentam reivindicar a essência da justiça, o que torna o conceito não exato. E isso não é interessante para a toeira do direito que se pretende “pura”, dogmática e exata.

    FONTE:

    KELSEN, Hans. O que é justiça? : a justiça, o direito e a política no espelho da

    ciência. Tradução de Luís Carlos Borges. 2. ed. São Paulo: Martins Fontes, 1998.

    MONTORO, André Franco. Introdução à ciência do direito. 25. ed. São Paulo:

    Revista dos Tribunais, 2000.
  • "Em "o que é justiça? A justiça , o direito e a política no espelho da ciência, as doutrinas sobre a justiça são divididas em racionalistas (Aristóteles, Kant, direito natural, etc.) e metafísico-religiosas (Platao, Jesus entre outros), e recebem pormenorizado estudo. Já nos estudos preambulares da obra, Kelsen declara sua opinião de que a justiça não pode ser comcebida de forma absoluta, e que, portanto, não pode ser entendida como lugar estanque, comum a todos os homens, de conteúdo inefável, tratando-se, pelo contrário, de algo extremamente mutável, variável. Para Kelsen, é esse relativismo que deve induzir à tolerância, e a tolerância à aceitação. Sua teoria de justiça resume-se a isso."

    Fonte: Eduardo Bittar, Curso de Filosofia do Direito

  • Primeiro momento: Teocentrismo/ jusnaturalismo clássico/ senso de justiça explicada com base em leis universais metafísico-religioso.

    Segundo momento: Antropocentrismo/ jusnaturalismo racionalista / senso de justiça explicada pela razão humana.

    Para o juspositivista Hans Kelsen, crítico do jusnaturalismo, tanto o jusnaturalismo clássico pautado no metafísico-religioso quanto o jusnaturalismo racional apresentam uma concepção "irreal" da ideia de justiça, e em sua obra, designa (rotula) o jusnaturalismo racional de pseudo racional.


ID
1138048
Banca
FCC
Órgão
DPE-SP
Ano
2013
Provas
Disciplina
Filosofia do Direito
Assuntos

Considere as seguintes afirmações sobre a Teoria Pura do Direito de Hans Kelsen:

I. A Teoria Pura do Direito trata o Direito como um sis- tema de normas válidas criadas por atos de seres humanos.

II. A Teoria Pura do Direito, assumindo o sincretismo metodológico, pretende ser a única ciência do Direito possível ou legítima.

III. A Teoria Pura do Direito limita-se a uma análise estrutural do Direito positivo.

Está correto APENAS o que se afirma em:

Alternativas
Comentários
  • A base da Teoria Pura do Direito é a distinção fundamental elaborada por Kelsen entre o que ele denomina "ser" e "dever ser".

    O âmbito do ser seria o mundo natural, explicado pelas ciências naturais com base nas premissas de verdadeiro/falso. Este domínio obedeceria ao princípio da causalidade, segundo o qual uma causa conduz a um efeito (quando A é, B é), sendo que o número de elos de uma série causal seria ilimitado. As leis naturais predizem eventos futuros e podem ser confirmadas ou não. Em não sendo aplicáveis, são falsas e devem ser substituídas.

    Já o âmbito do dever ser diria respeito às normas, enquanto atos de vontade que se dirigem intencionalmente a uma conduta considerada obrigatória tanto pelos indivíduos que põe as regras quanto do ponto de vista de um terceiro interessado, e que vinculam seus destinatários. O dever ser insere-se no domínio das ciências sociais e se explica não com base nas premissas de verdadeiro/falso, mas de válido/inválido. Este domínio obedeceria ao princípio da imputação (quando A é, B deve ser), sendo que o número de elos de uma série imputativa é necessariamente limitado. As leis jurídicas prescrevem, autorizam ou permitem condutas e admitem um certo grau de não aplicação, ou ineficácia, que não conduz à sua anulação.

    Segundo a Teoria Pura, a ciência jurídica não pretende com as proposições jurídicas por ela formuladas mostrar a conexão causal, mas a conexão de imputação entre os elementos de seu objeto.

    A conduta humana (ser) só adquire uma significação jurídica quando coincide com uma previsão normativa válida (dever ser). A conduta humana pode se conformar ou contrariar uma norma e, dessa forma, pode ser avaliada como positiva ou negativa. Já as normas são estabelecidas por atos de vontade humana e, por este motivo, os valores através delas constituídos são arbitrários e relativos. Com efeito, outros atos de vontade humana poderiam produzir outras normas, diversas das primeiras e, assim, constituir outros valores. A separação entre "ser" e "dever ser" permite, assim, que a teoria jurídica desenvolvida por Kelsen independa do conteúdo material das normas jurídicas.


  • Em relação às afirmativas apontadas, pode-se dizer que: A Teoria Pura do Direito trata o Direito como um sistema de normas válidas criadas por atos de seres humanos. Essa assertiva encontra-se correta. A teoria pura do direito, em sua “pureza” considera como direito válido as normas jurídicas advindas da produção estatal. Ela não considera como pertinentes de análise as normas metajurídicas ou advindas de uma esfera não estatal como as normas do direito natural. Nesse sentido, também encontra-se correta a afirmativa de número III, a qual sustenta que a Teoria Pura do Direito limita-se a uma análise estrutural do Direito positivo. O próprio Kelsen (2012, p.1), ao apresentar sua obra, informa que  sua teoria, “quando a si própria se designa como “pura” teoria do Direito, isto significa que ela se propõe garantir um conhecimento apenas dirigido ao Direito e excluir deste conhecimento tudo quanto não pertença ao seu objeto (qual seja, o direito positivo – acréscimo do professor), tudo quanto não se possa, rigorosamente, determinar como direito. Quer dizer que ela pretende libertar a ciência jurídica de todos os elementos que lhe são estranhos. Esse é seu princípio metodológico fundamental”.

    A assertiva II está incorreta por acusar a Teoria Pura de cometer sincretismo metodológico. O sincretismo metodológico, explicando de forma sucinta, consiste na adoção de teorias incompatíveis, como se compatíveis fossem. Kelsen não se utiliza desse aparato e, além disso, não pretende ser a única ciência do direito possível ou legítima.

    A alternativa correta, nesse sentido, é a letra “A”.

    FONTE:

    KELSEN, Hans. Teoria Pura do Direito. São Paulo: Martins Fontes, 2012. 8ª edição, 3ª tiragem.    


  • Pode até parecer "mi mi mi", mas pelo que vejo dessas questões de filosofia do direito e sociologia jurídica, especialmente cobradas pela FCC, o que pretendem é que decoremos os livros, pois não há margem para responder com o senso comum (bom senso). Há questões em que são extraídos ipsis litteris trechos dos livros indicados na bibliografia da Banca. Isso não avalia e não seleciona, porque entra no campo da sorte e do chutômetro, beirando a arbitrariedade...

  • Ba leoni, você copia e cola esse seu comentário em todas as questões de filosofia! Tente contribuir positivamente para as questões e para de mi mi mi!

  • GABARITO LETRA A

    A resposta da questão pode ser extraída de uma página da obra mencionada no seguinte trecho: "A Teoria Pura do Direito limita-se a uma análise estrutural do Direito positivo, baseada em um estudo comparativo das ordens sociais que efetivamente existem e existiram historicamente sob o nome de Direito. Portanto, o problema da origem do Direito - o Direito em geral ou uma ordem jurídica particular - isto é, das causas da existência do Direito em geral ou de uma ordem jurídica particular, com seu conteúdo específico, ultrapassa o escopo desta teoria. São problemas da sociologia e da história e, como tais, exigem métodos totalmente diferentes dos de uma análise estrutural de ordens jurídicas dadas. [...] A Teoria Pura do Direito trata o Direito como um sistema de normas válidas criadas por atos de seres humanos. É uma abordagem jurídica do problema do Direito. [...] ideias. É evidente que o pensamento jurídico difere do pensamento sociológico e histórico. A "pureza" de uma teoria do Direito que se propõe uma análise estrutural de ordens jurídicas positivas consiste em nada mais que eliminar de sua esfera problemas que exijam um método diferente do que é adequado ao seu problema específico. O postulado da pureza é a exigência indispensável de evitar o sincretismo de métodos, um postulado que a jurisprudência tradicional não respeita ou não respeita suficientemente. A eliminação de um problema da esfera da Teoria Pura do Direito não implica, é claro, negar a legitimidade desse problema ou da ciência que dele trata. O Direito pode ser objeto de diversas ciências; a Teoria Pura do Direito nunca pretendeu ser a única ciência do Direito possível ou legítima. A sociologia do Direito e a história do Direito são outras. Elas, juntamente com a análise estrutural do Direito, são necessárias para uma compreensão completa do fenómeno complexo do Direito. Dizer que não pode existir uma teoria pura do Direito, porque uma análise estrutural do Direito restrita ao seu problema específico não é suficiente para uma compreensão completa do Direito equivale a dizer que uma ciência da lógica não pode existir, porque uma compreensão completa do fenômeno psíquico do pensamento não é possível sem a psicologia" (KELSEN, Hans. Direito, Estado e justiça na teoria pura do direito. In: _____. O que é justiça. São Paulo: Martins Fontes, 2001, p. 291-292).

  • Gosto muito da Teoria Pura do Direito de Kelsen e, sinceramente, acredito que a II está correta.

    Pelo menos beira muito a correção.

    Abraços.

  • Lucio, o problema do Item II é a afirmação de ser um "sincretismo metodológico". É justamente este sincretismo que Kelsen pretende evitar. 

  • Comentário do item II

    O erro da questão está na expressão sincretismo metodológico.

    Sincretismo significa fusão, união, mistura de algo. No caso da questão refere-se a mistura de metódos. E isso na prática significa trabalhar com a ideia da adoção de métodos de várias ciências (Economia, Sociologia, Psicologia...) para se encontrar o Direito. Daí o ponto central, porque Kelsen buscou foi afastar tal análise sincretista (TPD = Teoria Pura do Direito).

    Para aprofundamentos:

    Segundo Arnaldo Vasconcelos (2010, p.125): “Tal conceito rigoroso, como proposto, deverá ser alcançado mediante um processo de purificação, isto é, de exclusão do âmbito do Direito de tudo aquilo que não lhe seja essencial. Nas palavras de Kelsen; ‘Quer isto dizer que ela pretende liberar a ciência jurídica de todos os elementos que lhe são estranhos. Esse é o seu princípio metodológico fundamental’ (1974:17). Ao final do processo, deverá o Direito estar reduzido a ele mesmo, ao estritamente jurídico, e nada mais. Seu objetivo programático final é este: ‘Evitar um sincretismo metodológico que obscurece a essência da ciência e dilui os limites que lhe são impostos pela natureza do seu objeto’ (1974:18). Trata-se, pois, de impedir que a ciência do Direito se confunda com a teoria política ou a ética, com a sociologia ou a psicologia. Sob outro prisma, o que Kelsen agora chama ciência jurídica livre ou genuína ciência do espírito, deveria surgir purificada: a) ‘de toda ideologia política’ e b) ‘ de todos os elementos da ciência natural’ ( 1974:04). Parece ter-se fechado o círculo da pureza.”

    Fonte - http://ambitojuridico.com.br/site/?n_link=revista_artigos_leitura&artigo_id=11440

    Uma excelente caminhada na busca pelo conhecimento.

    :)

     

     

     

  • levanta mão quem interpretou sincretismo com tendo a ver com singular...


    afff..kkkk



  • Para adicionar uma discussão.

    Com a devida licença, penso que a alternativa III também está incorreta (III. A Teoria Pura do Direito limita-se a uma análise estrutural do Direito positivo.).

    Explico: Em que pense Kelsen ter analisado a estrutura do Direito positivo, ele não deixou de lado analises acerca da fundamentação deste direito (elemento legitimador desta estrutura, mas que com ela não se confunde). Deste modo, com fulcro na existência de uma "Norma Hipotética Fundamental", afastou - ou tentou afastar -  do debate jurídico-filosófico a ideia de que o ordenamento jurídico estava fundamentado em elementos externos ao Direito.

     

    Assim, penso que a afirmação simplista de que a Teoria Pura do Direito se limita a uma análise estrutural do Direito Positivo está incompleta, tendo em vista que além disso, também buscou analisar o elemento fundante deste Direito Positivo

     

    O que acham colegas do QC?

    Vida longa e prospera! 


ID
1138051
Banca
FCC
Órgão
DPE-SP
Ano
2013
Provas
Disciplina
Filosofia do Direito
Assuntos

Na obra O que é justiça?, Hans Kelsen explicita que sua Teoria Pura do Direito formula a regra de Direito (usando o termo em sentido descritivo) como um juízo .

Alternativas
Comentários
  • RESPOSTA: LETRA A. 

    O que a Teoria Pura procura identificar como relevante para a pesquisa jurídica é o estudo da validade (existência de uma norma jurídica), a vigência (a produção de efeitos de uma norma jurídica), a eficácia (condutas obedientes e observantes a uma norma jurídica). Toda pesquisa da Teoria Pura se resume e se baseia no estudo da norma jurídica.

    "A Teoria Pura do Direito está apenas tirando uma conclusão óbvia quando formula a regra de Direito (usando o termo em sentido descritivo) como um juízo hipotético em que o delito surge como condição essencial e a sanção como a conseqüência" (Kelsen, O que é Justiça? A Justiça, o Direito e a política no espelho da ciência,1998, p.271).

     “As sanções do Direito têm o caráter de atos coercitivos” (Kelsen, 2005, p.71); 

     “atos específicos de coerção, como sanções, são previstos em casos específicos pelas regras que formam a ordem jurídica” (Kelsen, 2005, p. 42); 

    "A sanção é a reação da ordem jurídica contra o delito” (Kelsen, 2005, p. 29); 

    “A sanção é conseqüência do ilícito; o ilícito (ou delito) é um pressuposto da sanção” (Kelsen, 2003, p. 43); 

    “sanções consistem na realização compulsória de um mal” (Kelsen, 2003, p. 122); 

    “o sentido da ordem jurídica é que certos males devem, sob certos pressupostos, ser aplicados” (Kelsen, 2003, p. 48-9). 

    “elas [as normas válidas] dão a um determinado indivíduo poder ou competência para aplicar a um outro indivíduo um ato coativo como sanção” (Kelsen, 2003, p. 36); 

    “é a ordem jurídica que, taxativamente, determina as condições sob as quais a coação física deverá ser  aplicada e os  indivíduos que a devem aplicar” (Kelsen, 2003, p. 40).

  • q horror essa prova, q horror...

  • Sucintamente, o juizo hipotético, caucado na imputação, pode ser sintetizado na máxima "se é A, deve ser B" (na qual A é o ilícito e B é a sanção). Logo, o ilícito é a condição, enquanto a sanção é a sua consequência.

  • GABARITO LETRA A

    Norma jurídica e proposição jurídica

    Kelsen, no livro Teoria Pura do Direito, diferencia a norma jurídica da proposição jurídica

    “Na medida em que a ciência jurídica apenas apreende a conduta humana enquanto esta constitui conteúdo de normas jurídicas, isto é, enquanto é determinada por normas jurídicas, representa uma interpretação normativa destes fatos de conduta. Descreve as normas jurídicas produzidas através de atos de conduta humana e que hão de ser aplicadas e observadas também por atos de conduta e, conseqüentemente, descreve as relações constituídas, através dessas normas jurídicas, entre os fatos por elas determinados. As proposições ou enunciados nos quais a ciência jurídica descreve estas relações devem, como proposições jurídicas, ser distinguidas das normas jurídicas que são produzidas pelos órgãos jurídicos a fim de por eles serem aplicadas e serem observadas pelos destinatários do Direito. Proposições jurídicas são juízos hipotéticos que enunciam ou traduzem que, de conformidade com o sentido de uma ordem jurídica - nacional ou internacional - dada ao conhecimento jurídico, sob certas condições ou pressupostos fixados por esse ordenamento, devem intervir certas conseqüências pelo mesmo ordenamentO determinadas. As normas jurídicas, por seu lado, não são juízos, isto é, enunciados sobre um objeto dado ao conhecimento. Elas são antes, de acordo com o seu sentido, mandamentos e, como tais, comandos, imperativos. Mas não são apenas comandos, pois também são permissões e atribuições de poder ou competência. Em todo o caso, não são - como, por vezes, identificando Direito com ciência jurídica, se afirma - instruções (ensinamentos). O Direito prescreve, permite, confere poder ou competência - não “ensina” nada. Na medida, porém, em que as normas jurídicas são expressas em linguagem, isto é, em palavras e proposições, podem elas aparecer sob a forma de enunciados do mesmo tipo daqueles através dos quais se constatam fatos” (KELSEN, Hans. Teoria Pura do Direito. São Paulo: Martins Fontes, 2006, pp. 80-81).

  • Por que as normas jurídicas são consideradas por Kelsen como normas hipotéticas?

    Porque “As ordens sociais positivas têm sempre de estatuir condições sob as quais não é proibido matar, subtrair a propriedade alheia, mentir. Também isto mostra que todas as normas gerais de uma ordem social empírica, incluindo as normas gerais de omissão, apenas podem prescrever uma determinada conduta sob condições ou pressupostos bem determinados, e que, por isso, toda norma geral produz uma conexão entre dois fatos, conexão essa que pode ser descrita pelo enunciado segundo o qual, sob um determinado pressuposto, deve realizar-se uma determinada conseqüência. E esta, como se mostrou, a expressão verbal do princípio da imputação, diversa da que convém ao princípio da causalidade” (KELSEN, Hans. Teoria Pura do Direito. São Paulo: Martins Fontes, 2006, p. 112).

    Ele afirma que somente as “Apenas as normas individuais podem ser categóricas, no sentido de que prescrevem, autorizam ou positivamente permitem uma dada conduta de determinado indivíduo sem a vincular a determinado pressuposto” (KELSEN, Hans. Teoria Pura do Direito. São Paulo: Martins Fontes, 2006, p. 112).

    Contudo ele reconhece a possibilidade de normas individuais serem também hipotéticas, “quer dizer, fixar apenas como condicional mente devida a especificada conduta de um determinado indivíduo. Assim acontece por exemplo, quando o tribunal ordena a execução no patrimônio do devedor em mora, apenas sob a condição de o devedor não pagar a soma devida dentro de um determinado prazo ou quando o tribunal ordena a execução da pena aplicada a um determinado indivíduo, apenas na hipótese de este indivíduo cometer de novo um delito punível, dentro de um determinado prazo” (KELSEN, Hans. Teoria Pura do Direito. São Paulo: Martins Fontes, 2006, p. 113).

     

  • A questão aborda o tema relacionado à imputação, desenvolvida por Hans Kelsen tanto em “o que é justiça” quanto em a teoria pura do direito.

    Conforme Kelsen: "Esse princípio tem, nas regras de Direito, uma função análoga a que tem o princípio da causalidade nas leis naturais por meio das quais a ciência natural descreve a natureza. Uma regra de direito, por exemplo, é a afirmação de que, se um homem cometeu um crime, uma punição deve ser infligida a ele, ou a afirmação de que, se um homem não paga uma dívida contraída por ele, uma execução civil deve ser dirigida contra sua propriedade. Formulando de um modo mais geral: se um delito for cometido, uma sanção deve ser executada".

    Segundo Kelsen (1998, p. 91) “Se designa como “imputação” a ligação de pressuposto e conseqüência expressa na proposição jurídica com a palavra “dever ser”. Enquanto a ciência jurídica trata de normas jurídicas através da imputação, a ciência natural utiliza a lei da causalidade para estudar seu objeto. “A ciência jurídica, com efeito, não pretende, com as proposições jurídicas por ela formuladas, mostrar a conexão causal, mas a conexão de imputação, entre os elementos do seu objeto” (Kelsen, 1998, p. 100).

    Portanto, Kelsen explicita que sua Teoria Pura do Direito formula a regra de Direito (usando o termo em sentido descritivo) como um juízo hipotético em que o delito surge como condição essencial e a sanção como a consequência.

    Gabarito do professor: letra a.

    Referência:

    KELSEN, H. Teoria Pura do Direito. Tradução João Baptista Machado. 6.ed. São Paulo: Martins Fontes, 1998.

    KELSEN, H. O que é Justiça?.


  • Normas Categóricas X Normas hipotéticas

    Norma categórica é aquela que estabelece que uma determinada ação deve ser realizadas; norma hipotética é aquela que estabelece que uma determinada ação deve ser realiza caso se verifique uma determinada condição.


ID
1245694
Banca
MPE-SC
Órgão
MPE-SC
Ano
2014
Provas
Disciplina
Filosofia do Direito
Assuntos

Analise o enunciado da questão abaixo e assinale se ele é Certo ou Errado.


Na obra de Jean-Jacques Rousseau, nota-se a preocupação com o respeito à vontade geral dos indivíduos que compõem o Estado, além de uma constante crença na bondade da natureza humana, ao contrário do que defendeu Hobbes. Para Rousseau o homem nasce bom, porém, com as disputas existentes no meio em que se encontra inserido, acaba se degenerando. Enfatizando o valor ao predomínio da vontade individual, defende que direitos essenciais possam ser renunciados, como a liberdade e igualdade.

Alternativas
Comentários
  • O erro encontra-se no final: quando alega que podem ser renunciados os direitos de liberdade e igualdade,de fato o começo está correto:

    "Rousseau, diferentemente de Hobbes, afirmava que o homem, antes do contrato social, vivia no Estado da Natureza feliz, alegre e saltitante, desfrutando de liberdade e igualdade. Sem comandos políticos, o homem vivia no livre exercício de seus direitos naturais, em uma Idade de Ouro, onde não havia propriedade privada nem corrupção."

    "Em sua obra “O Contrato Social” (1.762), o homem, visando recuperar seu bem estar primitivo, teria transferido seus direitos naturais ao Estado em troca de direitos civis (vê-se aqui a dualidade rousseauniana entres direitos naturais e direitos civis, sendo que estes somente seriam justos e legítimos se fundados naqueles). Direitos naturais, então, seriam os direitos civis sob a tutela do Estado. Não haveria renúncia à liberdade, pois tal ato seria incompatível com a natureza humana. Toda a noção de contrato social deste filósofo está baseada no bem comum, na união de forças destinada à utilidade geral, que não se limita ao somatório das vontades particulares."


    Bons Estudos

  • FALSA.

  • A única parte errada é "defende que direitos essenciais possam ser renunciados, como a liberdade e igualdade", como muito bem explicou o colega José Júnior.

    Ao contrário de Rousseau, Hobbes achava que o homem já nascia mau. Este vídeo explica muito bem isso.

    https://www.youtube.com/watch?v=jz3AcQLx2_0

     

  • A questão demanda compreender axiomas de Rousseau.

    Liberdade e igualdade, ao contrário do exposto, para Rousseau, são irrenunciáveis.

    Outro equívoco no enunciado é o predomínio da vontade individual, quando, em verdade, para Rousseau predomina a vontade geral.

    Logo, a assertiva é INCORRETA.

    GABARITO DO PROFESSOR: ERRADO.


ID
1298653
Banca
NC-UFPR
Órgão
DPE-PR
Ano
2014
Provas
Disciplina
Filosofia do Direito
Assuntos

De acordo com a jurista francesa Simone Goyard-Fabre, a questão central da filosofia jurídica é a discussão acerca do fundamento para o ordenamento jurídico, que historicamente foi tomada pelo debate entre naturalismo e positivismo. Acerca do tema, é correto afirmar: 


1. Na filosofia hegeliana, a realização da razão no direito prescinde do Estado, eis que o direito positivo é arbitrário e naturalmente tendente à alienação, e a verdadeira cidadania só pode aflorar de uma sociedade autônoma face ao poder estatal.
2. A teoria autopoiética de Luhmann necessita da referência a um sistema jurídico “aberto”, cuja redefinição dos elementos é possível somente por meio de um referencial externo social, político ou cultural.
3. O advento do constitucionalismo anda lado a lado com o triunfo do racionalismo moderno, eis que possibilitou a interpretação racional do ordenamento jurídico enquanto “sistema” hierarquizado. 

Assinale a alternativa correta.

Alternativas
Comentários
  • Assertiva 1: incorreta: o Estado para Hegel é o que é em-si e para-si e, portanto, tem a efetividade de sua universalidade ou totalidade plena. Esta totalidade refere-se à união do espírito objetivo e o espírito subjetivo em que o indivíduo tem sua realidade e objetividade moral sendo parte do todo ético. Dessa forma, o indivíduo tem uma relação jurídica para com o Estado, isto é, tem um tribunal acima de si que realiza o direito enquanto liberdade. 

    Assertiva 2 : incorreta: A concepção luhmanniana representa uma cisão com o modelo clássico de ciência, que pretendeu descrever a vida social orientando-se pela ideia de insatisfação com a realidade, tão característica dos séculos XIX e XX, para, na espera de um melhor futuro, fundamentar suas teorias sociais na ideia de evolução da civilização, sendo o homem o operador central desse sempre aperfeiçoamento da sociedade. Os subsistemas sociais nascem em razão de sua função redutora de complexidade do sistema social, contudo, uma vez constituído um subsistema, este permanece como produto de si mesmo. Neste sentido diz-se que os sistemas sociais são autopoiéticos. Dizer que os sistemas sociais são autopoiéticos, portanto, significa que estes produzem seus próprios elementos, possibilitando a identificação do próprio sistema como unidade. A autopoiese é, dessa forma, um pressuposto para o fechamento operacional ao mesmo tempo em que esta recursividade de autorreprodução (autopoiesis) é condição de abertura do sistema. Isso quer dizer que aquele pode se relacionar com o seu meio, contudo é o próprio sistema que preordena a forma como essa relação se opera. Sendo assim, toda operação é uma operação dentro do sistema. Dito em outras palavras, não haverá referência externa sem autorreferência. 

    Assertiva 3 - correta: o constitucionalismo, em termos genéricos e supranacionais, constitui-se parte do estabelecimento de normas fundamentais de um ordenamento jurídico de um Estado, localizadas no topo da pirâmide normativa, ou seja, sua constituição.



  • o sistema jurídico é operativamente fechado, aberto cognitivamente.

  • Sobre o item 3:

    Segundo Simone Goyarde-Fabre, “Pode-se, portanto, dizer que a inflação humanista que atravessava o século do Iluminismo não só fazia do homem o condutor da vida política, como, mais precisamente, dava tamanho ímpeto ao indivíduo e à razão de que é portador que este, tornado o único agente da potência e da autoridade políticas, era saudado, em sua autonomia, como o criador das normas da ordem jurídica”. Mas o avanço das idéias não se limitava apenas a construções teóricas e especulativas fruto da racionalidade humana. O racionalismo associou a necessidade de ordem pública às reivindicações de progresso e liberdade, no seu caráter prático.

    Simone Goyarde-Fabre lembra que “a filosofia do direito orientou a reflexão sobre o poder para uma concepção constitucionalista que realmente construiu o pórtico do Estado Moderno”, citando como referência desta concepção publicações de diversos juristas e filósofos, entre outros, Burlamaqui, Rousseau e Kant. Destaca-se, assim, no século XVIII a evolução do racionalismo consagrando o conceito de “Constituição” pelo Direito político, condensando-se a capacidade normativa fundamental do Poder do Estado sobrevivendo ainda na teoria constitucional contemporânea.

    Fonte:

  • A questão em comento requer análise de cada uma das assertivas e conhecimento de axiomas de Hegel, Luhman e do constitucionalismo moderno.

    A assertiva I está INCORRETA.

    Hegel não nega o Estado.

    Hegel não fixa o Direito como algo arbitrário e capaz de gerar alienar. Em verdade, o Direito, para Hegel, promove a emancipação.

    Hegel não pensa na cidadania como uma alternativa crítica diante do Poder Estatal.

    A assertiva II está INCORRETA.

    A autopoiese do Luhman não é aferida tão somente por elementos externos, mas também com base em elementos internos.

    A assertiva III está CORRETA.

    De fato, o Constitucionalismo Moderno é o triunfo do racionalismo, de maneira que o Direito passa a ter a hipótese de uma leitura sistemática.

    Logo, só a assertiva III está CORRETA.

    Diante do exposto, vamos analisar as alternativas da questão.

    LETRA A- INCORRETA. A assertiva 3 é a única correta.

    LETRA B- CORRETA. A assertiva 3 é a única correta.

    LETRA C- INCORRETA. A assertiva 3 é a única correta.

    LETRA D- INCORRETA. A assertiva 3 é a única correta.

    LETRA E- INCORRETA. A assertiva 3 é a única correta.

    GABARITO DO PROFESSOR: LETRA B


ID
1370563
Banca
FCC
Órgão
AL-SP
Ano
2010
Provas
Disciplina
Filosofia do Direito
Assuntos

Constituem características do controle social, por meio do direito, sob a ótica funcionalista,

Alternativas
Comentários
  • Qual óptica funcionalista?

    1 - Günther Jakobs?

    2 - Antonio Castanheira Neves?

    3 - Niklas Luhmann?

  • A questão em comento demanda conhecimento acerca do funcionalismo.

    No funcionalismo, em síntese rápida, a sociedade é explicada com base no funcionamento de instituições específicas e o não funcionamento gera desregulação e desagregação. No funcionalismo, segundo Durkheim, a sociedade busca solidariedade e estabilidade. As estruturas de uma sociedade não devem laborar fragmentadas e trabalhando em conjunto geram estabilidade social.

    Na perspectiva funcionalista, o Direito atua como instrumento coercitivo que garante o bem comum e a paz social, e isto funciona na medida em que o Direito possui normas coercitivas, cogentes, imperativas, exigíveis, que condicionam o agir na vida social.

    Feitas tais ponderações, nos cabe comentar as alternativas da questão.

    LETRA A- INCORRETA. Incompatível com a perspectiva do controle social.

    LETRA B- INCORRETA. Para uma perspectiva funcionalista do Direito, embora a tradição seja importante, a validade das normas e a ideia de bem comum são dinâmicas e admitem mutação. Basta que a interação entre os agentes que geram estabilidade social almejem isto.

    LETRA C- INCORRETA. A alternativa só aponta desarmonia e desequilíbrios sociais.

    LETRA D- INCORRETA. Não necessariamente a perspectiva funcionalista do Direito e o controle social lidam com a construção do bem comum e da paz social através da espontaneidade.

    LETRA E- CORRETA. Alternativa mais compatível com a conjugação de funcionalismo e controle social.

    GABARITO DO PROFESSOR: LETRA E


ID
1370569
Banca
FCC
Órgão
AL-SP
Ano
2010
Provas
Disciplina
Filosofia do Direito
Assuntos

No que diz respeito às formas de atuação do direito como fator de mudança social, aquela que se caracteriza pela aplicação de um novo sistema jurídico para substituir o direito do Estado e responder às necessidades sociais é denominada direito

Alternativas
Comentários
  • O direito alternativo possui, entre suas características, a ênfase de atuar de forma indutiva, ou seja, o julgador, a partir de sua concepção de justiça, decide inicialmente e apenas depois buscará no ordenamento jurídico sua fundamentação, chegando a ter decisões fundamentadas até contra legem. Tais decisões, muitas vezes, poiam-se nos princípios do direito que por serem gerais são mais maleáveis ao operador.

  • "A busca de um direito livre, insurgente, marcado pelo conflito, transformador, de matriz sociológica, ocupado da legitimidade fática, integrou o ideário que alimentou uma geração de juristas e estudantes (em sua maior parte, sim, assumidamente “de esquerda”), os quais queriam ser protagonistas de um direito melhor e mais justo do que aquele que está positivamente posto, sentimentos que certamente ainda pairam no ar rarefeito do simbólico “ordenamento jurídico” repleto de egoísmo e carente de alteridade. Um direito interpretado por uma matriz e um sistema adequado, constitucional, político e socialmente orientado, autopoiético e sempre focado em novas possibilidades. Essas, em suma, algumas das notas e dos indicativos desta corrente de pensamento que muito contribuiu para forjar a cultura jurídica brasileira.

    Forte “pegada” nos direitos humanos, defesa da bandeira da luta por reforma agrária, rejeição da exclusão social, crítica da criminalização seletiva, pauta de incremento da cidadania e aprofundamento da democracia substancial para obter maior justiça social aos oprimidos, essas e outras questões orbitavam a esfera de interesse daqueles sujeitos históricos ativistas, juristas orgânicos que, corajosamente, apesar de todos os rótulos e pré-conceitos dados por quem ignorava as bases epistêmicas do movimento, flertaram com a fileira do direito alternativo, diferenciando-se, não raro, pela qualidade e autenticidade de seus inovadores e progressistas posicionamentos. Partir do dissenso para construir novos consensos, abalar o que está posto como “verdade”, essa a árdua missão do “alternativismo” na versão tupiniquim."

    FONTE: http://www.justificando.com/2014/08/25/o-direito-alternativo-continua-vivo/


ID
1427506
Banca
CESPE / CEBRASPE
Órgão
DPU
Ano
2015
Provas
Disciplina
Filosofia do Direito
Assuntos

Quanto à sociologia jurídica, julgue o  item  subsequente.

O positivismo jurídico representa o conjunto das teorias da Escola da Exegese. De acordo com essa escola, o direito só pode ser considerado como fruto do trabalho do legislador (direito estatal) e as leis devem ser interpretadas racional e logicamente.

Alternativas
Comentários
  • ITEM CORRETO

    O positivismo jurídico ou juspositivismo é uma corrente da teoria do direito que procura explicar o fenômeno jurídico a partir do estudo das normas positivas, ou seja, daquelas normas postas pela autoridade soberana de determinada sociedade. Ao definir o direito, o positivismo identifica, portanto, o conceito de direito com o direito efetivamente posto pelas autoridades que possuem o poder político de impor as normas jurídicas. Metodologicamente, o positivismo jurídico representa uma opção pela neutralidade do intérprete do direito, sustentando que ele não deve se posicionar relativamente aos conteúdos das normas, mas apenas descrevê-los, de modo a preservar a vontade política expressa por aqueles que criaram as normas.

  • A Teoria Subjetiva defendida pela Escola da Exegese, entende que ao se interpretar o texto legal deve-se buscar a vontade de quem a elaborou.

    Para a Teoria Objetiva, defendida pela Escola Histórica do Direito, o trabalho do interprete é pesquisar a vontade da lei.

  • Análise da questão:

    Embora a banca tenha considerado a assertiva correta, discordo parcialmente. Acredito que a denominada “Escola da exegese" seja apenas uma faceta de muitos movimentos teóricos que podem ser enquadrados no positivismo jurídico.

    Assim, não podemos ignorar os múltiplos movimentos do positivismo jurídico, como, por exemplo, a distinção entre positivismo exclusivo e positivismo inclusivo. O primeiro, nas lições de Eduardo Ribeiro Moreira (2010), trabalha a concepção jurídica por uma teoria do ordenamento, fundada na teoria das fontes, pensadas a partir do direito privado como vetor maior de aplicabilidade, não admitindo nenhuma relação do direito com a moral. A sanção e a autoridade competente são os principais elementos desse modelo.

    Por outro lado, o positivismo inclusivo trabalha a concepção jurídica por uma teoria do ordenamento, fundado na antiga teoria das fontes, dissociadas do direito constitucional. Diz-se positivista pela primazia de uma teoria da norma jurídica, porém flexibiliza o pensamento de Hans Kelsen. Entretanto, esse positivismo inclusivo que tem como representante, por exemplo,
    Herbert Hart, aceita uma relação entre direito e moral. A moral é contingente, dependendo do que o autor denominou de regra de reconhecimento. O fator social, ligado às normas é o principal elemento do direito (MOREIRA, 2010, p. 450/451).

    Em relação à escola da exegese e sua conexão com o positivismo, temos como elucidativa a lição do filósofo do direito Paulo Nader:

    “A atitude assumida pelos juristas franceses, ao considerarem Direito Positivo apenas o Código Napoleão e entenderem que o Código não possuía lacunas, originou a formação da Escola da Exegese. Esta crença na infalibilidade do Código Civil, que satisfazia, segundo os juristas da época, a todas as necessidades da vida social, desde que o intérprete examinasse o seu conteúdo e tirasse as conclusões lógicas, gerou a necessidade de reconstrução do pensamento do legislador. A técnica de revelação da vontade do legislador exigia que o intérprete examinasse bem o valor semântico de todas as palavras, comparando o texto a ser interpretado com outros, para evitar os conflitos e contradições. Pelos subsídios da gramática o intérprete vai descobrir o pensamento do legislador, que deve ser acatado incondicionalmente, qualquer que seja o resultado da interpretação, ainda que iníquo e absurdo. A lógica formal será uti lizada de acordo com os elementos obtidos no texto, sem dele afastar-se. Contudo, admite-se a pesquisa dos elementos históricos, na medida em que esclareça a intenção do legislador. Permite-se ainda ao intérprete recorrer às obras doutrinárias que serviram de base ao legislador" (NADER, 2014)

    “O pensamento predominante da Escola era codicista, de supervalorização do código. Pensavam os seus adeptos que o código encerrava todo o Direito. Não há veria qualquer outra fonte jurídica. Além do código, o intérprete não deveria pesquisar o Direito na organização social, política ou econômica. A sua função limitava-se ao estudo das disposições legais. Em seu teor, o código era considerado absoluto, com regras para qualquer problema social.

    Na da havia, no social, que houvesse escapado à previsão do legislador. O código não apresentava lacunas. Laurent afirmou que os códigos nada deixavam ao arbítrio do intérprete e o Direito estava escrito nos textos autênticos. Para Demolombe o lema era “os textos acima de tudo!". Aubry sentenciou: “Toda a lei, mas nada além da lei!" Estas exclamações dão bem a medida do apego ao código e da rejeição às outras fontes vivas do Direito. A ideia norteadora da Escola da Exegese foi sintetizada por F. Laurent, um de seus corifeus: “ Se uma teoria não tem as suas raízes nos textos, nem no espírito da lei, deve ser rejeitada; ao contrário, será jurídica se expressa na letra da lei e nos trabalhos preparatórios. Neste caso, deve ser aceita, não se recuando diante de alguma consequência".

    O principal objetivo da Exegese era revelar a vontade do legislador, daquele que planejou e fez a lei. A única interpretação correta seria a que traduzisse o pensamento de seu autor. Consequência dos postulados expressos pela Escola foi o entendimento de que o Estado era o único autor do Direito, pois detinha o monopólio da lei e do código. Como os tradicionalistas não admitiram outra fonte normativa, a sociedade ficava impedida de criar o Direito costumeiro. Em resumo, os postulados básicos da Escola da Exegese foram:

    a) Dogmatismo Legal;

    b) Subordinação à Vontade do Legislador;

    c) O Estado como Único Autor do Direito (Destaque do professor)" (NADER, 2014, capítulo 27).

    Gabrito: CERTO
  • Excelente a análise do professor no tocante a presente questão, marquei a questão como errada justamente por que entendo, com base no conhecimento comum reproduzido nos manuais acerca do positivismo, que não se pode afirmar que "o positivismo jurídico representa o conjunto das teorias da escola da exegese". A Escola da Exegese é apenas uma expressão mais primitiva do positivismo jurídico, que engloba também, e de forma muito mais relevante na modernidade, o positivismo analítico de Kelsen, que absolutamente não é da escola exegética, o positivismo de Hart no direito consuetudinário, e as variadas vertentes de positivismo inclusivo e exclusivo que vieram do confronto moderno entre positivismo, jusnaturalismo e jusrealismo. A Escola da Exegese realmente é historicamente ligada o início da identificação dessa posição jurídica, embora os autores já mencionem a existência de ideário positivista mesmo antes da escola da exegese, nos fundamentos filosóficos do estado absolutista e das monarquias "racionais" e constitucionais na fase do iluminismo. Com certeza teria recorrido do gabarito, que só poderia ser considerado correto se a afirmação fosse inversa: "o conjunto  das teorias da escola da exegese representa uma expressão do positivismo jurídico". Na forma regidida a assertiva é falsa.

  • Concordo com tudo que Vinícius Gonçalves falou. Sem lógica ser "correta" essa assertiva rsrs.

    Sobre o Juspositivismo, vale ressaltar: "O positivismo é um movimento de interpretação e aplicação do direito, de estudo do direito que nasce no século XIX e tem a sua maior projeção na primeira metade do século XX. Existem várias vertentes no Positivismo.Uma, chamada vertente do Positivismo Normativista, cujo principal autor é Hans Kelsen, que praticamente iguala o conceito de Estado ao conceito de Direito. Para Kelsen não há Direito fora do Estado e esse Estado para ele é norma. Estado não é uma instituição, mas um conjunto de normas. Só é Direito aquilo que oficialmente emana do Estado, pelo devido processo legislativo.Existem outras vertentes, uma das quais a mais importante é do Positivismo Sociológico, um dos principais autores é o Norberto Bobbio, que vê o Estado como um movimento social que, no entanto, só toma relevância a partir das normas que são positivadas pelo Estado.

    Outra, a inicial, é o Positivismo Exegético, forte na escola da exegese pós-revolução francesa, que defendia que o juiz deveria ser simplesmente le bouche de loi, sem lhe ser permitido interpretar. Assim, sempre que dúvida houvesse, deveria ele buscar a solução no próprio Direito posto, pois buscava-se uma codificação plena e perfeita de todas as possíveis situações.Então, a marca mais importante do Positivismo é dar um realce exacerbado à norma produzida pelo Estado sempre e isso é o que é o mais importante na visão positivista". (RESUMO DE HUMANÍSTICA).

  • Está correto porque o  positivismo surgiu para responder a abstração do Direito Natural e trouxe consigo forma rígidas de interpretação.  A escola da Exegese, junto ao Código de Napoleão, é o ápice dessa forma de enxergar o Direito, de forma literal, racional e gramatical. Ao juiz cabia apenas a aplicação da lei, de forma superficial, sua vontade era a vontade do legislador. 

    O positivismo surgiu como uma forma prática e realista à abstração e ao idealismo do Direito Natural (supostamente imutável e eterno), expressando-se por meio das normas válidas de um determinado espaço e tempo. 

    A Escola da Exegese surgiu como uma das consequências da criação do Código de Napoleão (1804), forma de interpretação que ocorria mediante privilégio dos aspectos gramaticais e lógicos. Com ela, tem-se o ápice do positivismo jurídico.

     

  • GABARITO: CERTO

    A Escola da Exegese também pregava o Estado com a única fonte do direito, pois todo o ordenamento jurídico seria originado da lei e, esta, por ser proveniente do legislador, teria como origem o Estado, ou seja, das fontes formais do direito atualmente aceitas pelo ordenamento jurídico brasileiro, somente a lei era admitida como fonte do direito.

    Fonte: CHAVES, Daniel Rodrigues. A Escola da Exegese: origem, características e contribuições. Revista Jus Navigandi, ISSN 1518-4862, Teresina, ano 17, n. 3440, 1 dez. 2012. Disponível em: https://jus.com.br/artigos/23137. Acesso em: 18 out. 2021.


ID
1472470
Banca
FGV
Órgão
OAB
Ano
2015
Provas
Disciplina
Filosofia do Direito
Assuntos

Rudolf Von Ihering, em A Luta pelo Direito, afirma que “ O fim do direito é a paz, o meio de atingi-lo,a luta.” Assinale a afirmativa que melhor expressa o pensamento desse autor

Alternativas
Comentários
  • O fim do Direito é a paz; o meio de atingi-lo, a luta. O Direito não é uma simples idéia, é força viva. Por isso a justiça sustenta, em uma das mãos, a balança, com que pesa o Direito, enquanto na outra segura a espada, por meio da qual se defende. A espada sem a balança é a força bruta, a balança sem a espada é a impotência do Direito. Uma completa a outra. O verdadeiro Estado de Direito só pode existir quando a justiça bradir a espada com a mesma habilidade com que manipula a balança.

    Rudolf Von Ihering

  • resposta: a)O Direito de uma sociedade é a expressão dos conflitos sociais desta sociedade, e ele resulta de uma luta de pessoas e grupos pelos seus próprios direitos subjetivos. Por isso, o Direito é uma força viva e não uma ideia.

  • O direito definido pelo autor decorre da constante luta das classes e indivíduos, dos povos, do poder estatal e pela constante tentativa de manutenção “da ordem”. Para o autor, as consequências destes conflitos resultam na criação de novos conceitos e ordenamentos, ou o aprimoramento destes, e esses fatos representam a evolução dos institutos jurídicos mais relevantes para o aprimoramento das sociedades. Não existe direito (norma) sem essa inquietação, sem essa constante luta.  

  • A alternativa correta é a letra “a". A ideia de direito como “força viva" é bem delimitada por Ihering em “A Luta pelo Direito". Para o autor, “A idéia do direito encerra uma antítese que se origina nesta idéia, da qual jamais se pode, absolutamente, separar: a luta e a paz; a paz é o termo do direito, a luta é o meio de obtê-lo". Portanto, só se alcança a paz (fim último do direito) por meio da luta.

    Ainda, para enfatizar essa ideia, temos que:

    “Esta luta perdurará tanto como o mundo, porque o direito terá de precaver-se sempre contra os ataques da injustiça. A luta não é, pois, um elemento estranho ao direito, mas sim uma parte integrante de sua natureza e uma condição de sua ideia. Todo direito no mundo foi adquirido pela luta; esses princípios de direito que estão hoje em vigor foi indispensável impô-los pela luta àqueles que não os aceitavam; assim, todo o direito, tanto o de um povo, como o de um indivíduo, pressupõe que estão o indivíduo e o povo dispostos a defendê-lo. O direito não é uma idéia lógica, porém idéia de força; é a razão porque a justiça, que sustenta em uma das mãos a balança em que pesa o direito, empunha na outra a espada que serve para fazê-lo valer. A espada sem a balança é a força bruta, a balança sem a espada é o direito impotente; completam-se mutuamente: e, na realidade, o direito só reina quando a força dispendida pela justiça para empunhar a espada corresponde à habilidade que emprega em manejar a balança" (IHERING, p. 22).

    Fonte: IHERING, Rudolf Von. A luta pelo direito. Versão para ebook. Ano: 2000. 64 páginas.


  • Obrigado......acertei

  •  O fim do DIREITO é a paz (é uma força viva) o meio de atingi-lo, a luta (pessoas e grupos pelos seus próprios direitos).”


ID
1592227
Banca
FGV
Órgão
OAB
Ano
2015
Provas
Disciplina
Filosofia do Direito
Assuntos

“Mister é não olvidar que a compreensão do direito como 'fato histórico-cultural' implica o conhecimento de que estamos perante uma realidade essencialmente dialética, isto é, que não é concebível senão como 'processus', cujos elementos ou momentos constitutivos são fato, valor e norma (...)"

                                                                                              (Miguel Reale, in Teoria Tridimensional do Direito)  

Assinale a opção que corretamente explica a natureza da dialética de complementaridade que, segundo Miguel Reale, caracteriza a Teoria Tridimensional do Direito.  


Alternativas
Comentários
  • LETRA B

    Nos termos da preleção do próprio Miguel Reale, jusfilósofo, artífice da Teoria Tridimensional do Direito: 


    "Por força do princípio de complementaridade, opera-se um raciocínio dialético, também denominado 'dialética de implicação e polaridade', segundo a qual os elementos em contraste não se fundem*mas, ao contrário, se correlacionam, mantendo-se distintos.


    [...]


    Daí a necessidade de ser a norma jurídica sempre objeto de interpretação, não como um objeto ideal, - como se fosse uma asserção lógico-sintética – mas sim como um enunciado em necessária correlação com a base fático-axiológica"



    *Em razão disso, excluem-se as alternativas "a" e "c", porquanto não há que se falar em síntese ou fusão, mas, antes, em uma relação de complementaridade entre elementos que se mantêm distintos, embora sejam reciprocamente suplementares (se houvesse fusão ou síntese, não faria sentido cogitar em complementação mútua). Para o entendimento figurativo dessa linha de raciocínio, basta trazer a resgate mnemônico o relacionamento entre indivíduos de personalidades opostas, que, segundo se diz, "se complementam". 



    A alternativa "d" pode ser descartada à luz do próprio excerto trazido no enunciado da questão, que fala da inviabilidade de concepção da realidade dialética de outra forma que não a de "um 'processus'", inconciliável com a ideia de "estrutura estática".

  • A alternativa correta está na letra “b”. Na compreensão do direito como “fato histórico-cultural”, segundo Miguel Reale, ressalta-se a necessidade de perceber a nossa inserção em uma realidade dialética. Acontece que, nesse âmbito dialético, apesar da “descoberta inevitável de lados opostos” que se revelam (desocultam) em aparente contradição, não há que se falar em conflito no sentido de independência. Na verdade, há de se falar em uma interdependência. Esses lados opostos, segundo REALE (p. 72) estão em mútua e necessária relação. Nesse sentido:

    “Dessa colocação do problema resulta o caráter dialético do conhecimento, que é sempre de natureza relacional, aberto sempre a novas possibilidades de síntese, sem que esta jamais se conclua, em virtude da essencial irredutibilidade dos dois termos relacionados ou relacionáveis. É a esse tipo de dialética, que denomina dialética de complementaridade, da qual a dialética dos opostos, de tipo marxista ou hegeliano, não é senão uma expressão particular, com as modificações resultantes da análise fenomenológica de seus termos, notadamente para se desfazer a confusão entre contrários e contraditórios. No âmbito da dialética de complementaridade, dá-se a implicação dos opostos na medida em que se desoculta e se revela a aparência da contradição, sem que com este desocultamento os termos cessem de ser contrários, cada qual idêntico a si mesmo e ambos em mútua e necessária correlação.

    É sobretudo no mundo dos valores e da praxis que mais se evidencia a existência de certos aspectos da realidade humana que não podem ser determinados sem serem referidos a outros aspectos distintos, funcionais, ou até mesmo opostos, mas ainda sim essencialmente complementares. Tal correlação de implicação não pode jamais se resolver mediante a redução de uns aspectos nos outros: na unidade concreta da relação instituída, tais aspectos mantém-se distintos e irredutíveis, daí resultando a sua dialeticidade, através de sínteses relacionais progressivas que traduzem a crescente e sempre renovada interdependência dos elementos que nela se integram.” (REALE, p. 72)

    Fonte: MIGUEL REALE, Teoria Tridimensional do Direito, São Paulo, 1994.


  • Até concordo com a letra B, discordo somente da afirmativa que são contrários...são ciências distintas porém que se correlacionam e não que estão na contramão uma das outras....

  • Questão inútil...

  • CONCORDO COM A CAROLINE FERRI...

  • Que questão ridícula !

     

  • Agora entendi de onde saem os discursos de Marina Silva (nem contra e nem a favor, muito pelo contrário) !

  • T. Franco, parabéns pelo espetacular comentário! Fantástico!

  • Teoria tirada do fabuloso gerador de lero lero
  • Não consigo entender a aplicação de algo completamente sem utilidade num exame.

  • Na moral, alguém dedica seu tempo estudando essa matéria? Só serve pra chutar e passar raiva!

  • Tirando a D, as outras 3 poderiam estar corretas a depender do significado que se dê para determinados termos como síntese, ocultamento, desocultamento, sobreposição, enfim, uma viagem na maionese que nunca chega a lugar nenhum.

  • Não percebo que fato, valor e norma sejam contrários. São distintos, complementares e até se retroalimentam, levando, em tese, à evolução da norma e do Direito. Em nenhum momento percebo qualquer ideia no que tange à contrariedade entre esses elementos. Questão um tanto quanto mal formulada.

  • "E por dialética se entendia a da linha de Hegel, com a contraposição entre uma tese e uma antítese, de cuja oposição surgiria, como terceiro termo, uma síntese, expressão compreensiva da verdade almejada.

    [...]

    Em seu livro Experiência e Cultura, Reale refere-se a diversas formas de dialética diferentes do modelo hegeliano ou marxista , formas essas que culminam, por assim dizer, em sínteses abertas, que representam, não o superamento da contradição, mas a correlação tensional entre elementos contrários.

    Dentre esses tipos de dialética não hegeliana merece destaque a chamada dialética de complementaridade [...]" (Enciclopédia Jurídica da PUCSP, tomo I (recurso eletrônico): teoria geral e filosofia do direito / coords. Celso Fernandes Campilongo, Alvaro Gonzaga, André Luiz Freire - São Paulo: Pontifícia Universidade Católica de São Paulo, 2017)

    Letra A - ERRADA

    A relação entre os polos opostos que são o fato, a norma e o valor, produz uma síntese conclusiva entre tais polos.

    A afirmação é da dialética de Hegel e não da dialética da complementariedade ("sínteses abertas").

    Letra B - CORRETA

    A implicação dos opostos na medida em que se desoculta e se revela a aparência da contradição, sem que, com esse desocultamento, os termos cessem de ser contrários.

    A dialética da complementariedade busca revelar a forma tridimensional que os pontos de tensão fato, valor e norma no tempo e no espaça fazem revelar a concepção do Direito para aquele espaciotemporal, ou seja, nada definitivo como almeja Hegel.("representam, não o superamento da contradição, mas a correlação tensional entre elementos contrários").

    Letra C - ERRADA

    A síntese conclusiva que se estabelece entre diferentes termos, conforme o modelo hegeliano de tese, antítese e síntese.

    A dialética da complementariedade ("sínteses abertas") contraria a dialética na forma de Hegel ("síntese, expressão compreensiva da verdade almejada").

    Letra D - ERRADA

    A estrutura estática que resulta da lógica de subsunção entre os três termos que constituem a experiência jurídica: fato, norma e valor.

    Não há estrutura estática na dialética da complementariedade. Veja: "sínteses abertas, que representam, não o superamento da contradição, mas a correlação tensional entre elementos contrários". Se a síntese é aberta, não há falar em estrutura estática.

    Isso se alinha aos dizeres de Miguel reale sobre a conceituação de Direito:

    Direito é a realização ordenada e garantida do bem comum numa estrutura tridimensional bilateral atributiva, ou, de uma forma analítica: Direito é a ordenação heterônoma, coercível e

    bilateral atributiva das relações de convivência, segundo uma integração normativa de fatos segundo valores. (REALE, Miguel. Lições preliminares de Direito. — 27. ed. — São Paulo : Saraiva, 2002).

  • Tá errado, é a alternativa C. Tese, antítese e síntese.

  • Que questão absurda vamos pular essa matéria aff

  • Será que fizeram um pacto com o Miguel Reale para continuar a cobrar essa matéria e só com questões dele?


ID
1767547
Banca
FCC
Órgão
MPE-RN
Ano
2012
Provas
Disciplina
Filosofia do Direito
Assuntos

No tocante às lacunas, a teoria que defende a inexistência de lacunas, aplicando-se a norma do tudo o que não está proibido está juridicamente permitido é a teoria do

Alternativas
Comentários
  • Vixe. Alguém poderia informar algum doutrinador que fale sobre o tema?? Marquei D como a maioria das pessoas. Qconcursos, uma explicação de professor viria a calhar aqui.
  • Modernamente a doutrina jurídica registra cinco teorias, no que se refere ao problema da existência das lacunas, catalogadas por Carlos Cóssio: realismo ingênuo, empirismo científico, ecletismo, pragmatismo e apriorismo filosófico.

    Realismo ingênuo: a evolução social cria espaços vazios não apenas na lei, mas no próprio sistema jurídico, de tal sorte que muitos não podem ser resolvidos com base em normas preexistentes.

    Empirismo científico: defende a inexistência de lacunas baseando-se na chamada norma de liberdade, pela qual tudo o que não está proibido está juridicamente permitido.

    Ecletismo: corrente majoritária. Enquanto a lei apresenta lacunas, a ordem jurídica não as possui. Isto porque o Direito se apresenta como um ordenamento que não se forma pelo simples agregado de leis, mas que as sistematiza, estabelecendo ainda critérios gerais para a sua aplicação.

    Pragmatismo: na prática, é seguida por muitos juízes e tribunais. Reconhece a existência de lacunas no ordenamento jurídico, contudo, que o Direito dispõe de fórmulas para regular todos os casos emergentes na vida social.

    Apriorismo filosófico: concepção defendida por Carlos Cóssio. Segundo ela, a ordem jurídica não apresenta lacunas. O Direito é uma estrutura totalizadora e, conseqüentemente, não há casos fora do todo.

     

    Fonte: http://informesdedireito.blogspot.com.br/2012/10/processos-de-integracao-do-direito.html

  • Nessa questão segui a intuição e acertei hahahaha..Mas que questão OSSO! Quem é o precursor dessas teorias?
  • A questão quer o conhecimento sobre as teorias que defendem a existência ou não de lacunas na lei.

    A) Apriorismo Filosófico.

    Apriorismo filosófico. Esta é a concepção defendida por Carlos Cossio, segundo a qual a ordem jurídica não apresenta lacunas. O seu pensamento está em concordância com o empirismo científico, mas dele se diferencia na fundamentação. Enquanto para o empirismo científico, na expressão de Cossio, o Direito é tomado como justaposição ou soma de regras jurídicas, o apriorismo filosófico o concebe “como una estructura totalizadora, de donde resulta que un régimen de Derecho positivo es una totalidad y, por consiguiente, que no hay casos fuera del todo porque, de lo contrario el todo no sería tal todo”. (Nader, Paulo. Introdução ao estudo do direito. 36.a ed. – Rio de Janeiro: Forense, 2014).

    Incorreta letra “A”.

    B) Ecletismo.

    “Ecletismo. Para os adeptos desta corrente, que é majoritária, enquanto a lei apresenta lacunas, a ordem jurídica não as possui. Isto porque o Direito se apresenta como um ordenamento que não se forma pelo simples agregado de leis, mas que as sistematiza, estabelecendo ainda critérios gerais para a sua aplicação.” (Nader, Paulo. Introdução ao estudo do direito. 36.a ed. – Rio de Janeiro: Forense, 2014).

    Incorreta letra “B”.

    C) Realismo Ingênuo.

    “Realismo ingênuo. A evolução social cria, de acordo com esta concepção, espaços vazios, brancos, não apenas na lei, mas no próprio sistema jurídico, de tal sorte que muitos casos não podem ser resolvidos com base em normas preexistentes.” (Nader, Paulo. Introdução ao estudo do direito. 36.a ed. – Rio de Janeiro: Forense, 2014).

    Incorreta letra “C”.

    D) Pragmatismo.

    “Pragmatismo. Esta corrente reconhece a existência de lacunas no ordenamento jurídico, mas entende ser necessário se convencionar, para efeitos práticos, que o Direito sempre dispõe de fórmulas para regular todos os casos emergentes na vida social.” (Nader, Paulo. Introdução ao estudo do direito. 36.a ed. – Rio de Janeiro: Forense, 2014).

    Incorreta letra “D”.


    E) Empirismo Científico.

    “Empirismo científico. Com base na norma de liberdade, pela qual tudo o que não está proibido está juridicamente permitido, Zitelmann e Donati, entre outros, defendem a inexistência de lacunas. Assim, não haveria vácuos no ordenamento.” (Nader, Paulo. Introdução ao estudo do direito. 36.a ed. – Rio de Janeiro: Forense, 2014).

    Correta letra “E”. Gabarito da questão.

    Resposta: E

    Gabarito do Professor letra E.

  • REALISMO INGÊNUO = admite espaços vazios, provocados pela evolução social, tanto na lei como no ordenamento jurídico.

     

    EMPIRISMO CIENTÍFICO = com base na lei da liberdade, não concebe lacunas no ordenamento jurídico, pois  tudo que não está proibido na lei, encontra-se juridicamente permitido no ordenamento jurídico.

     

    ECLETISMO = existem lacunas na lei, mas o ordenamento jurídico disponibiliza instrumento para colmatá-las. É a teoria adotada em nosso ordenamento jurídico - art. 4 da LINDB.

     

    PRAGMATISMO = as lacunas existem no ordenamento jurídico, contudo, o mesmo disponibiliza fórmulas para regular todos os casos emergentes na vida social.

     

    APRIORISMO FILOSÓFICO = concebida por Carlos Cóssio, o Direito é uma estrutura totalizadora, inexistindo casos fora do todo.

  • É muita vontade de inventar teoria para vender livro

  • FALA SÉRIO,

    FCC DEVE PAGAR UMA PESSOA PARA QUE ELA POSSA, EXCLUSIVAMENTE, INVENTAR TEORIAS

  • pelo amor de deus


ID
2001268
Banca
CESPE / CEBRASPE
Órgão
PM-CE
Ano
2012
Provas
Disciplina
Filosofia do Direito
Assuntos

A respeito das disciplinas jurídicas, da sociabilidade humana e dos instrumentos de controle social, julgue o item a seguir.
A norma moral é diferente da norma ética.

Alternativas
Comentários
  • “O direito, malgrado distinguir-se da moral, é grandemente influenciado por esta, de quem recebe valiosa substância. A norma moral é a norma ética, pois se funda num juízo de valor, expressando-se na experiência humana, no mundo exterior, através da atividade de assimilação de conceitos, que é atividade interna" (página 10) (Destaque do professor).

    “Direito e Moral estão numa constante relação de complementação, haja vista vez que a legitimidade de um ordenamento jurídico é aferida quando não conflitar com os princípios morais" (HABERMAS, 2003).

    A assertiva está certa.

    Fontes:

    Apostila: AESP/CE – Governo do Estado do Ceará - Curso de Formação Profissional para a Carreira de Praças Policiais Militares

    HABERMAS, Jürgen. Direito e Democracia entre facticidade e validade.Volumes I e II. 2ª ed. Trad. Flávio Beno Siebeneichler. Rio de Janeiro: Tempo Brasileiro, 2003


  • Questão anulada pela banca. Norma moral pode abranger a norma ética, mas não é o mesmo que ela.


ID
2395051
Banca
FGV
Órgão
OAB
Ano
2017
Provas
Disciplina
Filosofia do Direito
Assuntos

A principal tese sustentada pelo paradigma do positivismo jurídico é a validade da norma jurídica, independentemente de um juízo moral que se possa fazer sobre o seu conteúdo. No entanto, um dos mais influentes filósofos do direito juspositivista, Herbert Hart, no seu pós-escrito ao livro O Conceito de Direito, sustenta a possibilidade de um positivismo brando, eventualmente chamado de positivismo inclusivo ou soft positivism.
Assinale a opção que apresenta, segundo o autor na obra em referência, o conceito de positivismo brando.

Alternativas
Comentários
  • O positivismo inclusivo, segundo Herbert Hart, constitui a corrente positivista que acolhe a ideai de separação entre o direito e a moral. Entretanto, entende que essa distinção é flexível por existir sistemas jurídicos que apresentariam a incorporação de critérios morais.

    Logo a alternativa D é a correta e gabarito da questão.

    FONTE: Estratégia OAB 

    http://www.estrategiaoab.com.br/xxii-exame-de-ordem-comentarios-filosofia-do-direito/

  • O positivismo “moderado” pode ser entendido, conforme o próprio Hart, como aquele que aceita a possibilidade de que a norma de reconhecimento de um ordenamento jurídico incorpore, como critério de validade jurídica, a obediência a princípios morais ou valores substantivos. Esse apontamento foi feito em seu pós-escrito em resposta a Ronald Dworkin.   De acordo com Hart (p. 312), “Em primeiro lugar, igonora (Dworkin) o meu reconhecimento explícito de que a regra de reconhecimento pode incorporar, como critérios de validade jurídica, a conformidade com princípios morais ou com valores substantivos; por isso a minha doutrina é aquilo que tem sido designado como ‘positivismo moderado’ e não, como na versão de Dworkin acerca da mesma, positivismo meramente factual”.

    Gabarito do professor: letra d.

    Fonte: HART, H.L.A. O conceito de direito. 3. ed. Lisboa: Fundação Calouste Gulbenkian, 1994. 348 p.


  • Complementando: "Tal teoria afirma que a regra de reconhecimento socialmente aceita e praticada (portanto, conforme a tese positivista das fontes sociais do Direito), pode incorporar padrões morais como critérios de validade jurídica. Contudo, isso seria um acontecimento circunstancial, não havendo conexão necessária entre o Direito e a Moral. Assim, tal formulação teórica mantém a possibilidade de separação conceitual entre Direito e Moral."

     

    Fonte: Rodrigo de Souza Tavares (NEOCONSTITUCIONALISMO E POSITIVISMO INCLUSIVO: duas visões sobre a incorporação de critérios morais no direito. Disponível em )

  • Aprofundamento: [..] Herbert Hart compreendia que o fundamento do Direito repousava na aceitação, por uma comunidade, de uma regra de reconhecimento. Esta regra, por sua vez, indicava quem legitimamente poderia legislar. Percebe-se a importância do elemento convencional na proposta de Hart. A pergunta nuclear é: a lei foi elaborada por pessoas reconhecidas como competente por uma Constituição que, por sua vez, foi aceita pela comunidade? Fonte: Samuel Sales Fonteles, Hermenêutica Constitucional, pg.81.


ID
2402236
Banca
FCC
Órgão
DPE-PR
Ano
2017
Provas
Disciplina
Filosofia do Direito
Assuntos

Ao julgar a concessão de ordem liminar na Ação Popular n° 0013857-51.2017.4.02.5101, cujo pedido era a sustação de ato de nomeação para cargo em comissão pelo Chefe do Executivo, a autoridade judiciária, na motivação de sua decisão, expôs um pedido de desculpa por decidir contrariamente à autoridade administrativa com fundamento no que esta autoridade escreveu enquanto doutrinador. Em referido caso judicial, como em outros de natureza similar, houve uma afetação da espera política por uma decisão judicial, a qual teve que ser combatida por outra decisão judicial para reestabelecimento da decisão política.

Segundo a lição de Niklas Luhmann, o sistema "processo judicial" é marcado pela diferenciação com o ambiente, consolidando limites com este e, assim, existindo com autonomia. Dentre os mecanismos que permitem a operacionalização deste sistema está a adoção de papéis por seus atores. Desse modo, é correto afirmar:

Alternativas
Comentários
  • resposta correta letra E

  • Niklas Luhmann é bastante conhecido pela sua famosa "Teoria dos Sistemas".

    Nela ele basicamente demonstra que a sociedade é um sistema intercalado e autopoético. O Direito seria um subsistema desse "Sistema geral", bem como a Religião, Economia etc..

    O sucesso do Direito dependeria da coerência de suas normas. Quanto mais coerentes, mais provável que o subsistema jurídico tenha sucesso.

    Luhmann sofre fortes críticas, principalmente por ignorar o papel individual de uma pessoa, acreditando que uma pessoa não teria a capacidade de modificar o sistema. O autor alemão ignora que existem pessoas que efetivamente influênciam no sistema, como o carismático político, por exemplo.

    No que tange a "Legitimação pelo procedimento", Luhmann sustentam que as decisões judiciais somente teriam legitimidade na medida em que respeitassem os procedimentos legais (devido processo legal) previamente estipulado pelo Legislador. Na realidade, segue a mesma linha da sua Teoria do sistema, haja vista que se o Direito é coerente e organizado, o juiz deve respeitar esse subsistema social.

     

    Pelo que eu li sobre o Luhmann foi isso que eu entendi. Se me equivoquei em algo, por favor me digam, pois estamos nessa caminhada juntos.

     

    Abraço a todos, e boa sorte!

  • A autonomia do sistema jurídico teorizada por Luhamann é bem explicada por Néviton Guedes. Segundo GUEDES (2013) O Direito, como qualquer sistema social, só pode existir a partir da diferenciação funcional em relação ao seu meio ambiente, isto é, em relação aos demais subsistemas sociais — mídia, economia, política, religião, moral etc. Melhor explicando, “como todos os sistemas, os procedimentos judiciais constituem-se pela diferenciação, pela consolidação de limites diante de um meio ambiente. (...) Diferenciação não significa isolamento causal ou comunicativo. Tribunais não são prisões. Antes, trata-se apenas de construir uma esfera de sentido para si, de tal forma que processos seletivos de tratamento de informações colhidas do meio ambiente possam ser orientados por regras e decisões próprias do sistema. Portanto, que estruturas e eventos do meio ambiente não tenham validade imediata no sistema, mas apenas depois de serem reconhecidos pela filtragem (Filterung) de informações. A diferenciação só pode com isso ser executada pelo estabelecimento autônomo do processo.”

    Além disso, os procedimentos judiciais não podem ser deixados ao acaso: “não podem ser descobertos e compatibilizados caso a caso” (Luhmann). O Direito deve diferenciar-se dos demais subsistemas para estabilizar. Seria terrível que se promovesse a sua indiferenciação com intuito, ainda que inocente, de instabilizar. No dizer ainda de Luhmann, os procedimentos judiciais e as decisões deles resultantes não podem depender dos humores e dos interesses sociais momentâneos e aleatórios. Um cidadão não pode ser considerado culpado ou inocente, por exemplo, pelas relações de sub ou de sobreintegração que mantenha com o poder, com a economia ou com os meios de comunicação. No mais possível, e isso é básico a uma democracia, um julgamento — sobretudo de caráter subjetivo, em que estão envolvidas vidas de pessoas — não pode depender dos azares do momento, do humor da opinião pública, de suas relações de poder ou de sua condição econômica.

    Portanto, considerando a teorização de Luhmann e as lições de Guedes, é correto afirmar que há redução de legitimidade do procedimento judicial quando há confusão do papel de “juiz” com o papel de “estudante”, pois ocorre indesejada pessoalização do primeiro.

    Gabarito do professor: letra e.

    Referências:

    GUEDES, Néviton. A importância da autonomia e diferenciação do Direito. 2013. Disponível em: <http://www.conjur.com.br/2013-jan-07/constituicao-poder-importancia-autonomia-diferenciacao-direito>. Acesso em: 29 jul. 2017.

    Niklas Luhmann. Legitimation durch Verfahren. Frankfurt am Main: Suhrkamp, 1983, p. 59.


  • pessoa que fez esta questão, te admiro!

  • Interessante que a questão trata de uma AP tirada do mundo real: https://www.conjur.com.br/dl/posse-moreira-franco-ministro-estado.pdf.

  • Trecho da Ação Popular descrita:

    "Peço, humildemente perdão ao Presidente Temer pela insurgência, mas por pura lealdade as suas lições de Direito Constitucional. Perdoe-me por ser fiel aos seus ensinamentos ainda gravados na minha memória, mas também nos livros que editou e nos quais estudei. Não só aprendi com elas, mas, também acreditei nelas e essa é a verdadeira forma de aprendizado. Por outro lado, também não se afigura coerente, que suas promessas ao assumir o mais alto posto da Republica sejam traídas, exatamente por quem as lançou no rol de esperança dos brasileiros, que hoje encontram-se indignados e perplexos ao ver o seu Presidente, adotar a mesma postura da ex-Presidente impedida e que pretendia também, blindar o ex-presidente Luiz Ignácio Lula da Silva. Ao mestre com carinho."

  • Como respondi:

    No comando da questão, há o trecho "Dentre os mecanismos que permitem a operacionalização deste sistema está a adoção de papéis por seus atores."

    A partir disso, busquei a alternativa que tivesse algo próximo do trazido no enunciado.

    A letra "E", portanto, é a que mais se encaixa na ideia de "adoção de papéis", ela, inclusive, usa expressamente o termo "papel", vejam:

    "Há redução de legitimidade do procedimento judicial quando há confusão do papel de “juiz” com o papel de “estudante”, pois ocorre indesejada pessoalização do primeiro".

  • Aqui mostra primeiro a realização das ilusões do mercado de troca da sociedade (colocado como necessidade da realidade), a realização dos próprios interesses, mesmo que em detrimento do interesse dos outros. Após um ressentimento de ressaca, de ter extrapolado a moral, o que gera uma necessidade de proteção da consequência que surge socialmente, que é a absoluta frieza social, a falta de esperança e a tendência da extinção da raça humana, devido à fuga pelo individualismo e escassez dos recursos naturais.

    Falta do olhar e cumprimento da lógica dos sistemas, do todo (como teórico, estudante).

    Ao julgar a concessão de ordem liminar na Ação Popular n° 0013857-51.2017.4.02.5101, cujo pedido era a sustação de ato de nomeação para cargo em comissão pelo Chefe do Executivo, a autoridade judiciária, na motivação de sua decisão, expôs um pedido de desculpa por decidir contrariamente à autoridade administrativa (contradição do que é e o ser) com fundamento no que esta autoridade escreveu enquanto doutrinador. Em referido caso judicial, como em outros de natureza similar, houve uma afetação da espera política por uma decisão judicial (desconsidera as afetações macroestruturais, estudadas pelos teóricos do todo, do sistema, daí o mundo não muda, continua essa catástrofe, contradição entre teoria e prática), a qual teve que ser combatida por outra decisão judicial para reestabelecimento da decisão política.

  • A Ainda que se exija do juiz impessoalidade, há um acréscimo de legitimidade no procedimento (JURGEN HABERMAS) se o motivo da sentença for pessoal, desde que fundado no princípio democrático. Errado. Essa é uma tentativa de legitimar as promessas do iluminismo burguês, que ao final legitima o absurdo, devido a prática do "jeitinho".

    B Não há perda de legitimidade um procedimento judicial conduzido por um juiz que, numa conferência pública, afirmar que, por ser magistrado de carreira, pode fundamentar uma decisão em tudo o que quisesse. Errado. Nesse caso há perda de legitimidade.

    C O dever de impessoalidade estende-se a todos os atores do processo, inclusive às testemunhas e às partes, sob pena de perda de legitimidade da decisão. Errado. Não se estende.

    D Com a complexificação das sociedades democráticas, os procedimentos de escolha política terão maior legitimidade se ocorrem via sistema judicial. Errado. Não necessariamente pela via judicial, no sentido das teorias sociais afirmativas é necessário uma visão ampla, do sistema.

    E Há redução de legitimidade do procedimento judicial quando há confusão do papel dejuiz” com o papel de “estudante, pois ocorre indesejada pessoalização do primeiro. Correto. Contradição daquilo que é postulado (enquanto estudante) e a realidade que prevalece (papel de juiz), enquanto não houver real responsabilização do que é o causador da crise, viveremos no faz de conta, daquilo que é distante daquilo que deve ser.


ID
2422444
Banca
IFB
Órgão
IFB
Ano
2017
Provas
Disciplina
Filosofia do Direito
Assuntos

Kant explica na “Fundamentação da Metafísica dos Costumes” que o imperativo pode ser hipotético ou categórico, porém a distinção depende de a relação da lei com a vontade ser dirigida ou não para realizar um fim.
Leia as afirmações sobre os imperativos.
I) Nos imperativos hipotéticos, a relação entre lei e vontade é dirigida para alcançar um fim que se queira sendo real ou possível.
II) Os imperativos hipotéticos podem ser descritos como conselho de prudência, porém jamais como regras de habilidade.
III) O imperativo categórico declara ser a ação necessária por si mesma, em virtude de um fim irrealizável.
IV) No imperativo categórico a necessidade que relaciona a lei objetiva com a vontade vale como princípio apodítico prático.
Assinale a alternativa que apresenta somente as afirmações CORRETAS de acordo com a teoria moral de Kant.

Alternativas
Comentários
  • A questão em comento exige conhecimentos basilares sobre Kant e as ideias de imperativo hipotético e categórico.

    O imperativo categórico fixa maneiras para certos fins específicos sejam alcançados.

    O imperativo categórico, algo bom em si, diz que, independente do fim que se busca alcançar, há certas maneiras de agir.

    O imperativo categórico é absoluto. Ele ordena um dever pelo simples fato de ser um dever, de maneira que nosso agir, ao cumprir um imperativo categórico, é sempre impessoal e desinteressado.

    O imperativo hipotético é uma ordem condicionada a algo proveitoso ou satisfatório, e não um dever em si. As ações tomadas com base em imperativo hipotético levam em conta as consequências ou resultados no fazer aquilo que é devido. As máximas do imperativo hipotético não podem ser universalizadas.

    Assim sendo, os imperativos hipotéticos tendem a ser condicionados e particulares, variando conforme pessoas e circunstâncias, ao passo que os imperativos categóricos são obrigatórios, universais e incondicionados.

    Diante destas ponderações, cabe analisar cada uma das assertivas.

    A assertiva I está CORRETA.

    De fato, nos imperativos hipotéticos as ações são dirigidas para a busca de um fim, diferente dos imperativos categóricos, nos quais a ação se dá com base em deveres de agir, e não em fins específicos. Estes fins são possíveis ou impossíveis, mas dirigem o agir nos imperativos categóricos.

    A assertiva II está INCORRETA.

    Como se tratam de imperativos contingentes e com fins específicos, os imperativos hipotéticos levam em conta determinadas habilidades.

    A assertiva III está INCORRETA.

    Ao contrário do exposto, o fim da ação não é algo realmente indagado e relevante quando falamos em imperativo categórico.

    A assertiva IV está CORRETA.

    Como o imperativo hipotético age com finalidade específica, as ações buscam adequar a vontade à determinadas leis.

    Diante do exposto, estão corretas as assertivas I e IV.

    Cabe comentar as alternativas da questão.

    LETRA A- INCORRETA. As assertivas I e IV estão corretas.

    LETRA B- INCORRETA. As assertivas I e IV estão corretas.

    LETRA C- INCORRETA. As assertivas I e IV estão corretas.

    LETRA D- CORRETA. As assertivas I e IV estão corretas.

    LETRA E- INCORRETA. As assertivas I e IV estão corretas.

    GABARITO DO PROFESSOR: LETRA D


ID
2422447
Banca
IFB
Órgão
IFB
Ano
2017
Provas
Disciplina
Filosofia do Direito
Assuntos

Nos “Princípios da Filosofia do Direito”, Hegel apresenta o conceito de direito a partir das três fases da vontade, são elas: o direito abstrato, a moralidade (moralidade subjetiva) e a eticidade (moralidade objetiva).
Assinale a alternativa que indica como a vontade encontra-se nas três fases, respectivamente: direito abstrato, moralidade e eticidade.

Alternativas
Comentários
  • “O conteúdo objetivo da moralidade que se substitui ao bem abstrato é, através da subjetividade como forma infinita, a substância concreta. Em si mesma, portanto, estabelece ela diferenças que, assim, são pelo conceito ao mesmo tempo determinadas; por elas a realidade moral objetiva obtém um conteúdo fixo, necessário para si, e que está acima da opinião e da subjetiva boa vontade. É a firmeza que mantém as leis e instituições, que existe em si e para si.” (HEGEL 1997 p. 142-143)

  • Tô começando a achar que filosofia do direito não tem nada como nada. É uma merd@.

  • THE FUCK WAS THAT

    NEXT

  • Hegel, de fato, trata de Direito Abstrato, moralidade e eticidade.

    A questão exige como a vontade é encarada nestas três instâncias.

    No Direito Abstrato, segundo Hegel, temos uma instância de objetividade do Direito, efetivamente consubstanciada em um objeto externo, bem distante de reflexões correlacionadas com o imediato.

    Na moralidade, segundo Hegel, há uma instância subjetiva que se ocupa de fazer uma análise interna da vontade, ou seja, a vontade reflui de volta a si mesma, mas faz isto buscando a liberdade e o alcance da eticidade, isto é, se consubstancia em um objeto externo.

    Na eticidade, uma instância, objetiva, existe, segundo Hegel, a mediação social da liberdade, a real concretização do conceito do Direito. Trata-se de uma instância de reflexão sobre a moralidade, ou seja, trata-se de uma análise da vontade que reflui de volta a si mesma

    Devemos, pois, compreender que os passos da vontade em Hegel, são: estar consubstanciada em um objeto externo, de tal maneira que, em um instante subjetivo, seja capaz de refluir de volta a si mesma, e tendo um laço objetivo com a eticidade, uma instância objetiva, mas que também faz com que a vontade reflua de volta a si mesma.

    Feitas tais ponderações, vamos comentar as alternativas.

    LETRA A- INCORRETA. No trajeto da vontade em Hegel não há que se falar que a vontade seja elevada ao imediato.

    LETRA B- INCORRETA. No trajeto da vontade em Hegel não há que se falar que a vontade seja elevada ao imediato.

    LETRA C- INCORRETA. Não podemos, em Hegel, em momento algum tratar a vontade como um dever concreto.

    LETRA D- CORRETA. Representa o iter da vontade em Hegel. Devemos, pois, compreender que os passos da vontade em Hegel, são: estar consubstanciada em um objeto externo, de tal maneira que, em um instante subjetivo, seja capaz de refluir de volta a si mesma, e tendo um laço objetivo com a eticidade, uma instância objetiva, mas que também faz com que a vontade reflua de volta a si mesma.

    LETRA E-INCORRETA. No trajeto da vontade em Hegel não há que se falar que a vontade seja elevada ao imediato.



    GABARITO DO PROFESSOR: LETRA D


ID
2422450
Banca
IFB
Órgão
IFB
Ano
2017
Provas
Disciplina
Filosofia do Direito
Assuntos

Em sua obra “Princípios da Filosofia do Direito”, Hegel explica que não há eticidade, moralidade objetiva, no plano da vontade meramente natural e imediata, mas que se requer a sua mediação. As instituições são os momentos dos desdobramentos da eticidade.
Assinale a alternativa que apresenta em sequência os três desdobramentos da eticidade, de acordo com Hegel.

Alternativas
Comentários
  • http://www.ambito-juridico.com.br/site/index.php?n_link=revista_artigos_leitura&artigo_id=2602

     

    Moralidade Objetiva

    Constitui o momento em que a liberdade torna-se realidade. É a moralidade palpável, acima da opinião e da boa vontade, fortalecendo as leis e as instituições. É a totalidade de determinações morais da família, da sociedade civil e do Estado, incidindo (aparecendo) entre seus membros – existência, manifestação e realidade. Assim dizendo, fica expressa no caráter do indivíduo – probidade, honradez, integridade e honestidade.

    É tomado como substância moral, não sou eu como pessoa (tal qual o direito abstrato) tampouco o direito da consciência, mas sim o direito enquanto Espírito real de um povo que percorre, a fim de realizar a liberdade, diferentes momentos:

    - Família – como espírito moral objetivo, imediato e natural;

    - Sociedade Civil – associação com o fim de satisfazer carências, necessidades e dar garantia à propriedade privada;

    - Estado – consagração universal da vida pública.

  • A questão em comento exige conhecimento basilar de Hegel.

    Com efeito, a eticidade se desdobra em 03 planos, quais sejam:

    ·         Família- síntese;

    ·         Sociedade civil- antítese;

    ·         Estado- tese, consagração do absoluto, da vida pública.

    Estes elementos precisam estar em ordem, isto é, família, sociedade civil e Estado.

    Diante do exposto, cabe comentar as alternativas da questão.

    LETRA A- INCORRETA. Não traduz a sequência correta da questão.

    LETRA B- CORRETA. De fato, Hegel pensa na tríade família, sociedade civil e Estado.

    LETRA C- INCORRETA. Não traduz a sequência correta da questão.

    LETRA D- INCORRETA. Não traduz a sequência correta da questão.

    LETRA E- INCORRETA. Não traduz a sequência correta da questão.

    GABARITO DO PROFESSOR: LETRA B


ID
2449537
Banca
UFMT
Órgão
POLITEC-MT
Ano
2017
Provas
Disciplina
Filosofia do Direito
Assuntos

"O modelo contratualista [...] é essencialmente um método para dar uma resposta racional à pergunta que num certo sentido faz unidade com o pensamento político, a saber: como deve ser organizado um estado legítimo, ao qual todos os cidadãos sejam obrigados a dar o seu assentimento".

                   (Stefano Petruccini. Modelos de Filosofia Política. São Paulo: Paulus, 2014.)

A partir das informações do texto, analise as afirmativas.

I - O modelo contratualista apresenta uma contraposição entre um estado de natureza (pré-político) e um estado civil (político).

II - O estado de natureza, para Thomas Hobbes, é caracterizado como uma "guerra de todos contra todos".

III - Na concepção lockeana do contrato social, a propriedade privada não constitui um direito inviolável.

IV - Para Rousseau, no pacto social, o indivíduo abdica de sua liberdade em prol da segurança de sua vida.

Está correto o que se afirma em 

Alternativas
Comentários
  • Uma pergunta dessas para seleção de um profissional papiloscopista talvez reflita uma postura sensivelmente esguia de uma instituição que não pretende selecionar pessoas para o mero desempenho do cargo, mas pretende, também, colocar pessoas que sejam dotados de alto senso crítico. O estudo desses temas leva a crer que o candidato assuma uma postura crítica. Desconheço o motivo, porém arrisco deduzir que o Estado do Mato Grosso queira colocar em seus quadros pessoas de elevado nível intelectual, mesmo que essa exacerbada intelectualidade não venha a interferir potencialmente no desempenho do cargo.

    Mas de nada adiante lamentar. Se as regras do jogo são essas, então, não há como chegar lá sem enfrentá-las.

    O texto apresentado à reflexão é subscrito por Petruccini, porém se reporta à ideia de Estado concebida por Thomas Hobbes em seu livro O Leviatã. Em linhas gerais, Hobbes vivia em um período crítico do século XVII no qual seu país passava por uma profunda crise política travada entre a nobreza e o surgimento da elite burguesa. O poder político na época, que era arraigado na religião, governava por meio de normas reputadas por Hobbes como irracionais, pois aquela cultura religiosa adotava posturas excessivamente dogmáticas. A evolução das tecnologias e o crescente aumento de pessoas que se enriqueciam fora dos modelos anteriores entravam em conflito com o sistema secular adotado pela coroa religiosa.

    Visando propor um fim para os conflitos, Hobbes sugere que a sociedade em geral necessitava de um contrato social, no qual as pessoas renunciavam parte de suas liberdades individuais para entregá-las a um abstrato ser soberano (uma pessoa ou uma assembleia) e, a partir daí, todos os conflitos seriam dirimidos por ele, evitando o uso da força direta uns contra os outros.

    Logo, para responder esta questão, bastava o conhecimento das teorias de Hobbes para acertar. Aos nobres estudantes que pretendem conhecê-lo, mas não dispõem de tempo para ler as suas obras, recomendo uma dar uma visualizada no vídeo extremamente didático do Prof. André Pinho, no Youtube: https://www.youtube.com/watch?v=-lhNC3Pvfn4

    Boa sorte a todos nós!

     

  • alternativa: D

    I e II.

  • A questão em comento demanda análise de cada uma das assertivas à luz da discussão inerente ao Contrato Social.

    A assertiva I está correta.

    Com efeito, em regra, o Estado de Natureza é a alegoria de um Estado pré político e o Estado Civil, a civilização, é o Estado organizado, político.

    A assertiva II também está correta.

    De fato, para Thomas Hobbes, o Estado de Natureza é a guerra de todos contra todos, a anomia, a total ausência de segurança e paz.

    A assertiva III está incorreta.

    Ao contrário do exposto, segundo Locke, a propriedade é central na construção de Contrato Social.

    A assertiva IV está incorreta.

    Em instante algum Rousseau constrói a ideia de Contrato Social fazendo o homem abdicar da liberdade.

    Logo, são corretas as assertivas I e II.

    LETRA A- INCORRETA. Não constitui a sequência correta.

    LETRA B- INCORRETA. Não constitui a sequência correta.

    LETRA C- INCORRETA. Não constitui a sequência correta.

    LETRA D- CORRETA. Com efeito, são corretas as assertivas I e II.

    GABARITO DO PROFESSOR: LETRA D


ID
2449540
Banca
UFMT
Órgão
POLITEC-MT
Ano
2017
Provas
Disciplina
Filosofia do Direito
Assuntos

De acordo com a filósofa Hannah Arendt, o totalitarismo é uma forma de governo essencialmente diferente de outras formas de opressão política conhecidas, como o despotismo, a tirania e a ditadura. Considerando as características e as expressões históricas do totalitarismo no século XX, assinale a afirmativa INCORRETA.

Alternativas
Comentários
  • Gabarito: A. Em regimes totalitário o poder é concentrado no Estado (executivo cria leis e julga ou interfere de forma expressiva nesses poderes), ou seja, não há uma distinção entre privado e público.

  • De fato, como poucos, Hannah Arendt visualizou e escreveu sobre o totalitarismo. Ao escrever sobre este fenômeno, infelizmente forte no século XX, ela observou o seguinte:

    ·         Manifestação forte de totalitarismo nos regimes nazistas, stalinista e facistas;

    ·         Alta difusão do totalitarismo através de propaganda oficial estatal;

    ·         Manejo do terror, da tortura, do medo, a morte do inimigo, a eliminação do adversário político ou do “estranho" aos ideais totalitaristas.

    Diante do exposto, cabe responder a questão em tela, apontando a alternativa incorreta.

    LETRA A- INCORRETA, LOGO RESPONDE A QUESTÃO. O totalitarismo não preserva uma esfera de divisão saudável entre público e privado. No totalitarismo, a liberdade, a iniciativa privada são extirpadas.

    LETRA B- CORRETA, LOGO NÃO RESPONDE A QUESTÃO. Com efeito, nazismo e stalinismo são exemplos de manifestação do totalitarismo.

    LETRA C- CORRETA, LOGO NÃO RESPONDE A QUESTÃO. Com efeito, a propaganda estatal é uma tática de disseminação do totalitarismo.

    LETRA D- CORRETA, LOGO NÃO RESPONDE A QUESTÃO. De fato, o terror é parte integrante fundamental do totalitarismo.

    GABARITO DO PROFESSOR: LETRA A

  • Questão pede o Item INCORRETO!

    GAB: A

    Totalitarismo é um sistema político ou uma forma de governo que proíbe partidos de oposição, que restringe a oposição individual ao Estado e às suas alegações e que exerce um elevado grau de controle na vida pública e privada dos cidadãos. É considerado a forma mais extrema e completa de autoritarismo.

  • LETRA A - INCORRETA

    b, c e D são as características do totalitarismo

  • O argumento de Arendt não é só dela mas incluiu diversos intelectuais europeus de esquerda como George Orwell, que fugiu da Espanha depois de saber que tinha uma ordem de execução contra ele por parte dos comunistas, e Gertrude Stein, uma das principais organizadoras da intelectualidade de esquerda na França. O argumento desses autores, muito mais destacados que Arendt na época, é que o stalinismo incluía processos manipulados de destruição de seus inimigos, um argumento que pode ser expandido a partir do que sabemos sobre a atuação do stalinismo durante a guerra, quando até mesmo durante o sítio de Leningrado eliminava opositores que ajudavam na defesa da cidade. Na época, a realidade dos Gulags era do conhecimento de poucos e iria mais tarde radicalizar esse argumento.

    Tudo isso leva a uma questão que Hannah Arendt já havia observado em relação ao nazismo e que pode ser denominado do argumento da disponibilidade ou do anti-utilitarismo dos campos de extermínio. Arendt fez a seguinte afirmação: “… não é apenas o caráter não utilitário dos campos – o punitivismo sem sentido de pessoas completamente inocentes, a incapacidade de mantê-los em condições para que pudessem gerar alguma capacidade de apropriação do trabalho, a superfluosidade de uma população completamente dominada – que lhes dá qualidades distintas e absolutamente perturbadoras. Sua função anti-utilitária assenta-se no fato de que eles não podiam nem ao menos contribuir na emergência militar ou interferir no enorme desequilíbrio demográfico.”

    Muitos anos depois ficou claro que a estrutura do Gulag era homóloga à estrutura dos campos nazistas.

    Stalinismo e nazismo se aproximam na forma de eliminação de pessoas inocentes, na maneira como se manipula a verdade para torná-las culpadas de crimes que elas não cometerem ou nem ao menos sabiam que eram crimes. Mas existe uma diferença fundamental entre stalinismo e nazismo. O nazismo é um movimento centrado na ideia da transformação do demos em ethnos e no emprego da violência com esse objetivo. O projeto nazista era intransigente na questão étnico-política, o que explica que a Alemanha tenha sido capaz de firmar um pacto com a União Soviética ou mesmo incorporar antigos comunistas no partido nazista, mas seguido na tentativa de exterminar os judeus até o último dia da guerra. Já o stalinismo é um projeto semimarxista associado às formas despotismo asiático, no qual os indivíduos sempre foram elimináveis, mas por motivos estritamente políticos. O argumento de classe, nesse caso, é relativizado, mas ainda assim integrado em uma profilaxia que tinha o Gulag como local privilegiado do extermínio dos inimigos.

    A autora entende o totalitarismo como formas de repressão da pluralidade humana e da manipulação de massas. Essas características do stalinismo e do nazismo permitiram amplas formas de repressão e de eliminação de ativistas políticos ou de simples cidadãos.

    https://aterraeredonda.com.br/hannah-arendt-totalitarismo-e-stalinismo/


ID
2488408
Banca
FGV
Órgão
OAB
Ano
2017
Provas
Disciplina
Filosofia do Direito
Assuntos

...só a vontade geral pode dirigir as forças do Estado de acordo com a finalidade de suas instituições, que é o bem comum...

Jean-Jacques Rousseau


A ideia de vontade geral, apresentada por Rousseau em seu livro Do Contrato Social, foi fundamental para o amadurecimento do conceito moderno de lei e de democracia.

Assinale a opção que melhor expressa essa ideia conforme concebida por Rousseau no livro citado.

Alternativas
Comentários
  • Segundo Rousseau, a vontade geral é atribuída à sociedade civil como pessoa, sendo distinta da simples soma das vontades individuais e privadas dos seus membros, algumas dessas, inclusive, podem ser contrárias àquela. Essa autonomia da vontade geral, bem como a sua unidade, ficam evidenciadas nesse trecho Do Contrato Social: "Se quando o povo, bem informado, toma deliberações, e os cidadãos não comunicam entre si, a soma das pequenas diferenças daria sempre a vontade geral e a decisão seria boa”. Logo, a ideia de vontade geral em Rousseau se expressa como "o interesse em comum ou o substrato em comum das diferenças", qual seja: "o bem comum".

  • A vontade geral de Rousseau não é a vontade de todos, e sim uma decisão coletiva, ou seja, O INTERESSE COMUM, da qual até mesmo os governantes se submetem.

     

     

  • A vontade geral é o somatório do interesse comum ou do substrato das diferenças.

     

     

  • A questão exige conhecimento relacionado à teoria contratualista de Rousseau. A ideia de vontade geral, apresentada por Rousseau em seu livro Do Contrato Social, relaciona-se ao interesse em comum ou ao substrato em comum das diferenças.  

    Segundo Rousseau (Livro II, I- A soberania é inalienável), “A primeira e mais importante consequência dos princípios acima estabelecidos está em que somente a vontade geral tem possibilidade de dirigir as forças do Estado, segundo o fim de sua instituição, isto é, o bem comum; pois, se a oposição dos interesses particulares tomou necessário o estabelecimento das sociedades, foi a conciliação desses mesmos interesses que a tornou possível. Eis o que há de comum nesses diferentes interesses fornecedores do laço social; e, se não houvesse algum ponto em torno do qual todos os interesses se harmonizam, sociedade nenhuma poderia existir. Ora, é unicamente à base desse interesse comum que a sociedade deve ser governada".

    Gabarito do professor: letra d.

     

    Fonte: Rousseau, Jean-Jacques. Do Contrato Social. Tradução de Ricardo Rodrigues da Gama. 1ª ed. São Paulo: Russel, 2006.


  • Povo o gabarito  oficial é a letra D

  • Desculpa, mas filosofia não desce! Meu Deus! 

  • Marquei B e errei. Mas refletindo posteriormente consigo encontrar  sentido, porque a vontade de todos não significa que é a mesma para cada um. Todos tem vontades, o que não significa que sejam equânimes. Agora interesse em comum sim, significa que é o mesmo ponto desejado coletivamente.

  • Não diz coisa com coisa esse Contrato Social!!!.

  • Ainda bem que só são duas questões de filosofia viu rsrs.

  • Questão ridícula, óbvio que é a B. A vontade geral é diferente da vontade de todos? Apegar-se à diferença entre o significado de geral e todos seria um raciocínio simplista.

  • RESPOSTA CORRETA: LETRA D

  • Segundo Rousseau existe uma diferença entre a vontade de todos e a vontade geral, pois a vontade geral não é a mera soma da vontade de todos, mas pretende ser a vontade do interesse comum (Do Contrato Social). Em Rousseau existem vários níveis de vontade:

    a vontade geral, que se trata da vontade do corpo formado por toda a comunidade política (por todos os cidadãos);

    a vontade particular de um indivíduo ou de um grupo formado apenas por uma pequena parcela dos indivíduos da sociedade;

    e a vontade de todos, que é a soma de todas as vontades particulares e que não deve ser confundida com a vontade geral.

    Fonte: https://www.sabedoriapolitica.com.br/news/vontade-geral-essencia-da-soberania-popular-e-da-democracia/

  • Vontade geral = interesses que cada pessoa tem em comum com todas as demais

    É aquela que traduz o que há de comum em todas as vontades individuais, o “substrato coletivo das consciências”.

    Fonte: A noção de vontade geral e seu papel no pensamento político de Jean-Jacques Rousseau - Marcio Morena Pinto

  • Viajei nessa.

  • PESSOAL A CORRETA É A LETRA (D) O interesse em comum ou o substrato em comum das diferenças.

  • A vontade de todos leva em consideração a VONTADE DE CADA UM (algo particularizado), mesmo em conjunto.

    Já a vontade geral leva em consideração AQUILO QUE INTERESSA A TODOS CONJUNTAMENTE. E mesmo havendo divergência, procurar-se-à retirar algo que seja COMUM nessas divergências (substrato comum das diferenças).

    GABARITO: LETRA D.

  • essa questão tem duas repostas corretas, nao foi anulada pq ninguem entrou com recurso

  • ...só a vontade geral pode dirigir as forças do Estado de acordo com a finalidade de suas instituições, que é o bem comum...

    O interesse em comum ou o substrato em comum das diferenças.


ID
2488411
Banca
FGV
Órgão
OAB
Ano
2017
Provas
Disciplina
Filosofia do Direito
Assuntos

A igualdade de recursos é uma questão de igualdade de quaisquer recursos que os indivíduos possuam privadamente.

Ronald Dworkin


A igualdade é um dos valores supremos presentes na Constituição da República e, também, objeto de um debate profundo no âmbito da Filosofia do Direito.

Assinale a alternativa que apresenta a concepção de igualdade distributiva, defendida por Ronald Dworkin em seu livro A Virtude Soberana.

Alternativas
Comentários
  • Gabarito letra A

  • GABARITO LETRA A. 

    "Dworkin (2005, p.79) trata da liberdade no seguinte trecho: “Admitirei, para esse fim, que a igualdade de recursos é uma questão de igualdade de quaisquer recursos que os indivíduos possuam privativamente”.

    Como dito anteriormente, Dworkin (2005) critica a visão utilitarista de justiça - que calcula a igualdade pelo bem-estar das pessoas -, pois segundo ele, o bem-estar não pode ser utilizado como único critério para uma análise social bem sucedida.

    A teoria de Dworkin (2005) sobre a igualdade e distribuição de recursos tem dois pilares fundamentais: A igual importância e a responsabilidade especial.

    A igual importância tem em vista um ponto de vista objetivo, o de que a vida humana seja bem sucedida ao invés de desperdiçada. Esse princípio não quer afirmar que os seres humanos são iguais, ou que as vidas que geram são igualmente valiosas, o que o princípio denota é que a vida de cada pessoa deve ter alguma importância. Esse princípio não requer que as pessoas ajam igualmente em relação a todas as pessoas- não requer, por exemplo, que as pessoas tenham o mesmo interesse pelos objetivos de seus filhos na esfera privada e pelos objetivos dos filhos de outras pessoas- mas que diante de certas categorias e em certas circunstâncias disponham da mesma consideração para com todos. Assim, os políticos devem tratar os cidadãos objetivamente com igual consideração, que é a virtude soberana e indispensável à vida política.

    O outro princípio do individualismo ético utilizado por Dworkin (2005) é o da responsabilidade especial, este relaciona a escolha à responsabilidade. Admite que justiça na distribuição dos bens em uma sociedade deve refletir a escolha de cada um. Assim, contanto que as pessoas tenham tido liberdade de fazer o que quiseram de suas vidas devem assumir a responsabilidade pelo que delas fizeram. Dworkin (2005) considera injusto retirar recurso de quem escolheu trabalhar e repassá-los a outra pessoa que preferiu uma vida sem maiores esforços."

    PARA OS QUE QUEREM APROFUNDAR A LEITURA, SEGUE O LINK PARA ACESSO:

    http://www.publicadireito.com.br/artigos/?cod=0631ba089fcd29d7

    ( IGUALDADE: UM DEBATE ENTRE DWORKIN E AMARTYA SEN EQUALITY : A DEBATE BETWEEN DWORKIN AND AMARTYA SEN), feito por Marcella Regina Gruppi Rodrigues.

     

     

     

  • A questão aborda posicionamento defendido pelo justeórico Ronald Dworkin, em especial o que diz respeito a sua concepção de igualdade distributiva, defendida na obra “A Virtude Soberana". Segundo Dworkin (2013, p.85-86), “Na igualdade de bem-estar, as pessoas devem decidir que tipo de vida querem, independentemente das informações pertinentes para decidir o quanto suas escolhas reduzirão ou aumentarão a capacidade de outros terem o que querem. Esse tipo de informação só se torna importante em um segundo nível, político, no qual os administradores coletam todas as escolhas feitas no primeiro nível para ver qual distribuição dará a cada uma dessas escolhas êxito igual em alguma concepção de bem-estar interpretada como a dimensão correta do êxito. Na igualdade de recursos, porém, as pessoas decidem que tipo de vida procurar munidas de um conjunto de informações sobre o custo real que as suas escolhas impõem a outras pessoas e, consequentemente, ao estoque total de recursos que pode ser equitativamente utilizado por elas. As informações que sob a igualdade de bem-estar passam a um nível político independente são, sob a igualdade de recursos, levadas ao nível inicial da escolha individual".

    Portanto, a alternativa que apresenta a concepção de igualdade distributiva, defendida por Ronald Dworkin em seu livro A Virtude Soberana é: Circunstâncias segundo as quais as pessoas não são iguais em bem-estar, mas nos recursos de que dispõem.

    Gabarito do professor: letra a.

    Fonte: Ronald Dworkin. A virtude soberana: a teoria e a prática da igualdade. São Paulo: Martins Fontes, 2013. 689 p. 2ª triagem ; tradução de Jussara Simoes.


  • Carmeligia.. colega... sacanagem colocarem um frase feita do livro pra gente saber!

  • Assim, apesar de ser um expoente de uma corrente de pensamento denominada igualitarismo liberal, difere-se do igualitarismo puro ou radical pois aceita a premissa de que a distribuição das riquezas sociais deve expressar de alguma forma as escolhas das pessoas e que, portanto uma distribuição idêntica de riquezas – como defendido pelo liberalismo puro –, não é necessariamente uma distribuição justa e igualitária. Por outro lado, defende que as desigualdades puramente materiais, aquelas que não têm relação com as escolhas dos indivíduos, devem ter uma distribuição igualitária. Trata-se da aplicação do princípio da responsabilidade no campo da justiça distributiva.

    Assim, Dworkin (2005) defende a concepção de igualdade de recursos disponíveis. Os recursos são entendidos de modo amplo, como quaisquer recursos que os indivíduos possuam privativamente, não são assim somente a riqueza das pessoas, mas também suas qualidades pessoais de força, talento, caráter, aspirações e oportunidades legais ou outros tipos de oportunidades.

    Para Dworkin (2005) as pessoas são responsáveis pelas escolhas que fazem em suas vidas, no entanto esta premissa não é suficiente para uma distribuição justa de bens, uma vez que apesar de todos serem responsáveis por suas escolhas, estas também sofrem influência de atributos naturais, como o talento e a inteligência, que afetam sobremaneira a distribuição de recursos na sociedade. A igualdade em Dworkin (2005) se traduz nos recursos que as pessoas devem dispor para que possam realizar suas escolhas pessoais. 


    Fonte: http://www.publicadireito.com.br/artigos/?cod=0631ba089fcd29d7

  • A igualdade de recursos é uma questão de igualdade de quaisquer recursos que os indivíduos possuam privadamente.

    Circunstâncias segundo as quais as pessoas não são iguais em bem-estar, mas nos recursos de que dispõem.

    Gabarito A.

  • Só para mim não entra na cabeça a necessidade de estudar essa "matéria"?


ID
2537233
Banca
PM-SC
Órgão
PM-SC
Ano
2017
Provas
Disciplina
Filosofia do Direito
Assuntos

Segundo o livro “Polícia Comunitária, construindo segurança nas comunidades”, assinale a alternativa INCORRETA em relação às colunas ou eixos estruturantes da filosofia de Polícia Comunitária:

Alternativas
Comentários
  • Pronta resposta é eixos estruturantes da filosofia de Polícia Comunitária:

  • A questão em comento exige conhecimento de premissas basilares de Segurança Pública comunitária, modelo diferente do tradicional.

    A resposta adequada para a questão é a alternativa INCORRETA.

    A Polícia Comunitária:


    I-                    Exige parceria com o cidadão e com a comunidade;

    II-                  Não busca números e resultados imediatos tal como as metas da Segurança Pública tradicional;

    III-                 Simboliza um modelo de proximidade do policial com o cidadão, ou seja, o policial é parceiro do cidadão, não seu adversário, não alguém a impor temor;

    IV-                Exige um policial proativo, comunicativo, que interage de maneira diferenciada com o cidadão e a comunidade, que dialoga, que está conectado com exigências da comunidade e reclames de direitos humanos.

    Observadas tais premissas, vamos analisar as alternativas da questão.

    LETRA A- CORRETA, LOGO NÃO RESPONDE A QUESTÃO. De fato, o modelo de Polícia comunitária não se debruça apenas sobre os efeitos dos problemas da Segurança Pública, mas sim uma leitura mais cautelosa de questões sociais, direitos humanos e causas dos problemas de Segurança Pública.

    LETRA B- INCORRETA, LOGO RESPONDE A QUESTÃO. Pronta resposta não é uma das alternativas traçadas pela Polícia Comunitária.

    LETRA C- CORRETA, LOGO NÃO RESPONDE A QUESTÃO. De fato, a Polícia Comunitária demanda parcerias com outros órgãos públicos e a iniciativa privada.

    LETRA D- CORRETA, LOGO NÃO RESPONDE A QUESTÃO. Com efeito, a Polícia Comunitária demanda proatividade policial.

    LETRA E- CORRETA, LOGO NÃO RESPONDE A QUESTÃO. Com efeito, a Polícia Comunitária exige proximidade do policial com o cidadão.

    GABARITO DO PROFESSOR: LETRA B



ID
2557090
Banca
FGV
Órgão
OAB
Ano
2017
Provas
Disciplina
Filosofia do Direito
Assuntos

O povo maltratado em geral, e contrariamente ao que é justo, estará disposto em qualquer ocasião a livrar-se do peso que o esmaga.

John Locke


O Art. 1º, parágrafo único, da Constituição Federal de 1988 afirma que “todo o poder emana do povo, que o exerce por meio de representantes eleitos ou diretamente”. Muitos autores associam tal disposição ao conceito de direito de resistência, um dos mais importantes da Filosofia do Direito, de John Locke.


Assinale a opção que melhor expressa tal conceito, conforme desenvolvido por Locke na sua obra Segundo Tratado sobre o Governo Civil.

Alternativas
Comentários
  • B - Sempre que os governantes agirem de forma a tentar tirar e destruir a propriedade do povo ou deixando-o miserável e exposto aos seus maus tratos, ele poderá resistir.

  • Alternativa CORRETA B. É exatamente isso que Locke defende. Ao estabelecer um acordo por meio das leis, não se está a conceder um cheque em branco para o poder público fazer o que quiser. Ele foi criado com propósitos específicos e, uma vez não observados, surge o direito de resistir à ordem estatal. Simples assim.

  • Os direitos políticos são direitos públicos subjetivos fundamentais conferidos aos cidadãos para participarem dos negócios políticos do Estado. Tais direitos decorrem do princípio democrático, positivado Constituição, segundo o qual art. 1º, parágrafo único, CF/88 – “Todo o poder emana do povo, que o exerce por meio de representantes eleitos ou diretamente, nos termos desta Constituição”.

    A opção que melhor expressa tal conceito, conforme desenvolvido por Locke na sua obra Segundo Tratado sobre o Governo Civil, é a de que sempre que os governantes agirem de forma a tentar tirar e destruir a propriedade do povo ou deixando-o miserável e exposto aos seus maus tratos, ele poderá resistir.

    Segundo LOCKE, “Cada vez que os legisladores tentam tomar ou destruir a propriedade do povo, ou reduzi-lo à escravidão sob um poder arbitrário, estão se colocando em um estado de guerra contra o povo, que fica, portanto, dispensado de qualquer obediência e é então deixado ao refúgio comum que Deus deu a todos os homens contra a força e a violência. Sempre que o legislativo transgredir esta regra fundamental da sociedade, e, seja por ambição, por medo, por tolice ou por corrupção, tentar dominar a si mesmo ou pôr as mãos em qualquer outro poder absoluto sobre as vidas, as liberdades e os bens do povo, por este abuso de confiança ele confisca o poder que o povo depositou em suas mãos, para fins absolutamente contrários, e o devolve ao povo, que tem o direito de retomar sua liberdade original, e pelo estabelecimento de um novo legislativo (o que ele considerar adequado) promover sua própria segurança e tranqüilidade, que é o objetivo pelo qual estão em sociedade”.

    Gabarito do professor: letra b.

    Referência:

    LOCKE, John. Segundo tratado sobre o governo In: Carta acerca tolerância; Segundo tratado sobre o governo; Ensaio acerca do entendimento humano. 2. ed. São Paulo: Abril. Cultural, 1978.


  • GABARITO B 

    (...) cada vez que os legisladores  tentam tomar ou destruir a propriedade do povo, ou reduzi-lo à escravidão sob um poder arbitrário, estão se colocando em um estado de guerra contra o povo, que fica, portanto, dispensado de qualquer obediência e é então deixado ao refúgio comum que Deus deu a todos os homens contra a força e a violência. (p. 95/96; SEGUNDO TRATADO DO GOVERNO CIVIL/ John Locke​)

     

  •  b)

    Sempre que os governantes agirem de forma a tentar tirar e destruir a propriedade do povo ou deixando-o miserável e exposto aos seus maus tratos, ele poderá resistir. 

  • Deveriam tirar as questoes de Filosofia Juridica do Exame da Ordem. Essa materia e uma palhacada e nao reflete nada na vida advocaticia.

  • Locke era defensor do chamado direito natural. No entanto, ele temia o chamado Estado de Natureza, uma vez que nessas condições não há uma ordem coletiva a ser imposta, ou seja, é cada um por si e Deus por todos. Diante disso, o cometimento de injustiças era praticamente certo, o que acabava com qualquer hipótese de manutenção da ordem coletiva.

     

    Conforme afirma o próprio Locke, “A legislatura é um poder supremo, e pode ser chamado, em um certo sentido, um poder absoluto, mas em nenhum sentido um poder arbitrário. É limitada ao bem público da sociedade”, e em último recurso, em caso de abuso, o povo tem o direito de resistir ao poder supremo1.


     

    Daí a discussão sobre o direito de resistência, que passa a existir quando há abuso por meio do Estado em não cumprir com o acordo em seus exatos termos.

  • Excelente questão sobre um dos filósofos mais importantes da história da humanidade! Deveriam haver mais questões assim no Exame! São essenciais para a formação de JURISTAS (e não meros rábulas)! Já dizia Pontes de Miranda: "Quem só direito sabe, nem direito sabe!"

  • Galera, alguém poderia me dizer qual errou incorreu na alternativa A)? Seria porque essa resistência foi descrita de forma genérica; e não refereido-se a  governantes e ambiente político, como redigido na alternativa B)?

  • Eduardo, obrigado pela dica do material pra OAb. Minha intenção é me submeter ao próximo exame. Um abraço!

  • Hélia, acredito que o erro da alternativa A seja por quê não condiz com o pensamento do John Locke:

    ''O povo maltratado em geral, e contrariamente ao que é justo, estará disposto em qualquer ocasião a livrar-se do peso que o esmaga.''

    Veja que na alternativa A "a natureza humana é capaz de resistir às mais poderosas investidas morais e humilhações[...]", o que traz uma espécie de ''aceitação'' que não encontramos no pensamento de John Locke.

    Boa sorte no Exame.

  • Segundo LOCKE, “Cada vez que os legisladores tentam tomar ou destruir a propriedade do povo, ou reduzi-lo à escravidão sob um poder arbitrário, estão se colocando em um estado de guerra contra o povo, que fica, portanto, dispensado de qualquer obediência e é então deixado ao refúgio comum que Deus deu a todos os homens contra a força e a violência."

  • De acordo: Deveriam tirar as questoes de Filosofia Juridica do Exame da Ordem. Essa materia e uma palhacada e nao reflete nada na vida advocaticia.

    Deveria se manter nas carteiras escolares como formação do advogado! Como podemos levar isso a OAB, seria possível?

  • Sempre que os governantes agirem de forma a tentar tirar e destruir a propriedade do povo ou deixando-o miserável e exposto aos seus maus tratos, ele poderá resistir.

    O Segundo Tratado Sobre o Governo Civil  é um relato do estado de natureza. Afirma que homens são livres e iguais, e o estado de natureza, no qual viviam inicialmente, possui uma lei para governá-lo, a razão, a que todos estão sujeitos. Locke defende que sendo todos iguais e independentes, ninguém deve prejudicar o outro em sua vida, saúde, liberdade ou posses. Todos são obras do criador onipotente e infinitamente sábio, enviados ao mundo por sua ordem e a seu serviço. Um homem que transgride a lei da natureza declara viver sob outra regra que não aquela da razão e da equidade comum, e assim, torna-se perigoso ao gênero humano, podendo, então, sofrer punição.

  • A) O contratualismo moderno é uma forma de ruptura com a concepção aristotélico-tomista que explicava a sociedade e seus arquétipos baseados no conceito de natureza humana. Logo, a desobediência civil, para Locke, não encontra respaldo nem fundamento na natureza humana.

    B) GABARITO. Locke concebe o Estado de Natureza como portador de direitos fundamentais, dentre os quais, o direito à vida, à propriedade e à liberdade. O Estado social não é senão uma forma mais qualificada de garantir e preservar tais direitos. Contudo, a violação destes direitos por parte do Soberano, dá ao povo a legitimidade do direito de resistência à tirania.

    C) A questão exige a compreensão do conceito de direito de resistência à luz dos governados. A alternativa em tela, ao contrário, refere-se aos governantes e, se fosse essa a perspectiva solicitada no enunciado, a afirmativa estaria correta.

    D) O pensamento de Locke está inserido em uma base contratualista e busca pensar a configuração do Estado social à luz do contrato. A alternativa em comento trata, ao contrário, da estrutura do positivismo jurídico e da separação que este promove entre o direito e os demais aspectos da razão prática (ética e política), visando dar ao direito um tratamento normativo independente de valores éticos e de ideologias políticas.

    Fonte: LENZA, Pedro. OAB primeira fase: volume único. Saraiva Educação SA, 2020.

  • CORRELAÇÃO:

    LETRA B) Sempre que os governantes agirem de forma a tentar tirar e destruir a propriedade do povo ou deixando-o miserável e exposto aos seus maus tratos, ele poderá resistir.

    O povo maltratado em geral, e contrariamente ao que é justo, estará disposto em qualquer ocasião a livrar-se do peso que o esmaga.


ID
2557093
Banca
FGV
Órgão
OAB
Ano
2017
Provas
Disciplina
Filosofia do Direito
Assuntos

É verdade que nas democracias o povo parece fazer o que quer, mas a liberdade política não consiste nisso.

Montesquieu


No preâmbulo da Constituição da República, os constituintes afirmaram instituir um Estado Democrático destinado a assegurar, dentre outras coisas, a liberdade. Esse é um conceito de fundamental importância para a Filosofia do Direito, muito debatido por inúmeros autores. Uma importante definição utilizada no mundo jurídico é a que foi dada por Montesquieu em seu Do Espírito das Leis.


Assinale a opção que apresenta a definição desse autor na obra citada.

Alternativas
Comentários
  • Assertiva correta: letra C

    "Liberdade é o direito de fazer tudo o que as leis permitem".

  • A questão exigiu conhecimento de uma obra clássica estudada na faculdade de direito logo no primeiro semestre do curso: O espírito das leis, de autoria de Montesquieu.


     

    No livro décimo, em seu capítulo III, Montesquieu cravou a célebre passagem que o eternizaria: A liberdade é o direito de fazer tudo o que as leis permitem; e se um cidadão pudesse fazer o que elas proíbem ele já não teria liberdade, porque os outros também teriam este poder.


     

    Desta forma, o gabarito está na alternativa C.

  • A questão exige conhecimento relacionado aos conceitos de liberdade e democracia em Montesquieu. Para o autor, uma das principais características da democracia reside no condicionamento da liberdade dos cidadãos à conformidade às leis. Segundo Montesquieu “É verdade que nas democracias o povo parece fazer o que quer; mas a liberdade política não consiste nisso. Deve-se ter sempre presente em mente o que é independência e o que é liberdade. A liberdade é o direito de fazer tudo o que as leis permitem; se um cidadão pudesse fazer tudo o que elas proíbem, não teria mais liberdade, porque os outros também teriam tal poder”.

    Portanto, segundo o autor, Liberdade é o direito de fazer tudo o que as leis permitem. 

     

    Gabarito do professor: letra c.

    Referência: MONTESQUIEU. Do Espírito das Leis. São Paulo: Editora Nova Cultural, 1997.


  • "Liberdade é o direito de fazer tudo o que as leis permitem".

     

  • Não sou boa em filosofia, uso o “raciocínio lógico literário” e quase sempre, quando não acerto, elimino duas erradas:

    1.       O povo parece fazer o que quer, mas a liberdade [...] não consiste nisso.

    2.       Os constituintes instituíram (POR LEI) uma Democracia asseguradora da liberdade.

    3.       Definição do mundo jurídico, por Montesquieu em “Do Espirito das Leis”

    Ora, fazer o que quer não é liberdade (1), nossa CF assegura a liberdade (2) e estamos cassando um conceito de liberdade no mundo jurídico (3) no espirito das leis,

    soa razoável “chutar” a letra

    c) Liberdade é o direito de fazer tudo o que as leis permitem. 

  • Questoes de Filosofia Juridica sao uma palahacada da OAB.

  • Palhaçada é você ficar vagabundando o curso todo e no final não ter lido nem uma obra basilar do Direito. É uma prova técnica, não uma discussão em mesa de bar. Calouro.

  • "A liberdade é o direito de fazer tudo o que as leis permitem; se um cidadão pudesse fazer tudo o que elas proíbem, não teria mais liberdade, porque os outros também teriam tal poder”.

  • Brian Gentil, concordo em número, gênero e grau!

    Chato ver a turma banalizar um curso tão técnico.

  • A questão exige conhecimento relacionado aos conceitos de liberdade e democracia em Montesquieu. Para o autor, uma das principais características da democracia reside no condicionamento da liberdade dos cidadãos à conformidade às leis. Segundo Montesquieu “É verdade que nas democracias o povo parece fazer o que quer; mas a liberdade política não consiste nisso. Deve-se ter sempre presente em mente o que é independência e o que é liberdade. A liberdade é o direito de fazer tudo o que as leis permitem; se um cidadão pudesse fazer tudo o que elas proíbem, não teria mais liberdade, porque os outros também teriam tal poder”.

    Portanto, segundo o autor, Liberdade é o direito de fazer tudo o que as leis permitem. 

     

    Gabarito do professor: letra c.

    Referência: MONTESQUIEU. Do Espírito das Leis. São Paulo: Editora Nova Cultural, 1997.

  • As questões de Filosofia na OAB são dadas de graça para o candidato.

    As questões de filosofia são de interpretação. Não precisa conhecer a obra para responder à questão.

    Veja só:

    Texto: É verdade que nas democracias o povo parece fazer o que quer, mas a liberdade política não consiste

    nisso.

    A) Errada. A liberdade consiste na forma de governo dos homens, e não no governo das leis. (errado, pois nas democracias o povo parece fazer o que quer).

    B) Errada. A disposição de espírito pela qual a alma humana nunca pode ser aprisionada é o que chamamos de liberdade. (veja que aqui a questão extrapolou o que foi dito no enunciado)

    C) Correta. Liberdade é o direito de fazer tudo o que as leis permitem. (ou seja, na democracia o povo parece fazer o que quer. Se ''parece'' não podem fazer tudo, mas pode fazer o que as leis permitem. A liberdade política, inclusive, consiste nisso. É exatamente o que foi dito na segunda parte da frase.

    Viu! Basta interpretar. A resposta está no texto.

    D) Errada. O direito de resistência aos governos injustos é a expressão maior da liberdade. Como assim? Não foi falado nada de governo injusto. Extrapolou.

  • A) A liberdade política consiste na possibilidade de os homens serem governados por leis e, no caso de Montesquieu, esta liberdade se desponta na separação dos poderes executivo, legislativo e judiciário, o que assegura a existência de governos moderados.

    B) A redação da alternativa refere-se à liberdade filosófica, mais precisamente às condições volitivas de o ser humano agir livremente. Refere-se a um conceito abstrato de liberdade que, segundo Montesquieu, não corresponde à liberdade política.

    C) GABARITO. A concepção de liberdade política em Montesquieu, extraída de sua obra O Espírito da Leis, é apresentada de forma literal: “liberdade é o direito de fazer tudo o que as leis permitem”. Significa que os homens, ao renunciarem à sua independência natural (liberdade filosófica), passaram a viver sob a égide de leis políticas (liberdade política).

    D) O direito de resistência como manifestação da liberdade foge ao comando exigido no enunciado da questão. O foco principal, o conceito de liberdade política, perpassa a justificativa da liberdade sob o ponto de vista da lei e da separação dos poderes.

    Lenza, Pedro. OAB primeira fase: volume único. Saraiva Educação SA, 2020.

  • Observei, que na maioria das questões de filosofia do Direito a resposta, está no próprio enunciado.


ID
2713876
Banca
VUNESP
Órgão
PGE-SP
Ano
2018
Provas
Disciplina
Filosofia do Direito
Assuntos

A ausência de norma justa, caracterizada pela existência de um preceito normativo, que, se aplicado, resultará solução insatisfatória ou injusta, caracteriza lacuna

Alternativas
Comentários
  • Essa questão é pra quem sabe latim.

    Primeiramente, perceber que a lacuna é axiológica, pois existe uma norma aplicável (presença de algo que existe, presença ontológica, antônimo de lacuna ontológica) e há ideologia aplicável também, no caso, a da própria norma existente. No entanto, há lacuna no valor (axio) da norma existente, pois não contempla solução justa para o caso sob decisão. 

    Quanto ao segundo termo, é preciso saber que condo em latim é construo. 

    Em latim existem particípios de verbos:

    1- particípio presentr ativo: condens, o pedreiro é condens significa que o pedreiro está construindo neste momento.

    2 - passado passivo: condito, o prédio é condito, significa que alguém construiu o prédio no passado.

    3 - futuro ativo: conditurus, o pedreiro é conditurus, ele vai construir. (Nasciturus é quem vai nascet)

    4 - futuro passivo: condendo, o prédio é condendo, o prédio deve ser construído.

    Iure condito é o direito construído, o direito que já criaram, o direito positivo, direito ontológico.

    Iure condendo é o direito que deve ser construído. No caso, não há norma justa, uma nova norma deve ser construída para este caso específico pois aqui é injusta a que existe. 

    A lacuna é portanto axiológica, o que significa que há um iure condendo, um direito que deve ser feito para este caso.

  • Lacuna Ontológica: há lei para o caso concreto, porém a norma está desligada da realidade social, de modo que não tem aplicação prática.

    Lacuna Axiológica : Na lacuna axiológica há lei para o caso concreto, porém sua aplicação se revela injusta ou insatisfatória.

    ____________________________________________________________________________________________________________________

     

    Iure Condendo:  De jure condendo/constituendo(Lê-se: dê iúre condéndo/constituêndo.) Do direito a constituir; nos moldes do direito que deve ser estabelecido/constituído. Esta expressão é o propósito de matérias ou situações jurídicas não previstas em leis vigentes (ausência de norma justa), mas que podem ou poderão, com o tempo, constituir normas de direito objetivo ( Enciclopédia Juridica - http://www.enciclopedia-juridica.biz14.com/pt/d/de-jure-condendo-constituendo/de-jure-condendo-constituendo.htm)

     

    Iure Conditio : (Lê-se: dê iúre cóndito/constitúto.) Nos moldes do direito vigente/ constituído; o mesmo que direito adquirido.

     

  • Bobbio (1995), de outro turno, estabelece a existência de lacunas reais (iure conditio) e lacunas ideológicas ou impróprias (iure condendo). Nessa classificação, as lacunas reais são lacunas propriamente ditas, enquanto as lacunas ideológicas surgem a partir de uma confrontação entre o que é um sistema real e um sistema ideal, significando a ausência de norma justa.

    http://www.ambito-juridico.com.br/site/index.php?n_link=revista_artigos_leitura&artigo_id=7291

  • Essa é dose pra elefante.

  • GABARITO: B

     

    Lacunas na classificação de Mª Helena Diniz:

     

    Normativa: ausencia de lei para o caso concreto

    Ontológica: exite lei, mas sem eficácia social

    Axiológica: existe lei, mas sua aplicação é injusta/insatisfatória

  • Que nível de questão... As bancas estão cada vez mais sofisticadas e inusitadas em suas cobranças...

     

  • A questão é excelente, salvo a parte de ter que saber latim

  • Lamentável esse tipo de questão, pois premia o chute. A maior parte das pessoas capacitadas consegue chegar no raciocínio de ser uma lacuna axiológica (poderia parar por aí), mas quase ninguém vai saber ou lembrar se é "condendo" ou "condito", então vai ser premiado quem chutar na certa.

  • Lamentável esse tipo de questão, pois premia o chute. A maior parte das pessoas capacitadas consegue chegar no raciocínio de ser uma lacuna axiológica (poderia parar por aí), mas quase ninguém vai saber ou lembrar se é "condendo" ou "condito", então vai ser premiado quem chutar na certa.

  • (Procurador do Estado/SP - 2018 - VUNESP) A ausência de norma justa, caracterizada pela existência de um preceito normativo, que, se aplicado, resultará solução insatisfatória ou injusta, caracteriza lacuna (A)​ontológica ou iure condendo.

    Calareso, Alice Satin. Como passar concursos jurídicos . Editora Foco. Edição do Kindle. 

    A tradicional noção de lacuna do Direito envolve a ausência de norma para solucionar uma situação fática, o que é comum tendo em vista que as relações sociais são mais ágeis do que o processo de

    Calareso, Alice Satin. Como passar concursos jurídicos . Editora Foco. Edição do Kindle. 

    criação de leis. Sob as luzes da doutrina de Norberto Bobbio e Maria Helena Diniz, a lacuna axiológica não é rigorosamente uma lacuna nesse sentido. A lacuna axiológica envolve a ideia de que existe uma norma, mas a sua aplicação ao caso concreto levaria a uma solução insatisfatória ou injusta. Ainda sob a mesma doutrina, a lacuna ontológica envolve a ideia da existência de uma norma, mas que já se encontra obsoleta em vista do dinâmico progresso das relações sociais e avanço tecnológico. A lacuna ideológica consiste na falta de uma norma justa, que enseje uma solução satisfatória ao caso concreto.O “iure condito” é o Direito já constituído, já em vigor numa sociedade, significando que a solução do caso se daria com o Direito em vigor. Já a expressão “iure condendo” refere-se ao Direito ainda a ser construído.GN

    Calareso, Alice Satin. Como passar concursos jurídicos . Editora Foco. Edição do Kindle. 

  • A questão em comento demanda conhecimento basilar dos estudos de Bobbio.

    A ideia basilar do Direito é não ter lacunas e ser completo.

    Em havendo lacunas, em geral, são de ordem normativa, ou seja, a ausência de normas para regrar determinadas condutas sociais.

    As lacunas axiológicas não dizem respeito à ausência de normas, mas sim à ausência de normas tidas como justas para atender certos valores.

    Ademais, relembrando Bobbio, o mesmo classifica as lacunas em reais (Iuri conditio) e ideológicas (iure condendendo).

    Feitas estas observações, cabe analisar as alternativas das questões.

    LETRA A- INCORRETA. Não se trata de uma lacuna ontológica.

    LETRA B- CORRETA. Trata-se de lacuna axiológica e de ordem imprópria, ideológica, ou seja, iure condendendo.

    LETRA C- INCORRETA. Não se trata de lacuna real ou Iuri conditio.

    LETRA D- INCORRETA. Uma lacuna ideológica não é real, mas sim de ordem iure condendendo.

    LETRA E- INCORRETA. Não é caso de lacuna ontológica.

    GABARITO DO PROFESSOR: LETRA B


ID
2725108
Banca
FCC
Órgão
DPE-AM
Ano
2018
Provas
Disciplina
Filosofia do Direito
Assuntos

No que concerne à relação entre Direito e Estado, tal como a tematiza Hans Kelsen na obra O que é a justiça?, é correto afirmar que o Estado

Alternativas
Comentários
  • Quando Kelsen se pergunta: o que é justiça; ele responde não o sabemos e nem unca saberemos (é algo subjetivo).

    Abraços

  • PALAVRAS-CHAVE:

     

    Hans Kelsen - JurídiKo

    Norma pura; concepção formal;

    Dever ser; vontade racional; não há leis naturais.

    Teoria Pura do Direito;

    Jurídico-positivo: norma posta, positivada e suprema;

    Lógico-jurídico: norma fundamental hipotética; plano do suposto; fundamento lógico transcendental.

  • “... o Estado é um ordenamento jurídico. Mas nem todo ordenamento jurídico pode ser designado como Estado; só o é quando o ordenamento jurídico estabelece, para a produção e execução das normas que o integram, órgãos que funcionam de acordo com a divisão do trabalho. Estado significa ordenamento jurídico quando já alcançou certo grau de centralização...”. (KELSEN) 

    Fonte: https://www.jurisway.org.br/v2/dhall.asp?id_dh=949

     

  • Gabarito A (para aqueles que n sao assinantes)

  • a) é uma ordem jurídica relativamente centralizada. CORRETO. Essa ideia está inclusive na obra "Teoria Pura do Direito".

    b) é uma entidade metajurídica que precede a criação do Direito. ERRADO, pois é Kelsen entende ser uma ordem jurídica, não metajurídica.

    c) considerado democrático, e somente este, é legítimo para produzir normas jurídicas, pois reflete a justiça. ERRADO. Kelsen, na Teoria Pura do Direito, faz um elogio à democracia. Ele estrutura a liberdade e a pessoa livre em torno da democracia. Mas isso não significa que um Estado ditatorial não produza direito. Logo, para Kelsen, não é só o Estado democrático que produz normas jurídicas. O Estado ditatorial pode ser reconhecido do ponto de vista internacional como direito por exemplo. Esse Estado ditatorial, segundo Kelsen, também pode gerar sua própria ordem normativa. Isso porque, para Kelsen, não há ligação entre direito e justiça.

    d) é um grupo de pessoas unidas para a consecução de interesses comuns, e o Direito é um corpo normativo que reflete a moral do povo. ERRADO. Essa ideia de "moral do povo" é uma discussão sociológica e, portanto, alheia ao modelo kelseniano, que não está preocupado com discussões sociológicas e sim apenas com as discussões jurídicas.

    e) e Direito são duas coisas completamente distintas e não necessariamente relacionadas. ERRADO. Para Kelsen, Estado e Direito se confundem.

  • A questão exige conhecimento acerca das noções de Direito, Estado e Justiça na teoria pura do direito de Hans Kelsen, em especial dos conceitos definidos em sua obra “o que é justiça”. Sobre o tema, é correto afirmar que para Kelsen, o Estado é uma ordem jurídica relativamente centralizada. Vejamos:

     

    Segundo Kelsen (2001) “Se o Estado é uma comunidade, é uma comunidade jurídica. Ele é uma ordem jurídica que constitui a comunidade. Porém afirmar que o Estado como ordem social é idêntico ao Direito não é cor reto. Nem toda ordem jurídica é Estado. A ordem jurídica relativamente centralizada sim, é Estado. E Estado aqui é pessoa atuante, literalmente falando, como a autoridade, o poder por trás do Direito. Isto é o animismo que a Teoria Pura do Direito tenta, em vão, eliminar, por ser tautologicamente vazia”.

     

    O gabarito, portanto, é a letra “a”, sendo as demais alternativas incompatíveis com a obra jurídica do autor.

     

    Gabarito do professor: letra a.

     

    Referência:

    KELSEN, Hans; BORGES, Luís Carlos; KELSEN, Hans. O que é Justica? A justica, o Direito e a Política no espelho da ciência. 3.ed. Sao Paulo: Martins Fontes, 2001.

  • Para Kelsen, Estado é uma organização relativamente centralizada.

    Para Kelsen, o Estado é uma ordem jurídica centralizada.

    O examinador afirma que, para Kelsen, Estado seria uma ordem jurídica relativamente centralizada. Então, se ele for um pouco menos centralizado, comportando núcleos distintos de produção normativa (pluralismo), ainda assim se teria Estado e Direito em sentido moderno?

    Esquisita essa questão. Kelsen = monismo jurídico.


ID
2725117
Banca
FCC
Órgão
DPE-AM
Ano
2018
Provas
Disciplina
Filosofia do Direito
Assuntos

Relativamente à análise feita por Tercio Sampaio Ferraz Junior, na obra A ciência do direito, acerca da positivação do direito, é INCORRETO afirmar que, para o autor, o fenômeno da positivação

Alternativas
Comentários
  • A D e a E eram excludentes

    Abraços

  • Segundo Tercio Sampaio Ferraz Junior, na obra A ciência do direito, o fenômeno da positivação

    - estabelece o campo em que se move a ciência do direito moderno.

    - forçou a tematização do ser humano como objeto da ciência do direito.

     - representa uma legalização do câmbio do direito.

    - não faz do direito positivo o objeto único da ciência jurídica, mas condiciona a determinação do seu método e objeto.

  • A D e a E eram excludentes


  • Respondi por exclusão.

    Direito positivo = lei escrita

    A legislação não é a única fonte do direito, então, não é o único objeto da ciência jurídica.


    Mas encontrei o trecho do livro utilizado na questão na internet:

    [...]

    Entendemos que o fenômeno da positivação não só explica o papel ambíguo que o homem assume perante o Direito - fundamento de todas as positividades jurídicas, o homem é também o seu objeto central -, mas também o nascimento da moderna ciência jurídica, com suas imanentes ambigüidades. Queremos dizer, com isso, que a positivação forçou a tematização do ser humano como objeto da Ciência do Direito. Mesmo correntes modernas que procuram fazer da ciência jurídica uma ciência da norma não podem deixar de enfrentar o problema do comportamento humano e suas implicações na elaboração e aplicação do Direito. As recentes críticas que se fazem, nestes termos, ao kelsenísmo e sua Teoria Pura revelam que o objeto central da Ciência do Direito não é nem a positivação nem o conjunto das normas positivas, mas esse ser (o homem) que, do interior da positividade jurídica que o envolve, se representa, discursivamente, o sentido das normas ou proposições prescritivas que ele próprio estabelece, obtendo, afinal, uma representação da própria positivação.

    O que queremos dizer é que o fenômeno da positivação estabelece o campo em que se move a Ciência do Direito moderno. Note-se que isto não precisa ser entendido em termos positivistas, no sentido de que só o Direito positivo seja o seu objeto, mas simplesmente que a positivação envolve o ser humano de tal modo que toda e qualquer reflexão sobre o Direito tem de tomar posição perante ela. Ela não faz do direito positivo o objeto único da ciência jurídica, mas condiciona a determinação do seu método e objeto.

    [...]

    Ora, o fenômeno da positivação cortou a possibilidade de a Ciência do Direito trabalhar com este tipo de enunciado. Se o século XIX entendeu ingenuamente a positivação como uma relação causal entre a vontade do Legislador e o Direito como norma legislada ou posta, o século XX aprendeu rapidamente que o direito positivo não é criação da decisão legislativa (relação de causalidade), mas surge da imputação da validade do direito a certas decisões (legislativas, judiciárias, administrativas). Isto significa que o Direito prescinde, até certo ponto, de uma referência genética aos fatos que o produziram (um ato de uma vontade historicamente determinada) e sua positividade passa a decorrer da experiência atual e corrente, que se modifica a todo instante e determina a quem se devam endereçar sanções, obrigações, modificações etc. A positivação representa, assim, uma legalização do câmbio do direito.

  • Quanto tem alternativas excludentes assim facilita um pouco a vida... mas se não fosse isso. ai ai!!!

  • Essa foi leitura e intepretação, pra ninguém zerar filosofia do direito.

  • A questão exige conhecimento acerca do fenômeno da positivação do direito, na Perspectiva do autor Tércio Sampaio, em sua obra: a Ciência do Direito. Segundo o autor, temos que:

     

    “Entendemos que o fenômeno da positivação não só explica o papel ambíguo que o homem assume perante o Direito - fundamento de todas as positividades jurídicas, o homem é também o seu objeto central -, mas também o nascimento da moderna ciência jurídica, com suas imanentes ambiguidades. Queremos dizer, com isso, que a positivação forçou a tematização do ser humano como objeto da Ciência do Direito. Mesmo correntes modernas que procuram fazer da ciência jurídica uma ciência da norma não podem deixar de enfrentar o problema do comportamento humano e suas implicações na elaboração e aplicação do Direito. As recentes críticas que se fazem, nestes termos, ao kelsenísmo e sua Teoria Pura revelam que o objeto central da Ciência do Direito não é nem a positivação nem o conjunto das normas positivas, mas esse ser (o homem) que, do interior da positividade jurídica que o envolve, se representa, discursivamente, o sentido das normas ou proposições prescritivas que ele próprio estabelece, obtendo, afinal, uma representação da própria positivação. O que queremos dizer é que o fenômeno da positivação estabelece o campo em que se move a Ciência do Direito moderna. Note-se que isto não precisa ser entendido em termos positivistas, no sentido de que só o Direito positivo seja o seu objeto, mas simplesmente que a positivação envolve o ser humano de tal modo que toda e qualquer reflexão sobre o Direito tem de tomar posição perante ela. Ela não faz do direito positivo o objeto único da ciência jurídica, mas condiciona a determinação do seu método e objeto" (FERRAZ JÚNIOR, 1980).

     

    “A positivação representa, assim, uma legalização do câmbio do direito. Assim, por exemplo, a rescisão de um contrato de locação de imóveis pode ser proibida, de novo permitida, dificultada etc." (FERRAZ JÚNIOR, 1980).

     

    Tendo em vista os trechos supracitados em destaque, quando comparado com as alternativas, o gabarito, será a letra “d", sendo a única alternativa incompatíveis com a obra jurídica do autor.

     

    Gabarito do professor: letra d.

     

     

    Referência:

    FERRAZ JÚNIOR, Tércio Sampaio. A ciência do direito. São Paulo: Editora Atlas, 1980.

     


ID
2762935
Banca
FGV
Órgão
OAB
Ano
2018
Provas
Disciplina
Filosofia do Direito
Assuntos

Em tempos de mudanças e reformas, é comum assistirmos a diferentes tipos de lutas sociais, especialmente visando à garantia de direitos e à conquista de novos direitos. Em A Luta pelo Direito, o jurista alemão Rudolf Von Ihering afirma que o fim do Direito é a paz, mas o meio de atingi-lo é a luta.
Considerando essa afirmação e de acordo com o livro citado, assinale a opção que melhor caracteriza o pensamento jusfilosófico de Ihering.

Alternativas
Comentários
  • Ao longa de A Luta pelo Direito Ihering destaca e defende que a defesa do direito é um dever do interessado para consigo próprio (tema da primeira sessão ou, do primeiro capítulo) ou, de um modo mais abrangente, é também um dever para com a sociedade (tema da segunda sessão ou, do segundo capítulo). A frase de abertura da conferência (publicada em livro) é emblemática na caracterização de Ihering: “A paz é o fim que o direito tem em vista, a luta é o meio de que se serve para o conseguir”[10]. Isto é, o Direito busca a paz, fazendo-o por intermédio da luta. A premissa é válida para o indivíduo, sua classe e para o próprio Estado: “A vida do direito é uma luta: luta dos povos, do Estado, das classes, dos indivíduos”

    Luta e firmeza são as condições para a construção e a manutenção de direitos, que não são dados espontaneamente pela natureza; direitos são duramente conquistados e mais duramente ainda mantidos: “Todos os direitos da humanidade foram conquistados na luta: todas as regras importantes do direito devem ter sido, na sua origem, arrancadas àquelas que a elas se opunham, e todo o direito, direito de um povo ou direito de um particular, faz-se presumir que se esteja decidido a mantê-lo com firmeza”[12]. O Direito não se teoriza, se vive, é alcançado mediante força e luta: “O Direito não é uma pura teoria, mas uma força viva”[13]. É também de Ihering mais uma referência à conhecida metáfora da balança da Justiça, no sentido de que; “(...) a Justiça sustenta numa das mãos a balança e que pesa o Direito, e na outra a espada de que se serve para o defender. A espada sem a balança é a força brutal; a balança sem a espada é a impotência do Direito”[14].

    Fonte: Site Conjur.

  • Livro maravilhoso!

  • O Direito de uma sociedade é a expressão dos conflitos sociais dela e resulta de uma luta de pessoas e grupos pelos seus próprios direitos subjetivos. Por isso, o Direito é uma força viva, e não uma ideia.

  • A) Ideia de Volksgeist - espírito do povo - é de autoria de Savigny não de Ihering, este afirmara que o povo era como um só corpo, motivado por forças interiores e silenciosas. Afora é difícil afirmar que o direito romano é a melhor expressão do processo histórico-social que influencia o direito

    B) Correto

    C) O Direito resulta exclusivamente da ação institucional do Estado. O agente estatal é um dos diversos atores na formação da expressão jurídica auxiliando na criação de fontes formais, como as leis, ou na forma do poder judiciário julgando e criando sua jurisprudência com relação a muito desses conflitos, no entanto é inegável a influência de pessoas distintas do Estado,já que este edifica sua normatividade através dos ensejos, das vozes e dos conflitos de seus jurisdicionados.

    D) As ideias de superstrutura, infraestrutura, conflito de classes além do direito como uma expressão da burguesia para o controle da classe operária é de Karl Marx e não de Ihering, não havendo grandes traços convergentes entre as obras, apesar de ambos os autores serem contemporâneos, nascidos em 1818.

  • A questão exige conhecimento acerca das ideias do jurista alemão Rudolf Von Ihering, em sua obra A Luta pelo Direito. Para o autor, O Direito de uma sociedade é a expressão dos conflitos sociais dela e resulta de uma luta de pessoas e grupos pelos seus próprios direitos subjetivos. Por isso, o Direito é uma força viva, e não uma ideia. Nesse sentido, conforme o próprio Ihering "O direito não é mero pensamento, mas sim força viva. Por isso, a Justiça segura, numa das mãos, a balança, com a qual pesa o direito, e na outra a espada, com a qual o defende. A espada sem a balança é a força bruta, a balança sem a espada  a fraqueza do direito. Ambas se completam e o verdadeiro estado de direito só existe onde a força, com a qual a Justiça empunha a espada, usa ã mesma destreza com que maneja a balança".

    Gabarito do professor: letra b.

    Referência: IHERING, Rudolf Von. A luta pelo direito. Trad. João de Vasconcelos. São Paulo: Martin Claret, 2009. 100 p.

  • Como resolvi essa questão:

    A – A primeira coisa que salta aos olhos nessa questão é a expressão “sempre”, que é perigosa em questões objetivas, passando a ideia de totalidade, unicidade, as quais dificilmente se encaixam no Direito, visto a quantidade de exceções para tudo rsrs

    B – GABARITO. Percebe-se que a resposta nas questões de Filosofia estão no enunciado, que trata especificamente de lutas sociais e conquista de direitos, o que encaixa exatamente com essa alternativa. 

    C – Novamente um alerta vermelho com a expressão “exclusivamente”. Além disso, em momento nenhum a questão trouxe viés político ou Estatal.

    D – Essa alternativa é semelhante ao gabarito, mas ao olhar com atenção, o processo de luta de CLASSES especificamente, vem dos estudos Marxistas, juntamente com a ideia do uso do Direito como instrumento de dominação.

  • Sem sombra de dúvida a letra B palavras chaves direito subjetivo, luta e direito é um força viva não poderia ser outra alternativa pois são palavras próprias do jusfilósofo Ihering

  • Ihering possui uma visão realista do Direito. Não considera que os diretos – dentre eles, o direito do Estado, dos povos, das classes ou dos indivíduos – sejam dados pela natureza ou construídos sem esforços. O meio para se conseguir direitos é determinado pela luta, cujo fim é a concretização da paz. Por isso, o Direito, na visão de Ihering, não se teoriza, mas se vive. O direito é uma força viva que se obtém e se mantém mediante a luta.

    (OAB esquematizado - Pedro Lenza 2020)

  • ''Quem acertou,acertou. Quem não acertou, cocoricó. ''

  • CORRELAÇÃO:

    B) O Direito de uma sociedade é a expressão dos conflitos sociais dela e resulta de uma luta de pessoas e grupos pelos seus próprios direitos subjetivos. Por isso, o Direito é uma força viva, e não uma ideia.

    "Em tempos de mudanças e reformas, é comum assistirmos a diferentes tipos de lutas sociais, especialmente visando à garantia de direitos e à conquista de novos direitos. Em A Luta pelo Direito, o jurista alemão Rudolf Von Ihering afirma que o fim do Direito é a paz, mas o meio de atingi-lo é a luta. Considerando essa afirmação e de acordo com o livro citado, assinale a opção que melhor caracteriza o pensamento jusfilosófico de Ihering".

  • A importância de estudar provas passadas... Essa mesma pergunta foi feita no exame XVI - 2015. Quando li, logo me recordei da resposta.

    • Rudolf Von Ihering, em A Luta pelo Direito, afirma que “ O fim do direito é a paz, o meio de atingi-lo,a luta.” Assinale a afirmativa que melhor expressa o pensamento desse autor

    A) O Direito de uma sociedade é a expressão dos conflitos sociais desta sociedade, e ele resulta de uma luta de pessoas e grupos pelos seus próprios direitos subjetivos. Por isso, o Direito é uma força viva e não uma ideia.

  • Olá, colegas concurseiros!

    Passando pra deixar essa dica pra quem tá focado na prova da OAB.

    Serve tanto pra quem esta começando agora quanto pra quem já é avançado e só esta fazendo revisão.

     Baixe os 490 mapas mentais para prova da OAB.

    Link: https://go.hotmart.com/W62298174Y

     Estude 10 mapas mentais por dia.

     Resolva 10 questões sobre o assunto de cada mapa mental estudado.

    → Em 45 dias você terá estudado os 490 mapas e resolvido aproximadamente de 5000 questões.

    Faça esse procedimento e seu aproveitamento melhorará em até 85%!


ID
2762938
Banca
FGV
Órgão
OAB
Ano
2018
Provas
Disciplina
Filosofia do Direito
Assuntos

Em seu livro Levando os Direitos a Sério, Ronald Dworkin cita o caso “Riggs contra Palmer” em que um jovem matou o próprio avô para ficar com a herança. O Tribunal de Nova Iorque (em 1889), ao julgar o caso, deparou-se com o fato de que a legislação local de então não previa o homicídio como causa de exclusão da sucessão. Para solucionar o caso, o Tribunal aplicou o princípio do direito, não legislado, que diz que ninguém pode se beneficiar de sua própria iniquidade ou ilicitude. Assim, o assassino não recebeu sua herança.
Com base na obra citada, assinale a opção que melhor expressa uma das pretensões fundamentais da jusfilosofia de Ronald Dworkin.

Alternativas
Comentários
  • D)     Argumentar que regras e princípios são normas com características distintas, mas igualmente vinculantes e, em certos casos, os princípios poderão justificar, de forma mais razoável, a decisão judicial.

  • Veja o treço:

    ...o Tribunal aplicou o princípio do direito, não legislado, que diz que ninguém pode se beneficiar de sua própria iniquidade ou ilicitude. Assim, o assassino não recebeu sua herança.

    Resposta:

     d) Argumentar que regras e princípios são normas com características distintas, mas igualmente vinculantes e, em certos casos, os princípios poderão justificar, de forma mais razoável, a decisão judicial.

  • Argumentar que regras e princípios são normas com características distintas, mas igualmente vinculantes e, em certos casos, os princípios poderão justificar, de forma mais razoável, a decisão judicial.

  • Bacana

  • Fala Galera ! O próprio Dworking foi instituidor da chamada teoria da derrotabilidade, tendo em vista as diferenças entre regras e princípios (EXCLUA a "C"), o autor afirmava que a regra poderia ser afastada em determinados casos, sem que seja prejudicada sua vigência e utilização para conflitos ulteriores, sendo a aplicação de um princípio em determinados situações o mais adequado a se fazer, como exemplo temos o Direito Penal, uma vez que o Princípio da Insignificância, resultante dos Princípios da Fragmentariedade e da Última ratio, predomina ante a possibilidade aplicação exegética do Art 155 do CP, constituindo a ideia de tipicidade material. Diante disso é perceptível que o pensamento de Dworking que vai ao encontro da Alternativa "D".

  • A questão exige conhecimento relacionado à teoria das normas, em especial no que diz respeito à inquestionável contribuição do jusfilósofo Ronald Dworkin quanto à classificação dos princípios enquanto espécie de normas, em sua teoria dos princípios.
    O caso Rigg vs. Palmer resume-se na pretensão de um neto - demandante da ação - o qual era
    beneficiário do testamento de seu avô. A questão problemática do caso gira em torno do fato de o
    neto ter matado o avô justamente com a intenção de herdar a herança. Quando analisada a
    pretensão arguida em juízo - em 1889 – e tendo em vista a lacuna legislativa em relação ao
    testamento em situações como esta, a corte de apelação do Estado de Nova Iorque decretou que o
    demandante, Elmer Palmer, não tinha o direito de herdar, tendo em vista o princípio de que
    ninguém pode beneficiar-se da própria torpeza, favorecendo, assim, os demais herdeiros.
    Portanto, o caso exposto é essencial para argumentar que regras e princípios são normas com características distintas, mas igualmente vinculantes e, em certos casos, os princípios poderão justificar, de forma mais razoável, a decisão judicial.

    Gabarito do professor: letra d. 

  • A grande questão é tanto as regras como os princípios são normas jurídicas, entretanto cada uma com suas peculiaridades (princípio: geral, abstrato, axiológico) e as regras: (objetivas, limitadas "não conseguem prever tudo"). Sendo assim, em alguns casos a resolução apenas baseada na norma pode ser extremamente injusta, por isso são tão importantes os princípios.

  •  

    DWORKIN = princípios e regras são a mesma coisa? NÃO, são diferentes, embora em algumas situações aplica-se um como se fosse o outro, SÃO DIFERENTES!!!

  • Adoro essa matéria.

  • Parafraseando Ronald Dworkin Ninguém pode se beneficiar da sua própria torpeza se favorecendo sendo legítimo o entendimento que regras e princípios são normas com características distintas mas igualmente vinculantes e justificam a decisão judicial

  • Dworkin parte da concepção de que as normas são subdivididas em regras e princípios. As regras se estruturam de forma binária, sendo válidas ou inválidas. Os princípios, ao contrário, são indeterminados do ponto de vista semântico, permitindo que o sistema jurídico seja permeável por conteúdos morais, sobretudo quando se trata da resolução de casos difíceis.

  • CORRELAÇÃO:

    LETRA D) Argumentar que regras e princípios são normas com características distintas, mas igualmente vinculantes e, em certos casos, os princípios poderão justificar, de forma mais razoável, a decisão judicial.

    "Em seu livro Levando os Direitos a Sério, Ronald Dworkin cita o caso “Riggs contra Palmer” em que um jovem matou o próprio avô para ficar com a herança. O Tribunal de Nova Iorque (em 1889), ao julgar o caso, deparou-se com o fato de que a legislação local de então não previa o homicídio como causa de exclusão da sucessão. Para solucionar o caso, o Tribunal aplicou o princípio do direito, não legislado, que diz que ninguém pode se beneficiar de sua própria iniquidade ou ilicitude. Assim, o assassino não recebeu sua herança. Com base na obra citada, assinale a opção que melhor expressa uma das pretensões fundamentais da jusfilosofia de Ronald Dworkin".

  • Comentário do Professor do QC:

    A questão exige conhecimento relacionado à teoria das normas, em especial no que diz respeito à inquestionável contribuição do jusfilósofo Ronald Dworkin quanto à classificação dos princípios enquanto espécie de normas, em sua teoria dos princípios.

    O caso Rigg vs. Palmer resume-se na pretensão de um neto - demandante da ação - o qual era beneficiário do testamento de seu avô. A questão problemática do caso gira em torno do fato de o neto ter matado o avô justamente com a intenção de herdar a herança. Quando analisada a pretensão arguida em juízo - em 1889 – e tendo em vista a lacuna legislativa em relação ao testamento em situações como esta, a corte de apelação do Estado de Nova Iorque decretou que o demandante, Elmer Palmer, não tinha o direito de herdar, tendo em vista o princípio de que ninguém pode beneficiar-se da própria torpeza, favorecendo, assim, os demais herdeiros.

    Portanto, o caso exposto é essencial para argumentar que regras e princípios são normas com características distintas, mas igualmente vinculantes e, em certos casos, os princípios poderão justificar, de forma mais razoável, a decisão judicial.

    Gabarito do professor: letra d. 

  • Olá, colegas concurseiros!

    Passando pra deixar essa dica pra quem tá focado na prova da OAB.

    Serve tanto pra quem esta começando agora quanto pra quem já é avançado e só esta fazendo revisão.

     Baixe os 490 mapas mentais para prova da OAB.

    Link: https://go.hotmart.com/W62298174Y

     Estude 10 mapas mentais por dia.

     Resolva 10 questões sobre o assunto de cada mapa mental estudado.

    → Em 45 dias você terá estudado os 490 mapas e resolvido aproximadamente de 5000 questões.

    Faça esse procedimento e seu aproveitamento melhorará em até 85%!


ID
2863000
Banca
FCC
Órgão
DPE-MA
Ano
2018
Provas
Disciplina
Filosofia do Direito
Assuntos

Sobre as reflexões jusfilosóficas acerca do estado de exceção, é correto afirmar:

Alternativas
Comentários
  • Alemão: grundnorm

    Português: norma básica

    Abraços

  • B. Errada.

    Kelsen desenvolveu dois sentidos para a palavra Constituição: o sentido lógico-jurídico e o sentido jurídico-positivo. Em sentido lógico-jurídico, a Constituição significa a norma fundamental hipotética, cuja função é servir de fundamento lógico transcendental de validade da Constituição em sentido jurídico-positivo. Kelsen não admitia como fundamento de validade da Constituição positiva algo de real, de índole sociológica, política ou filosófica. Assim, foi obrigado a desenvolver um fundamento formal (normativo) para a Constituição em seu sentido jurídico-positivo – a norma fundamental hipotética, também chamada de norma pensada ou pressuposta –, que existiria, segundo ele, apenas como pressuposto lógico de validade das normas constitucionais positivadas.

  • Uma das mais célebres frases de Carl Schmitt é a que dá início à sua obra "Teologia política: “soberano é quem decide sobre o estado de exceção”.Nessa afirmação se dá o extrato daquilo que é mais rico em termos de análise do direito a partir do poder. O cumpridor da regra não revela a verdade do direito: apenas demonstra seu caráter burocrático. O poder nu, soberano, é aquele que passa por cima das normas e instaura, portanto, a decisão original. Daí o soberano

    ser o que decide sobre a exceção.

    Para Schmitt, pensar a exceção é pensar um quadro no qual não há uma ordem mecanicamente estabelecida. Trata-se de uma quimera, para o seu pensamento, imaginar que o conjunto normativo constitua e estabeleça o processamento da ordem. É justamente a exceção que instaura a ordem, a partir de uma desorientação inicial. A decisão não é o último momento de uma cadeia normativa, como pensa o juspositivismo; é o primeiro, pois é o que dá base à ordem.

    Filosofia do Direito / Alysson Leandro Mascaro. – 5. ed. rev., atual. e ampl. – São Paulo: Atlas, 2016. pg. 362.




    Um pensador atual – Giorgio Agamben. Nascido em Roma, em 1942, Giorgio Agamben pode ser visto como um grande nome do pensamento político atual. Lecionou nos Estados Unidos por algum tempo, mas renunciou o cargo de docente em protesto à política do governo norte-americano. Setembro de 2001 e o estado de exceção. Alguns textos políticos produzidos por

    Agamben são respostas, se não teorias, ao ocorrido no dia 11 de setembro nos Estados Unidos, com a queda das torres gêmeas e o suposto ataque ao Pentágono. Acredita o autor que acaba uma era com esses ataques, instaurando-se uma política de retorno ao estado de exceção, como existia nos campos nazistas em que era permitido reduzir os direitos de pessoas em nome de uma suposta “segurança”. Essas pessoas, que tem seus direitos reduzidos, são os chamados homo sacer.

    Homo Sacer. O homo sacer ou homem sagrado é uma figura do direito romano. É aquele que tendo cometido um crime não pode ser sacrificado segundo os ritos de punição. É aquele que pode ter sua condição reduzida e que pode ter seus direitos suspensos, sendo este o conceito biopolítico de Estado de Exceção para Agamben.

    Exemplo: Em 13 de novembro de 2001 foi criada a detenção infinita; ou, o ato patriótico dos Estados Unidos em que é possível se manter preso indefinidamente o estrangeiro suspeito de atividades que ponham em risco a segurança dos EUA; ou, anular radicalmente o ordenamento jurídico do indivíduo, podendo se criar um ser inominável e inclassificável.

    (Vade mecum humanístico / coordenação Alvaro de Azevedo Gonzaga, Nathaly Campitelli Roque. – 4. ed. – Rio de

    Janeiro: Forense; São Paulo: MÉTODO: 2014.)


  • GABARITO: A.

  • Tem algum livro que tem a resposta dessas questões?

  • Sobre a letra A:

    "A expansão totalitária em escala global provoca preocupações para com a continuidade do estado de exceção, que se faz regra, justamente porque se tornou permanente e autojustificativo. A política dos Estados Unidos da América ao longo do combate ao terrorismo, conceitualmente sustentada por John Yoo, professor em Berkeley, é da premissa um emblemático exemplo. O tema do estado de exceção, explorado por Carl Schmitt, para quem é no estado de exceção que se define um soberano, porque é este quem diz o direito, é assunto com o qual também se ocupou Giorgio Agamben, filósofo italiano, que já lecionou nos Estados Unidos. Para Agamben, o estado de exceção fez-se um paradigma de governo, no sentido que a expressão sugere na língua grega: um modelo.

    O estado de exceção, assim, seria instrumento de combate a uma necessidade e é justamente aí que reside o perigo. Ainda que em sua feição contemporânea decorra da teorização do direito público da tradição democrático-revolucionária, e não da tradição absolutista - é um tema de Robespierre, e não de Hobbes - - o estado de exceção, “impunemente, mediante a violência governamental, afasta o aspecto normativo do direito, eliminando-o”. Assim, para Agamben, o totalitarismo faz do estado de exceção uma situação que apresenta um conteúdo aparente de legalidade. Por isso, assusta-nos a passagem de Agamben, para quem, “o estado de exceção apresenta-se (...) como um patamar de indeterminação entre democracia e absolutismo”

    Fonte: https://www.conjur.com.br/2014-set-21/embargos-culturais-giorgio-agamben-perigo-estado-excecao-torne-regra

  • Agamben não é muito otimista quanto à possibilidade de retorno ao estado de direito me vigor antes da adoção da exceção como regra, pois o que está em questão agora são os próprios conceitos de „estado‟ e de „direito‟. No entanto, “se é possível tentar deter a máquina, mostrar sua ficção central, é porque, entre violência e direito, entre a vida e a norma, não existe nenhuma articulação substancial” . Isso significa que no centro do poder sempre constará o estado de exceção, embora nem sempre visível, pois este é um espaço vazio, onde uma ação humana sem relação com o direito se coloca em frente a uma norma sem relação com a vida. O que não quer dizer que a máquina, com seu centro vazio, não seja eficaz; ao contrário, o que ele procura mostrar é justamente, que ela continuou a funcionar quase sem interrupção a partir da Primeira Guerra Mundial, por meio do fascismo e do nacional-socialismo, até nossos dias: “O estado de exceção, hoje, atingiu exatamente seu máximo desdobramento planetário” . O aspecto normativo do direito pode ser, assim, impunemente eliminado e contestado por uma violência governamental que, ao ignorar no âmbito externo o direito internacional e produzir no âmbito interno um estado de exceção permanente, pretende, no entanto, ainda aplicar o direito.

    Nesse sentido, o estado de exceção em Agamben se configura como estrutura política fundamental presente em diversas sociedades que se intitulam democráticas, mas que adotam o instituto para os casos de conflitos mais extremos. no centro do poder sempre constará o estado de exceção, por mais invisível que seja a percepção deste, pois este é um espaço vazio, onde uma ação humana sem relação com o direito se coloca em frente a uma norma sem relação com a vida. Essa relação, portanto, mostra que “o direito em sua não relação com a vida e a vida em sua não relação com o direito significa abrir entre eles um espaço para a ação humana que, há algum tempo, reivindicava para si o nome „política.

    AGAMBEN, Giorgio. Estado de Exceção. Tradução de Iraci D. Poleti. São Paulo: Boitempo, 2004. P. 131.

  • A questão em comento requer conhecimento de premissas basilares dos estudos de Agamben, Schmidt, Kelsen e Estado de Exceção.

    Agamben tem uma leitura peculiar e bem crítica do Estado de Exceção, quase de conteúdo niilista.

    Para tal pensador, o totalitarismo ganhou escala global e uma perene “normalização", “naturalização" do Estado de Exceção.

    Logo, o Direito e sua normatividade, por mais que seja bem intencionado, pode ser superado pela vontade arbitrária e volátil dos Governos.

    Segundo Agamben, mesmo em democracias supostamente bem instauradas, há um espaço nevrálgico de ações e decisões não normativas, não pautadas em normas jurídicas, fulcradas tão somente na vontade dos detentores de situações políticas privilegiadas.

    Após tais definições, nos cabe comentar as alternativas da questão.

    LETRA A- CORRETA. Reproduz, com acuro, o pensamento de Agamben, ou seja, do totalitarismo ganhando expansão global, suplantando regras normativas, o Direito, e tornando a democracia cada vez mais frágil diante de ações invisíveis de grupos que efetivamente ocupam posições políticas governamentais privilegiadas.

    LETRA B- INCORRETA. A norma fundamental é uma construção de Kelsen para instauração do sistema jurídico. É pressuposta e hipotética. Não foi uma categoria bem trabalhada e digerida por Carl Schmidt.

    LETRA C- INCORRETA. Agambem, ao contrário do exposto, não trabalha, com frequência, o conceito de soberano como ordenador do Direito. Em Carl Schmidt é que tal categoria é mais frequente.

    LETRA D- INCORRETA. Embora Carl Schmidt tenha admitido expressamente o Estado de Exceção, ele vê tal contexto apenas quando necessário e indispensável, ou seja, quando a ordem esteja abusiva e a desordem reinstaure a ordem.

    LETRA E- INCORRETA. Kelsen, um democrata, não trabalhou as figuras do soberano e do Estado de Exceção.

    GABARITO DO PROFESSOR: LETRA A

  • o livro do Filipe augusto do ouse saber


ID
2863009
Banca
FCC
Órgão
DPE-MA
Ano
2018
Provas
Disciplina
Filosofia do Direito
Assuntos

Sobre as correntes do pensamento jurídico, é correto afirmar que para o

Alternativas
Comentários
  • Naturalismo é justamente o contrário da A, que trata do positivismo

    Abraços

  • Egologismo é a escola de pensamento jurídico argentina fundada por Carlos Cossio na primeira metade do século XX. Ela se baseia na construção de uma normatividade jurídica a partir da análise da relação sujeito existencial com a norma, frisando que direito é a tutela da conduta humana em sociedade, conduta objetivada em atos e tutelável pelo Estado.

    Para Cossio, deve-se frisar a análise da conduta humana na normatividade, da conduta como meio de se formar a norma jurídica, a qual não tem razão de ser senão na relação com a pessoa tutelada.     O termo egologismo remete exatamente à análise, pelo direito, da natureza humana em sua esfera do eu, da egoidade, portanto, da existência psicológica e livre do homem em sociedade.

    O egologismo foi uma escola de pensamento que proliferou basicamente na América Latina e no mundo hispânico em geral.


  • O tridimensionalismo é de Miguel Reale.

  • Os estudos de Eugeny Pashukanis acerca da origem, desenvolvimento e extinção do direito expressos em sua obra A Teoria Gera! do Direito e o Marxismo. Jurista respeitado, líder e dirigente da Revolução Russa de 1917, Pashukanis elaborou seu texto visando o esclarecimento pessoal e como meio de sistematização de suas idéias para uma intervenção mais qualificada no debate travado à época, quando, em pleno período de implantação do socialismo na Rússia, era discutido o caráter do direito na nova sociedade. Utilizando o método materialista de Karl Marx, Pashukanis estuda a gênese da forma jurídica a partir do aparecimento da forma mercadoria; aponta para a inevitável ligação entre direito e capitalismo e afirma a impossibilidade da existência de um "direito socialista", como queria Piotr Stucka, jurista bolchevique que com ele polemizou. O direito continua existindo no período de transição (socialismo) e conserva elementos burgueses. Por isso, só com a definitiva extinção das formas mercantis advindas do capitalismo é possível falar na extinção do direito. 

  • Este direito natural tem validade em si, é anterior e superior ao direito positivo e, em caso de conflito, é ele que deve prevalecer. O Jusnaturalismo é, por isso, uma doutrina antitética à do “positivismo jurídico”, segundo a qual só há um direito, o estabelecido pelo Estado, cuja validade independe de qualquer referência a valores éticos.

    FASSÒ, Guido. Jusnaturalismo. In: BOBBIO, Norberto. (Org.). Dicionário de política. Brasília: Editora UnB, 2010.


  • As regras, que por excelência emanam do 

    Poder Legislativo, concebidas como padrão, coexistem 

    com políticas, que podem ser definidas como outro tipo 

    de padrão que estabelece um objetivo a ser alcançado, em 

    geral, uma melhoria em algum aspecto econômico, político ou social da comunidade; as regras também coexistem 

    com os princípios, padrão a ser observado como exigên-

    cia de justiça e equidade ou alguma outra dimensão da 

    moralidade (DWORKIN, 2007, p. 60)

    A despeito de algumas divergências entre Alexy 

    e Dworkin, que não iremos aprofundar no presente tra-

    balho, ambos são taxativos em afirmar, por meio de suas 

    construções teóricas, que os princípios partilham, junta-

    mente com as regras, do espaço no mundo jurídico.

  • Errei a questão e pesquisei pra tentar entender, vou compartilhar as observações, então, se tiver algo errado, me ajudaria muito saber :)

    LETRA A

    Acredito que a ideia de Bobbio não era bem essa não.  Segundo o autor: “O Jusnaturalismo é uma concepção segundo a qual existe e pode ser conhecido um ‘direito natural’ (ius naturale), ou seja, um sistema de normas de conduta intersubjetiva diverso do sistema constituído pelas normas fixadas pelo Estado (direito positivo) ”. Alternativa A diz: só é valido direito estatal que se identifica com a justiça. Bobbio diz: o direito natural e o direito estatal existem, ambos, com perspectivas diferentes.

    (+)

  • LETRA B

    OK, a gente sabe que tridimensionalismo (Direito = FATO+ VALOR+ NORMA) está ligado ao Miguel Reale (com a observação de que Reale não foi o primeiro a pensar dessa forma, mas foi com ele (Reale) que a ideia foi sistematizada). Mas e Carlos Cóssio?

    Cóssio era argentino, partiu das ideias de Kelsen para chegar a outra perspectiva de Direito. Na obra "Ideologia e Direito" Cossio desenvolve a idéia de que o  não é apenas a norma, como preceitua Kelsen, mas possui um elemento basilar, que é a conduta humana.

    *obs- o primeiro autor a tratar do tridimensionalismo teria sido IcilioVanni,

    LETRA C

    Esse safadinho que me derrubou. Mas era isso aí mesmo, me faltou conhecimento para saber que o Pachukanis foi o maior jurista da União Soviética e que, basicamente, sistematizou as ideias de Marx no campo jurídico. Foi ele, inclusive, o autor de: A teoria geral do direito e o marxismo.

  • LETRA D ( de Deus e de Dworkin)

    A gente sabe que Dworkin levava os princípios a sério. Eu acho que essa alternativa pode ter dois erros. 1. Normativismo não é com Dworkin. Como o nome sugere, Normativismo é para quem investia muito no conceito de ... norma, e aí surge a pista de que o Normativismo (ou Teoria pura do Direito) está ligado com Kelsen. 2. Não tenho convicção, então é bom checar essa conclusão, mas a gente se confunde com a ideia de que princípios são regras por toda a discussão atual sobre a força normativa dos princípios (ou eu me confundo, pelo menos). Vale a lembrança que Dworkin separava princípios (que poderiam ser sopesados em cada caso) das regras (lógica do tudo ou nada). Logo, em Dworkin, são conceitos contrapostos (acho).

    ** Letra E já tá meio comentada pelos outros itens.

    É isso ;)

    Fontes:

  • FONTES

    https://enciclopediajuridica.pucsp.br/verbete/63/edicao-1/direito-natural-e-jusnaturalismo (letra A)

    https://jus.com.br/artigos/50721/o-tridimensionalismo-de-miguel-reale (letra B)

    https://pt.wikipedia.org/wiki/Teoria_egol%C3%B3gica_do_direito (letra B)

    https://pt.wikipedia.org/wiki/Evgeni_Pachukanis ( letra C)

    http://www.egov.ufsc.br/portal/sites/default/files/anexos/25014-25016-1-PB.htm (letra D)

  • Gosto do Lúcio, mas tem uns comentários dele que são desnecessários.

  • Sobre as correntes do pensamento jurídico, é correto afirmar que para o:

    A) Jusnaturalismo de Norberto Bobbio, só é válido o direito estatal, de modo que a justiça se identifica com a própria ordem jurídica positivada.

    Errado. O italiano Norberto Bobbio (1960) era defensor do positivismo e não do jusnaturalismo. O restante da assertiva está correto, se assimilado ao positivismo.

    B) Tridimensionalismo jurídico de Carlos Cóssio, o direito se constitui pela união dialética dos elementos fato, valor e norma.

    Errado. O Tridimensionalismo Jurídico ou a Teoria Tridimensional do Direito foi criada pelo brasileiro Miguel Reale (1950), que dispõe que o direito é composto pela tríade: fato, valor e norma.

    C) Marxismo de Evguiéni Pachukanis, há uma interdependência no capitalismo entre a forma mercadoria e a forma jurídica, o que leva a propugnar a própria extinção do direito.

    Correto. O russo Evguiéni Pachukanis (1920) foi um dos líderes da escola dos juristas marxistas soviéticos. Ele procurou desenvolver no campo jurídico as indicações de Karl Marx, no que concluiu que o direito é uma forma burguesa que atinge o máximo de seu desenvolvimento no capitalismo e que deve ser extinta quando da superação deste modo de produção.

    D) Normativismo de Ronald Dworkin, os princípios são regras jurídicas.

    Errado. O Normativismo foi uma grande contribuição do austríaco Hans Kelsen (1930) para o direito, sedimentando a ideia do ordenamento jurídico como sendo um conjunto hierarquizado de normas jurídicas estruturadas. Para o americano Ronald Dworkin (1980), os princípios, em contraste com as regras em sentido estrito, afirmam razões (jurídico-morais) que justificam uma determinada decisão, atribuindo-lhes um peso diferenciado ante outras possíveis razões com os quais compete. Assim, a intencionalidade dos princípios é valorativo e exige uma atitude interpretativa.

    E) Egologismo existencial de Miguel Reale, há duas ordens jurídicas válidas, quais sejam a natural e a positiva.

    Errado. O argentino Carlos Cóssio (1940) elaborou a Teoria Egológica do Direito, que dá novo contorno ao positivismo do austríaco Hans Kelsen (1930), estabelecendo que a norma não deve ser apenas compreendida racionalmente, mas em toda a sua multiplicidade, em seus elementos formais e materiais.

  • A questão em comento demanda compreensão de axiomas básicos na atividade de pensadores maiores da Teoria do Direito e Filosofia do Direito. Bobbio, Dworkin, Miguel Reale, Carlos Cóssio, Pachukanis merecem registro nos estudos da questão.

    Contribuiu para a leitura marxista do Direito Pachukanis, ou seja, uma leitura crítica do Direito enquanto superestrutura de ocultação de relações de classe, opressão e instrumento de alienação e dominação.

    Pachukanis, pautado nos estudos de Marx, identificou, como poucos, mecanismos de reificação, lógicas mercantilistas de opressão e a necessidade da superação do Estado e seu instrumental, qual seja, o Direito, para superação das mazelas da luta de classes e vitória dos oprimidos.

    Feitas tais ponderações, nos cabe comentar as alternativas da questão.

    LETRA A- INCORRETO. Bobbio, um reprodutor de Kelsen, está bem longe de ser um jusnaturalista. Em Teoria da Norma e Teoria do Ordenamento justificou e apoiou uma leitura formal do Direito e todo arsenal positivista kelseniano.

    LETRA B- INCORRETO. O tridimensionalismo, ou seja, a relação fato, valor e norma, não pertence a Carlos Cóssio, mas sim foi uma perspectiva trabalhada por Miguel Reale.

    LETRA C- CORRETO. Com efeito, Pachukanis, uma leitura crítica e marxista do Direito, vislumbrou, de fato, as relações entre mercadoria e arsenal formal do Direito, e a necessidade de suplantar o Estado, seu arsenal jurídico e todos mecanismos de opressão e alienação do homem.

    LETRA D- INCORRETO. O normativismo é uma perspectiva de Kelsen, Hart, positivistas, e não Dworkin, visto por muitos como pós positivista, crítico do Positivismo, e inaugurador da ideia de Direito como integridade e um sistema fundado não só em regras, mas também em princípios.

    LETRA E- INCORRETO. Conforme já exposto, Miguel Reale defendeu a perspectiva do tridimensionalismo jurídico e da tríade fato-valor-norma.

    GABARITO DO PROFESSOR: C


ID
2881519
Banca
MPE-PR
Órgão
MPE-PR
Ano
2019
Provas
Disciplina
Filosofia do Direito
Assuntos

Assinale a alternativa incorreta:

Alternativas
Comentários
  • Crítica de Daniel Sarmento ao Ativismo Judicial: carnavalização dos princípios constitucionais.

    Parece com essa E

    Abraços

  • Gab.: D

    Método de hermenêutica constitucional advindo do direito norte- americano.

    Interpretativismo = Afirma que a interpretação da CF deve ser pautada por elementos contidos no texto constitucional (textualismo - mens legis)

    Não interpretativismo = É uma visão progressista, afirma que cada geração te o direito de viver a CF do seu modo, cabendo ao judiciário adaptar os valores consagrados aos dias atuais. o papel do juiz seria desenvolver atualizar o valores constitucionais, atuando como verdadeiro protagonista.

  •  liberalismo igualitário

    Sem dúvida, a vertente mais cultuada do liberalismo político da atualidade é a do liberalismo igualitário que tem como pensadores john Raw e Ronald Dworkin.

    Para essa corrente, temos a prevalência (prioridade) dos direitos individuais sobre os

    interesses da coletividade ou do Estado (viés liberal); todavia, além da liberdade, existe uma preocupação com a igualdade material (com a justiça social e a distribuição

    de recursos). Nesse sentido, temos que "o liberalismo igualitário sustenta que não é papel do Estado promover valores hegemônicos na sociedade, (ou seja, uma concepção heterônoma de vida), interferindo nas liberdades individuais.Com isso, o liberalismo igualitário reconhece a existência de um amplo pluralismo na sociedade (pluralismo razoável), na medida em que as pessoas têm diferentes concepções sobre a vida digna (religiosas, morais, políticas, de projetos de vida e etc). Nesses termos, mediante o igual respeito e consideração a todos (Dworkin), defendem os liberais igualitários, a "tolerância" e o "dever de neutralidade estatal" diante das diversas concepções sobre o bem existentes na sociedade"m, além, como já dito, da necessidade de distribuição de recursos básicos que produzam igualdade material.

    Fonte: Direito Constitucional - Bernardo Gonçalves

  • Correntes Hermenêuticas Americanas

    São divididas em duas correntes:

    Interpretativista: Os juízes ao interpretarem a constituição devem se limitar a captar o sentido dos preceitos expressos na constituição, ou, pelo menos, nela claramente. Alinha-se à postura passiva do Juiz, ou seja, à concepção tradicional ou formalista, com o legislador como o centro da hermenêutica.

    Não-interpretativista: Diferentemente dos interpretativistas, os não-interpretativistas defendem uma maior autonomia do juiz ao seinterpretar a norma, com aplicação de “valores e princípios substantivos”– princípios da liberdade e da justiça. Assim, importa mais os valores, como a igualdade, a justiça e a liberdade demandados pela sociedade, do que a estrita vontade do legislador. Admite-se o ativismo judicial ou papel criativo do juiz.

    Fonte: http://www.hugomeira.com.br/hermeneutica-constitucional/

    Portanto, a letra A está correta, não sendo o gabarito da questão, já que se pede a alternativa incorreta.

  • “Embora uma das principais características do neoconstitucionalismo seja a defesa da constitucionalização do Direito, parte da doutrina defende que tal fenômeno deva ocorrer com parcimônia. Isso porque a excessiva constitucionalização do Direito, conhecida como panconstitucionalização, poderia gerar um viés antidemocrático no ordenamento jurídico de determinado Estado; afinal, se tudo já está decidido e definido pela Constituição, é pequeno ou quase nulo o espaço de liberdade e conformação do legislador. Nessa linha, os representantes do povo seriam meros executores de medidas já impostas pelo constituinte, o que atentaria contra o regime democrático. Logo, para que a constitucionalização do Direito ocorra de forma democrática, é necessário que se respeite a liberdade de conformação do legislador.”

    Fonte: caderno sistematizado

  • nem li

  • d) O liberalismo igualitário supera a noção de individualismo, pois seu foco se centra em entidades supraindividuais como o Estado, a Nação, a Sociedade, os grupos étnicos e outros conjuntos de pessoas. (ALTERNATIVA INCORRETA)

    Liberalismo é uma teoria política e social que enfatiza fundamentalmente os VALORES INDIVIDUAIS da liberdade e da igualdade. Para os liberais, todo indivíduo têm direitos humanos inatos. O governo tem o dever de respeitar tais direitos e deve atuar principalmente para resolver disputas quando os interesses dos indivíduos se chocam

    Liberalismo igualitário consiste na noção de que a justiça deve manter-se neutra às concepções da vida boa, refletindo um conceito sobre as pessoas como seres dotados de livre escolha e sem amarras morais preexistentes. Liberdade de escolha e Estado neutro, dessa forma, são indissociáveis, pois seres livres e independentes requerem uma estrutura de direitos neutra quanto às finalidades, que se recuse a tomar partido em controvérsias morais e religiosas, que deixe os cidadãos livres para escolher os próprios valores. Isto é, nada tem a ver com entidades supraindividuais.

    FONTE: https://medium.com/@lucaspereira_93691/liberalismo-igualit%C3%A1rio-c6b76f71065f

  • onde estuda essas coisas? hahahahhahahaha

  • Já dizia Tadeu Schmidt do Fantástico: "Que, que isso rapaz?"

  • A questão aborda temáticas diversas, mesclando hermenêutica constitucional, filosofia política e filosofia do direito. Analisemos as assertivas:

    Alternativa “a": está correta. Conforme CANOTILHO (p. 206), as correntes interpretativistas consideram que os juízes, ao interpretarem a constituição, devem limitar-se a captar o sentido dos preceitos expressos na constituição, ou, pelo menos, nela claramente implícitos. O interpretativismo, embora nâo se confunda com o literalismo — a competência interpre-tativa dos juízes vai apenas até onde o texto claro da interpretação lhes permite —, aponta como limites de competência interpretativa a textura semântica e a vontade do legislador.

    Alternativa “b": está correta. A teoria da democracia dualista, defendida por Bruce Ackerman (1991), sustenta que as decisões adotadas pelo próprio povo, em contextos de grande mobilização cívica, devem ser protegidas do alcance da vontade dos representantes do povo, formada em momentos em que a cidadania não esteja intensamente envolvida. Esta teoria distingue a política extraordinária, correspondente àqueles "momentos constitucionais", da política ordinária, que se realiza através das deliberações do dia a dia dos órgãos representativos. Para a perspectiva ackermaniana, a política extraordinária - que não exige, necessariamente, formalização procedimental através de assembléia constituinte ou de emenda constitucional - se situa em patamar superior à política ordinária, e pode legitimamente impor limites a esta.

    Alternativa “c": está incorreta. O correto seria fazer a comparação com a norma e o texto (ou texto normativo) e não com o dispositivo. Conforme ÁVILA (2004), não há uma necessária correspondência entre norma e seus dispositivos. Essa análise permite afastar a falsa imagem que determina uma relação simultânea entre texto e norma, pois, nem sempre a norma é derivada de um "texto", assim como nem sempre é possível derivar uma norma de um texto.

    Alternativa “d": está incorreta.  Não há que se falar em superação, mas sim em “não contradição". Essa é a perspectiva, por exemplo, do liberalismo igualitário de Will Kymlicka. O individualismo liberal, para o autor, é calcado no universalismo de base kantiana, não conflitando com o ideal de comunidade, sendo que o objetivo da concepção de responsabilidade e autonomia individual não é colocar as pessoas umas contra as outras, mas possibilitar a união de todos os cidadãos pelo respeito mútuo.

    Alternativa “e": está correta.  Conforme SARMENTO, com efeito, quem defende que tudo ou quase tudo já está decidido pela Constituição, e que o legislador é um mero executor das medidas já impostas pelo constituinte, nega, por consequência, a autonomia política ao povo para, em cada momento da sua história, realizar as suas próprias escolhas. O excesso de constitucionalização do Direito reveste-se, portanto, de um viés antidemocrático.

    Gabarito do professor: letra d.

    Referências:

    ACKERMAN, Bruce. We the people: Foundations. Cambridge: The Belknap Press, 1991.

    ÁVILA, Humberto. Teoria dos Princípios: da definição à aplicação dos princípios jurídicos. 3. ed. São Paulo: Malheiros, 2004.

    CANOTILHO, J. J. Gomes. Direito constitucional e teoria da constituição. 6ª ed. Coimbra, 1993. Editora Almedina.

    KYMLICKA, Will. Liberalism, community and culture. New York: Oxford Press, 2002.

    SARMENTO, Daniel. O Neoconstitucionalismo no Brasil: riscos e possibilidades.

  • A questão trabalhou a Hermenêutica Constitucional no interior do debate jurídico norte-americano. Sugiro a leitura do livro do Bernardo Gonçalves Fernandes, nessa parte, ele trabalha esses pontos.

  • NÃO INTERPRETATIVISMO =     “A possibilidade e a necessidade de os juízes invocarem e aplicarem valores e   princípios substantivos (princípios da liberdade e da justiça) contra atos da responsabilidade do legislativo em desconformidade com o projeto de constituição” (Canotilho).

    Admite-se uma função criativa do juiz (ATIVISMO JUDICIAL) = o juiz não é mais boca da lei, como queria Montesquieu.

    A interpretação eleita deve ser a que dá a maior efetividade possível ao direito,

    para que cumpra a SUA FUNÇÃO SOCIAL (STF, AgRg na Rcl 2600/SE). Trata-se de princípio

    vinculado especificamente aos direitos fundamentais (implícito no art. 5º, § 1º, da CF).

  • A) As correntes Hermenêuticas Americanas são divididas em duas correntes:

    Interpretativista: Os juízes ao interpretarem a constituição devem se limitar a captar o sentido dos preceitos expressos na constituição, ou, pelo menos, nela claramente.

    Não-interpretativista: Diferentemente dos interpretativistas, os não-interpretativistas defendem uma maior autonomia do juiz ao se interpretar a norma, com aplicação de “valores e princípios substantivos”– princípios da liberdade e da justiça. Admite-se o ativismo judicial ou papel criativo do juiz.

    B) A teoria da democracia dualista, defendida por Bruce Ackerman, sustenta que as decisões adotadas pelo próprio povo, em contextos de grande mobilização cívica, devem ser protegidas do alcance da vontade dos representantes do povo, formada em momentos em que a cidadania não esteja intensamente envolvida. Esta teoria distingue a política extraordinária, correspondente àqueles "momentos constitucionais", da política ordinária, que se realiza através das deliberações do dia a dia dos órgãos representativos. Não exige, necessariamente, formalização procedimental através de assembléia constituinte ou de emenda constitucional - se situa em patamar superior à política ordinária, e pode legitimamente impor limites a esta.

    C) A qualificação de determinadas normas como princípios ou como regras, depende da colaboração constitutiva do intérprete. Para tanto, como bem salientado pelo autor Humberto Ávila, é preciso ter em mente que "normas não são textos nem o conjunto deles, mas os sentidos construídos a partir da interpretação sistemática de textos normativos".Não existe correspondência entre norma e dispositivo. Isto é, nem sempre que houver um dispositivo haverá uma norma e vice versa. O autor sustenta sua posição invocando os princípios da segurança jurídica e certeza do direito, os quais são normas que não têm dispositivos específicos para lhes dar suporte físico.

    D) O liberalismo igualitário sustenta que não é papel do Estado promover valores hegemônicos na sociedade, (ou seja, uma concepção heterônoma de vida), interferindo nas liberdades individuais. Cada pessoa deve ter a liberdade para eleger os seus planos de vida, além do acesso aos meios necessários para persegui-los, desde que isso não fira direito de terceiros. (...) Com isso, o liberalismo igualitário reconhece a existência de um amplo pluralismo na sociedade (pluralismo razoável), na medida em que as pessoas têm diferentes concepções sobre a vida digna (religiosas, morais, políticas, de projetos de vida e etc.). Livro do Bernardo Gonçalves Fernandes.

    E) Daniel Sarmento reconhece o papel extremamente importante que a filtragem constitucional tem desempenhado nos diversos ramos do direito, impondo uma releitura dos mais importantes institutos, porém chama atenção para o que ele denomina de "Panconstitucionalização". A constitucionalização em excesso é antidemocrática na medida em que retira da maioria o poder de decisão.

  • pesado pra caramba esse concurso!

  • concurseira determinada

    Direito constitucional: teoria, história e métodos de trabalho. - Daniel Sarmento

  • A alternativa incorreta foi retirada do livro de Daniel Sarmento, bem melhor explicado do que o citado Bernardo Gonçalves. Segue o trecho:

    A tradição liberal de defesa da liberdade manifesta-se tanto na esfera política quanto no campo econômico. Na esfera política, o liberalismo está vinculado à defesa de liberdades públicas e existenciais, como a liberdade de expressão, de religião e a privacidade. Já na esfera econômica, o liberalismo significa rejeição à intervenção estatal no mercado e defesa da livre-iniciativa e da propriedade privada. O liberalismo político pode endossar ou não as teses do liberalismo econômico. É possível, por exemplo,defender intransigentemente as liberdades 'públicas, mas apoiar, simultaneamente, enérgicas intervenções do Estado no campo econômico voltadas à promoção da igualdade material. Essa é a característica central do liberalismo igualitário, que tem como grandes expoentes o filósofo John Rawls e o jurista Ronald Dworkin, ambos norte-americanos.

    Com efeito, o que distingue o liberalismo igualitário do liberalismo tradicional é que o primeiro tem um forte compromisso não só com a liberdade, mas também com a igualdade material. O liberalismo igualitário contemporâneo legitima o "Estado de Direito", não o "Estado mínimo". Portanto, quanto à intervenção estatal no domínio econômico, ele está muito mais próximo da social democracia europeia do que do liberalismo clássico ou do neoliberalismo.

    Nada obstante, o liberalismo igualitário, como antes consignado, é uma vertente do liberalismo político. Daí a sua dimensão liberal que se exprime no reconhecimento da prioridade dos direitos individuais diante dos interesses do Estado ou da coletividade. Esta ideia foi bem sintetizada por John Rawls, na abertura da sua obra clássica sobre a Teoria da Justiça: "Cada pessoa possui uma inviolabilidade fundada na justiça que nem o bem-estar da sociedade inteira pode sobrepujar (...). Portanto, numa sociedade justa as liberdades decorrentes da igual cidadania são garantidas, e os direitos assegurados por razões de justiça não se sujeitam à barganha política ou a cálculos de interesse social" .

    Pode-se falar, neste sentido, que o liberalismo igualitário, como todo liberalismo, é individualista, pois o seu foco prioritário se centra no indivíduo, e não em qualquer entidade supraindividual como o Estado, a Nação, a classe social ou o grupo étnico..

  • Acertei por ter lido sobre o tema no livro de Daniel Sarmento. A questão simplesmente copiou e colou um trecho da página 207 e modificou algumas palavras. Ela só parece inteligente pelo tema, mas o examinador foi tão preguiçoso como o que cobra literalidade de lei.

  • Não to nesse nível.

  • CÊ TA DOIIIIIIIIIDO....

  • Questão dentro do conteúdo programático do Cargo de Promotor de Justiça. 2019 é o meu 1º ano de estudo para as Provas de Promotor, percebo que os cursos especialziados e o material de estudo estar em consonância entre o que é estudo e o que estar caindo nas provas.

    É uma longo jornada até o dia da tão sonhada aprovação!! 

    Não desistam amigos!! A luta é árdua, mas a vitória é certa.

     

    VQV!!

  • Pessoal, pra MPPR tem que estudar a Comissão de Concurso .

    Tem que ler o que os promotores do Paraná estão publicando ou sobre o que já fizeram estudos.

    O MPPR publica a comissão. Você vai lá no grupo que você está estudando e joga os nomes no google. E aí lê tudo que eles escreveram, participaram etc =).

    Não contem pra ngm esse segredo ;).

    GRUPO 1 – Procurador de Justiça

    FRANCISCO JOSÉ ALBUQUERQUE DE SIQUEIRA BRANCO e suplente Promotor

    de Justiça MARCELO ADOLFO RODRIGUES, para as disciplinas de Direito Penal,

    Direito Eleitoral e Legislação do Ministério Público;

    GRUPO 2 – Procurador de Justiça

    JOSÉ DELIBERADOR NETO e suplente Promotor de Justiça GUSTAVO HENRIQUE

    ROCHA  DE  MACEDO,  para  as  disciplinas  de  Direito  Constitucional,  Direito

    Administrativo, Direito  Tributário,  Filosofia  do  Direito,  Sociologia  Jurídica  e  Direito

    Previdenciário; 

    GRUPO 3 – Representante da Ordem dos Advogados do Brasil, na

    qualidade de Examinador, o Advogado EROULTHS CORTIANO JUNIOR, e suplente o

    Advogado WILLIAN SOARES PUGLIESE, para as disciplinas de Direito Civil, Direito

    Empresarial e Direito Processual Civil; GRUPO

    4 – Promotor de JOELSON LUIS

    PEREIRA e suplente Promotora de Justiça SIMONE MARIA TAVARNARO PEREIRA,

    para as disciplinas de Direito Processual Penal, Execução Penal, Medicina Legal,

    Direito  do  Consumidor  e  Direito  Sanitário; 

    GRUPO  5 –  Promotor  de  Justiça

    FERNANDO  DA  SILVA  MATTOS  e  suplente  Promotor  de  Justiça  WILLIAN

    BUCHMANN, para as disciplinas de Direito da Infância e da Juventude e/ou Direito à

    Educação,  Proteção  ao  Patrimônio  Público,  Direito  Ambiental  e  Habitação  e

    Urbanismo, Ação Civil Pública, Ações Coletivas e/ou procedimentos extrajudiciais de

    atribuição  do  Ministério  Público  (Inquérito  Civil,  Procedimento  Preparatório,

    Procedimento Administrativo e Notícia de Fato), Direitos da Pessoa com Deficiência,

    Direitos do Idoso e Direitos Humanos.

  • Tô muito lascada, meu Deus..

  • me consolei nos comentários.. rs

  • Corrente hermenêutica, são dividas em:

    1 - CORRENTE INTERPRETATIVA - Defende que, os juízes ao interpretarem a constituição, deve-se limitar aos preceitos nela expressos ou que pelo menos nela estejam claramente implícitos, pois os magistrados não podem, a pretexto de defender a constituição, suprimir a vontade política do poder político democrático, porquanto os juízes não tem legitimidade para impor seus valores à sociedade.

    2 - CORRENTE NÃO - INTERPRETATIVA - Por sua vez, esta corrente reconhece como legitima a atividade criadora dos juízes (atavismo judicial), ao argumento de que a interpretação não se limita a revelar o sentido do texto normativo, mas também ajuda a construir norma, podendo recorrer a elementos externo ao texto como (a realidade social, valores morais).

    fonte: Teoria da Constituição - TOMO I - Sinopses, pag. 231/232.

  • Alguém pode chamar a prof. Fabiana Coutinho?? ???

  • Isso me parece algum tipo de piada de mau gosto

  • Questão level very hard hahaha

  • 5 bocejos, zero ideia da resposta....

  • A letra c afirma que a norma é produto da interpretação do texto. Normas são construídas a partir de regras e princípios segundo entendimento doutrinário majoritário. Falar que existe norma sem dispositivo e vice versa é atentar para opinião ou estudo acadêmico singularizado. Não achei a questão inteligente, pelo contrário. Mal elaborada, perniciosa e sem atentar para o objetivo principal de um concurso público que é buscar pessoas capazes de contribuir com a instituição na resolução das demandas da sociedade, triste e desanimador.

  • Acho legal estudar sobre, desde que seja aplicado posteriormente no cargo, e não essa patifaria que vemos diariamente.

  • (CESPE- PETROBRÁS - ADVOGADO - 2007 )Entre as correntes de interpretação constitucional, pode-se apontar uma bipolaridade que se concentra entre as correntes interpretativistas e não interpretativistas das constituições. As correntes interpretativistas se confundem com o literalismo e permitem ao juiz que este invoque e aplique valores e princípios substantivos, como a liberdade e a justiça contra atos da responsabilidade do Poder Legislativo em desconformidade com a constituição.

    - Gabarito: Errado.

    Link: https://www.qconcursos.com/questoes-de-concursos/questoes/72281e34-2e?compartilhamento_id=29581

  • Uma complementação da letra B:

    B) Segundo a concepção dualista de democracia, há dois tipos de decisão que podem ser tomadas nesse regime: o primeiro tipo são as decisões do povo, que estabelecem a norma constitucional; o segundo tipo são as decisões dos governantes, que ocorrem pelas leis, decretos e demais atos regulares do governo.

    Ackerman da dicotomia conhecida como a política ordinária (normal lawmaking) e a política extraordinária (higher lawmaking). 

    A política ordinária: é visualizada através dos mecanismos ordinários que regulam o processo legislativo, cuja legitimidade se concretiza pela realização de eleições livres e regulares.

    Política Extraordinária: A segunda via é mais tortuosa e mais rara de ser visualizada na práxis: trata-se de um longo processo travado entre a sociedade civil e os demais poderes, cuja regulação não se encontra claramente definida. Apenas a política extraordinária consegue canalizar a vontade do Povo, o que explica porque se trata de um processo tão oneroso e que requer uma ação coordenada das diversas instituições que compõem o Estado para levar a cabo as requisições propostas. Por tal razão, eventuais entraves que possam comprometer a realização das reivindicações, como o princípio da separação de poderes, são deixados de lado em favor da formação de um momento constitucional.

    Extraído da tese do artigo: A saga da igualdade e os momentos constitucionais: o papel da Suprema Corte no Constitucionalismo Norte-americana.

    Autores: Guilherme Ozório e Katya Kozicki.

    Salvo melhor juízo, Essa política extraordinária é a consolidação da vontade do povo em momentos constitucionais (normas com teor constitucional), mesmo diante de entraves (obstáculos), tais como a separação de poderes, por exemplo. Agora, a Política ordinária: É aquela feita pelos legisladores democraticamente eleitos.

    Qualquer erro me notifica, por favor.

  • Eu acertei, mas escorrendo suor na testa. Eu me considero um social-democrata, politicamente falando.

    Eu sequer conhecia o termo liberalismo igualitário. Porém, Dworkin e John Rawls são muito citados. A questão foi feita no contexto do Direito Constitucional norte-americano.

    Inclusive, o Daniel Sarmento tem Doutorado na Universidade de Yale. Então, está explicado as referências.

    P.S. Aparentemente, os examinadores guardaram o livro do Barroso e estão usando mais o do Daniel Sarmento.

  • Recomendo o livro do professor Daniel Sarmento e Cláudio Pereira.

  • Gabarito do professor é letra "B" no corpo do texto da justificativa

    Mas ao final ele marca a letra "D".

    Qual o problema da letra "B" , afinal de contas?

  • É uma questão fora da curva em nível de conhecimento. É normal que em provas de alto nível haja questões como essa.

    Bons estudos a todos!

  • Breve comentário sobre a letra C

    "C) Embora se costume afirmar que a norma é o produto da interpretação do texto, não existe correspondência necessária entre norma e um dispositivo, pois há normas que não encontram suporte físico em um dispositivo específico, e há dispositivos a partir dos quais não se constrói norma alguma."

    Exemplo de norma que não encontra suporte físico em um dispositivo específico: Princípio da Proporcionalidade

    Exemplo de dispositivo a partir do qual não se constrói nenhuma norma: Preâmbulo da CF/88

  • Putz, depois que li a resposta certa, fez todo sentido hehe!

  • Entendi a B errada pois não cita que os representantes exercem o poder decorrente. Achei incompleta a assertiva.

  • O liberalismo igualitário, ao invés do dito na alternativa, exprime no reconhecimento da prioridade dos direitos individuais diante de interesses do Estado ou da coletividade. Não seria papel do Estado promover valores hegemônicos na sociedade, interferindo nas liberdades individuais.

    O liberalismo igualitário é uma vertente do liberalismo tradicional. O que diferencia é que naquele, além de um compromisso com a liberdade, há um forte compromisso também com a igualdade material dos indivíduos.

    Fonte: Daniel Sarmento. Direito Constitucional: teoria, história e métodos de trabalho.

  • "Liberalismo" superando "individualismo", focando em "entidades supraindividuais", "grupos étnicos"...

    Não tem como isso estar certo kkk

  • Olha, eu já gravei que é ao contrário essa parada.

    Mas alguém de verdade entender porque os interpretativistas dizem que não pode interpretar o texto da CT e os não-interpretativistas dizem que deve interpretar (transcender) na aplicação?

    Pensa num conteúdo que não tem lógica nos materiais que leio e o examinador adora.

  • D

    QUESTÃO INUTEEEEEL

  • A. CORRETO.

    B. CORRETO. Dualidade dos momentos: tem-se os de política extraordinária (momentos constitucionais) e os de política ordinária (atividade legiferante comum)

    C. CORRETO. Inclusive, remonta à técnica de interpretação constitucional (normativo-estruturante)

    D. ERRADO. Liberalismo igualitário continua sendo individualista que, nesse caso, refere-se ao foco no indivíduo em seu poder para determinar/eleger seus planos de vida

    E. CORRETO.

  • A resposta mais curtida faz a pergunta: "onde estuda essas coisas?"

    O ponto é: não se estuda essas coisas. Você perderia tempo estudando doutrina densa e ainda assim com grande chance de errar a questão, dado que é um tema bem subjetivo. Além disso, vai deixar de estudar grande parte do conteúdo que garantiria sua aprovação.

    O custo para acertar uma questão dessa é imenso. Você precisa, praticamente, ler os mesmos livros que um doutorando em constitucional tem que ler (Canotilho e sua turma). O ponto é que o doutorando em constitucional só faz isso da vida dele, e provavelmente já é concursado. Você não. Você precisa dominar o essencial de pelo menos 14 matérias diferentes. Não se aprofunde demais. Não perca tempo. Ligue o f0da-C para algumas coisas. Tenha estratégia!

  • depois dessa, pego minhas malas e vou para Azkaban

  • Foi difícil. Eu só acertei por saber uma coisinha. Lembrei da origem do liberalismo e dos direitos de primeira geração, que são essencialmente individuais e que preconizam a retirada do Estado da esfera de determinação do indivíduo. Não seria congruente com uma superação da noção de indivíduo.

  • Sobre a alternativa A - Assim como o colega Daniel, que comentou abaixo, eu sempre confundia a corrente interpretativista X não-interpretativista. Ora, pensava eu, se alguém precisa absorver valores e princípios para apurar o sentido da norma, essa pessoa com certeza está interpretando . Contudo, não é essa a logica. É o seguinte: se o cara está diante da lei e ele está retirando o sentido DIRETAMENTE da norma, ou seja, sem o uso de "ferramentas" que transcendem o próprio texto, ele estará interpretando essa norma.(corrente interpretativista)

    De outro lado, se para trazer sentido à norma lanço mão de valores e princípios que sequer encontram-se implícitos ou explícitos na norma, não há interpretação direta da norma.( não-interpretativista).

  • A letra D está incorreta, pois Dworkin entende que a igualdade liberal ou liberalismo igualitário rejeita a igualdade de bem-estar material, que neutralizaria as consequências das decisões éticas tomadas pelo indivíduo. O liberalismo igualitário trabalha com um valor de neutralidade:

    Essa forma de liberalismo (baseado na igualdade) insiste que o governo deve tratar as pessoas como iguais no seguinte sentido: não deve impor sacrifícios nem restrições a nenhum cidadão com base em um argumento que o cidadão não poderia aceitar sem abandonar seu senso de igual valor. [DWORKIN, p. 194, 1995 (tradução livre)]

    Portanto, Dworkin não abandona o individualismo, mas vincula-o ao respeito ao livre arbítrio do indivíduo de tomar suas decisões, a cavaleiro de interferências externas. Entretanto, ele reconhece que limitações econômicas podem interferir nos parâmetros de "boa vida" de um indivíduo (por óbvio), e assevera que:

    Certamente, os recursos devem figurar como parâmetros de alguma forma, porque não podemos descrever o desafio de viver bem sem fazer algumas suposições sobre os recursos que uma boa vida deve ter disponíveis. Os recursos não podem contar apenas como limitações, porque não podemos fazer qualquer sentido da melhor vida possível, abstraindo-se completamente de suas circunstâncias econômicas. [IBID., p. 259 (tradução livre)]

    Fonte: CP IURIS


ID
2881582
Banca
MPE-PR
Órgão
MPE-PR
Ano
2019
Provas
Disciplina
Filosofia do Direito
Assuntos

Segundo o grupo de teorias críticas do direito, é incorreto afirmar:

Alternativas
Comentários
  • "Sob a designação genérica de teoria crítica do direito, abriga-se um conjunto de movimentos e de ideias que questionam o saber jurídico tradicional na maior parte de suas premissas: cientificidade, objetividade, neutralidade,estatalidade, completude. (...)

    A teoria crítica, portanto, enfatiza o caráter ideológico do direito, equiparando-o à política, a um discurso de legitimação do poder. (...)

    A produção filosófica de pensadores como Horkheimer, Marcuse, Adorno e, mais recentemente, Jurgen Habermas, terá sido a principal influência pós-marxista da teoria crítica."

     

    Fonte: Curso de Direito Constitucional Contemporâneo: Os conceitos fundamentais e a construção do novo modelo. 

    Luís Roberto Barroso.

  • Lúcio Weber, o texto fala que a educação pode ser usada como INSTRUMENTO. Ou seja, da mesma forma que uma faca pode ser usada para algo bom ou ruim, a educação também é uma arma/instrumento. Existem pessoas que fazem doutorado, mas escolhem fazer bombas. Infelizmente.

    Dependendo da idade da pessoa, do seu conhecimento (ou falta dele), da confiança que deposita no seu líder, da carência de recursos, entre diversos outros meios, essa pessoa pode ser usada como massa de manobra para questões políticas, religiosas, educacionais, sociais...

    Toda educação possui um viés ideológico. Até aquelas que pretendem não ter: usam a "alienação" ( um conceito que designa indivíduos que estão alheios a si próprios ou a outrem tornando-se escravos de atividades ou instituições humanas, devido a questões econômicas, sociais ou ideológicas.).

  • Quando o examinador se refere à educação como meio de manipular, OBVIAMENTE, o faz em alusão à educação sem pensamento crítico, mecanicista e que (de)forma burocratas. Evidente que é a educação ontológica e não deontológica.

    Quanto ao gabarito (c), basta lembrar que diversos seguimentos que conhecem a Lei são "alienados" por não (re)conhecerem as mazelas de um Estado que serve apenas aos donos do poder (político e econômico).

  • GABARITO: LETRA C

    1. Teoria crítica do Direito e vertentes do pensamento crítico

    A teoria crítica do Direito é um movimento de pensamento aberto e composto de várias correntes teóricas que têm, como causa comum, a apresentação de uma concepção emancipadora em torno do Direito, de forma a desmistificar outras concepções teóricas que representem a manutenção de uma realidade socialmente injusta ou possam provocar retrocessos em relação às conquistas democráticas da sociedade ou impedir a evolução do processo democrático de mudanças sociais

    (...)

    4. Categorias da teoria crítica do Direito

    Esclarece Luiz Fernando Coelho que, no plano epistemológico, a teoria crítica do Direito possui categorias próprias, as quais não constituem um a priori formal ou material e sim estruturas de pensamento que foram construídas para o fenômeno jurídico como seu objeto reflexivo. São, assim, categorias da teoria crítica: sociedade; ideologia; alienação e práxis. O Direito, assim, passa a ser compreendido em função da sociedade, da ideologia, da alienação e da práxis, diversamente da concepção positivista. Elas não são estudadas como objeto do Direito; este é que é estudado pelo ponto de vista da sociedade, da ideologia, da alienação e da práxis.

    A sociedade não é concebida como ordem e progresso, mas como movimento social.

    A ideologia é compreendida como uma imagem manipulada que a sociedade tem sobre ela mesma.

    A alienação é o próprio produto da ideologia dominante, configurando-se como situação de inconsciência da grande maioria dos integrantes da sociedade sobre o papel que nela desempenham bem como sobre seus direitos fundamentais.

    Por fim, a práxis, apresentada como a dimensão ética da teoria crítica do Direito, seria a união do saber com o fazer, visando, precipuamente, à transformação da realidade social.

    Com efeito, a teoria crítica do Direito, por intermédio de uma visão libertadora e emancipadora, construtiva e prospectiva, propõe a revisão e a superação da hermenêutica jurídica tradicional.

    Em relação ao que foi analisado, observa-se que a teoria crítica do Direito é uma concepção teórica aberta e flexível. Ela propõe uma visão teórica emancipadora, livre de preconceitos ou de barreiras artificiais da racionalidade, bem como uma práxis transformadora da realidade social. Teoria e práxis são compreendidas em conjunto. A dialética da participação é sua proposta metodológica, a qual exige uma interação interdisciplinar efetiva, que tenha o condão de abranger várias dimensões teóricas num compromisso não só de compreender e interpretar, mas principalmente de compreender e interpretar para transformar a realidade.

    Extraído de https://aplicacao.mpmg.mp.br/xmlui/bitstream/handle/123456789/460/

    Teoria%20cr%C3%ADtica%20direito%20acesso%20justi%C3%A7a_Almeida.pdf?sequence=3

  • Isso é filosofia marxista do direito, sem qualquer isenção ou compromisso científico
  • Teoria crítica do direito nada mais é do que o bom e velho marxismo. Portanto, necessita-se pensar em termos marxistas para responder à questão. Alienação, para o intelectual alemão, é a realidade pela qual os trabalhadores não detêm os próprios bens que eles mesmos produzem. Nesse sentido especifico, o conceito de alienação vincula-se à ideia de trabalho (por isso, denomina-se de "entfremdete Arbeit" ou até mesmo de "entäußerte Arbeit"), de classes e de mais-valia (Mehrwert).

    NEXT

  • A questão em comento demanda conhecimento de bases da teoria crítica do Direito, ou seja, leituras singulares da aplicação dos estudos de Marx ao Direito.

    Em verdade, o legado de Marx nos leva a leituras até certo modo pessimistas do fenômeno jurídico.

    Wayne Morrison, comentando a obra de Marx, explica o seguinte:
    “ O Estado é tanto uma organização política, algo funcionalmente determinado para processos sociais, quanto uma ilusão. Para funcionar, para assumir sua forma, o Estado depende do Direito e da ideologia. (...) O Estado foi criado pelo desenvolvimento da divisão de trabalho e pela correspondente ascensão das relações de classe. Instaurou o conflito, e os indivíduos desenvolveram a ideia de propriedade privada para mediarem o conflito entre si próprios e sua vida em grupo" (MORRISON, Wayne. Tradução Jefferson Luiz Camargo. Revisão técnica Gildo de Sá Leitão Rios. São Paulo: Martins Fontes, 2006, p. 311)"

    Feitas tais considerações, resta claro em Marx o Estado como superestrutura, como  ente de dominação ideológica a permitir a opressão das classes mais favorecidas sobre as mais fracas e o Direito como mecanismo de perpetuação de tal ideologia.

    A partir destas explicações, cabe comentar as alternativas da questão. (A alternativa INCORRETA é a que responde a questão).

    LETRA A - CORRETO, LOGO NÃO RESPONDE A QUESTÃO. Não vigora o ideário positivista de ordem e progresso na leitura crítica do Direito de Marx. A sociedade, tal como o Estado, é teatro de conflitos e luta de classes, ou seja, grupos marginalizados vão estabelecer conflitos com grupos em posição de elite.

    LETRA B - CORRETO, LOGO NÃO RESPONDE A QUESTÃO. De fato, a ideologia, no pensamento crítico marxista, se presta a mascarar opressões e relações de classe. A ideologia atinge suas vítimas não no campo do consciente, mas sim gerando ilusões. Com efeito, para as perspectivas críticas do Direito influenciadas pelo marxismo, mídia, cultura e educação fazem parte do processo de alienação acima exposto.

    LETRA C - INCORRETO, LOGO RESPONDE A QUESTÃO. Não corresponde ao conceito de ideologia na perspectiva crítica marxista. Os vários comentários acima feitos demonstram claramente que alienação não coincide com o afirmado na letra C.

    LETRA D - CORRETO, LOGO NÃO RESPONDE A QUESTÃO. De fato, no campo da práxis, cabe ao jurista, munido de conhecimento e informações sobre as ideologias dominantes perpetradas pelo Estado e Direito, realizar perspectivas éticas de emancipação do homem, luta contra iniquidades, resistência diante de abusos e reconstrução do Direito enquanto Justiça.

    LETRA E - CORRETO, LOGO NÃO RESPONDE A QUESTÃO. De fato, a dialética é o campo do atrito, da contraposição de síntese e antítese, tudo no escopo da real transformação, de forma que seja possível um projeto de emancipação do homem nas searas jurídica, social e econômica, ou seja, um despertar de consciência diante dos flagelos da alienação e da luta de classes.


    GABARITO DO PROFESSOR: LETRA C.
  • Teoria crítica do Direito é afeta ao Marxismo. A categoria da alienação no marxismo consiste na apropriação do produto do trabalho pelo dono do meio de produção; o trabalhador fica "alheio", apartado do fruto do seu trabalho.


ID
2959846
Banca
FCC
Órgão
DPE-SP
Ano
2019
Provas
Disciplina
Filosofia do Direito
Assuntos

Relativamente ao que Émile Durkheim afirma acerca do crime e da pena em seu livro Da divisão do trabalho social, é INCORRETO afirmar que:  

Alternativas
Comentários
  • Frase: "NÃO REPROVAMOS PORQUE É UM CRIME, MAIS É UM CRIME PORQUE O REPROVAMOS" Durkheim.

    Abraços

  • Fico admirado pelos comentarios de Lúcio Weber, apesar de não acrescentar em mts das vezes, confesso sentir falta de sua eloquência. rsrsrsrsrs!!!!!!

  • Em defesa do Lúcio, muitos comentários dele (inclusive nessa questão), ajudam sim a entender o que a banca pediu.

    Claro que alguns são meio deslocados, mas pelo número de questões comentadas, acho isso até normal.

  • Émile Durkheim (1858-1917).

    Durkheim pode ser considerado como o fundador da sociologia jurídica (Iembrando que o fundador da "sociologia" é Augusto Comte). Determinou, a partir de seus estudos, as diretrizes que iriam seguir o pensamento sobre a sociologia, com sua crítica à sociologia positivista e naturalista que impregnava o pensamento sobre o fenômeno jurídico. 

    O objeto da Sociologia, para Durkheim, são os fatos sociais, encarados como coisas, com os quais o sociólogo deve estabelecer um autêntico aprendizado. Como fato social, o sociólogo francês entende “toda maneira de fazer, fixada ou não, suscetível de exercer sobre o indivíduo uma coerção exterior; ou ainda, toda maneira de fazer que é geral na extensão de uma sociedade dada e, ao mesmo tempo, possui uma existência própria, independente de suas manifestações individuais”. Os fatos sociais, por sua vez, têm como características essenciais a coerção social, a exterioridade aos indivíduos e a generalidade. A coerção social é evidenciada pelas sanções, sejam legais ou espontâneas, exercendo força sobre os indivíduos, nela devendo inserir-se também a educação como elemento de conformação do indivíduo à sociedade. Os fatos sociais são ainda dotados de exterioridade, no sentido de que se mostram independentes da vontade ou adesão dos indivíduos. Resulta daí a concepção do Direito como um fato social, constituído por normas impostas pela sociedade aos indivíduos. Por último, no dizer de Durkheim, os fatos sociais possuem natureza coletiva, com características exteriores comuns a uma universalidade de indivíduos. 

    O crime seria enquadrado na categoria dos fatos sociais normais. O autor afirma que o crime é normal porque uma sociedade que dele estivesse isenta seria inteiramente impossível. O crime não se observa apenas na maior parte das sociedades desta ou daquela espécie, mas em todas as sociedades de todos os tipos. Não há nenhuma onde não exista criminalidade.

    Assim sendo, não existe fenômeno que apresente da maneira mais irrecusável todos os sintomas da normalidade, pois ele se mostra intimamente ligado às condições de toda vida coletiva. Portanto, o crime é necessário, por estar ligado às condições fundamentais de toda vida social. Do mesmo modo, ele é útil, pois as condições de que ele é solidário são elas mesmas indispensáveis à evolução normal da moral e do direito.

    Os fatos fundamentais da criminalidade apresentam-se sob um aspecto de novidade. Contrariamente às idéias correntes, o criminoso não mais aparece como um ser radicalmente insociável, como uma espécie de elemento parasitária, corpo estranho e inassimilável, introduzido no seio da sociedade. Com efeito, ele é apenas um agente regular da vida social. Por sua vez, o crime não deve mais ser concebido como um mal que não possa ser contido dentro de limites demasiado estreitos.

    Resumo: o crime é normal, necessário e útil, sendo que muita vezes, ele constitui uma simples antecipação da moral futura. 

  • Continuação:

    Para Durkheim a sanção é uma resposta pré-estabelecida à uma ação, podendo ser positiva ou negativa. A sanção negativa constitui o que conhecemos como “pena”, que é uma desaprovação social do ato realizado.

    Weiss adverte que as contribuições de Durkheim no estudo da pena são centrais para toda a sociologia jurídica. O estudo durkheimiano sobre a pena se faz voltado para a solidariedade social, que, segundo o autor, poderia ser orgânica, isto é, resultante da divisão social do trabalho, na qual há maior interdependência entre os sujeitos; ou mecânica, que resulta de relações de similitude entre os indivíduos, nela há menor intensidade nas relações de dependência entre os homens.

    Durkheim conclui que nas sociedades com vínculos mecânicos, o desrespeito aos ideais sociais configura um crime, uma ofensa grave aos mais altos valores da comunidade, que acarreta uma pena. A pena surge na história como uma reação passional, que é mais intensa em função da menor instrução dos indivíduos da sociedade. Isso não quer dizer que nas sociedades contemporâneas, tidas como mais ilustradas, a pena tenha perdido seu caráter passional, de vingança, ela apenas se transformou, tornando-se mais humana, menos degradante e procura justificar verdadeiramente a si.

    Portanto, o direito Penal não perdeu sua razão de ser nas sociedades modernas. A reação passional continua compondo uma razão para pena, muito embora a tenham vestido com uma roupagem mais humana, mais racionalmente embasada, menos cega pelas paixões. O direito penal continua como guardião dos valores sagrados de uma sociedade. Contudo, a mera função retributiva por si só não sustenta a existência de penalidades, assim como não o faz a função preventiva, pois a inibição do crime pela punição, ainda que possa ocorrer, não exprime uma relação consecutiva necessária.

    A razão de ser da pena, em última análise, diz Durkheim, é a manutenção da coesão social e da vitalidade da consciência comum. Isto é, reestabelecer o respeito pela lei e pelos ideais sociais. Assim, a pena serve mais para manter o valor dos ideais coletivos, acalmando os cidadãos, que para punir e causar dor ao infrator. Por outro lado, quando não aplicada a pena, acumulando um histórico de impunidade, é possível que os nervos sociais aflorem, causando reações passionais por parte dos indivíduos, como acontece no Brasil de “justiceiros”, em virtude da perda de sentido ou validade dos sagrados ideais.

    Fonte:

    https://www.ebah.com.br/content/ABAAAgoqYAH/resumo-durkheim-rodriguez

  • A questão em comento demanda conhecimento da obra de Durkheim e sua visão sobre a pena e o crime.

    O escopo da pena é manter a ordem, a coesão social, a racionalidade.

    A consciência coletiva precisa ser preservada e é criminoso todo ato que atente contra tal estado.

    A pena revela-se uma reação passional diante do crime.

    Mesmo com o avançar da civilização, a pena não perdeu seu caráter de vingança e sua natureza passional.

    A pena é uma reação necessária diante de posturas que ofendam regras de conduta.

    A pena não tem caráter racional.

    Feitas tais observações, vamos comentar as alternativas da questão (a resposta adequada é a alternativa incorreta).

    LETRA A- CORRETA, LOGO NÃO RESPONDE A QUESTÃO. Com efeito, para Durkheim, a pena mantém a coesão social e a vitalidade da consciência comum.

    LETRA B- CORRETA, LOGO NÃO RESPONDE A QUESTÃO. Para Durkheim, de fato, um ato criminoso ofende estados fortes da consciência coletiva.

    LETRA C- CORRETA, LOGO NÃO RESPONDE A QUESTÃO. Para Durkheim, com efeito, a pena tem caráter passional.

    LETRA D- INCORRETA, LOGO RESPONDE A QUESTÃO. A pena não tem caráter racional. A pena tem natureza passional.

    LETRA E- CORRETA, LOGO NÃO RESPONDE A QUESTÃO. Reproduz a ideia da pena como reação de natureza passional, com intensidade graduada, exercida contra membros que violam certas regras de conduta.

    GABARITO DO PROFESSOR: LETRA D


ID
2959849
Banca
FCC
Órgão
DPE-SP
Ano
2019
Provas
Disciplina
Filosofia do Direito
Assuntos

Considere as assertivas abaixo acerca do conceito de órgão do Estado, explicitado por Hans Kelsen em sua obra Teoria Geral do Direito e do Estado.

I. Quem quer que cumpra uma função determinada pela ordem jurídica é um órgão.
II. A qualidade de órgão de um indivíduo é constituída por sua função.
III. Os cidadãos que elegem o parlamento não são órgãos do Estado.
IV. O juiz que sentencia o criminoso é um órgão de Estado.

Está correto o que se afirma APENAS em:

Alternativas
Comentários
  • Para Hans Kelsen, a norma jurídica comporta mais de uma compreensão, competindo à Ciência do Direito apontar as alternativas que se abrem ao operador jurídico. A aplicação da norma seria um ato de política jurídica.21

    A perspectiva da Teoria Pura do Direito de Hans Kelsen, é possível distinguir uma ?jurisprudência? que trata da validade do Direito de outra que considera a eficácia do Direito. Jurisprudência normativa e jurisprudência sociológica.

    John Austin constitui-se como patrono do moderno positivismo jurídico ao apartar o Direito do ?contexto social?, o que foi seguido por Hart e Hans Kelsen ao escrever a Teoria pura do direito. Jhon Austin é o pai de Kelsen e Hart (brincadeira, mas bom para lembrar!) Não é John Rawl é John Austin; parecido com Austin Powers!!!

    Abraços

  • I. Quem quer que cumpra uma função determinada pela ordem jurídica é um órgão. - Correta

    II. A qualidade de órgão de um indivíduo é constituída por sua função. - Correta

    III. Os cidadãos que elegem o parlamento não são órgãos do Estado. Errada . Para Kelsen funcionam como órgão legislativo. Se justifica pelas alternativas I e II. O cidadão, cumprindo sua função de eleger o parlamento, funciona como órgão do Estado.

    IV. O juiz que sentencia o criminoso é um órgão de Estado. - Correta. Para Kelsen o Juíz funciona como órgão aplicador do Direito.

  • Faltei esta aula de kelsen, mas fiz pós em lógica. Logo,

    1 - se a I é verdadeira a IV não pode ser falsa e (fiquei entre a B e a C)

    2 - se cumprir uma função é determinante para ser órgão então não posso deixar a II de fora.

  • A questão em comento demanda compreensão da obra de Kelsen e da ideia de órgão.

    Vamos analisar cada uma das assertivas.

    A assertiva I está CORRETA.

    De fato, segundo Kelsen, todo aquele que cumpre uma função determinada pela ordem jurídica é um órgão.

    É importante termos em mente que, a partir do momento que consideramos a assertiva I como correta, a dedução e a lógica responderão as demais assertivas.

    A assertiva II também é CORRETA.

    O fato da assertiva I ser reputada como verdadeira gera como consequência lógica que a II também seja.

    Ora, de fato, em Kelsen a qualidade de órgão de um indivíduo é talhada por sua função.

    A assertiva III é FALSA.

    Ela retira a condição de órgão de quem elege um Parlamento.

    Ora, quem elege um Parlamento exerce uma função.

    Ora, quem exerce uma função é um órgão.

    Por fim, a assertiva IV é CORRETA.

    O juiz exerce uma função.

    Quem exerce função, na teoria de Kelsen, é órgão.

    Uma linha de silogismos lógicos no leva, desde a assertiva I, raciocinando com lógica, a analisar cada uma das assertivas.

    De fato, o juiz que sentencia um criminoso é um órgão do Estado.

    Assim sendo, são verídicas as assertivas I, II e IV.

    Cabe comentar as alternativas da questão diante disto.

    LETRA A- INCORRETA. Não traduz a sequência correta da questão.

    LETRA B- CORRETA. Traduz a sequência da questão, qual seja, I, II e IV.

    LETRA C- INCORRETA. Não traduz a sequência correta da questão.

    LETRA D- INCORRETA. Não traduz a sequência correta da questão.

    LETRA E- INCORRETA. Não traduz a sequência correta da questão.

    GABARITO DO PROFESSOR: LETRA B

  • "...Em seguida Kelsen apresenta alguns aspectos sobre órgãos do Estado e diz que são todos aqueles que cumprem uma função determinada pela ordem jurídica, seja no âmbito legislativo, executivo ou judiciário. Qualquer pessoa que firma contrato é um órgão estatal, pois, está produzindo Direito, bem como o juiz que emite uma sanção e, também, um parlamentar que criou aquele Direito. Assim, todos os seus atos, consequentemente, são destinados também ao Estado. A renda, o patrimônio do Estado é proveniente dos impostos que são arrecadados por indivíduos no cumprimento de ordens jurídicas válidas, quando membros de órgãos estatais, estes terão suas ações imputadas ao Estado. As ações do Estado serão executadas pelos seus órgãos, ou seja, o próprio Estado cria e aplica a ordem jurídica e estabelece quem está qualificado e o procedimento para a execução das suas ordens. Dentro desse contexto, o Estado pode determinar que haja órgãos com mais obrigações que outros e, portanto, sejam superiores a estes, exercendo funções compostas. Um exemplo característico de função composta de atos com conteúdos diferentes é o processo. Todos os órgãos devem estar em sincronia nas execuções de suas funções parciais para que haja eficiência e eficácia na função total, a saber: a função total do Estado como ordem jurídica. (...)"

    fonte: https://ambitojuridico.com.br/edicoes/revista-136/o-direito-e-o-estado-segundo-hans-kelsen-x-direito-achado-na-rua-defendido-por-jose-geraldo-de-souza-junior/


ID
2959852
Banca
FCC
Órgão
DPE-SP
Ano
2019
Provas
Disciplina
Filosofia do Direito
Assuntos

A partir da leitura da tragédia Édipo-Rei, segundo Michel Foucault, em A Verdade e as formas jurídicas, o inquérito, considerado como um procedimento de produção da verdade judiciária na Antiguidade, inclui, necessariamente, a 

Alternativas
Comentários
  • Teoria Crítica: é mais uma teoria dentro do grande grupo TEORIAS DO CONFLITO, tem influência dno pensamento MARXISTA, mas teve portas abertas pela própria teoria do Labbeling Aproach (vide supra). MICHEL FOUCAULT converge mais para as teorias do CONFLITO (ver também Escola de Frankfurt no pós guerra).

    Um dos instrumentos do poder disciplinar, caracterizado por Michel Foucault em seu livro Vigiar e Punir, consiste em uma forma de punição que é, ao mesmo tempo, um exercício das condutas dos indivíduos. Este instrumento da disciplina é denominado, pelo autor, sanção normalizadora.

    Em Vigiar e Punir, Michel Foucault explicita os mecanismos disciplinares de poder que, segundo o filósofo, caracterizam a forma institucional da prisão do início do século XIX. De acordo com as análises deste autor, pode-se afirmar que a modalidade panóptica do poder disciplinar não está na dependência imediata nem é o prolongamento direto das estruturas jurídico-políticas de uma sociedade e, entretanto, não é absolutamente independente destas estruturas.

    Abraços

  • A história de Édipo significa, para Michel Foucault, o momento de surgimento da prática jurídica do inquérito. Para ele, com o passar do tempo, tal prática foi racionalizada até se tornar decisiva para a história do Ocidente, seja para a criação dos modelos judiciários que se seguiram, seja para o nascimento de outros saberes, tais como os filosóficos, os retóricos e os empíricos. Diferentemente da prova , que se fundamenta no desafio lançado de um guerreiro a outro e na qual a verdade é definida de maneira mecânica, seguindo métodos ordálicos, o inquérito tem sua base no testemunho; na presença de alguém capaz de relatar o fato e confirmar, a partir de sua própria memória, a verdade.

    Foucault fala de pelo menos quatro consequências diretas, no interior da democracia grega, advindas da possibilidade de testemunhar:

    1) a modificação nas relações de poder, na medida em que o testemunho transforma-se na possibilidade de opor a verdade ao poder estabelecido;

    2) a elaboração e o aprimoramento das formas tradicionais de prova e demonstração, que implicam, em suma, o como, em que condições e com quais regras se produz a verdade;

    3) o desenvolvimento da retórica, ou seja, da arte de persuadir;

    4) o desenvolvimento de um novo modelo de conhecimento que tem seu fundamento na memória e na investigação.

  • "Para Foucault, a tragédia de Édipo, escrita por Sófacles, é o primeiro testemunho das práticas judiciárias gregas: o mecanismo de estabelecimento da verdade obedece a chamada "lei das metades", ou seja, é por metades que se ajustam e se encaixam que a descoberta da verdade se procede em Édipo." - Curso Ênfase, professor Felipe Penteado.

  • Foucalt, ao estudar a tragédia grega e investigar Edipo Rei trouxe à baila a necessidade de uma apuração mais clara do inquérito.

    O inquérito, enquanto mecanismo da “verdade" judiciária na Antiguidade Clássica, segundo Foucalt, exalta a necessidade da TESTEMUNHA, figura diferente da apuração da verdade através das ordálias.

    A testemunha é alguém capaz de reproduzir fatos conforme sua memória e convicções. A testemunha é capaz de gerar novas conformações ao poder estabelecido, gerar a necessidade do aprimoramento da retórica e forçar um novo modelo de convencimento e persuasão baseado na apuração, memória e investigação.

    Feitas tais observações, nos cabe comentar as alternativas da questão.

    LETRA A- CORRETA. Testemunha é o termo adequado para a noção de inquérito de Foucalt

    LETRA B- INCORRETO. Não reproduz o pensar de Foucalt sobre inquérito

    LETRA C- INCORRETO. Não reproduz o pensar de Foucalt sobre inquérito

    LETRA D- INCORRETO. Não reproduz o pensar de Foucalt sobre inquérito

    LETRA E- INCORRETO. Não reproduz o pensar de Foucalt sobre inquérito

    GABARITO DO PROFESSOR: LETRA A


ID
2959855
Banca
FCC
Órgão
DPE-SP
Ano
2019
Provas
Disciplina
Filosofia do Direito
Assuntos

Referindo-se ao conceito de direito e à ideia de justiça em seu livro Teoria geral do direito e do Estado, Hans Kelsen afirma que “libertar o conceito de Direito da ideia de justiça é difícil porque ambos são constantemente confundidos no pensamento político não científico, assim como na linguagem comum, e porque essa confusão corresponde à tendência ideológica de dar aparência de justiça ao Direito positivo. [...] É uma tendência política, não científica”.

Tendo em conta a situação relatada no excerto acima, é correto afirmar que, para Hans Kelsen, do ponto de vista de uma “teoria pura do Direito”,

Alternativas
Comentários
  • Acertei usando do seguinte raciocínio: Se direito esta impregnado da ideia de justiça sendo difícil estabelecer os limites de ambos, e se a tendência ideológica é dar aparência de justiça ao direito, logo justiça corresponde a legalidade, ou seja, ao direito posto ao positivismo. Assim, "apenas com o sentido de legalidade é que a justiça pode fazer parte de uma ciência do Direito."

    Alternativa "A"

  • Para Hans Kelsen, a norma jurídica comporta mais de uma compreensão, competindo à Ciência do Direito apontar as alternativas que se abrem ao operador jurídico. A aplicação da norma seria um ato de política jurídica.21

    A perspectiva da Teoria Pura do Direito de Hans Kelsen, é possível distinguir uma ?jurisprudência? que trata da validade do Direito de outra que considera a eficácia do Direito. Jurisprudência normativa e jurisprudência sociológica.

    John Austin constitui-se como patrono do moderno positivismo jurídico ao apartar o Direito do ?contexto social?, o que foi seguido por Hart e Hans Kelsen ao escrever a Teoria pura do direito. Jhon Austin é o pai de Kelsen e Hart (brincadeira, mas bom para lembrar!) Não é John Rawl é John Austin; parecido com Austin Powers!!!

    Abraços

  • A questão fala sobre a JUSTIÇA PARA KELSEN :

    "Para Kelsen a validade das normas de direito positivo não depende da relação em que se encontram com a norma de justiça. Assim, o direito positivo vale enquanto tal, ou seja, retira a sua validade da objetividade, da norma posta. Em outras palavras, a sua validade se justifica no próprio sistema de normas positivas instituído (é um sistema fechado que encontra validade no próprio sistema e não em um ideal de justiça). Se uma norma entrou com regularidade no sistema jurídico, automaticamente ela retira dele ( do próprio sistema) a sua validade subjetiva, sendo desnecessário pedir a sua adequação a um ideal de justiça.

    Portanto, para Kelsen, não se pode deduzir de um ideal, que se quer absoluto, uma norma do dever-ser. O mundo do ser - de onde se pode pensar e imaginar um valor "universal" - não se comunica com o mundo do dever-ser. Logo, diante da perspectiva da ciência positiva surge o sentido do relativo, uma vez que para Kelsen, a ciência "não tem de decidir o que é justo, isto é, prescrever como devemos tratar os seres humanos, mas DESCREVER aquilo que de fato é valorado como justo, sem se identificar a si própria com um destes juízos de valor".

    -> LEGALIDADE É O ATRIBUTO JURÍDICO DE IDENTIFICAR SE DETERMINADO ATO ESTÁ OU NÃO DENTRO DO PERMITO PELO SISTEMA JURÍDICO. Nesse sentido, para Kelsen a justiça só pode existir em uma ciência do direito enquanto legalidade (Dentro do sistema normativo é descrito o que é justo para que determinado ato possa ser valorado como justo ou injusto. O que é justo deve estar posto no ordenamento para garantir a segurança jurídica )

  • Pessoal cuidado com comentário de Thais Moreira Santos, a linha de raciocínio dela está correta com relação somente à tendência ideológica (que justiça corresponde ao direito positivado), mas a questão pede a posição de Hans Kelsen e não é pelo motivo que nossa colega citou de conclusão lógica que é considerada a alternativa "a" correta, pois Hans Kelsen como diz também na questão, acredita exatamente de modo diverso: que justiça e direito não andam juntos do ponto de vista científico. Para Hans Kelsen a lei é válida e legal se foi positivada (aprovada), independente de ser justa ou não.

  • Gabarito: Letra "A" - "apenas com o sentido de legalidade é que a justiça pode fazer parte de uma ciência do Direito".

    Inicialmente cumpre ressaltar que, para Kelsen, "o problema da justiça é um problema ético e é distinto do problema jurídico de validade. Portanto, o problema do justo e do direito se colocam em planos diferentes para seu positivismo jurídico. [...]"

    Portanto, vimos que justiça e validade não conceitos distintos. Na linha do gabarito da questão, acrescenta-se que "é necessário que o conteúdo e algo que aconteça no mundo dos fatos coincida com o conteúdo de uma norma considerada válida [...]".

    Basta lembrar que Kelsen é o "louco" do positivismo e que só considerada o que está prescrito no ordenamento jurídico, se está fora, nem merece ser olhado.

    Fonte: Curso CEI de Filosofia do Direito e Sociologia Jurídica, Aula 04. O positivismo normativista de Hans Kelsen, pág. 05.

  • A questão em comento demanda conhecimento de Kelsen, da Teoria Pura do Direito e da sua ideia de Justiça.

    Em Kelsen, a Teoria Pura do Direito confere ao Direito cientificidade.

    O Direito, enquanto ciência, não deve se misturar com elementos da Moral, valores, acepções de Justiça.

    O Direito, enquanto Direito Positivo, não pode estar contaminado de aspirações metafísicas da Justiça.

    A Justiça está adstrita ao campo da política do Direito, ou seja, na aplicação, na interpretação do Direito.

    Teoria Pura do Direito e Justiça não se confundem. A Teoria Pura do Direito é uma construção científica e universal, ao passo que a Justiça é metafísica e oscilante, variando conforme padrões de cada ordenamento jurídico, suas conjunturas, suas nuances.

    Diante do exposto, vamos apreciar as alternativas da questão.

    LETRA A- CORRETO. Com efeito, o instante de apreciação da Justiça se dá, de fato, com a edição do sentido de legalidade, ou seja, quando cada ordenamento jurídico incorpora os elementos legais próprios do contexto, da conjuntura onde está inserido.

    LETRA B- INCORRETO. Kelsen não é um jusnaturalista e não pretende a Justiça como uma instância crítico reflexiva do Direito.

    LETRA C- INCORRETO. Confunde Direito e Justiça, e aposta apenas na Justiça como transcendente, não admitindo a Justiça no campo político, prático do Direito.

    LETRA D- INCORRETO. Incorre no mesmo vício da letra C, isto é, aposta na Justiça como apenas transcendente, não admitindo a Justiça no campo político, prático do Direito.

    LETRA E- INCORRETO. Incorre no mesmo vício da letra C, isto é, aposta na Justiça como apenas transcendente, não admitindo a Justiça no campo político, prático do Direito.

    GABARITO DO PROFESSOR: LETRA A

  • A assertiva "A" pode ser assim resumida: "Para Kelsen, se está na lei (direito posto) então está justo". Apenas dessa maneira pode-se conceber "Justiça e Direito" para a Teoria Pura do Direito de Kelsen.

    Lembre-se: Kelsen queria conferir o status de Ciência Autônoma para o Direito, de modo que o Direito deveria ter seu próprio método, e o método cunhado por Kelsen é a análise da norma jurídica como posta no ordenamento (daí o termo "positivismo").

    Discussões políticas, filosóficas, sociais, psicológicas não devem ser levadas em consideração quando da análise do que está contido na norma jurídica (para tal teoria). Sua fundamentação e validade dar-se-ia apenas no próprio ordenamento: uma norma será válida quando se fundamentar numa norma válida anterior (pirâmide de Kelsen).

    Kelsen não achava as discussões políticas, filosóficas, sociais, etc. como irrelevantes. Não é isso. Ele apenas quis criar uma Teoria PURA do Direito - ou seja, na qual pudesse-se estudar o Direito por si só, como Ciência Autônoma.


ID
2959858
Banca
FCC
Órgão
DPE-SP
Ano
2019
Provas
Disciplina
Filosofia do Direito
Assuntos

Considerando a crítica que Ronald Dworkin endereça ao positivismo jurídico no livro Levando os direitos a sério, é INCORRETO afirmar que, segundo o autor, para o positivismo: 

Alternativas
Comentários
  • D

    Jusnaturalismo é direitos antes das regras

    Abraços

  • Está incorreta a acertiva "D" porque Dworkin entende que a visão positivista dos direitos jurídicos se referem somente ao que está positivado (ou seja -escrito-aprovado), não existindo nada que possua validade além do direito positivado. Dworkin é um grande crítico do positivismo porque pensa ao contrário deles. E é exatamente isso que a questão cobra- o posicionamento de Dworkin quanto ao positivismo.

    Dworkin diz que os positivistas concebem um modelo de sistema de regras, onde os princípios não podem ser validados como pertencentes a este sistema - ou seja: somente as regras é que tem validade, não existindo direito jurídico pré-existente. Sendo que quando o juiz no positivismo deixar de aplicar alguma regra está usando de seu poder discricionário criando uma nova lei.

    Já, Dworkin, em todas suas obras defende que regras são diferente de princípios, porque regra se refere apenas ao que está escrito, (então quanto à regra, ou você aplica ou não aplica-"tudo ou nada") mas na hora de o juiz aplicar a lei ele deve se ater principalmente aos princípios, que são as razões de ser das regras. Então, se uma regra estiver contrariando algum princípio (que tem mais valor) o juiz pode utilizar de seu poder discricionário (mas deve para isso basear-se nos princípios que originaram a regra) e deixar de aplicar referida regra, com base num princípio, então neste caso, não estaria usando de sua livre discricionariedade, tampouco criando nova lei.

    São conclusões que tirei de estudos que já fiz sobre este filósofo, então me corrijam se eventualmente errei em algo

  • Para responder a questão, pensei que os princípios são as árvores; as regras, os frutos.

    Desta forma, para Dworkin, não há como o direito jurídico ser anterior a "qualquer legislação" como afirma a questão. Isto porque as regras são criadas para, em tese, efetivar os princípios e dar maior segurança jurídica.

    Por exemplo: as regras que tratam dos direitos dos idosos, deficientes, mulheres, crianças, etc, são elaboradas a partir do princípio da igualdade material.

    Espero ter ajudado.

  • Não tenho extensa leitura em Dworkin, mas meu método para responder a esta questão foi chute embasado. Como um dos fundamentos do positivismo é justamente a observância estrita do que está disposto na lei, não poderia nunca o positivismo aceitar que existem normas jurídicas sem que haja positivação em algum texto legislativo, pois isso é uma postura do jusnaturalismo.

  • Tomar o cuidado com o enunciado. Dworkin, como pós-positivista, questiona preceitos do positivismo, e a questão pede a compreensão dele em torno do positivismo (E NÃO DO PÓS-POSITIVISMO), mas a partir de uma assertiva INCORRETA. Vejam, ainda, que as alternativas A e E não poderiam ser assinaladas, em vista de que partem da mesma premissa e, ao fim e ao cabo, significam a mesma coisa, isto é, que a obrigação jurídica diz com a norma jurídica, desde que válida.

  • GABARITO: C.

    A fim de esboçar uma justificatica para o gabarito, segue o raciocínio.

    O positivismo jurídico clássico, que tem em Hans Kelsen o seu maior expoente, compreende a legitimação do ordenamento jurídico a partir de uma estrutura escalonada de normas, que extraem o seu parâmetro de validade das normas hierarquicamente superiores. Trata-se de uma perspectiva formal do Direito, em que sua validade circunscreve-se à obediância de normas hierarquicamente superiores, afastando-se a avaliação moral do Direito de seus critérios de legitimação.

    Para Kelsen, a legitimidade da Constituição é extraída de uma norma jurídica hipotética e transcendental, anterior ao próprio documento constitucional, cujo conteúdo mandatório seria "cumpra-se a Constituição", compreendida esta norma pressuposta como constituição em sentido lógico-jurídico. A Constituição, por sua vez, seria o ápice do ordenamento juídico, concebida como constituição em sentido jurídico-positivo, e todo o sistema jurídico nela se fundaria, e sua validade seria avaliada a partir da obediência aos parâmetros encartados nesta Constituição e no sistema hierarquizado de normas que ela estabelece.

    Assim, a juridicidade dos direitos é extraível da sua previsão, em um sistema formalmente estruturado, não se reconhecendo juridicidade a postulações que lhe sejam anteriores - a exemplo do jusnaturalismo -, ou mesmo sem previsão neste mesmo ordenamento.

    Oportuno destacar, ademais, que o positivismo jurídico clássico baseava-se no paradigma da completude do sistema, e supunha que da literalidade dos textos legais eram extraíveis todos os direitos, sendo eventuais omissões supríveis pela analogia ou costumes, a exemplo da previsão na LINDB. Essa postura legicêntrica do Direito, que marca o positivismo clássico, conclui que direitos jurídicos não preexistem a sua previsão legal, advindo desta a sua existência e exigibilidade.

  • A questão em comento demanda compreender axiomas de Ronald Dworkin.

    Reputado, em essência, como um pós positivista (embora o próprio autor rejeitasse rótulos de escolas), Dworkin é um rompimento com a tradição clássica do Positivismo, e, dentre outras construções, se destacou por enxergar o Direito como integridade e ver a necessidade do encadeamento de normas em princípios e regras.

    As regras fixam balizas, obrigações, comportamentos a serem seguidos de maneira mais direta, ao passo que os princípios fixam um estado ideal de coisas, otimizando perspectivas, ideais.

    Diante do breve exposto, cabe comentar as alternativas da questão (DESTACANDO QUE A RESPOSTA ADEQUADA É A ALTERNATIVA INCORRETA):

    LETRA A- CORRETA, LOGO NÃO RESPONDE A QUESTÃO. Há, de fato, uma correlação lógica entre obrigações jurídicas e regras jurídicas.

    LETRA B- CORRETA, LOGO NÃO RESPONDE A QUESTÃO. De fato, o direito de uma comunidade é estabelecido, segundo Dworkin, por regras especiais, conforme os critérios específicos desta comunidade.

    LETRA C- CORRETA, LOGO NÃO RESPONDE A QUESTÃO. Regras jurídico e o Direito, de fato, existem de forma concomitante, são coexistentes.

    LETRA D- INCORRETA, LOGO RESPONDE A QUESTÃO. Não é possível pensar em direitos sem uma legislação anterior, sem um estatuto jurídico próprio, sem alguma espécie plausível de regulação pela comunidade jurídica.

    LETRA E- CORRETA, LOGO NÃO RESPONDE A QUESTÃO. Já que regras e obrigações coexistem, se não temos uma regra específica, não surge obrigação específica.

    GABARITO DO PROFESSOR: LETRA D

  • ##Atenção: Ronald Dworkin explica que há três características centrais do positivismo jurídico: “ (a) O direito de uma comunidade é um conjunto de regras especiais utilizado direta ou indiretamente pela comunidade com o propósito de determinar qual comportamento será punido ou coagido pelo poder público. Essas regras especiais podem ser identificadas e distinguidas com auxílio de critérios específicos, de testes que não têm a ver com o seu conteúdo, mas com seu pedigree ou maneira pela qual foram adotadas ou formuladas. [...] (b) O conjunto dessas regras jurídicas é coextensivo com “o direito”, de modo que se o caso de alguma pessoa não estiver coberto por uma regra dessas (porque não existe nenhuma que pareça apropriada ou porque as que parecem apropriadas são vagas ou por alguma outra razão), então esse caso não pode ser decidido mediante a “aplicação do direito”. Ele deve ser decidido por alguma autoridade pública, como um juiz, “exercendo seu discernimento pessoal”, o que significa ir além do direito na busca por algum outro tipo de padrão [...]. e (c) Dizer que alguém tem uma “obrigação jurídica” é dizer que seu caso se enquadra em alguma regra jurídica válida que exige que ele faça ou se abstenha de fazer alguma coisa.” (DWORKIN, Ronald. Levando os Direitos a Sério. São Paulo: Martins Fontes, 2002, p. 50). No mesmo sentido, Luísa Giuliani Bernsts e Giovanna Dias explicam: “O debate Hart-Dworkin inicia, como visto, com as críticas formuladas pelo segundo autor, que delineou três principais preceitos comuns que fundamentam o positivismo jurídico na sua concepção. Primeiro, o Direito seria "um conjunto de regras especiais utilizado direta ou indiretamente pela comunidade com o propósito de determinar qual comportamento será punido ou coagido pelo poder público". Essas regras poderiam ser identificadas e distinguidas com o auxílio de critérios específicos, de testes que dizem respeito ao seu pedigree, também revelando-se coextensivas ao Direito. E aqui, chega-se ao segundo preceito: se algum caso não estiver coberto por uma das regras, ele poderá ser decidido por alguma autoridade pública, a partir de seu discernimento pessoal, oportunidade em que dispensada a aplicação do Direito. Finalmente, como terceiro preceito, na ausência de uma regra jurídica válida, não existiria obrigação jurídica, tendo em vista que a última apenas existe na ocorrência da primeira [4].” (Fonte: ) Mas atenção: Dworkin nega de forma categórica o positivismo jurídico e no início de sua obra, Taking Rights Seriously, defende uma teoria liberal do direito. Para ele o positivismo não admite a ideia de que os direitos jurídicos possam preexistir a qualquer forma de legislação e as regras seriam aplicadas na maneira “tudo ou nada”. (DWORKIN, 1978:24) (Fonte: ).


ID
2959861
Banca
FCC
Órgão
DPE-SP
Ano
2019
Provas
Disciplina
Filosofia do Direito
Assuntos

Relativamente ao que Ronald Dworkin afirma acerca das regras e dos princípios no livro Levando os direitos a sério, considere as assertivas abaixo.

I. As regras são aplicáveis à maneira do “tudo ou nada”, ou seja, dados os fatos que uma regra estipula, ou a regra é válida e neste caso deve ser aplicada, ou não é válida e neste caso não se aplica.
II. Os princípios enunciam razões que conduzem o argumento para uma certa direção.
III. Os princípios possuem uma dimensão de peso ou importância que as regras não têm.
IV. Se duas regras entram em conflito, apenas uma delas pode ser considerada válida.

Está correto o que se afirma em 

Alternativas
Comentários
  • Dworkin: filósofo do direito norte-americano; defendeu que o sistema jurídico não está restrito às leis e que os princípios não funcionam simplesmente como meio de integração.

    Dworkin: as normas podem ser classificadas como princípios, regras e políticas (policy) - estas metas a serem alcançadas, geralmente relacionados a um incremento econômico, político ou social de exigências da sociedade; isso as diferenciariam dos princípios, cuja observância decorre do fato de serem exigências da moralidade e não servir para incrementar bem-estar social - a categoria das policies não foi adotada por Alexy, autor cuja obra influenciou toda a tradição brasileira.

    Abraços

  • Princípios

    Constituem Mandados de

    Otimização

    Vários Graus de Concretização

    Convivência Conflitual: princípios

    coexistem

    Permitem Ponderação

    (Balanceamento)

    Regras

    Aplicação Tudo-ou-Nada

    (Dworkin: applicable in all-or-nothing fashion)

    Imperativos de Imposição, Permissão ou

    Proibição

    Convivência Antinômica: regras excluem-se

    Contêm fixações Normativas definitivas (a regra

    é válida ou não)

  • _ Regras: Aplicam-se por tudo ou nada. Verifica-se se a regra é válida ou inválida. Na colisão entre regras, uma delas deve ser considerada inválida.

     

    - Princípios: Possuem uma dimensão de peso. Na colisão entre princípios, sobrepõe aquele que possui maior dimensão de peso, sem que isso acarreta a perda da validade do outro princípio.

    Os princípios servem como instrumentos de auxílio para a decisão do julgador (norteiam a tomada de decisões, especialmente nos hard cases).

    Os princípios e as regras possuem características distintas, mas ambos são vinculantes e justificam a decisão de maneira mais adequada em determinados casos.

    Os princípios podem ser utilizados para quebrar o rigorismo da lei.

  • I - As regras são aplicáveis à maneira do “tudo ou nada”, ou seja, dados os fatos que uma regra estipula, ou a regra é válida e neste caso deve ser aplicada, ou não é válida e neste caso não se aplica.

    O fato de a norma não ser aplicada por não ocorrer o preenchimento de seu Tatbestand, não quer dizer que ela não seja válida. Isso só quer dizer que ela não é aplicável ao caso concreto.

    III. Os princípios possuem uma dimensão de peso ou importância que as regras não têm.

    O princípios "possuem" peso, sim. Tanto que Robert Alexy posteriormente desenvolve a teoria da "ponderação de princípios" com base nisso. Agora, daí para dizer que "princípios têm mais importância do que regras", acredito que seja um tanto quanto transbordar indevidamente a obra tanto de Dworkin, quanto de Alexy (aqui posso estar enganado).

    IV - Se duas regras entram em conflito, apenas uma delas pode ser considerada válida. 

    Válida em que sentido? Se for válida diante do caso concreto, aí uma é afastada para dar lugar a outra (e é por isso que existem os parâmetros da antinomia). Agora, se se trata de validade dentro do ordenamento jurídico, ou seja, uma norma inválida deve ser afastada do ordenamento, aí não. De qual caso estaria o examinar falando?

    GATO ESCALDADO TEM MEDO ATÉ DE ÁGUA FRIA.

    NEXT

  • Palestra no youtube sobre o assunto. https://www.youtube.com/watch?v=5AGAGSdsVVE II Jornada de Teoria do Direito: As Contribuições de Ronald Dworkin para o pensamento jurídico contemporâneo Realização: Grupo de Estudos Pura Teoria do Direito

    Veja em 1:10:00

  • Gabarito: Letra "E" - todas corretas.

    "[...] Dworkin afirma que as regras são aplicadas ao modo de "tudo ou nada", de modo que se a hipótese de incidência de uma regra é preenchida, ou a regra é válida e a consequência normativa deve ser aceita, ou ela não é considerada válida. (Item I). No caso de colisão entre essas regras, uma delas deve ser considerada válida (Item IV). Já o princípios não determinam uma decisão de forma absoluta, mas contém fundamentos que devem ser conjugados com outros fundamentos decorrentes de outros princípios (Item II). Portanto, possuem o que Dworkin chama de uma dimensão de peso (Item III), de modo que na colisão, um pode se sobrepor ao outro sem ocasionar uma perda de validade. A diferença é, portanto, do ponto de vista da estrutura lógica da norma".

    Fonte: Curso CEI de Filosofia do Direito e Sociologia Jurídica, Aula 07. O pós-positivismo: uma introdução Ronald Dworkin e Robert Alexy, pág. 05.

  • A questão em comento demanda conhecimento de Ronald Dworkin e das diferenciações entre princípios e regras.

    Cabe analisar cada uma das assertivas.

    A assertiva I está CORRETA.

    De fato, regras são aplicáveis na base do “tudo ou nada", isto é, na colisão de regras, uma delas é excluída.

    A assertiva II está CORRETA.

    Com efeito, os princípios fixam aspirações, estado de coisas, otimizam razões e argumentos para uma dada direção.

    A assertiva III está CORRETA.

    Princípios, havendo colisão, são ponderados. Um princípio não deixa de existir em relação a outro. Princípios são harmonizados. Regras não passíveis de ponderação, não tem dimensão de peso, e, no conflito, vigora o “tudo ou nada", ou seja, uma delas é invalidada.

    A assertiva IV está CORRETA.

    Conforme já exposto, já que regras vigoram na lógica do “tudo ou nada", ou seja, as regras, havendo confronto, geram um impasse onde apenas uma delas é validada.

    Logo, as assertivas I, II, III e IV são verdadeiras.

    Diante do exposto, cabe apreciar as alternativas.

    LETRA A- INCORRETA. Todas as assertivas estão verdadeiras.

    LETRA B- INCORRETA. Todas as assertivas estão verdadeiras.

    LETRA C- INCORRETA. Todas as assertivas estão verdadeiras.

    LETRA D- INCORRETA. Todas as assertivas estão verdadeiras.

    LETRA E- CORRETA. Todas as assertivas estão verdadeiras.

    GABARITO DO PROFESSOR: LETRA E


ID
2959867
Banca
FCC
Órgão
DPE-SP
Ano
2019
Provas
Disciplina
Filosofia do Direito
Assuntos

No livro Da divisão do trabalho social, Émile Durkheim estabelece uma relação entre direito e solidariedade social, na qual 

Alternativas
Comentários
  • "Considerado por muitos como o grande fundador das Ciências Sociais, Émile Durkheim nasceu na França em 1858 e lá viveu até sua morte, em 1917. Diretamente influenciado pelo  de , dedicou sua trajetória intelectual a elaborar uma ciência que possibilitasse o entendimento dos comportamentos coletivos. Sua grande preocupação era explicar os elementos capazes de manter coesa a nova sociedade que ia se configurando após a  e a 

    Durkheim determina o  como objeto central de investigação deste novo campo científico. Entende-se o fato social como uma ?coisa? que exerce força de coerção sobre os sujeitos, independente de sua vontade ou ação individual. O fato social se impõem na direção da sociedade para o indivíduo e se estabelece de forma a homogenizar e padronizar os comportamentos particulares, garantindo que sejam coletivos. Propositalmente, Durkheim chama o fato social de ?coisa? para ressaltar que ele é um objeto no sentido científico, isso é, algo que pode ser observado, definido e explicado pelo cientista social."

    Abraços

  • A solidariedade social para Durkheim se dá pela consciência coletiva, pois essa é responsável pela coesão (ligação) entre as pessoas.

    Para Durkheim, há dois tipos de solidariedade: a mecânica e a orgânica.

    Solidariedade Mecânica Solidariedade Orgânica

    Sociedades pré capitalistas (simples) Sociedades Capitalistas (complexas)

    Funções sociais semelhantes Funções sociais especializadas e interdependentes

    Sem significativa divisão do trabalho Divisão do trabalho complexa

    Coerção imediata, violenta e punitiva Coerção formal e mediada (Direito)

    Direito Punitivo Direito Restitutivo

    Economia simples e pouco diversificada Economias complexas e bastante diversificadas

    Há uma correlação entre os tipos de consciência que determina o tipo de solidariedade e de direito ( o direito simboliza a solidariedade social, que é a consciência coletiva, a noção coletiva maior ou menor - mais individualizada - do que é certo ou errado, da moral e da ética).

    O fato social é externo ao indivíduo. Ele é definido de acordo com as regras de cada sociedade ( Direito é um fato social e externo ao indivíduo), por exemplo a maneira como os indivíduos devem se vestir e se comportar em determinados eventos.

  • Para Durkheim,  fato social  são instrumentos sociais e culturais que determinam as maneiras de agir, pensar e sentir na vida de um indivíduo. Assim, o fato social o obriga a se adaptar às regras da sociedade, já que o indivíduo se vê no mundo ao nascer e deve se comportar segundo as regras pre-existentes.

    Portanto o direito é um fato social.

    O examinador automaticamente descartou as alternativas "A', "D" e "E", pois se a "B" for incorreta, tornará a "C" automaticamente correta e vice-versa, já que uma é o espelho da outra. Como o direito é fruto da coletividade e resposta só poderia ser a "B".

  • A solidariedade social é um fenômeno totalmente moral, que, por si, não se presta à observação exata, nem, sobretudo, à medida. Como a solidariedade social é um fenômeno que, por si, não é observável, Durkheim procurará um fato exterior que a simbolize. Esse símbolo visível é o direito. Através do direito, fato externo visível, consegue-se estudar a própria solidariedade.

    Durkheim encara o direito como o que ele chama de um fato social (aspectos da vida social que moldam ações individuais). Esses fatos sociais são externos aos indivíduos, possuindo um caráter de coerção.

  • Gabarito: letra B

    “o direito é concebido como um fato externo que simboliza a solidariedade social.”

    Para Durkheim, a harmonia social (o porquê de vivermos em sociedade) é explicada a partir da ideia de COESÃO SOCIAL (uma relação entre a consciência particular e a consciência coletiva).

    Para entender melhor a coesão social, Durkheim voltou seus estudos para a SOLIDARIEDADE (divide-se em 02 espécies: mecânica e orgânica), que nada mais seria como uma forma de explicar a autorregulação (convívio em sociedade) – como a consciência particular pode se alinhar com a coletiva.

    A SOLIDARIEDADE MECÂNICA estava ligada a uma “sociedade primitiva”, onde o direito é PRIMITIVO e PUNITIVO (como espécie de educação ao agressor, servindo este de exemplo para que os demais não sigam siga conduta transgressora).

    A SOLIDARIEDADE ORGÂNICA, por sua vez, está inserida em sociedade “mais complexas”, onde o direito é RESTRITIVO (porque existe uma relação de interdependência entre os cidadãos, logo o melhor caminho seria a recuperação, tendo em vista que a punição poderia trazer prejuízos para o retorno das atividades. Ex.: sem uma mão, não teria como voltar a fazer manutenção nas máquinas de uma fábrica).

    Assim, parte-se do pressuposto que a solidariedade se confunde com o direito.

    O direito é uma fato externo (aqui já é a teoria do fato social, propriamente dita, de Durkeim), porque trata-se de algo preexistente, no sentido de existir antes mesmo do nascimento do homem, influenciando e regulando condutas.

    Por isso que se fala que FATOS SOCIAIS são GERAIS, INDEPENDENTES, COERCITIVOS e EXTERNOS. São sociais, pois independem da manifestação individual.

  • O Direito é considerado um fato social externo (normas de conduta produzidas pela sociedade) com significativa força coercitiva, bem como é um conjunto de padrões de comportamento que orienta a ação social.

    Possui a função de ampliar o coletivo em relação ao indivíduo (produz uniformidade no coletivo), bem como apresenta-se como símbolo visível da solidariedade social.

  • Gente, alguem pode indicar um bom livro esquematizado de filosofia do direito para concursos onde se encontram essas teorias?

    Ou vcs estudam esses livros de filosofia um por um?

  • A questão em comento demanda conhecimento basilar da obra de Durkheim.

    Para tal autor, o Direito, com efeito, produz um fato social externo, reproduzindo normas de conduta produzidas socialmente, simbolizando padrões de conduta, isto tudo em nome da construção de uma solidariedade social.

    Em outras palavras, o Direito, uma produção social (portanto externo ao indivíduo), se impõe coercitivamente, e obtém coesão social.

    Feitas tais considerações, nos cabe comentar as alternativas da questão.

    LETRA A- INCORRETA. O Direito é um ícone, um símbolo da solidariedade social de Durkheim.

    LETRA B- CORRETA. Reproduz o pensar de Durkheim, ou seja, o Direito como fato social externo, alheio ao homem, produzindo coesão, solidariedade social.

    LETRA C- INCORRETA. O Direito produz solidariedade social em Durkheim.

    LETRA D- INCORRETA. Não é a solidariedade, mas sim o Direito que é concebido em Durkheim como fato externo.

    LETRA E- INCORRETA. Não é a solidariedade, mas sim o Direito que é concebido em Durkheim como fato externo.

    GABARITO DO PROFESSOR: LETRA B

  • Não estudei Durkheim mas estudei lógica


ID
3010885
Banca
FGV
Órgão
OAB
Ano
2019
Provas
Disciplina
Filosofia do Direito
Assuntos

Costuma-se dizer que o ordenamento jurídico regula a própria produção normativa. Existem normas de comportamento ao lado de normas de estrutura... elas não regulam um comportamento, mas o modo de regular um comportamento...

BOBBIO, Norberto. Teoria do Ordenamento Jurídico. São Paulo: Polis; Brasília EdUnB, 1989.


A atuação de um advogado deve se dar com base no ordenamento jurídico. Por isso, não basta conhecer as leis; é preciso compreender o conceito e o funcionamento do ordenamento. Bobbio, em seu livro Teoria do Ordenamento Jurídico, afirma que a unidade do ordenamento jurídico é assegurada por suas fontes.

Assinale a opção que indica o fato que, para esse autor, interessa notar para uma teoria geral do ordenamento jurídico, em relação às fontes do Direito.

Alternativas
Comentários
  • regulam um comportamento, mas o modo de regular um comportamento...

    BOBBIO, Norberto. Teoria do Ordenamento Jurídico. São Paulo: Polis; Brasília EdUnB, 1989.

    No mesmo momento em que se reconhece existirem atos ou fatos dos quais se faz depender a produção de normas jurídicas, reconhece-se que o ordenamento jurídico, além de regular o comportamento das pessoas, regula também o modo pelo qual se devem produzir as regras.

    alternativa A

  • A questão exige conhecimento acerca da obra “Teoria do Ordenamento Jurídico", de Norberto Bobbio. Conforme o autor, “fontes do direito" são aqueles fatos ou atos dos quais o ordenamento jurídico faz depender a produção de normas jurídicas (p. 45). A respeito das fontes, o que importa notar é que o ordenamento jurídico moderno, além de regular o comportamento das pessoas, regula também o modo pelo qual se devem produzir as regras. Assim, o ordenamento jurídico regula a própria produção normativa. Isso indica a existência de normas de comportamento ao lado de normas de estrutura. Além das normas dirigidas diretamente aos cidadãos, há grande número de normas que têm a finalidade de oferecer aos juízes instruções sobre o modo através do qual se devem produzir as normas individuais e concretas que são as sentenças. É justamente a presença e frequência dessas normas para a produção de outras normas (normas de estrutura) que constituem a complexidade do ordenamento jurídico. É nesse ponto que a teoria do ordenamento ultrapassa a teoria da norma, ao menos numa questão formal: enquanto a teoria da norma havia parado na consideração da norma como imperativos – no sentido de ordem de fazer ou não fazer – a teoria do ordenamento vai além e afirma que as normas de conduta são imperativos de primeira instância, mas há ainda as normas imperativas de segunda instância, que consistem em comandos de comandar.

    Portanto, conforme o autor, existem normas jurídicas denominadas “normas de estrutura" que, ao invés de regular diretamente o comportamento, regula o modo pelo qual se devem produzir as regras.

    Referência: BOBBIO, Norberto. Teoria do ordenamento jurídico. 6ª ed. Brasília: Editora UNB, 1995.

    Gabarito do professor: letra a.



  • TENTEM LER PRIMEIRO AS ALTERNATIVAS E DEPOIS O ENUNCIADO, GARANTO QUE VÃO CONSEGUIR RESPONDER SEM NUNCA TER LIDO UMA OBRA DE FILOSOFIA.

     

    GAB. A

  • Fiquei em dúvida entre a alternativa A e a D, porém marquei a D.

    Não entendi que foi a A porque contém o seguinte trecho"...além de regular o comportamento das pessoas...", mas no enunciado da questão está "...elas não regulam um comportamento,...".

    Não vi sentido no gabarito. Alguém poderia me explicar onde está meu erro?

  • adoro quando eu acho que é uma resposta, fico um tempão nela, ai depois mudo de ideia, marco outra e a que tinha pensado antes era a correta, aiai.....

  • A questão exige conhecimento acerca da obra “Teoria do Ordenamento Jurídico", de Norberto Bobbio. Conforme o autor, “fontes do direito" são aqueles fatos ou atos dos quais o ordenamento jurídico faz depender a produção de normas jurídicas (p. 45). A respeito das fontes, o que importa notar é que o ordenamento jurídico moderno, além de regular o comportamento das pessoas, regula também o modo pelo qual se devem produzir as regras. Assim, o ordenamento jurídico regula a própria produção normativa. Isso indica a existência de normas de comportamento ao lado de normas de estrutura. Além das normas dirigidas diretamente aos cidadãos, há grande número de normas que têm a finalidade de oferecer aos juízes instruções sobre o modo através do qual se devem produzir as normas individuais e concretas que são as sentenças. É justamente a presença e frequência dessas normas para a produção de outras normas (normas de estrutura) que constituem a complexidade do ordenamento jurídico. É nesse ponto que a teoria do ordenamento ultrapassa a teoria da norma, ao menos numa questão formal: enquanto a teoria da norma havia parado na consideração da norma como imperativos – no sentido de ordem de fazer ou não fazer – a teoria do ordenamento vai além e afirma que as normas de conduta são imperativos de primeira instância, mas há ainda as normas imperativas de segunda instância, que consistem em comandos de comandar.

    Portanto, conforme o autor, existem normas jurídicas denominadas “normas de estrutura" que, ao invés de regular diretamente o comportamento, regula o modo pelo qual se devem produzir as regras.

    Referência: BOBBIO, Norberto. Teoria do ordenamento jurídico. 6ª ed. Brasília: Editora UNB, 1995.

    Gabarito do professor do QC: Letra A.

  • Para Norberto Bobbio, fontes do direito são "fatos" ou "atos" dos quais o ordenamento jurídico faz depender a produção de normas jurídicas.

    letra A.

  • Gabriel, o erro não está em você. É a própria OAB que apenas quer tirar 260 de você a cada vez que faz a prova. A ideia é você reprovar sempre. Entendeu?

  • As respostas das questões de Filosofia sempre estão no enunciado. Observem abaixo:

    Costuma-se dizer que o ordenamento jurídico

    regula a própria produção normativa. Existem

    normas de comportamento ao lado de normas de

    estrutura... elas não regulam um

    comportamento, mas o modo de regular um

    comportamento...

    Resposta: A

    "No mesmo momento em que se reconhece existirem

    atos ou fatos dos quais se faz depender a produção de

    normas jurídicas, reconhece-se que o ordenamento

    jurídico, além de regular o comportamento das pessoas,

    regula também o modo pelo qual se devem produzir as

    regras.

  • LETRA A) No mesmo momento em que se reconhece existirem atos ou fatos dos quais se faz depender a produção de normas jurídicas, reconhece-se que o ordenamento jurídico, além de regular o comportamento das pessoas, regula também o modo pelo qual se devem produzir as regras.

    "Costuma-se dizer que o ordenamento jurídico regula a própria produção normativa. Existem normas de comportamento ao lado de normas de estrutura... elas não regulam um comportamento, mas o modo de regular um comportamento"...

  • Fiquei em dúvida entre a alternativa A e a D, porém marquei a D.

    Não entendi que foi a A porque contém o seguinte trecho"...além de regular o comportamento das pessoas...", mas no enunciado da questão está "...elas não regulam um comportamento,...".

    Não vi sentido no gabarito. Alguém poderia me explicar onde está meu erro?

    exatamente o que o Gabriel escreveu

    foi o que pensei e respondi D

  • Para Bobbio existe o objetivo de regular como serão produzidas as regras, o qual é chamado de ''normas de estrutura''

  • a resposta está na pgta

  • No gabarito da prova do concurso Prova da Ordem, está certa a letra C.

    Fiquei na duvida agora.

    Porque acredito que é a letra A.

  • Olá, colegas concurseiros!

    Passando pra deixar essa dica pra quem tá focado na prova da OAB.

    Serve tanto pra quem esta começando agora quanto pra quem já é avançado e só esta fazendo revisão.

     Baixe os 490 mapas mentais para prova da OAB.

    Link: https://go.hotmart.com/W62298174Y

     Estude 10 mapas mentais por dia.

     Resolva 10 questões sobre o assunto de cada mapa mental estudado.

    → Em 45 dias você terá estudado os 490 mapas e resolvido aproximadamente de 5000 questões.

    Faça esse procedimento e seu aproveitamento melhorará em até 85%!


ID
3021055
Banca
CESPE / CEBRASPE
Órgão
DPE-DF
Ano
2019
Provas
Disciplina
Filosofia do Direito
Assuntos

A respeito da estrutura da norma jurídica, dos modelos teóricos do direito e da interpretação jurídica, julgue o item seguinte.


Para Hans Kelsen, ao justificar o seu modelo de direito positivo, os fatos são o fundamento de validade das normas jurídicas.

Alternativas
Comentários
  • GABARITO ERRADO.

    A afirmação vai no sentido diametralmente oposto ao defendido por Hans Kelsen, em Teoria Pura do Direito: “Se o Direito é concebido como uma ordem normativa, como um sistema de normas que regulam a conduta de homens, surge a questão: O que é que fundamenta a unidade de uma pluralidade de normas, por que é que uma norma determinada pertence a uma determinada ordem? E esta questão está intimamente relacionada com esta outra: Por que é que uma norma vale, o que é que constitui o seu fundamento de validade? Dizer que uma norma que se refere à conduta de um indivíduo “vale” (é “vigente”), significa que ela é vinculativa, que o indivíduo se deve conduzir do modo prescrito pela norma. Já anteriormente num outro contexto, explicamos que a questão de porque é que a norma vale - quer dizer: por que é que o indivíduo se deve conduzir de tal forma - não pode ser respondida com a simples verificação de um fato da ordem do ser, que o fundamento de validade de uma norma não pode ser um tal fato. Do fato de algo ser não pode seguir-se que algo deve ser; assim como do fato de algo dever ser se não pode seguir que algo é. O fundamento de validade de uma norma apenas pode ser a validade de uma outra norma. Uma norma que representa o fundamento de validade de uma outra norma é figurativamente designada como norma superior, por confronto com uma norma que é, em relação a ela, a norma inferior.” (KELSEN, Hans. Teoria Pura do Direito. São Paulo Martins Fontes, 1999, p. 215).

    FONTE: CESPE

  • 182 ? Para Hans Kelsen, ao justificar o seu modelo de direito positivo, os fatos são o fundamento de validade das normas jurídicas.

    ERRADO: as normas encontram seu fundamento em uma norma superior, regressando até a norma hipotética fundamental. O ordenamento como um todo, por sua vez, encontra validade na eficácia social. Os fatos, de forma alguma, fundamentam as normas.

    Abraços

  • Então, a validade do direito positivo é extraída da norma fundamental, a qual não se trata de norma posta, mas de norma pressuposta e possui como função validar todo o ordenamento jurídico.

     

    O sistema jurídico para Kelsen é unitário, orgânico, fechado, completo e autossuficiente, nada falta para seu aperfeiçoamento. Normas hierarquicamente inferiores buscam seu fundamento de validade em normas hierarquicamente superiores.

     

    O ordenamento jurídico se resume a esse complexo emaranhado de relações normativas. Qualquer abertura para fatores externos ao direito comprometeria sua rigidez e completude, de modo que a norma fundamental desempenha esse papel importante de fechamento do sistema normativo escalonado.

     

    Assim, o ponto de apoio de todo o sistema jurídico positivista kelsiano está numa estrutura escalonada de normas, na qual a última aparece como norma fundamental, no ápice de uma pirâmide de relações normativas e contém o comando de que todos devem obediência a Constituição.

  • Alguns conceitos.

    Direito é ordem coativa da conduta humana.

    Fato Jurídico é ato + significado. Ato seria "A" mata "B" e significado seria "homicídio".

    Ato subdivide-se em objetivo e subjetivo.

    Objetivo é a interpretação dada pelo direito ao fato.

    Subjetivo é dotado de significado para os demais indivíduos.

  • Mesmo complexo, o ordenamento jurídico é uno ou unitário. Isso porque suas di- versas normas não ocupam um mesmo plano. Aqui Bobbio aplica a teoria da hierarquia das normas jurídicas de Hans Kelsen. Tal teoria permite caracterizar como uno o ordenamento jurídico, na medida em que toda norma desse ordenamento encontra seu fundamento de validade na norma hierarquicamente superior, numa cadeia que levará até à pressuposta norma hipotética fundamental. 

  • Com base na visão Kelsen, o fundamento de validade das normas é extraído dentro do próprio sistema jurídico, de modo que uma norma inferior extrai seu fundamento em uma norma superior, regressando até a norma hipotética fundamental.

    Mas o que vem a ser essa norma hipotética fundamental? É a Constituição no sentido lógico jurídico.

    Deve-se lembrar que Hans Kelsen distingue Constituição em sentido lógico jurídico e Constituição em sentido jurídico positivo, sendo que esta extrai daquela seu fundamento de validade.

    A Constituição em sentido jurídico positivo é a Constituição posta (positivada), ou seja, o documento constitucional, por exemplo, a CRFB/88. De seu turno, a Constituição em sentido lógico jurídico é a norma fundamental hipotética, que só existe em tese, ou seja, não é posta pelo Estado, mas sim pressuposta, fruto de uma convenção social.  

     

  • O fundamento é a norma hipotética fundamental.

  • Lembrar da "Pirâmide de Kelsen" (101 de Direito em qualquer faculdade), que consiste nas normas inferiores no na pirâmide retirarem o seu fundamento de validade das normas hierarquicamente superiores.

  • A questão em comento demanda conhecimento de Kelsen e seu sistema dinâmico de validade de normas.

    As normas jurídicas, em Kelsen, fazem parte de uma pirâmide jurídica (que tem no ápice a norma fundamental- pressuposta, oca, hipotética), de forma que as normas inferiores extraem validade das normas superiores.

    A validade das normas se funda na normas superiores, abstratas, e não em aspectos fáticos.

    Logo, a assertiva está incorreta.

    GABARITO DO PROFESSOR: ERRADO


ID
3021061
Banca
CESPE / CEBRASPE
Órgão
DPE-DF
Ano
2019
Provas
Disciplina
Filosofia do Direito
Assuntos

A respeito da estrutura da norma jurídica, dos modelos teóricos do direito e da interpretação jurídica, julgue o item seguinte.


Por autofundamentação constitucional do direito, não se deve compreender a hierarquização linear do sistema jurídico.

Alternativas
Comentários
  • GABARITO CERTO.

    A afirmação deve ser avaliada sob o paradigma do modelo sistêmico de descrição do direito que, a exemplo do modelo kelseniano, nega a existência de um fundamento de validade externo do direito, indicando a constituição como forma fundante do sistema jurídico a partir do acoplamento estrutural entre o direito e a política. O que não significa a consequência de hierarquização linear do ordenamento, já que a organização interna do sistema é complexa e circular. Nesse sentido, a explanação de Marcelo Neves: “A questão da autofundamentação apresenta-se, em primeiro lugar, no plano da dinâmica jurídica, no sentido da tradição kelseniana. Além da distinção entre legislação e jurisdição ou execução, torna-se, então, relevante a distinção entre constituição e lei. Dessa maneira, há uma duplicação do código "lícito/ilícito" mediante a introdução da diferença interna "constitucional/inconstitucional". A autofundamentação constitucional do sistema não deve, porém, levar a uma compreensão hierárquica linear do sistema jurídico, pois, embora as normas constitucionais sirvam como fundamento de validade das demais normas jurídicas, os sentidos normativos da Constituição dependem tanto da legislação quanto dos atos de interpretação-aplicação da Constituição. Portanto, a partir da própria autofundamentação constitucional do sistema jurídico, desenvolve-se uma hierarquia entrelaçada no processo dinâmico de concretização normativa.”

    FONTE: CESPE

  • Por autofundamentação constitucional do direito, não se deve compreender a hierarquização linear do sistema jurídico. (certo)

    Resumindo o texto da colega Maria Júlia: "A afirmação deve ser avaliada sob o paradigma do modelo sistêmico de descrição do direito que, a exemplo do modelo kelseniano, nega a existência de um fundamento de validade externo do direito, indicando a constituição como norma fundante do sistema jurídico a partir do acoplamento estrutural entre o direito e a política. O que não significa a consequência de hierarquização linear do ordenamento, já que a organização interna do sistema é complexa e circular."

  • 184 ? Por autofundamentação constituição do direito não se deve compreender a hierarquização do sistema jurídico.

    CORRETO: não necessariamente. Mas o que é autofundamentação? ?Só quando o direito passa a ser posto basicamente por decisões, ou seja, com a sua positivação na sociedade moderna, ele se torna permanentemente alterável. À decidibilidade e à mutabilidade do direito associa-se o problema de sua diferenciação funcional e autonomia na sociedade moderna. Diferenciado o direito da moral, superado, institucionalmente, o seu apoio nas noções jusnaturalistas, surge a questão de sua fundamentação como sistema operativamente autônomo? (NEVES, Marcelo. Entre Hidra e Hércules: princípios e regras constitucionais. São Paulo: Martins Fontes, 2013, p. 113-114). Autundamentação é a fundamentação em si próprio, descolando-se da necessidade de elementos externos.

    Abraços

  • "CORRETO: não necessariamente. Mas o que é autofundamentação? 'Só quando o direito passa a ser posto basicamente por decisões, ou seja, com a sua positivação na sociedade moderna, ele se torna permanentemente alterável. À decidibilidade e à mutabilidade do direito associa-se o problema de sua diferenciação funcional e autonomia na sociedade moderna. Diferenciado o direito da moral, superado, institucionalmente, o seu apoio nas noções jusnaturalistas, surge a questão de sua fundamentação como sistema operativamente autônomo' (NEVES, Marcelo. Entre Hidra e Hércules: princípios e regras constitucionais. São Paulo: Martins Fontes, 2013, p. 113-114). Autofundamentação é a fundamentação em si próprio, descolando-se da necessidade de elementos externos."

    bons estudos

  • EU PRECISO SER ESTUDADA PELA FILOSOFIA...PRECISO ENTENDER A MESMA..MAS NÃO SEI COMO...

  • Bate um desânimo quando você não entende a pergunta. Não entende a resposta. Não entende os comentários. Por que Cespe? Por que?
  • Por autofundamentação constitucional do direito, não se deve compreender a hierarquização linear do sistema jurídico. CERTO.

    Autofundamentação significa que ele está fundamentado em si mesmo dispensando qualquer forma de hierarquização linear.

    Hierarquização linear do sistema jurídico significa que a sua validade encontra fundamento externo, por exemplo: fundamentada no direito natural, entretanto, os sistema jurídico se fundamenta em si mesmo, pois a fundamentação é sistémica ou circular.

  • Autofundamentação do Direito se dá quando quando o Direito passa a ter como substrato de fundamentação suas próprias decisões, isto é, a partir de sua positivação, se diferencia de noções axiológicas, morais. O Direito, ao invés de fundamentações metafísicas, transcendentais, jusnaturalistas, se funda no normativo, no positivado. Esta é uma tese trabalhada por Marcelo Neves.

    Autofundamentação do Direito dispensa todo e qualquer processo de hierarquização linear do Direito, de tal maneira que, em palavras mais claras, o sistema normativo constitucional basta para fundamentar o Direito, inexistindo elementos externos necessários para este processo de fundamentação do Direito.

    A assertiva, portanto, está CORRETA.

    GABARITO DO PROFESSOR: CERTO

  • Sangue de jesus tem poder

ID
3021064
Banca
CESPE / CEBRASPE
Órgão
DPE-DF
Ano
2019
Provas
Disciplina
Filosofia do Direito
Assuntos

A respeito da estrutura da norma jurídica, dos modelos teóricos do direito e da interpretação jurídica, julgue o item seguinte.


Na concepção do direito como integridade, segundo Ronald Dworkin, a interpretação deve ser guiada pelas melhores regras para o futuro.

Alternativas
Comentários
  • Está certo sim Juarez, porque o backup (arquivos BKF e BKP) são compactados, e não apenas 'empacotados'.
  • Obrigado.
  • GABARITO ERRADO.

    O modelo conceitual do direito como integridade para Dworkin é especialmente interpretativo. Mas focado numa avaliação histórica, em cadeia, das melhores práticas jurídicas de uma comunidade, o que exige do intérprete a atenção a diversas fontes do direito e não que ele se oriente, pragmaticamente, pelas melhores regras para o futuro. Assim, o próprio Dworkin afirma: “O pragmatismo exige que os juízes pensem de modo instrumental sobre as melhores regras para o futuro. Esse exercício pode pedir a interpretação de alguma coisa que extrapola a matéria jurídica: um pragmático utilitarista talvez precise preocupar-se com a melhor maneira de entender a ideia de bem-estar comunitário, por exemplo. Uma vez mais, porém, um juiz que aceite o pragmatismo não mais poderá interpretar a prática jurídica em sua totalidade. O direito como integridade é diferente: é tanto o produto da interpretação abrangente da prática jurídica quanto sua fonte de inspiração. O programa que apresenta aos juízes que decidem casos difíceis é essencialmente, não apenas contingentemente, interpretativo; o direito como integridade pedelhes que continuem interpretando o mesmo material que ele próprio afirma ter interpretado com sucesso. Oferece-se como a continuidade - e como origem – das interpretações mais detalhadas que recomenda.” (DWORKIN, Ronald. Império do direito. São Paulo: Martins Fontes, 1999, p. 273). 

    FONTE: CESPE

  • Na concepção do direito como integridade, segundo Ronald Dworkin, a interpretação deve ser guiada pelas melhores regras para o futuro. (errado)

    O modelo conceitual do direito como integridade para Dworkin é especialmente interpretativo. Mas focado numa avaliação histórica, em cadeia, das melhores práticas jurídicas de uma comunidade, o que exige do intérprete a atenção a diversas fontes do direito e não que ele se oriente, pragmaticamente, pelas melhores regras para o futuro. (baseado no texto da colega Maria Júlia)

  • Dworkin: filósofo do direito norte-americano; defendeu que o sistema jurídico não está restrito às leis e que os princípios não funcionam simplesmente como meio de integração.

    Alexy: filósofo do Direito Alemão; defendeu que o Direito é constituído não somente por regras, mas também por princípios e diretrizes políticas.

    Abraços

  • "ERRADO: nem só para o futuro, nem só para o passado. A integridade nega que as manifestações do Direito sejam meros relatos factuais voltados para o passado, como quer o convencionalismo; ou programas instrumentais voltados para o futuro, como pretende o pragmatismo. Para o Direito como integridade, as afirmações jurídicas são, ao mesmo tempo, posições interpretativas voltadas tanto para o passado quanto para o futuro."

    bons estudos

  • A questão em comento demanda conhecimento basilar da doutrina de Ronald Dworkin.

    A interpretação não leva em conta apenas “regras", também abraçando princípios.

    A ideia de integridade não é pensada apenas com “regras para o futuro", mas sim considera o Direito como romance em cadeia, escrito e reescrito a cada dia, promovendo perene diálogo entre tradição e renovação.

    GABARITO DO PROFESSOR: ERRADO

  • GABARITO: ERRADO

    O direito como integridade nega que as manifestações do direito sejam relatos factuais do convencionalismo, voltados para o passado, ou programas instrumentais do pragmatismo jurídico, voltados para o futuro. Insiste em que as afirmações jurídicas são opiniões interpretativas que, por esse motivo, combinam elementos que se voltam tanto para o passado quanto para o futuro; interpretam a prática jurídica contemporânea como uma política em processo de desenvolvimento. Assim, o direito como integridade rejeita, por considerar inútil, a questão de se os juízes descobrem ou inventam o direito; sugere que só entendemos o raciocínio jurídico tendo em vista que os juízes fazem as duas coisas e nenhuma delas.

    Fonte: https://ambitojuridico.com.br/cadernos/direito-constitucional/teoria-da-integridade-uma-abordagem-da-sistematizacao-de-ronald-dworkin/

  • RONALD DWORKIN (Pós-positivismo)

    1. Direito deve ser visto como um instrumento de justiça (condição de bem-estar dos indivíduos);

    2. Fonte do Direito é social;

    3. Direito tem natureza argumentativa

    4. Direito está enraizado em preceitos morais.

    5. Interpretação deve ser evolutiva (história e interpretação andam juntas)

    6. Direito = princípios (sistema de peso) e regras (sistema do tudo ou nada)

    7. Princípios: critérios norteadores.

    8. Estratégia da ponte: os princípios NÃO têm convivência autônoma (uns sem os outros), são todos ligados por uma ponte.

    9. Teoria dos direitos como trunfos: visa a identificar (de forma antecedente) a probabilidade de violações dos direitos fundamentais, de forma a proteger os indivíduos de possíveis ameaças ao igual tratamento e ao igual respeito (liberdade igualitária), limitando políticas utilitaristas extremadas.


ID
3021067
Banca
CESPE / CEBRASPE
Órgão
DPE-DF
Ano
2019
Provas
Disciplina
Filosofia do Direito
Assuntos

A respeito da estrutura da norma jurídica, dos modelos teóricos do direito e da interpretação jurídica, julgue o item seguinte.


Para Herbert Hart, a principal característica da regra de reconhecimento é que ela seja referenciada pelo grupo social como norma dotada de autoridade.

Alternativas
Comentários
  • GABARITO CERTO.

    A sua ampla aceitação e referência como norma dotada de autoridade é o principal elemento da regra de reconhecimento no conceito de direito desenvolvido por Hart. Nas lavras do próprio autor: “A forma mais simples de remédio para a incerteza do regime das regras primarias é a introdução daquilo a que chamaremos uma regra de reconhecimento. Esta especificará algum aspecto ou aspectos cuja existência de uma dada regra é tomada como uma indicação afirmativa e concludente de que é uma regra do grupo que deve ser apoiada pela pressão social que ele exerce. A existência de tal regra de reconhecimento pode tomar uma qualquer de entre uma vasta variedade de formas, simples ou complexas (...) o que é crucial é o reconhecimento da referência ao escrito ou à inscrição enquanto dotado de autoridade, isto é, como o modo adequado à eliminação das dúvidas acerca da existência da regra. Onde exista tal reconhecimento, existe uma forma muito simples de regra secundária: uma regra para a identificação concludente das regras primárias de obrigação. (HART, Herbert. O conceito de direito. Lisboa: Fundação Calouste Gulbenkian, 1994, p. 104).

    FONTE: CESPE

  • Para Herbert Hart, a principal característica da regra de reconhecimento é que ela seja referenciada pelo grupo social como norma dotada de autoridade. (certo)

    Errei por julgar que o autor seria Max Weber...

  • John Austin constitui-se como patrono do moderno positivismo jurídico ao apartar o Direito do ?contexto social?, o que foi seguido por Hart e Hans Kelsen ao escrever a Teoria pura do direito. Jhon Austin é o pai de Kelsen e Hart (brincadeira, mas bom para lembrar!) Não é John Rawl é John Austin; parecido com Austin Powers!!!

    Abraços

  • A prova foi assim? Alternativas isoladas? Ou o QC está querendo aumentar o número de questões?

  • O sistema jurídico complexo é dotado de regras primárias e secundárias. As regras primárias são aquelas que impõem deveres, obrigações, enquanto que as secundárias se subdividem em:

    1) Regras de alteração: permitem a modificação ou extinção das regras;

    2) Regras de julgamento ou adjudicação: identificam os indivíduos competentes para julgar e ainda definem o processo a seguir;

    3) Regras de reconhecimento: fornecem critérios para a existência da regra.

    Observar a similitude com a norma fundamental de Kelsen. Mas, diferente de Kelsen, Hart apresenta a regra de reconhecimento como uma questão de fato, não hipotética, em que pode ser provada através da prática social, da aceitação geral.

  • A questão em comento demanda conhecimento da regra de reconhecimento e dos estudos de Herbert Hart.

    Para Hart, a regra de reconhecimento é aquela utilizada pelos utentes da linguagem jurídica para aceitar uma norma como aderente, usual e legítima na prática social destes aderentes.

    Logo, por óbvio, a regra de reconhecimento é dotada de referencial de autoridade dentro desta comunidade de usuários do Direito.

    GABARITO DO PROFESSOR: CORRETO


ID
3021070
Banca
CESPE / CEBRASPE
Órgão
DPE-DF
Ano
2019
Provas
Disciplina
Filosofia do Direito
Assuntos

Acerca do direito, da moral e das modernas teorias da justiça, julgue o item a seguir.


De acordo com Herbert Hart, é aceitável a concepção de que a maioria tem o direito moral de impor a todos a maneira de se viver, pois esse fato é compatível com os princípios democráticos.

Alternativas
Comentários
  • GABARITO ERRADO.

    A concepção de Herbert Hart é no sentido contrário, ou seja, a de que não se afigura legítimo e adequado aos princípios da democracia que a maioria possa ser guiada pelo “populismo moral” para impor seu modo de vida à minoria, o que violaria a liberdade individual. Assim, diz Hart: “Parece realmente fácil acreditar que a fidelidade aos princípios democráticos impõe a aceitação do que se pode chamar de populismo moral: a concepção de que a maioria tem o direito moral de determinar como todos devem viver. Esta é uma má interpretação da democracia, a ameaçar a liberdade individual (...) Quaisquer outros argumentos devem ser para a coerção da moralidade. Ninguém pensaria, ainda qual a moral popular fosse mantida por uma ‘esmagadora maioria’ ou marcada pelo tríplice estigma da ‘intolerância, indignação e repulsa’, que a fidelidade aos princípios democráticos requer sejam admitidos e sua imposição sobre a minoria justificada.” (HART, Herbert. Direito, Liberdade e Moralidade. Porto Alegre: Fabris, 1987, p. 95 e 97)

    FONTE: CESPE

  • Outro forte positivista: Herbert Hart; autor de ?o conceito de direito?. 

    Abraços

  • No capítulo "o soberano e o súdito", em o "o conceito de direito", Hart desconstrói esse modelo embasado no hábito, demonstrando que ele não é capaz de explicar dois aspectos: 1) continuidade ou persistência do direito ante ao desaparecimento do legislador ou daqueles que lhe prestavam obediência; 2) estatuto juridicamente insuscetível de limitações é incompatível com os órgãos legislativos modernos.

  • A questão em comento demanda conhecimentos de axiomas de Hart e da fixação de regras morais sobre a vida em comunidade e a democracia.


    Hart não preconiza uma visão simplesmente aritmética de maioria, de modo que a vontade da “maioria" sempre irá ditar a vida em sociedade. Isto, por certo, acabaria por inibir e cancelar o dissenso, tornando o dizer das minorias meramente simbólico, ou seja, não estaríamos a falar de uma verdadeira democracia.


    Em instante algum os estudos de Hart permitem que a iniciativa privada e a liberdade individual sejam suprimidas. Não há espaços para totalitarismos camuflados em Hart.

    Hart não dá espaço para que populismos morais retirem da minoria a potestade de fixar suas próprias diretrizes de vida.



    GABARITO DO PROFESSOR: ERRADO

  • HERBERT HART (Positivismo mais aberto que o de KELSEN)

    1. O reconhecimento social e a força das estruturas institucionais podiam influenciar (como condição) a validade do ordenamento jurídico

    2. A moral NÃO pode ser considerada critério de validade das normas jurídicas.

    3. Textura aberta e poder discricionário: as lacunas na lei são inevitáveis. O julgador poderá utilizar interpretações anteriores (jurisprudência, hard cases) ou ainda tomar uma nova decisão, com discricionariedade, criando uma alternativa de interpretação. O intérprete tem o papel de criar e recriar o direito.

    4. Regra de reconhecimento: confere unidade e sistematicidade ao ordenamento jurídico, identificando as regras jurídicas por meio de um teste de qualidade (teste de pedigree).


ID
3021094
Banca
CESPE / CEBRASPE
Órgão
DPE-DF
Ano
2019
Provas
Disciplina
Filosofia do Direito
Assuntos

A respeito das correntes sociológicas do direito, da eficácia jurídica, da função simbólica do direito e da opinião pública, julgue o item seguinte.


É correto afirmar que o símbolo do “dever ser”, como obrigação positivada na norma jurídica, é uma expectativa de comportamento estabilizada de modo contrafático.

Alternativas
Comentários
  • GABARITO CERTO.

    A estabilização de determinada expectativa de comportamento social é o fundamento da positivação de uma norma jurídica, o que gera como consequência a positividade do próprio direito em termos contrafáticos.

    Ou seja, a sua validade e eficácia passam a não depender mais do mundo fático para que o próprio sistema jurídico seja considerado autônomo.

    Segundo Luhmann, “As normas são expectativas de comportamento estabilizadas em termos contrafáticos.

    Seu sentido implica na incondicionabilidade de sua vigência na medida em que a vigência é experimentada e, portanto, também institucionalizada, independentemente da satisfação fática ou não da norma.

    O símbolo do dever ser expressa principalmente a expectativa dessa vigência contrafática, sem colocar em discussão essa própria qualidade — aí estão o sentido e a função do dever ser”.

    FONTE: CESPE

  • Aquele que é o ?ser? pensa já ser o ?dever-ser?;

    Aquele que é o ?dever-ser? pensa ainda ser o ?ser?, mas busca o ?dever-ser?

    Abraços

  • CORRETO: as normas jurídicas são independentemente e, muitas vezes, contrárias aos fatos reais. Elas exprimem um dever ser que resulta em uma consequência jurídica (normalmente uma sanção). De toda forma, elas geram uma expectativa de comportamento, que pode se confirmar ou não. Dizer, por fim, que as normas jurídicas são contrafáticas é verificar que sua validade independente dos fatos (do cumprimento efetivo da norma).

    fonte estratégia concursos

  • Norma jurídica = "dever ser", é prescritiva.

    Ciência jurídica = "ser", é descritiva.

    Norma Jurídica é contrafática, pois ainda que o homicídio ocorra, a norma continuará existindo.

    Norma Jurídica não é "verdadeira" ou "falsa", mas válida ou inválida.

  • "Do ponto de vista psicossocial, portanto, diz-se que o direito nada mais é que uma estrutura de reiteração contrafática de expectativas de condutas, mutuamente referentes e expressas de forma transpessoal ou generalizável (Luhmann, 1983/1985). Significa dizer, portanto, que o que distingue a linguagem do direito da de outros campos normativos é o fato de se expressar de forma abstrata em relação às vivências imediatas, elaborando modelos extensíveis a diversos campos de regulação, seja em relação à matéria, às pessoas, lugar etc., permanecendo válido ainda quando tais normas sejam desmentidas pelos fatos (contrafaticidade), enquanto as normas da moral e da religião são veículos de preferências e vivências localizadas, não universalizáveis formalmente além desses limites".

    bons estudos

  • Norma jurídica = "dever ser", é prescritiva.

    Ciência jurídica = "ser", é descritiva.

    Norma Jurídica é contrafática, pois ainda que o homicídio ocorra, a norma continuará existindo.

    Norma Jurídica não é "verdadeira" ou "falsa", mas válida ou inválida.

  • A questão em comento demanda compreensão da ontologia da norma jurídica.

    A norma jurídica não está no plano dos fatos.

    A norma jurídica está no plano das abstrações.

    A norma jurídica é um dever ser, uma projeção, um ideal de comportamentos.

    O escopo da norma jurídica é evitar comportamentos desviantes dos ideias por ela colimados.

    O objetivo da norma jurídica é brecar posturas contrafáticas a seus comandos.

    Logo, a assertiva está correta.

    GABARITO DO PROFESSOR: CERTO

  • Gabarito: CERTO

     

    Notem que as normas jurídicas, em regra, são independentemente e, em muitos casos, contrárias aos fatos do mundo real.

    Vejamos: Uma sanção, é uma expressão da norma jurídica. Temos aí um dever-ser com consequência jurídica.

    Esse dever-ser cria uma expectativa de comportamento, que pode se confirmar ou não a depender da realidade apresentada.

    Ainda segundo Kelsen, a Ciência Jurídica pode ser caracterizada como uma ciência normativa, na medida em que toma seu objeto como norma, e isto significa dizer que estamos trabalhando no plano do dever- ser, e não do ser, porque a norma pode ser conceituada como o sentido de um ato por meio do qual uma conduta é prescrita, permitida ou facultada, não se confundindo com o ato de vontade cujo sentido constitui. A norma é dever-ser e o ato de vontade que a põe é o ser. Esta distinção é um dado imediato da consciência.

    Nesse sentido, ser e dever-ser são categorias lógicas que se caracterizam dentro de uma perspectiva de compreensão do fenômeno jurídico. A formalidade, ou, mais especificamente, o caráter formal da norma, aparece no dever-ser.

    De certo, que dizer que as normas jurídicas são contrafáticas, é justamente a apresentação de sua validade independer dos fatos, ou seja, do cumprimento efetivo da norma.

    Logo, gabarito é CERTO.

  • revisar

  • O fato de a norma jurídica ter validade mesmo quando violada, considera ilegítima ou inadequada, permite elaborar a teoria do caráter contrafático do direito. Contrafático significa contrário aos fatos reais. As normas jurídicas são contrárias aos fatos reais em quatro sentidos;

    → Primeiro, como já constatamos, a norma continua válida mesmo quando está sendo violada. Por tal razão, as autoridades e os cidadãos devem cobrar seu respeito alegando sua validade, que persiste mesmo quando a norma contraria a realidade.

    → Segundo, a norma jurídica é contrafática porque exprime um dever ser que objetiva mudar a realidade social, transformar o comportamento dos homens e as relações sociais. O Direito contraria frequentemente os fatos sociais porque deseja que estes sejam alterados.

    → Terceiro, as normas jurídicas têm uma função contrafática indireta, mas não menos importante. Manifestam a vontade de manutenção da atual situação, ou seja, das instituições políticas, das relações sociais e das posições dos indivíduos. Por isso afirma-se que o direito assume um papel conservador.

    → Quarto, o direito é contrafático porque seus mandamentos são válidos mesmo quando contrariam a lógica e o senso comum. Mesmo quando contrariam a realidade e suas tendências.


ID
3119338
Banca
FCC
Órgão
Câmara de Fortaleza - CE
Ano
2019
Provas
Disciplina
Filosofia do Direito
Assuntos

A teoria de Kelsen é "pura" em dois sentidos: (i) afirma-se livre de quaisquer considerações ideológicas, não se emitem juízos de valor sobre qualquer sistema jurídico, e a análise da "norma jurídica" não é afetada por nenhuma concepção da natureza do direito justo; (ii) o estudo sociológico da prática do direito e o estudo das influências políticas, econômicas ou históricas sobre o desenvolvimento do direito ficam além da esfera de ação da teoria pura. [...] Para Kelsen, as regras eram as características observáveis (na escrita etc.) de um sistema normativo. As regras eram, portanto, as características de superfície do direito, e as normas sua essência interior; conquanto elas possam ter dado origem aos atos de "vontade" de um Parlamento, ou à adoção de um costume por um juiz, uma vez aceitas como direito adquirem existência independente; sua validade não depende da vontade de um mandatário.

(MORRISON, Wayne. Filosofia do Direito: dos gregos ao pós-modernismo. São Paulo: Martins Fontes, 2006, p. 381, 382 e 392)


Considere as proposições abaixo acerca do texto:

I. O direito natural continua a fundamentar uma teoria pura do direito, ou seja, é base do direito positivo (norma jurídica).

II. O direito é perspectivado internamente por Kelsen e a norma jurídica é compreendida como uma idealidade, ou seja, um dever-ser, e não como tudo que é da natureza, ou seja, um ser.

III. Comporta a teoria de Kelsen uma validação da norma jurídica inferior pela norma jurídica superior, não cabendo, portanto, uma validação externa, de cunho sociológico.


Está correto o que se afirma APENAS em:

Alternativas
Comentários
  • A questão em comento requer conhecimentos basilares de Kelsen e sua teoria normativista, bem como do pequeno texto de introito da questão.

    Para entender Kelsen, é preciso ter em mente o seguinte:


    I-                    Kelsen é um positivista;

    II-                  Kelsen acredita que o Direito é um conjunto de normas. Logo, é visto como um normativista;

    III-                 A Teoria Pura do Direito é a forma como Kelsen confere ao Direito cientificidade, retirando de suas entranhas tudo o que lhe for estranho;

    IV-                Enquanto ciência, o Direito é puro, mas na política do Direito, isto é, no momento da prática e aplicação do Direito, pode receber caracterizações morais, sociológicas, tudo dentro do contexto onde está inserido e do esquadro, da moldura permitida pela Constituição.



    Diante destas exposições, é possível analisar as assertivas da questão.

    A assertiva I é FALSA. Kelsen é um positivista. Kelsen é refratário ao Direito Natural. A Teoria Pura do Direito, em momento algum, se confunde com o Direito.

    A assertiva II é VERDADEIRA. Kelsen fixa o Direito como ciência regida pela imputação, e não pela causalidade. As normas jurídicas são abstratas, não são empíricas. As normas jurídicas constituem dever ser.

    A assertiva III é VERDADEIRA. De fato, o sistema jurídico kelseniano faz com que as normas jurídicas busquem validade na pirâmide normativa, ou seja, o processo de criação, validade e aplicação de normas é pautado na lógica de que normas inferiores são compatíveis com normas superiores. Não há elementos externos, sociológicos, a firmar validade das normas no sistema kelseniano, de matiz lógico formal.


    Diante do exposto, as assertivas II e III são VERDADEIRAS.

    Nos cabe, pois, analisar as alternativas.

    LETRA A- INCORRETO. Em verdade, as assertivas II e III são verdadeiras.

    LETRA B- INCORRETO. Em verdade, as assertivas II e III são verdadeiras.

    LETRA C- INCORRETO. Em verdade, as assertivas II e III são verdadeiras.

    LETRA D- INCORRETO. Em verdade, as assertivas II e III são verdadeiras.

    LETRA E- CORRETO. Em verdade, as assertivas II e III são verdadeiras.




    GABARITO DO PROFESSOR: LETRA E

  • ASPECTOS FUNDAMENTAIS DO POSITIVISMO

    Validade jurídica norma juridicamente válida

    Lógica jurídica a ordem jurídica é estruturada cientificamente como um sistema jurídico

    Previsibilidade a disposição das normas num todo ordenado define o modus operandi do órgão estatal responsável pela aplicação da norma

    Segurança jurídica observância dos elementos de validade da norma garantem a segurança jurídica

    HANS KELSEN 1881 1973

    Corrente de pensamento Normativismo jurídico

    Obra principal Teoria Pura do Direito 1934

    Objetivo: análise e fundamentação do Direito como ciência

    PARA KELSEN: Justiça é um valor inconstante, relativo, dissolúvel e mutável

    Validade jurídica prepondera sobre o que é justo

    Justiça e legalidade estão em planos distintos, a primeira no campo da ética, a segunda no campo do direito.

    SLIDES AULA PROF ODAIR GRANCURSOS

  • Olá!

    Gabarito: E

    Bons estudos!

    -O sucesso é a soma de pequenos esforços repetidos dia após dia.

  • O pensamento de Hans Kelsen de baseia em NORMA PURA ele é positivista, logo toda vez que disser que a norma se baseia em outra coisa.. ignooooreeeeeeee! Lembre-se: norma é DEVER SER e não SER

    ("ser" é o mundo dos fatos, é a realidade,mas a norma nos traz uma perspectiva do mundo do "dever ser", isto é, como as coisas deveriam funcionar. Pronto, metade das questões desse jurista maravilhoso resolvidas :D


ID
3122851
Banca
FGV
Órgão
OAB
Ano
2019
Provas
Disciplina
Filosofia do Direito
Assuntos

Um juiz pode dar uma sentença favorável a uma querelante com um rostinho bonito ou proveniente de determinada classe social, na realidade porque gosta do rosto ou da classe, mas ostensivamente pelas razões que apresentar para sua decisão.

Neil MacCormick


Existem diferentes motivos pelos quais uma decisão é tomada, segundo MacCormick. Alguns argumentos podem ser até mesmo inconfessáveis, porém, de qualquer forma, a autoridade que decide precisa persuadir um auditório quanto à sua decisão.

Assinale a opção que, segundo Neil MacCormick, em seu livro Argumentação Jurídica e Teoria do Direito, apresenta a noção essencial daquilo que a fundamentação de uma decisão deve fazer.

Alternativas
Comentários
  • GABARITO: A

    Questão que trata da hermenêutica Jurídica: Professor de filosofia política escocês, Neil MacCormick (1941-2009), interessante, mas pouco lido no Brasil, sobretudo nos cursos de graduação em Direito.

    A citação pode ser ignorada. O enunciado observa a importância de uma boa argumentação. Argumentar significa justificar, fundamentar. Ainda que o candidato nunca tenha lido este professor escocês, poderia usar o raciocínio lógico. Boas razões = justificação = argumentação.

  • A questão exige conhecimento acerca das contribuições do jusfilósofo Neil MacCormick, em seu livro Argumentação Jurídica e Teoria do Direito. MacCormick, ao analisar as decisões judiciais prolatadas pelos tribunais inglês e escocês, constrói os pilares de sua teoria, afirmando a necessidade de uma justificação bem fundamentada.

    Logo no preâmbulo, MacCormick aponta a função precípua de sua obra: explicar a argumentação jurídica, considerando-a como uma ramificação da argumentação prática. Tal explanação é realizada com base na análise de jurisprudências britânicas, principalmente inglesas e escocesas, eis que o processo decisório no Reino Unido é vantajoso por envolver a prática de cada juiz apresentar seu parecer publicamente, engajando-se todos em uma discussão pública em si, que leva a uma exposição das melhores razões decisórias. O autor explica que seu estudo é fundado na visão do processo de argumentação como um processo de justificação, afinal, toda a decisão deve ser justificada com bons argumentos.

    Portanto, segundo Neil MacCormick, em seu livro Argumentação Jurídica e Teoria do Direito, a ideia que apresenta a noção essencial daquilo que a fundamentação de uma decisão deve fazer é: dar boas razões ostensivamente justificadoras em defesa da decisão, de modo que o processo de argumentação seja apresentado como processo de justificação.

    Gabarito do professor: letra a.

    Referências:

    MACCORMICK, Neil. Argumentação jurídica e teoria do direito. São Paulo: Martins Fontes, 2006.

    OLIVEIRA, Nayla Soares de; MORAIS, Evilanne Brandão de. Teoria da argumentação jurídica de Neil MacCormick. 2013. Disponível em: <https://jus.com.br/artigos/23733/teoria-da-argumentacao-juridica-de-neil-maccormick>. Acesso em: 04 dez. 2019.



  • Reparei que em algumas questões, algumas palavras que tem no enunciado podem ser encontradas em apenas uma das alternativas, no caso na correta.

  • gab. A

    Explicação:A falta de argumentação jurídica pelos operadores do direito, mostra a deficiência do conhecimento argumentativo pelo qual passamos na atualidade.

  • Resposta no próprio enunciado já que para esse jusfilósofo a argumentação como forma de justificar de forma ostensiva suas razões era a maneira que era defendida por ele. Gabarito A

  • CORRELAÇÃO:

    LETRA A) Dar boas razões ostensivamente justificadoras em defesa da decisão, de modo que o processo de argumentação seja apresentado como processo de justificação.

    "Um juiz pode dar uma sentença favorável a uma querelante com um rostinho bonito ou proveniente de determinada classe social, na realidade porque gosta do rosto ou da classe, mas ostensivamente pelas razões que apresentar para sua decisão".

  • A opção correta é a Letra "A"

  • Para Neil MacCormick os argumentos são os que tem a função de justificar as decisões

  • Duas questões perdidas na prova! Matéria que não serve para nada.

  • Hermenêutica Jurídica uma matéria de extrema valia, para a construção de pareceres de qualidade, não entendo como existem pessoas que classificam esta disciplina como inútil, ao invés de reclamar, deveriam ir estudar, talvez assim se tornassem melhores profissionais.

  • Comentário do Professor do QC:

    A questão exige conhecimento acerca das contribuições do jusfilósofo Neil MacCormick, em seu livro Argumentação Jurídica e Teoria do Direito. MacCormick, ao analisar as decisões judiciais prolatadas pelos tribunais inglês e escocês, constrói os pilares de sua teoria, afirmando a necessidade de uma justificação bem fundamentada.

    Logo no preâmbulo, MacCormick aponta a função precípua de sua obra: explicar a argumentação jurídica, considerando-a como uma ramificação da argumentação prática. Tal explanação é realizada com base na análise de jurisprudências britânicas, principalmente inglesas e escocesas, eis que o processo decisório no Reino Unido é vantajoso por envolver a prática de cada juiz apresentar seu parecer publicamente, engajando-se todos em uma discussão pública em si, que leva a uma exposição das melhores razões decisórias. O autor explica que seu estudo é fundado na visão do processo de argumentação como um processo de justificação, afinal, toda a decisão deve ser justificada com bons argumentos.

    Portanto, segundo Neil MacCormick, em seu livro Argumentação Jurídica e Teoria do Direito, a ideia que apresenta a noção essencial daquilo que a fundamentação de uma decisão deve fazer é: dar boas razões ostensivamente justificadoras em defesa da decisão, de modo que o processo de argumentação seja apresentado como processo de justificação.

    Gabarito do professor: letra a.

    Referências:

    MACCORMICK, Neil. Argumentação jurídica e teoria do direito. São Paulo: Martins Fontes, 2006.

    OLIVEIRA, Nayla Soares de; MORAIS, Evilanne Brandão de. Teoria da argumentação jurídica de Neil MacCormick. 2013. Disponível em: <https://jus.com.br/artigos/23733/teoria-da-argumentacao-juridica-de-neil-maccormick>. Acesso em: 04 dez. 2019.

  • a resposta das questões de filosofia tá no próprio enunciado, é só saber interpretar. dois pontinhos de mão beijada.

  • Olá, colegas concurseiros!

    Passando pra deixar essa dica pra quem tá focado na prova da OAB.

    Serve tanto pra quem esta começando agora quanto pra quem já é avançado e só esta fazendo revisão.

     Baixe os 490 mapas mentais para prova da OAB.

    Link:  https://go.hotmart.com/W62298174Y

     Estude 10 mapas mentais por dia.

     Resolva 10 questões sobre o assunto de cada mapa mental estudado.

    → Em 45 dias você terá estudado os 490 mapas e resolvido aproximadamente de 5000 questões.

    Faça esse procedimento e seu aproveitamento melhorará em até 85%!


ID
3146407
Banca
MPE-GO
Órgão
MPE-GO
Ano
2019
Provas
Disciplina
Filosofia do Direito
Assuntos

Com base nas lições trazidas pelo pós-positivismo e pelo neoconstitucionalismo, informe o item incorreto:

Alternativas
Comentários
  • Em síntese conclusiva, pode-ser dizer que as abordagens pós-positivistas pressupõem uma conexão necessária entre entre direito e moral, enquanto as abordagens neoconstitucionalistas reconhecem que, em determinados modelo constitucionais, existe uma conexão contingente entre as duas esferas. 

    -

    Neoconstitucionalismo: cunhado por Susanna Pozzolo em 1997, numa conferência proferida no XVIII Congresso Mundial de Filosofia Jurídica e Social realizado na Argentina, o termo neoconstitucionalismo ganhou projeção a partir das coletâneas organizadas por Miguel Carbonell. Na doutrina constitucional brasileira, diversos trabalhos foram dedicados ao estado do tempo na última década.

    Neoconstitucionalistas: ?mais ponderação do que subsunção?.

    O Neoconstitucionalismo valoriza a dimensão jurídica da Constituição Federal, enquanto para o Novo Constitucionalismo Democrático Latino Americano a busca da legitimidade democrática se dá pela maior e mais efetiva participação popular.

    Neoconstitucionalismo: b.1) teórico; b.2) ideológico; b.3) metodológico.

    Abraços

  • a) CORRETO. Segundo MARCELO NOVELINO (2016, p. 61), “O pós-positivismo, como via intermediária para a superação das concepções jusnaturalistas e juspositivistas, tem pretensão de universalidade, ou seja, pretende desempenhar o papel de autêntica teoria geral do direito aplicável a qualquer tipo de ordenamento jurídico, independentemente de suas características específicas. (…) O neoconstitudonalismo, por seu turno, tem pretensões mais especificas. Trata-se de uma teoria particularmente desenvolvida para explicar e dar conta das complexidades que envolvem um modelo específico de organização político-jurídica (o Estado constitucional democrático) e de constituição (“constituições normativas e garantistas”).”

    b) ERRADO. O examinador se fundamentou, novamente, em MARCELO NOVELINO. Segundo o constitucionalista (2016, p. 62), “as diferentes aspirações do pós-positivismo (teoria universal) e do neoconstitucionalismo (teoria particular) também se revelam nos planos em que a moral atua como uma instância crítica de valoração do direito. Na visão pós-positivista a normatividade dos princípios e a centralidade da argumentação jurídica são determinantes para se pensar o direito e a moral como esferas complementares e não totalmente autônomas. Embora sejam reconhecidas as especificidades de cada uma delas, não se admite um tratamento segmentado. Ao pressupor uma “conexão necessária” entre direito e moral, o pós-positivismo rompe com dualismo, característico das concepções juspositivistas, entre as duas esferas”. Em seguida, leciona: “O neoconstitucionalismo, por seu turno, adota uma premissa distinta ao considerar que a conexão identificativa entre direito e moral resulta da incorporação dos valores morais nas constituições por meio de princípios e direitos fundamentais, pontes necessárias entre as duas esferas. Sob esta óptica, a conexão entre o direito e a moral é apenas contingente – e não necessária, como no pós-positivismo -, isto é, decorre da incorporação de princípios morais nas constituições contemporâneas.”

    c) CORRETO. Segundo BARROSO (2009, p. 327-9), a existência dos princípios e seu eventual reconhecimento pela ordem jurídico não são novidades das últimas décadas, conforme se observa do direito romano. Contudo, o que há de singular no pós-positivismo é o reconhecimento da normatividade dos princípios.

    d) CORRETO. Aponta MARCELO NOVELINO (2016, p. 58), ao tratar do pós-positivismo ético, que “como postura ideológica perante o direito vigente, o pós-positivismo pode ser visto como uma via intermediária que busca preservar a segurança jurídica, mas sem adotar uma visão cética em relação à justiça material. O caráter prima fade atribuído à segurança jurídica, em casos extremos, pode ser afastado em nome da justiça”.

    Fonte: encurtador.com.br/abzEH

  • Neoconstitucionalismo: a conexão entre direito e moral é contingente.

    Pós-positivismo: a conexão entre direito e moral fica mais estreita, não sendo mera contingência causada pela positivação de valores morais no texto da Constituição.

    É interessante a posição do Lenio Streck, que caminha junto com Kelsen, ao alertar que a moral pode se transformar em uma predadora do direito e das garantias fundamentais.

    Toda essa "treta" relativa a teoria geral do direito ficou muito evidente no julgamento da questão da execução provisória da pena a partir da condenação em segunda instância.

  • Quando nem com os comentários dos colegas você consegue entender direito a questão, faz o quê?

  • Talvez com o significado da palavra contingente fique mais fácil entender. Vejamos:

    contingente

    con·tin·gen·te

    adj m+f

    1 Que pode, ou não, suceder ou existir; duvidoso, eventual, incerto.

    2 Que pode acontecer a alguém ou tocar-lhe por sorte; casual, fortuito, aleatório.

    3 Que, entre muitos, compete a cada um.

    4 FILOS Que não é necessário ou essencial, mas depende das circunstâncias.

    5 FILOS Na escolástica, diz-se daquilo que, apesar de possível em si, pode, não obstante, ser necessário em relação a outra coisa, isto é, àquilo que o faz ser.

    6 FILOS Para a filosofia contemporânea, diz-se daquilo que é livre, imprevisível e não determinado, e que, como tal, existe ou age no mundo natural.

    sm

    1 Parte que cabe a cada um quando se procede a uma distribuição; cota, fração, parcela, quinhão.

    2 Delimitação quantitativa; número, quantidade.

    3 Grupo de indivíduos que têm determinada função a desempenhar; agrupamento, coletividade, conjunto.

    4 MIL Conjunto organizado de soldados que se destaca de um batalhão para realizar uma diligência ou missão temporária; destacamento.

    5 FILOS O que não é necessário ou essencial.

    ETIMOLOGIA

    lat contingens-ntis.

    Fonte:

    ***

    Em outras palavras: para o neoconstitucionalismo a ligação entre direito e moral não é essencial, e não para os pós-positivistas, como a questão propõe.

    O correto seria: Segundo as abordagens neoconstitucionalistas, a conexão identificativa entre direito e moral é de caráter contingente, ou seja, ela é fruto da incorporação de valores morais nas constituições.

    Os pós-positivistas, ao contrário, atestam que a conexão entre essas duas esferas é necessária e independe de seu reconhecimento por meio da inscrição de princípios morais nas constituições.

  • A)C Enquanto o neoconstitucionalismo pretende ser uma teoria particular, aplicável a um modelo específico de organização política-jurídica (Estado constitucional democrático) e de Constituição, o pós-positivismo, a partir da descrença na neutralidade do sujeito, postula superar o positivismo metodológico e desempenhar o papel de autêntica teoria geral do direito aplicável a qualquer tipo de ordenamento jurídico, independentemente de suas características específicas, porquanto, as teorias pós-positivistas não foram desenvolvidas para o modelo de Constituição do Estado Constitucional democrático.” André Rufino do Vale, Aspectos do neoconstitucionalismo, p. 68-69, Revista Brasileira de Direito Constitucional – RBDC.

    B)E (GABARITO) Os conceitos vieram invertidos. Relação entre direito e moral:

    ÓTICA PÓS-POSITIVISTA:A normatividade entre os princípios e a centralidade da argumentação jurídica e dos direitos fundamentais são determinantes para que o direito e a moral sejam pensados não como esferas autônomas, mas complementares, posto que, ao pregar uma conexão necessária entre os dois, rompe com o dualismo entre as duas esferas defendido pelo positivismo metodológico. ” Assim, para o pós-positivismo, a conexão entre direito e moral é de caráter necessário e independente do seu reconhecimento por meio da inscrição de princípios morais nas constituições.

    ÓTICA NEOCONSTITUCIONALISTA: “Parte de uma premissa distinta. A conexão identificada entre direito e moral é decorrente da incorporação dos valores morais nas constituições por meio de princípios constitucionais e direitos fundamentais, pugnando que a conexão entre ambos é contingente (aquilo que pode ou não ocorrer; incerto) – e não necessária, como no pós-positivismo, isto é, uma relação decorrente das características das constituições contemporâneas, nas quais os princípios morais são incorporados pelo direito.”

    André Rufino do Vale, Aspectos do neoconstitucionalismo, p. 68-69, Revista Brasileira de Direito Constitucional – RBDC.

    C)C - O pós-positivismo traz a ideia de que princípios são espécies do gênero norma jurídica, juntamente com as regras. Por isso, está correto afirmar que a normatividade dos princípios compõe um dos aspectos centrais do pós-positivismo.

    D)C - No positivismo a questão central girava em torno da segurança jurídica, assim, o aplicador do direito, quando aplicava a lei, não analisava se os efeitos da aplicação da lei eram bons ou ruins, justos ou injustos. Já no pós-positivismo defende-se a ideia de que “há ou deve haver um relacionamento entre o direito e a justiça (moral). “O pós-positivismo busca ir além da legalidade estrita, mas não despreza o direito posto; procura empreender uma leitura moral do Direito, mas sem recorrer a categorias metafísicas. A interpretação e aplicação do ordenamento jurídico hão de ser inspiradas por uma teoria de justiça, mas não podem comportar voluntarismos ou personalismos, sobretudo os judiciais” (BARROSO, 2007, p. 22, grifo nosso).

  • "Os neoconstitucionalistas, ao contrário, atestam que a conexão entre essas duas esferas é necessária e INDEPENDE de seu reconhecimento por meio da inscrição de princípios morais nas constituições."

    O erro está aí em negrito. Não sei se é o único. Mas uma marca do neoconstitucionalismo é a insistência em reconhecer princípios morais (ou éticos) inscritos na própria constituição.


ID
3194155
Banca
FCC
Órgão
Câmara de Fortaleza - CE
Ano
2019
Provas
Disciplina
Filosofia do Direito
Assuntos

Para Hobbes, a mais alta expressão da justiça está no cumprimento das determinações do soberano, na medida em que os homens alienaram seus interesses pessoais àquele que lhe dá em troca a segurança e a paz. Mas ao mesmo tempo essa submissão ao poder estatal não nega o fato de que haja uma lei da natureza, que se expressa pela razão, e que, justamente pelas insuficiências dos homens em concretizá-la em estado de natureza, é suplantada pela lei civil do soberano. [...] O direito do homem à sua preservação resulta da razão mas, além disso, é uma condição humana concreta. Por direito natural, os homens podem se defender, mas, mesmo que não lhes fosse permitido, fariam-no do mesmo modo. [...] Por isso, a lei fundamental da natureza, que manda buscar e seguir a paz, é imediatamente acompanhada por um direito natural fundamental, procurar todos os meios de defesa própria em caso de ausência de paz.

(MASCARO, Alysson Leandro. Filosofia do Direito. São Paulo: Atlas, 2018, p. 167 e 169)


De acordo com as ideias expostas acima e em consonância com o pensamento de Thomas Hobbes,

Alternativas
Comentários
  • Para Hobbes, a vida em natureza não possibilita ao homem a convivência pacífica. Há a guerra de todos contra todos. A sociedade civil, e o Estado que lhe advém, garantem a ordem não havida no estado de natureza. 

    Segundo Hobbes, o homem em natureza não age segundo a razão, e sim de acordo com os seus interesses e pendores pessoais. Mesmo que se quisesse voluntariamente agir nesse sentido, não haveria meios de garantir que todos os demais também assim agissem. As leis naturais, para Hobbes, obrigam in foro interno, isto é, para a própria pessoa, sua vontade e consciência, mas não in foro externo, ou seja, na convivência concreta dos homens em sociedade. Por causa da inclinação do homem às suas paixões, não há de se esperar que a razão – que é a lei natural – venha a imperar. Por isso, para Hobbes, apesar das leis da natureza, é necessário um poder que mantenha a segurança. Resulta daí, para ele, a necessidade de que as leis naturais sejam concretizadas por meio do soberano, no Estado.

  • Gab c o Estado fundamenta-se na razão e no poder soberano.

    expressões do texto que justificam a resposta:

    "Para Hobbes, a mais alta expressão da justiça está no cumprimento das determinações do soberano, na medida em que os homens alienaram seus interesses pessoais àquele que lhe dá em troca a segurança e a paz".

    "Por isso, a lei fundamental da natureza, que manda buscar e seguir a paz, é imediatamente acompanhada por um direito natural fundamental, procurar todos os meios de defesa própria em caso de ausência de paz".

  • Diferentemente de John Locke, o Estado de Natureza, para Hobbes, é um Estado de liberdade irrestrita e de guerra, no qual os homens lesam, invadem, usurpam e prejudicam uns aos outros (não há controle racional do homem, nem estado idílico e bucólico de pleno deleite do estado de natureza) – homo homini lúpus = o homem é o lobo do próprio homem.

    Desse modo, Thomas Hobbes sustenta a necessidade de um pacto social para a preservação da paz e harmonia entre os homens, sob a autoridade de um soberano que está acima de todos.

    O Estado é criado a parti desse acordo de vontades (pacto), de forma que se inicia a vida civil (abolição da guerra e da impunidade) – o poder do soberano provém de um contrato entre os homens.

  • Para Hobbes, a mais alta expressão da justiça está no cumprimento das determinações do soberano, na medida em que os homens alienaram seus interesses pessoais àquele que lhe dá em troca a segurança e a paz".

    "Por isso, a lei fundamental da natureza, que manda buscar e seguir a paz, é imediatamente acompanhada por um direito natural fundamental, procurar todos os meios de defesa própria em caso de ausência de paz".

  • A questão em comento requer conhecimento basilar das premissas de estudo de Thomas Hobbes.

    O badalado autor de “Leviatã" explicou que:

    I- No Estado de Natureza, homens são livres, mas não há mecanismos que garantam a paz e a vida;
    II- “ O homem é o lobo do homem", ou seja, no Estado de Natureza o homem é mau e sua cobiça não tem limites racionais. A paixão e o apetite levam o desejo humano à depredação permanente;
    III- O Contrato Social é a possibilidade do estabelecimento de um Estado Civil onde reine a racionalidade e o homem entregue a plena liberdade ao soberano em troca da garantia da vida e da paz.

    Diante destas ponderações, cabe comentar a questão (também considerando o texto que serviu de introito para a pergunta formulada na questão):

    LETRA A - INCORRETA. Não há soberano no Estado de Natureza, ou seja, é a guerra de todos contra todos;

    LETRA B - INCORRETA. Não há soberano no Estado de Natureza.

    LETRA C - CORRETA. O soberano é uma figura do Estado Civil, uma reação ao Estado de Natureza, uma fixação da razão.

    LETRA D - INCORRETA. O texto faz alusões constantes ao Direito Natural e vincula o mesmo como algo a ser garantido na sociedade civil.

    LETRA E - INCORRETA. Estado de Natureza (terreno da guerra de todos contra todos e da ausência da razão), Direito Natural (direito inerente ao homem fruto da razão) e sociedade civil (criação a partir da razão e da entrega da liberdade ao soberano em troca da paz e da vida) são conceitos distintos.


    GABARITO DO PROFESSOR: LETRA C.
  • 03 expoentes da teria do JUSNATURALISMO CONTRATUAL (ao lado, suas palavras-chaves)

    HOBBES: garantir a SEGURANÇA

    LOCKE: garantir a PROPRIEDADE

    ROUSSEAU: garantir a LIBERDADE

    Todos eles entendem que o contrato é necessário e que o Estado é fruto de um pacto (contrato) que a sociedade faz. Mas eles justificam essa necessidade por motivos diferentes:

    1) HOBBES: na noção de igualdade material + perspectiva agressiva e egoísta do estado de natureza do homem = para conter essa agressividade = necessidade de um ESTADO ABSOLUTISTA.

    2) LOCKE: a noção de igualdade + estado natural do homem POLÍTICO + ESTADO LIBERAL.

    3) ROUSSEAU: noção de igualdade + perspectiva romântica do estado de natureza do homem + ESTADO SOCIAL (porque, ao contrário do LOCKE, Rousseau entende que o mal da sociedade veio com a ideia de propriedade - como uma fraude e que, não é o homem que precisa servir ao ESTADO LIBERAL, mas sim o ESTADO SOCIAL que deve servir ao homem) = BEM COMUM.

    FONTE: MINHAS ANOTAÇÕES DAS AULAS DO PROF ODAIR GRANCURSOS

  • 03 expoentes da teria do JUSNATURALISMO CONTRATUAL (ao lado, suas palavras-chaves)

    HOBBES: garantir a SEGURANÇA

    LOCKE: garantir a PROPRIEDADE

    ROUSSEAU: garantir a LIBERDADE

    Todos eles entendem que o contrato é necessário e que o Estado é fruto de um pacto (contrato) que a sociedade faz. Mas eles justificam essa necessidade por motivos diferentes:

    1) HOBBES: na noção de igualdade material + perspectiva agressiva e egoísta do estado de natureza do homem = para conter essa agressividade = necessidade de um ESTADO ABSOLUTISTA.

    2) LOCKE: a noção de igualdade + estado natural do homem POLÍTICO + ESTADO LIBERAL.

    3) ROUSSEAU: noção de igualdade + perspectiva romântica do estado de natureza do homem + ESTADO SOCIAL (porque, ao contrário do LOCKE, Rousseau entende que o mal da sociedade veio com a ideia de propriedade - como uma fraude e que, não é o homem que precisa servir ao ESTADO LIBERAL, mas sim o ESTADO SOCIAL que deve servir ao homem) = BEM COMUM.

    FONTE: MINHAS ANOTAÇÕES DAS AULAS DO PROF ODAIR GRANCURSOS


ID
3194161
Banca
FCC
Órgão
Câmara de Fortaleza - CE
Ano
2019
Provas
Disciplina
Filosofia do Direito
Assuntos

O raciocínio aristotélico remete a uma interpretação nova da ideia de natureza: se a natureza física é a mesma em todos os lugares (“O fogo queima tão bem aqui quanto entre os Persas”), a natureza humana é variável e sujeita a uma indeterminação essencial. A esse respeito, a "conformidade com a natureza" não se pode pensar em termos de universalidade, mas antes de variabilidade. Em suma, o direito natural não deve ser compreendido como uma "universalidade separada", sobre o modo platônico de uma supernatureza, mas antes como aquilo que acompanha a variabilidade do humano.

(BILLIER, Jean-Cassien; MARYIOLI, Aglaé. História da Filosofia do Direito. São Paulo: Manole, 2005, pp. 82-83)


Com base no texto acima,

Alternativas
Comentários
  • A questão em comento exige conhecimento basilar de padrões de Justiça e equidade em Aristóteles.

    A Justiça não é produzida de maneira hermética.

    O “dar a cada um o que é seu" pode representar a equidade, ou seja, a ideia de “Justiça no caso concreto".

    A Justiça não se manifesta em standarts prontos.

    A Justiça pode se amoldar à contingências de cada conflito e não estar sempre atrelada à legalidade estrita.

    O humano tem variações. O humano tem contingências próprias. O humano é o reino das paixões.

    Diante do exposto, cabe comentar as alternativas aqui elencadas.

    LETRA A - INCORRETO. Não é este o sentido do texto a ser interpretado. A norma jurídica, segundo os padrões de Aristóteles, quando submetida ao reino da equidade, possui variabilidade, mesmo a lei positiva. O contexto humano de aplicação da equidade deve ser levado em conta.

    LETRA B - CORRETO. De fato, Direito Natura e Justiça não são abstratos e estáticos, mas sim dinâmicos, contingentes, passíveis de relativizações, contextualizações, ou seja, situações concretas podem modificar a conformação do justo.

    LETRA C - INCORRETO. Justiça comutativa e Justiça distributiva não são termos sinônimos.  A Justiça distributiva aparece na ideia da proporcionalidade, do “dar a cada um o que é seu", fixando como bens devem ser distribuídos. Já a Justiça comutativa se revela de forma corretiva, de maneira a inibir e corrigir o injusto.

    LETRA D - INCORRETO. Ora, se a natureza humana é variável e o Direito regula condutas humanas, variável também será...

    LETRA E - INCORRETO. O texto deixa bem claro que Aristóteles pensa Justiça e equidade em bases mais sólidas, concretas, práticas, pragmáticas, diferentes das proposições metafísicas e de Supernatureza de Platão.


    GABARITO DO PROFESSOR: LETRA B.
  • COMENTARIO PROF QC

    A questão em comento exige conhecimento basilar de padrões de Justiça e equidade em Aristóteles.

    A Justiça não é produzida de maneira hermética. O “dar a cada um o que é seu" pode representar a equidade, ou seja, a ideia de “Justiça no caso concreto".

    A Justiça não se manifesta em standarts prontos. A Justiça pode se amoldar à contingências de cada conflito e não estar sempre atrelada à legalidade estrita.

    O humano tem variações. O humano tem contingências próprias. O humano é o reino das paixões.

    Diante do exposto, cabe comentar as alternativas aqui elencadas.

    LETRA A - INCORRETO. Não é este o sentido do texto a ser interpretado. A norma jurídica, segundo os padrões de Aristóteles, quando submetida ao reino da equidade, possui variabilidade, mesmo a lei positiva. O contexto humano de aplicação da equidade deve ser levado em conta.

    LETRA B - CORRETO. De fato, Direito Natural e Justiça não são abstratos e estáticos, mas sim dinâmicos, contingentes, passíveis de relativizações, contextualizações, ou seja, situações concretas podem modificar a conformação do justo.

    LETRA C - INCORRETO. Justiça comutativa e Justiça distributiva não são termos sinônimos.  A Justiça distributiva aparece na ideia da proporcionalidade, do “dar a cada um o que é seu", fixando como bens devem ser distribuídos. Já a Justiça comutativa se revela de forma corretiva, de maneira a inibir e corrigir o injusto.

    LETRA D - INCORRETO. Ora, se a natureza humana é variável e o Direito regula condutas humanas, variável também será...

    LETRA E - INCORRETO. O texto deixa bem claro que Aristóteles pensa Justiça e equidade em bases mais sólidas, concretas, práticas, pragmáticas, diferentes das proposições metafísicas e de Supernatureza de Platão.


ID
3195442
Banca
FCC
Órgão
Câmara de Fortaleza - CE
Ano
2019
Provas
Disciplina
Filosofia do Direito
Assuntos

Dizer que o direito não existe essencialmente para satisfazer a ordem e a segurança ou que o papel do jurista não é só aplicar ou estudar leis existentes, postas pelo Estado, mas, mais que isto, aplicar o Direito com base em uma justiça do filósofo e do sábio, do caso concreto, não se dissociando os conceitos de justiça e de um direito que seja justo, sendo que a lei injusta não deve ser considerada lei, contraria o positivismo jurídico e reflete o pensamento

Alternativas
Comentários
  • Na questão em comento, é importante levar em conta o texto que serve de introito da questão.

    O texto deixa claro que:

    I-                    O Direito não existe necessariamente para satisfazer apenas a ordem;

    II-                  O papel do jurista não é só aplicar a lei, algo que questiona o papel do legalismo e do positivismo;

    III-                 O Direito deve ser aplicado com base no justo;

    IV-                A Justiça do caso concreto demanda sabedoria;

    V-                  A lei injusta não é lei.

    Temos aqui axiomas do Jusnaturalismo e de uma leitura onde claramente não há que se falar em Direito se não falamos em Justiça.

    Diante do exposto, nos cabe comentar as alternativas da questão.

    LETRA A- INCORRETO. A ideia do Direito como uma convenção entre homens que promovem a passagem do Estado de Natureza para o Estado Civil não responde a questão em tela.

    LETRA B- INCORRETO. O texto não evoca a jurisprudência como a única e principal fonte do Direito.

    LETRA C- CORRETO. Reflete o melhor pensar da questão, qual seja, o jurista em busca do justo, da melhor solução, o Direito como arte.

    LETRA D- INCORRETO. A ideia do Direito como fato, valor e norma não reproduz o texto da questão.

    LETRA E- INCORRETO. O texto não fala da perspectiva dialógica argumentativa e democrática do Direito em Habermas.

    GABARITO DO PROFESSOR: LETRA C

  • COMENTÁRIO DO PROF QC

    LETRA A- INCORRETO. A ideia do Direito como uma convenção entre homens que promovem a passagem do Estado de Natureza para o Estado Civil não responde a questão em tela.

    LETRA B- INCORRETO. O texto não evoca a jurisprudência como a única e principal fonte do Direito.

    LETRA C- CORRETO. Reflete o melhor pensar da questão, qual seja, o jurista em busca do justo, da melhor solução, o Direito como arte.

    LETRA D- INCORRETO. A ideia do Direito como fato, valor e norma não reproduz o texto da questão.

    LETRA E- INCORRETO. O texto não fala da perspectiva dialógica argumentativa e democrática do Direito em Habermas.

  • Na verdade, já dava pra excluir as alternativas A, D e E porque as três proveem do mesmo tronco: CORRENTES DO PENSAMENTO JUSPOSITIVISTA

    1) Os juspositivismos ecléticos mesclam o campo da normatividade estatal às valorações sociais (Teoria Tridimensional do Direito Reale.

    2) Os juspositivismos estritos normativismo jurídico (Teoria Pura do Direito Kelsen)

    3) Os juspositivismo éticos ou pós positivismo instituições e diretrizes se ligam a valores éticos e sociais para além da normatividade pura e simples Dworkin Alexy Rawls Habermas

    Aula prof Odair José Torres de Araújo / GRANCURSOS


ID
3195448
Banca
FCC
Órgão
Câmara de Fortaleza - CE
Ano
2019
Provas
Disciplina
Filosofia do Direito
Assuntos

Considere as afirmativas abaixo:


I. O direito subjetivo de um sujeito é garantido pelo Estado, ou seja, pelo direito objetivo. Será direito subjetivo o direito que se encontra com a pessoa, na sua subjetividade, podendo ser exercido, porque também amparado por normas estatais.

II. O direito objetivo é a legislação posta pelo Poder Legislativo. Não será direito objetivo normas jurídicas que não sejam as legisladas, tais como os decretos, portarias e resoluções expedidas pelo Poder Executivo.

III. A liberdade é ao mesmo tempo um direito subjetivo, porque pertence ao indivíduo, instantaneamente, e um direito objetivo, porque decorre de normas jurídicas, ou seja, é garantida pelo Estado.


Está correto o que se afirma APENAS em

Alternativas
Comentários
  • A questão em comento demanda conhecimento de axiomas sobre Direito Objetivo e Direito Subjetivo.

    O Direito Objetivo está amparado pelo ordenamento jurídico, ou seja, é o conjunto de normas do Direito Positivado.

    O direito subjetivo é decorrência do ordenamento jurídico, ou seja, a potestade, facultade do indivíduo de fazer valer as normas que lhe geram vantagens e interesses.

    Diante do exposto, cabe comentar as assertivas da questão.


    A assertiva I está CORRETA.

    De fato, o direito subjetivo é uma decorrência lógica das previsões normativas externadas no ordenamento jurídico, ou seja, no Direito Objetivo.


    A assertiva II está INCORRETA.

    Não há previsão de que o Direito Objetivo não pode ser exteriorizado nas normativas tratadas na assertiva II.


    A assertiva III é CORRETA.

    A liberdade é expressamente prevista no ordenamento jurídico, ou seja, é positivada no Direito Objetivo, e representa um direito subjetivo inalienável do indivíduo.

    Diante do exposto, são verídicas as assertivas I e III.




    Cabe, diante do exposto, comentar as alternativas da questão.

    LETRA A- INCORRETO. As assertivas I e III são verdadeiras.

    LETRA B- INCORRETO. As assertivas I e III são verdadeiras.

    LETRA C- INCORRETO. As assertivas I e III são verdadeiras.

    LETRA D- INCORRETO. As assertivas I e III são verdadeiras.

    LETRA E- CORRETO. As assertivas I e III são verdadeiras.



    GABARITO DO PROFESSOR: LETRA E

  • A teoria de Kelsen é "pura" em dois sentidos:

    (i) afirma-se livre de quaisquer considerações ideológicas, não se emitem juízos de valor sobre qualquer sistema jurídico, e a análise da "norma jurídica" não é afetada por nenhuma concepção da natureza do direito justo;

    (ii) o estudo sociológico da prática do direito e o estudo das influências políticas, econômicas ou históricas sobre o desenvolvimento do direito ficam além da esfera de ação da teoria pura. [...] 

    Para Kelsen, as regras eram as características observáveis (na escrita etc.) de um sistema normativo. As regras eram, portanto, as características de superfície do direito, e as normas sua essência interior; conquanto elas possam ter dado origem aos atos de "vontade" de um Parlamento, ou à adoção de um costume por um juiz, uma vez aceitas como direito adquirem existência independente; sua validade não depende da vontade de um mandatário.

    I. O direito subjetivo de um sujeito é garantido pelo Estado, ou seja, pelo direito objetivo. Será direito subjetivo o direito que se encontra com a pessoa, na sua subjetividade, podendo ser exercido, porque também amparado por normas estatais.

    II. A liberdade é ao mesmo tempo um direito subjetivo, porque pertence ao indivíduo, instantaneamente, e um direito objetivo, porque decorre de normas jurídicas, ou seja, é garantida pelo Estado.

    O direito é perspectivado internamente por Kelsen e a norma jurídica é compreendida como uma idealidade, ou seja, um dever-ser, e não como tudo que é da natureza, ou seja, um ser.

    III. Comporta a teoria de Kelsen uma validação da norma jurídica inferior pela norma jurídica superior, não cabendo, portanto, uma validação externa, de cunho sociológico.


ID
3195622
Banca
FCC
Órgão
Câmara de Fortaleza - CE
Ano
2019
Provas
Disciplina
Filosofia do Direito
Assuntos

A nosso ver, a principal característica dessas doutrinas é serem extrovertidas: ou seja, não praticam a dedução a partir de princípios inatos, como tenta fazer o racionalismo, mas voltam-se para o exterior, tratam o homem como objeto de ciência. [...] observando o homem tal como ele é em vez de escrutinarem o dever-ser, acreditam que os atos dos homens estão instintivamente dirigidos pela vontade de bem-estar [...]. [...] o homem tende para a segurança. Essa necessidade será plenamente satisfeita pelo Estado [...]. Com efeito, é nesse momento que a política começa a se especializar; em vez de ser a ciência do justo, torna-se uma arte do útil, à qual o direito está subordinado.

(VILLEY, Michel. A formação do pensamento jurídico Moderno. São Paulo: Martins Fontes, 2009, passim)


As ideias expressas acima referem-se

Alternativas
Comentários
  • parabéns pra pessoa que acertou essa sem chutar

  • GABARITO: B

    Hobbes considera a mecânica (estudo do movimento na ciência natural ou física) como modelo para sua psicologia e também para sua sociologia. Ele parte do conceito de indivíduos isolados, como átomos (que são corpos inorgânicos imutáveis e eternos) e faz a analogia com os homens no estado real de natureza. É essa analogia que pode explicar as alterações sociais.

    Assim, cada indivíduo reage a movimentos exteriores numa necessidade incondicional. Vistas do interior, as reações humanas apresentam-se como vivências, sentimentos e impulsos. Para Hobbes, todos os afetos que sentimos são efeitos de fenômenos mecânicos no nosso corpo e também no mundo exterior.

    Seguindo uma tradição empirista que remonta a Aristóteles, Hobbes entende que a mente humana é totalmente desprovida de qualquer representação anterior à experiência

    Ainda segundo o autor, os sonhos são causados por perturbações de alguma parte do corpo (interna) que provocam sonhos diversos para perturbações diversas. Os sonhos são o reverso das imaginações despertas. Com isso, Hobbes critica as religiões e os costumes que estimulam imaginações fortes, tornando as pessoas supersticiosas e despreparadas para a obediência civil.

    Devemos entender, portanto, que, para Hobbes, fora da nossa mente há apenas matéria em movimento, como se fossem feixes de luzes desorganizados. Quando captamos esses feixes, a mente organiza esses dados, isto é, cria um mundo artificialmente através da linguagem (que também é artificial). A imaginação se dá pelas palavras, sinais e entendimento. Da mesma forma que se cria um mundo ilusório pra si, os indivíduos coletivamente podem criar um mundo comum para si. É a common wealth, termo inglês usado pelos filósofos para designar uma comunidade, sociedade civil organizada ou Estado.

    CABRAL, João Francisco Pereira. "O materialismo na teoria do conhecimento de Thomas Hobbes"; Brasil Escola. Disponível em: https://brasilescola.uol.com.br/filosofia/o-materialismo-na-teoria-conhecimento-thomas-hobbes.htm. Acesso em 20 de junho de 2021.

  • A questão em comento demanda leitura atenta do texto de introito da questão.

    Lendo tal texto, apuramos o seguinte:

    I-                    Apego a dedução com base no racionalismo, ou seja, sem processos de indução;

    II-                  Olhar do homem como objeto da ciência, isto é, apego ao cientificismo;

    III-                 Observar o homem tal como ele é, sem apegos metafísicos, sem alusão recorrente ao transcendente;

    IV-                Olhar empírico do homem;

    V-                  Os homens tendem para a ordem e a segurança, ou seja, saem do Estado de Natureza, com a liberdade sem freios, o mundo do “homem lobo do homem" para o Estado Civil, fruto do Contrato Social, onde o soberano garante a paz e o direito à vida, a segurança, isto tudo em função da renúncia pelo homem da liberdade plena;

    VI-                Olhar utilitarista para a Ciência Política, que sai da lógica do justo enquanto arte para o campo do útil, ou seja, apego ao pragmático, ao utilitarismo;

    VII-              O Direito não está subordinado a um justo metafísico, transcendental, mas sim ao posto pelo Soberano, pautado no útil, no empírico, na ordem, na segurança.

    Diante de tais proposições, podemos comentar as alternativas da questão.

    LETRA A- INCORRETO. Não há alusão na questão ao metafísico de Kant. Não falamos aqui no transcendental, mas sim no empírico.

    LETRA B- CORRETO. De fato, o texto tem a perspectiva materialista, empírica de Hobbes, seu desejo de ordem, segurança, ciência, um olhar para o homem como é;

    LETRA C- INCORRETO. O texto não fala da perspectiva de Justiça distributiva de Aristóteles. O justo é sublimado em prol do útil no texto em questão.

    LETRA D- INCORRETO. O justo é sublimado em prol do útil no texto em questão.

    LETRA E- INCORRETO. O texto até menciona o termo “dever ser", o que pode causar confusões. Não passa disto. A passagem para a segurança, o apego ao útil, o racionalismo, tudo isto são perspectivas teóricas incorporadas por Kelsen, mas anteriores a ele, típicas de Hobbes (Kelsen, em alguma medida, foi influenciado por Hobbes, bebeu das lições de um iluminista).

    GABARITO DO PROFESSOR: LETRA B

  • 03 expoentes da teria do JUSNATURALISMO CONTRATUAL (ao lado, suas palavras-chaves)

    HOBBES: garantir a SEGURANÇA

    LOCKE: garantir a PROPRIEDADE

    ROUSSEAU: garantir a LIBERDADE

    Todos eles entendem que o contrato é necessário e que o Estado é fruto de um pacto (contrato) que a sociedade faz. Mas eles justificam essa necessidade por motivos diferentes:

    1) HOBBES: na noção de igualdade material + perspectiva agressiva e egoísta do estado de natureza do homem = para conter essa agressividade = necessidade de um ESTADO ABSOLUTISTA.

    2) LOCKE: a noção de igualdade + estado natural do homem POLÍTICO + ESTADO LIBERAL.

    3) ROUSSEAU: noção de igualdade + perspectiva romântica do estado de natureza do homem + ESTADO SOCIAL (porque, ao contrário do LOCKE, Rousseau entende que o mal da sociedade veio com a ideia de propriedade - como uma fraude e que, não é o homem que precisa servir ao ESTADO LIBERAL, mas sim o ESTADO SOCIAL que deve servir ao homem) = BEM COMUM.

    aula prof of. Odair José Torres de Araújo do GRANCURSOS

  • GABARITO: B

    O materialismo é uma concepção filosófica que admite a origem e a existência humana a partir de uma condição concreta: a matéria. É uma corrente que acredita nas circunstâncias concretas e materiais como principal meio de explicação da realidade e seus fenômenos sociais, históricos e mentais.


ID
3195631
Banca
FCC
Órgão
Câmara de Fortaleza - CE
Ano
2019
Provas
Disciplina
Filosofia do Direito
Assuntos

A teoria de Kelsen é "pura" em dois sentidos: (i) afirma-se livre de quaisquer considerações ideológicas, não se emitem juízos de valor sobre qualquer sistema jurídico, e a análise da "norma jurídica" não é afetada por nenhuma concepção da natureza do direito justo; (ii) o estudo sociológico da prática do direito e o estudo das influências políticas, econômicas ou históricas sobre o desenvolvimento do direito ficam além da esfera de ação da teoria pura. [...] Para Kelsen, as regras eram as características observáveis (na escrita etc.) de um sistema normativo. As regras eram, portanto, as características de superfície do direito, e as normas sua essência interior; conquanto elas possam ter dado origem aos atos de "vontade" de um Parlamento, ou à adoção de um costume por um juiz, uma vez aceitas como direito adquirem existência independente; sua validade não depende da vontade de um mandatário.

(MORRISON, Wayne. Filosofia do Direito: dos gregos ao pós-modernismo. São Paulo: Martins Fontes, 2006, p. 381, 382 e 392)


Considere as proposições abaixo acerca do texto:

I. O direito natural continua a fundamentar uma teoria pura do direito, ou seja, é base do direito positivo (norma jurídica).

II. O direito é perspectivado internamente por Kelsen e a norma jurídica é compreendida como uma idealidade, ou seja, um dever-ser, e não como tudo que é da natureza, ou seja, um ser.

III. Comporta a teoria de Kelsen uma validação da norma jurídica inferior pela norma jurídica superior, não cabendo, portanto, uma validação externa, de cunho sociológico.


Está correto o que se afirma APENAS em:

Alternativas
Comentários
  • KELSEN ENTENDE QUE O DEVER-SER É MAIS IMPORTANTE QUE O SER. BJO DE LUZ DA ALICIA E LUANA.

  • I) Kelsen é um dos maiores representantes do Positivismo Jurídico, o qual se contrapõe ao Jusnaturalismo- Corrente filosófica que reconhece a existência de Leis naturais existentes antes mesmo da existência do estado e de suas codificações, leis naturais= Vida, Liberdade, Propriedade. A Teoria Pura do Direito, desenvolvida por Kelsen, é exatamente o contrário da teoria de um " direito natural" , para ela, o Direito é um fenômeno enxergado apenas na norma.

    II) As leis naturais e outros fenômenos seriam seres. A norma é um Dever- ser, e sendo as normas positivadas mais importantes, para Kelsen, que os aspectos externos, elas devem ser observadas com prioridade.

    III) Síntese do pensamento Kelsiano= O ordenamento é escalonado, uma norma inferior encontra validade em uma superior e a norma do topo da hierarquia( constituição) encontra validade em um conceito desenvolvido por ele chamado " Norma Hipotética Fundamental". Sociologia, história, e outras áreas são desprezadas ao se observar o Direito como um fenômeno suficiente em si mesmo. Porém, é interessante ressaltar que ao buscar justificar a validade da norma superior por meio da Norma Hipotética Fundamental, Kelsen recorreu a Filosofia( Não expressamente)= Imperativo Categórico Kantiano.

    Letra E correta.

    @VINCITIMPROBUS

  • A questão em comento requer conhecimentos basilares de Kelsen e sua teoria normativista, bem como do pequeno texto de introito da questão.

    Para entender Kelsen, é preciso ter em mente o seguinte:

    I-                    Kelsen é um positivista;

    II-                  Kelsen acredita que o Direito é um conjunto de normas. Logo, é visto como um normativista;

    III-                 A Teoria Pura do Direito é a forma como Kelsen confere ao Direito cientificidade, retirando de suas entranhas tudo o que lhe for estranho;

    IV-                Enquanto ciência, o Direito é puro, mas na política do Direito, isto é, no momento da prática e aplicação do Direito, pode receber caracterizações morais, sociológicas, tudo dentro do contexto onde está inserido e do esquadro, da moldura permitida pela Constituição.

    Diante destas exposições, é possível analisar as assertivas da questão.

    A assertiva I é FALSA. Kelsen é um positivista. Kelsen é refratário ao Direito Natural. A Teoria Pura do Direito, em momento algum, se confunde com o Direito.

    A assertiva II é VERDADEIRA. Kelsen fixa o Direito como ciência regida pela imputação, e não pela causalidade. As normas jurídicas são abstratas, não são empíricas. As normas jurídicas constituem dever ser.

    A assertiva III é VERDADEIRA. De fato, o sistema jurídico kelseniano faz com que as normas jurídicas busquem validade na pirâmide normativa, ou seja, o processo de criação, validade e aplicação de normas é pautado na lógica de que normas inferiores são compatíveis com normas superiores. Não há elementos externos, sociológicos, a firmar validade das normas no sistema kelseniano, de matiz lógico formal.



    Diante do exposto, as assertivas II e III são VERDADEIRAS.


    Nos cabe, pois, analisar as alternativas.

    LETRA A- INCORRETO. Em verdade, as assertivas II e III são verdadeiras.

    LETRA B- INCORRETO. Em verdade, as assertivas II e III são verdadeiras.

    LETRA C- INCORRETO. Em verdade, as assertivas II e III são verdadeiras.

    LETRA D- INCORRETO. Em verdade, as assertivas II e III são verdadeiras.

    LETRA E- CORRETO. Em verdade, as assertivas II e III são verdadeiras.



    GABARITO DO PROFESSOR: LETRA E


ID
3195634
Banca
FCC
Órgão
Câmara de Fortaleza - CE
Ano
2019
Provas
Disciplina
Filosofia do Direito
Assuntos

Tem o direito, como direito "subjetivo" (ou seja, o direito de um determinado sujeito), de ser distinguido da ordem jurídica, como Direito "objetivo". Na linguagem jurídica inglesa dispõe-se da palavra right quando se quer designar o direito (subjetivo), o direito de um determinado sujeito, para o distinguir da ordem jurídica, do Direito objetivo, da law.

(KELSEN, Hans. Teoria Pura do Direito. 8.ed., São Paulo: Martins Fontes, 2009, p. 140 e 141)


Tendo em vista o texto acima, é correto o que se afirma em:

Alternativas
Comentários
  • Teoria Pura de Hans Kelsen – POSITIVISMO: O direito é norma (jurídica).De acordo com o positivismo de Hans Kelsen, a escolha de uma interpretação dentro da moldura de possibilidades proporcionada pela norma jurídica realiza-se segundo a livre apreciação do tribunal, e não por meio de qualquer espécie de conhecimento do direito preexistente.

    POSITIVISMO é uma corrente filosófica que tem como pressuposto a mera descrição científica do direito, sem fazer qualquer juízo de valor a respeito de como o direito deve ser: o positivismo descreve apenas como o direito é (ou como o positivismo acha que ele é).

    A teoria de Kelsen é "pura" em dois sentidos:

    (i) afirma-se livre de quaisquer considerações ideológicas, não se emitem juízos de valor sobre qualquer sistema jurídico, e a análise da "norma jurídica" não é afetada por nenhuma concepção da natureza do direito justo;

    (ii) o estudo sociológico da prática do direito e o estudo das influências políticas, econômicas ou históricas sobre o desenvolvimento do direito ficam além da esfera de ação da teoria pura. [...] 

    Para Kelsen, as regras eram as características observáveis (na escrita etc.) de um sistema normativo. As regras eram, portanto, as características de superfície do direito, e as normas sua essência interior; conquanto elas possam ter dado origem aos atos de "vontade" de um Parlamento, ou à adoção de um costume por um juiz, uma vez aceitas como direito adquirem existência independente; sua validade não depende da vontade de um mandatário.

    I. O direito subjetivo de um sujeito é garantido pelo Estado, ou seja, pelo direito objetivo. Será direito subjetivo o direito que se encontra com a pessoa, na sua subjetividade, podendo ser exercido, porque também amparado por normas estatais.

    II. A liberdade é ao mesmo tempo um direito subjetivo, porque pertence ao indivíduo, instantaneamente, e um direito objetivo, porque decorre de normas jurídicas, ou seja, é garantida pelo Estado.

    O direito é perspectivado internamente por Kelsen e a norma jurídica é compreendida como uma idealidade, ou seja, um dever-ser, e não como tudo que é da natureza, ou seja, um ser.

    III. Comporta a teoria de Kelsen uma validação da norma jurídica inferior pela norma jurídica superior, não cabendo, portanto, uma validação externa, de cunho sociológico.

    FONTE: questões da FCC que se repetem falando sobre a TEORIA PURA de HANS KELSEN.

  • A questão em comento demanda conhecimento da diferenciação básica entre Direito Objetivo e Direito subjetivo.

    O Direito Objetivo representa o conjunto de normas, o “Direito", o ordenamento jurídico.

    O direito subjetivo representa a pretensão jurídica, a faculdade do indivíduo usar, a seu favor, o ordenamento, o “direito".

    Feita esta singela digressão, cabe comentar as alternativas.

    LETRA A- CORRETA. Representa, de fato, a distinção entre Direito Objetivo e direito subjetivo. O Direito Objetivo representa o conjunto de normas, o “Direito", o ordenamento jurídico.

    O direito subjetivo representa a pretensão jurídica, a faculdade do indivíduo usar, a seu favor, o ordenamento, o “direito".

    LETRA B- INCORRETA. Bem interpretado, o texto da questão não diz isto. Não há como dizer que o dever jurídico representa a expressão máxima do Direito Objetivo.

    LETRA C- INCORRETA. O direito subjetivo, sim, pressupõe a existência de um Direito Objetivo, ou seja, de um conjunto de normas.

    LETRA D-INCORRETA. O Direito Objetivo não é formado por um conjunto de normas apenas emanadas pelo Poder Legislativo.

    LETRA E- INCORRETA. Direito Objetivo e direito subjetivo não se confundem.

    GABARITO DO PROFESSOR: LETRA A

  • Direito objetivo (norma agendi) : Corresponde ao direito posto. É a própria norma ou o conjunto de normas de conduta positivadas, ou, ainda, a própria ordem jurídica,  fruto da atividade legislativa.

    Direito subjetivo (facultas agendi): É o poder que a ordem jurídica confere a alguém de  agir e de exigir de outrem determinado comportamento previsto na lei, isto é, no direito objetivo.

    Fonte: PDF Gran Cursos


ID
3464563
Banca
RBO
Órgão
Prefeitura de Porto Ferreira - SP
Ano
2012
Provas
Disciplina
Filosofia do Direito
Assuntos

Assinale a alternativa correta: Direito Objetivo, na concepção Maximilianus C.A Führer e de ÉDIS Milaré:

Alternativas
Comentários
  • A questão em comento demanda conhecimento basilar de termos como Direito Objetivo e direito subjetivo.
    O Direito Objetivo está corporificado no ordenamento jurídico, no sistema jurídico, no conjunto de normas.
    O direito subjetivo é a faculdade do sujeito, com base em previsões do sistema jurídico, fazer valer suas prerrogativas, seus direitos.


    Diante do exposto, cabe comentar as alternativas da questão.
    LETRA A- CORRETA. Com efeito, Direito Objetivo simboliza o conjunto de regras e normas que, em dado momento, regem a relação humana. Diferente de regras morais, as normas jurídicas tem aspecto cogente, devendo, por todos, serem obedecidas.

    LETRA B- INCORRETA. O conceito trabalhado na alternativa diz respeito ao direito subjetivo.

    LETRA C- INCORRETA. O Direito Objetivo, via de regra, decorre, sim, da lei, não sendo a vontade das partes a principal fonte ou o sinônimo do Direito Objetivo.

    LETRA D- INCORRETA. O equívoco nesta alternativa resta na afirmação de que as normas devem ser obedecidas de forma facultativa.



    GABARITO DO PROFESSOR: LETRA A

  • Gabarito letra A

    Direito Objetivo

    • É o conjunto de normas

    jurídicas de caráter geral e de

    observância obrigatória pelos

    indivíduos;

    •É imposto pelo Estado;

    •Refere-se ao conceito de Norma

    Agendi - são normas jurídicas

    comportamentais;

    •Estabele limites à conduta

    humana em sociedade;

    •Expressa a vontade geral;

    •Organiza a sociedade conforme

    o conjunto de regras jurídicas;

    •Ex.: O respeito à propriedade é

    uma imposição do Direito;

    Direito Subjetivo

    • É o poder atribuído ao indivíduo

    para agir conforme a sua

    faculdade para a satisfação dos

    seus interesses tutelados pela

    lei (Direito objetivo);

    •Refere-se à Facultas Agendis -

    faculdade individual de agir

    conforme o direito objetivo;

    •Expressa a vontade individual

    •É posterior à norma jurídica;

    •Deriva e nasce do Direito

    objetivo;

    •Ex.: O indivíduo poderá usar o

    seu direito à propriedade para

    protegê-la. Poder de coerção.

    Fonte: Profs. Wangney Ilco e Marcelo Tannuri


ID
3670072
Banca
CESPE / CEBRASPE
Órgão
DPU
Ano
2004
Disciplina
Filosofia do Direito
Assuntos

Com relação à filosofia do direito, julgue o próximo item.


O utilitarismo é uma espécie de ética normativa segundo a qual se considera correta uma ação se ela colaborar para promover a felicidade, de modo que um indivíduo egoísta, por exemplo, pode ser valorizado, com base nessa proposta.

Alternativas
Comentários
  • O utilitarismo é uma família de teorias consequencialistas, defendida principalmente por Jeremy Bentham e John Stuart Mill, que afirma que as ações são boas quando tendem a promover a felicidade e más quando tendem a promover o oposto da felicidade.

    Abraços

  • O Utilitarismo seria um desenvolvimento da teoria Hedonista sob um enfoque coletivista.

    Logo, o equivoco da assertiva está em dizer que um" indivíduo egoísta" seria valorizado. No Utilitarismo, o indivíduo e seus direitos são suprimidos em nome da coletividade. Uma ação é considerada correta se ela traz prazer/ bem para um maior número de pessoas.

    @vincitimprobus

  • GABARITO: ERRADO

    O utilitarismo é uma doutrina que avalia a moral e, sobretudo, as consequências dos atos humanos. Caracteriza-se pela ideia de que as condutas adotadas devem promover a felicidade ou prazer do coletivo, evitando assim as ações que levam ao sofrimento e a dor.

    Fonte: https://www.educamaisbrasil.com.br/enem/filosofia/utilitarismo


ID
3670357
Banca
CESPE / CEBRASPE
Órgão
DPU
Ano
2004
Disciplina
Filosofia do Direito
Assuntos

Com relação à filosofia do direito, julgue o próximo item.


Herbert Hart considera que o direito é identificado a partir de um critério de validade de regras, enquanto Ronald Dworkin entende ser o direito um conceito interpretativo.

Alternativas
Comentários
  • Para Herbert Hart, a principal característica da regra de reconhecimento é que ela seja referenciada pelo grupo social como norma dotada de autoridade. 

    Abraços

  • Questão semelhante à Q475829, DPU, 2015:

    "Herbert Hart considera que o direito é identificado a partir de um critério de validade de regras, enquanto Ronald Dworkin entende ser o direito um conceito interpretativo."

    Gabarito: certo.

  • Gabarito Certo

    Na teoria positivista de Herbert Hart, a validade de uma regra é extraída não de seu conteúdo, mas com base em sua origem, sua fonte. Para Hart, o direito pode ser identificado com as prescrições de conduta encontradas a partir de alguma regra de identificação das normas (regra de reconhecimento). Dworkin, ao criticar Hart e sua regra de reconhecimento, denominou que, no positivismo, o “teste de pedigree” seria o instrumental a ser usado para dizer se uma regra é válida ou não. Conforme Dworkin, ao caracterizar o positivismo de Hart: “O Direito de uma comunidade é um conjunto de regras especiais utilizado direta ou indiretamente pela comunidade com o propósito de determinar qual comportamento será punido ou coagido pelo poder público. Essas regras especiais podem ser identificadas e distinguidas com auxílio de critérios específicos, de testes que não tem haver (sic) com seu conteúdo, mas com seu pedigree ou maneira pela qual foram adotadas ou formuladas. Esses testes de pedigree podem ser usados para distinguir regras jurídicas válidas de regras jurídicas espúrias (regras que advogados e litigantes erroneamente argumentam ser regras de direito) e também de outros tipos de regras sociais (em geral agrupadas como "regras morais") que a comunidade segue mas não faz cumprir pelo poder público” (DWORKIN, 2002, p. 27-28).

    Por outro lado, para Ronald Dworkin, o direito é uma “prática social” de característica argumentativa e interpretativa. Essa tese é em grande medida lapidada na obra Laws empire (1986), traduzida como “O império do direito”. Dworkin chega a comparar a atividade interpretativa do direito a um “romance em cadeia”, expressão utilizada por ele para se referir à situação muito complexa na qual o intérprete deve aplicar uma norma na qual não é autor. Assim, o “romance em cadeia” é um projeto em que “cada romancista da cadeia [da obra que está sendo escrita] interpreta os capítulos recebidos para escrever um novo capítulo, que é então acrescentado ao que recebe o romancista seguinte, e assim sucessivamente. Cada um deve escrever seu capítulo de um modo a criar a melhor maneira possível o romance em elaboração” (DWORKIN, 2007).

    O romance em cadeia se assemelha à atividade interpretativa do direito, no qual os diversos autores – legisladores e juízes – como qualquer escritor de um romance em cadeia estão vinculados à prática revelada pela história e cuja presença implica compreender sua projeção também para outros casos futuros (SGARBI, 2009, p. 184).

    A assertiva, assim, é verdadeira.

    Referências:

    DWORKIN, Ronald. Levando os direitos a sério. São Paulo: Martins Fontes, 2002. 568 p.

    DWORKIN, Ronald. O império do Direito. 2. ed. São Paulo: Martins Fontes, 2007. Tradução de: Jefferson Luiz Camargo.

    SGARBI, Adrian. Clássicos de Teoria do Direito. 2. ed. Rio de Janeiro: Lumen Juris, 2009. 238 p.

    Fonte: comentário do professor do qconcursos na mesma questão na DPU de 2015


ID
3670609
Banca
CESPE / CEBRASPE
Órgão
DPU
Ano
2004
Disciplina
Filosofia do Direito
Assuntos

Quanto à sociologia jurídica, julgue o item subsequente.


O positivismo jurídico representa o conjunto das teorias da Escola da Exegese. De acordo com essa escola, o direito só pode ser considerado como fruto do trabalho do legislador (direito estatal) e as leis devem ser interpretadas racional e logicamente.

Alternativas
Comentários
  • ESCOLA DE EXEGESE: Surge a partir domovimento doutrinário que estudou o Código de Napoleão; Pretensão de se encontrar na lei a resposta para todos os conflitos; Utilização do silogismo; Movimento do fetichismo legal. Interessante: a escola da exegese prega a não-exegese (lúcio).

    Abraços

  • questão classificada errado

  • GABARITO: CERTO

    A Escola da Exegese também pregava o Estado com a única fonte do direito, pois todo o ordenamento jurídico seria originado da lei e, esta, por ser proveniente do legislador, teria como origem o Estado, ou seja, das fontes formais do direito atualmente aceitas pelo ordenamento jurídico brasileiro, somente a lei era admitida como fonte do direito.

    Fonte: CHAVES, Daniel Rodrigues. A Escola da Exegese: origem, características e contribuições. Revista Jus Navigandi, ISSN 1518-4862, Teresina, ano 17, n. 3440, 1 dez. 2012. Disponível em: https://jus.com.br/artigos/23137. Acesso em: 18 out. 2021.


ID
3670678
Banca
CESPE / CEBRASPE
Órgão
DPU
Ano
2004
Disciplina
Filosofia do Direito
Assuntos

Com relação à filosofia do direito, julgue o próximo item.


Na teoria pura do direito de Kelsen, a interpretação autêntica é realizada pelo órgão aplicador do direito, ou seja, tanto pelo Poder Judiciário quanto pelo Poder Legislativo.

Alternativas
Comentários
  • A perspectiva da Teoria Pura do Direito de Hans Kelsen, é possível distinguir uma ?jurisprudência? que trata da validade do Direito de outra que considera a eficácia do Direito. Jurisprudência normativa e jurisprudência sociológica.

    Abraços

  • Gabarito Certo

    Vejam um interessante trecho de um artigo que encontrei na internet:

    Kelsen distingue, com base nessas duas categorias, entre interpretação autêntica e não-autêntica. A primeira é realizada pelo órgão aplicador, enquanto a segunda, por uma pessoa privada, que não seja um órgão jurídico, e pela ciência jurídica.

    Como órgãos aplicadores do direito, Kelsen identifica o legislativo, que aplica a constituição e as normas superiores, o judicial, ao proferir sentenças, e o administrativo, na tarefa de editar resoluções administrativas em cumprimento de sentenças.

    Desse modo, pode-se perceber que kelsen identifica, além dos já conhecidos (juízes, Promotores, advogados, etc), o legislativo como aplicadores do direito.

  • A questão em comento demanda conhecimento de Kelsen e da intepretação autêntica.

    Para Kelsen, interpretação autêntica, em suma, é aquela feita pelo órgão aplicador da norma.

    Este órgão aplicador do Direito pode ser tanto o Poder Judiciário, quanto o Poder Legislativo.

    A interpretação da norma se confunde com a aplicação da norma em Kelsen e representa aquilo que o mesmo chama de “política do Direito", ou seja, o Direito em seu instante de aplicação e prática, diferente do Direito enquanto ciência (a Teoria Pura do Direito prega que o Direito, enquanto ciência, é puro e não é invadido por elementos sociológicos que lhe sejam estranhos).

    GABARITO DO PROFESSOR: CERTO

  • Para responder com tranquilidade a essa questão, basta entender o seguinte.

    Hans Kelsen, expoente do Positivismo Jurídico, defendia a total separação entre regra e moral. Segundo ele, o Juiz realizaria uma mera interpretação lógica ("Se ocorreu X, aplica-se a lei Y, que comina a sanção Z) ao decidir os casos concretos. Assim, não haveria que se analisar aspectos de justiça, pois o ordenamento jurídico seria um todo completo, sem lacunas e sem possibilidades de interferência de outras searas (como a religião, a política, a economia). Vigorava, para ele, a pureza científica do Direito.

    Alguém pode pensar. Sim, e daí?

    E daí que interpretação autêntica (também chamada de intepretação legislativa) é aquela realizada pelo órgão responsável por elaborar a própria lei que precisa ser interpretada, sendo este órgão o Legislativo. Assim, interpretação autêntica, para Kelsen, é aquela realizada pelo Legislativo (próprio conceito de 'interpretação autêntica') e pelo Judiciário, pois este apenas faz aquilo que está estabelecido expressamente na lei.

  • GABARITO: CERTO

    O jurista austríaco distingue a interpretação das normas jurídicas em duas espécies: autêntica, aquela é interpretada pelo órgão com competência para aplicá-la e não autêntica, interpretação realizada por pessoas estranhas ao órgão jurídico, quais sejam as pessoas em geral e a ciência jurídica.

    Fonte: https://jus.com.br/artigos/37261/a-interpretacao-sob-a-otica-da-teoria-pura-do-direito


ID
3808162
Banca
UFBA
Órgão
UFBA
Ano
2013
Provas
Disciplina
Filosofia do Direito
Assuntos

Ao analisar a concepção de ciência do direito da teoria pura, em Hans Kelsen, pode-se afirmar que a matriz de pensamento jurídico dogmática considera o direito com autossuficiência metodológica e trabalha com elementos internos e externos ao ordenamento jurídico, buscando equilíbrio entre o ser e e o dever ser.

Alternativas
Comentários
  • A questão em comento requer conhecimento basilar da obra de Kelsen e da Teoria Pura do Direito.

    Na Teoria Pura do Direito, nada que é estranho ao Direito deve ser referência se pensarmos o Direito enquanto ciência. Logo, para ter cientificidade, o Direito está alijado de elementos externos, tais como aferições axiológicas ou valorativas.

    A questão fala em uma Teoria Pura do Direito que leva em conta aspectos externos ao Direito e, neste sentido, se equivoca.

    Kelsen, um formalista, ao conceber o Direito, enquanto ciência, pensa a Teoria Pura do Direito isenta de concepções ideológicas até para que tenha real cientificidade e universalidade.

    Logo, a assertiva está errada.

    GABARITO DO PROFESSOR: ERRADO


ID
3808216
Banca
UFBA
Órgão
UFBA
Ano
2013
Provas
Disciplina
Filosofia do Direito
Assuntos

No dizer de Roberto Lyra Filho, pode-se afirmar como uma das principais teses da teoria crítica a da separação entre direito e lei, pois a legislação abrange, sempre, em maior ou menor grau, direito e antidireito, isto é, direito propriamente dito, legítimo, e a negação do direito, entortado pelos interesses de classe, privilégios arbítrios do poder estabelecido.

Alternativas
Comentários
  • A questão em comento requer conhecimento basilar dos estudos de Roberto Lyra Filho e da teoria crítica do Direito.

    Tal autor separa Direito de lei.

    O Direito é mais amplo que a lei.

    A lei é só uma manifestação do Direito e pode carregar consigo manifestações legítimas ou manipulações de elites que usam a lei para preservar privilégios, oprimir e conservar seu status quo.

    Logo, a assertiva está correta.

    GABARITO DO PROFESSOR: CERTO


ID
3808225
Banca
UFBA
Órgão
UFBA
Ano
2013
Provas
Disciplina
Filosofia do Direito
Assuntos

Os grandes princípios de um sistema jurídico são normalmente enunciados em algum texto de direito positivo, não obstante, tem-se, como fora de dúvida, que esses bens sociais supremos existem fora e acima da letra expressa das normas legais, sendo essa a defesa de uma das teses do jusnaturalismo.

Alternativas
Comentários
  • A questão em comento exige conhecimento basilar de máximas do Jusnaturalismo.

    Para o Jusnaturalismo, as grandes máximas e princípios do Direito podem até ser positivados, mas independem disto para existir, são anteriores à positivação e inspiram toda a codificação e sistematização do Direito.

    Para um pensar neste sentido, a dignidade da pessoa humana, por exemplo, não precisa estar positivada para ser um bem supremo e epicentro do Direito. O fato da dignidade da pessoa humana ser um dos pilares do constitucionalismo brasileiro e estar no art. 1º, III, da CF/88, lhe confere centralidade, força normativa, mas, independente disto e antes disto, a dignidade é algo intrínseco a todo ser humano e faz parte da construção do Direito, não havendo a necessidade de catalogação e positivação de tal assertiva axiológica para sua validade.

    Assim sendo, a assertiva está correta.

    GABARITO DO PROFESSOR: CERTO


ID
3808228
Banca
UFBA
Órgão
UFBA
Ano
2013
Provas
Disciplina
Filosofia do Direito
Assuntos

O Jusracionalismo, ou Escola Clássica do Direito Natural, estabelece-se como uma crítica ao Jusnaturalismo de cunho teológico, constituindo, através da razão iluminista, uma revolução em relação ao pensamento jusnaturalista anterior.

Alternativas
Comentários
  • A questão em comento requer conhecimento basilar acerca de Jusnaturalismo.

    O Jusnaturalismo clássico, de matizes religiosas, vê o Direito como criação divina e insere a lei divina acima da lei natural e da lei positiva.

    O Jusnaturalismo racional, fundado, por exemplo, nos estudos de Hugo Grócio, vê o homem como medida de todas as coisas, e insere a lei natural como algo fruto da razão, independente de juízos da Igreja.

    Logo, a assertiva está correta.

    GABARITO DO PROFESSOR: CERTO


ID
3902191
Banca
FEPESE
Órgão
Prefeitura de São José - SC
Ano
2018
Provas
Disciplina
Filosofia do Direito
Assuntos

Assinale a alternativa que indica corretamente o voto ou decreto passado em comício, originariamente obrigatório apenas para os plebeus. Hoje em dia, é convocado antes da criação da norma e são os cidadãos, por meio do voto, que vão aprovar ou não a questão que lhes for submetida.

Alternativas
Comentários
  • Alternativa A

    É importante saber diferenciar os institutos do plebiscito e do referendo.

    Plebiscito é convocado previamente à criação do ato legislativo ou administrativo que trate do assunto em pauta.

    Referendo o referendo é convocado posteriormente, cabendo ao povo ratificar ou rejeitar a proposta. O congresso apresenta o projeto já pronto e o povo acata ou rejeita.

    Dica: Lembre-se, o P vem antes do R. Fica fácil assim lembrar a diferença de plebiscito e referendo. Plebiscito é previamente ao ato, referendo é posterior.

    Fonte: site do TSE.

  • Referendo = Referendar, então é após o ato normativo.

    Consequentemente, o plebiscito é anterior à norma.

  • A questão em comento requer conhecimento sobre iniciativas de democracia direta, sem necessidade de eleição de governantes.

    Cabe falar do plebiscito.

    O plebiscito é um mecanismo de consulta prévia da população sobre determinadas leis que serão posteriormente votadas no Parlamento.

    Um exemplo de plebiscito no Brasil se deu em 1993, quando a população, devidamente consultada, optou pela mantença do presidencialismo e da República (ao invés das opções parlamentarismo e Monarquia).

    Plebiscito diferente de referendo.

    O plebiscito é uma consulta prévia à população sobre determinadas leis que serão posteriormente votadas e apreciadas pelo Parlamento.

    O referendo é uma consulta posterior sobre a aceitação de validade ou não de lei anteriormente votada no Parlamento.

    Diante do exposto, cabe comentar as alternativas.

    LETRA A- CORRETA. De fato, plebiscito corresponde à nomenclatura correta.

    LETRA B- INCORRETA. Não constitui a nomenclatura adequada para o fenômeno relatado na questão.

    LETRA C- INCORRETA. Não constitui a nomenclatura adequada para o fenômeno relatado na questão.

    LETRA D- INCORRETA. Não constitui a nomenclatura adequada para o fenômeno relatado na questão.

    GABARITO DO PROFESSOR: LETRA A


ID
3902194
Banca
FEPESE
Órgão
Prefeitura de São José - SC
Ano
2018
Provas
Disciplina
Filosofia do Direito
Assuntos

Qual a lei é frequentemente expressa pela máxima olho por olho, dente por dente?

Alternativas
Comentários
  • Alternativa C

    Questão abordando um conceito histórico acerca das fases de evolução do direito penal e as respectivas penas.

    "Na fase da vingança privada, era a chamada justiça pelas próprias mãos, surgiu a conhecida Lei de Talião (“olho por olho, dente por dente”), que visava conferir proporcionalidade entre a infração e a pena, de sorte que a vingança privada ficava limitada à gravidade do mal infligido pelo infrator." (Paulino, Lincoln - evolução histórica do Direito Penal - https://lincolnpaulino99.jusbrasil.com.br/artigos/873118780/evolucao-historica-do-direito-penal?ref=feed).

  • A questão em comento requer conhecimento basilar de noções arcaicas do Direito e da Filosofia.

    O Direito existe para impor ordem na vida social e evitar o arbítrio desmedido.

    Em um mundo sem regras racionais, “olho por olho, dente por dente” é típico da Lei de Talião.

    Cabe comentar as alternativas da questão.

    LETRA A- INCORRETA. Distante da ideia de Lei de Talião

    LETRA B- INCORRETA. Distante da ideia de Lei de Talião

    LETRA C- CORRETA. Com efeito, Lei de Talião é a resposta adequada.

    LETRA D- INCORRETA. Distante da ideia de Lei de Talião

    LETRA E- INCORRETA. Distante da ideia de Lei de Talião

    GABARITO DO PROFESSOR: LETRA C


ID
5504779
Banca
FGV
Órgão
OAB
Ano
2021
Provas
Disciplina
Filosofia do Direito
Assuntos

Norberto Bobbio, em seu livro Teoria da Norma Jurídica, considera a sanção uma das mais significativas características da norma jurídica. Ele diferencia a sanção jurídica da sanção moral e da sanção social, pelo fato de a sanção jurídica ser institucionalizada.


Assinale a opção que, segundo Bobbio na obra em referência, expressa as características da sanção institucionalizada. 

Alternativas
Comentários
  • A sanção institucionalizada é aquela que não tem relação com vontade divina, direito natural, mas sim o que está positivado na lei, é uma aplicação da sanção para uma violação à uma regra. Conforme traz a alternativa C.

  • ALTERNATIVA C

    Nas palavras do próprio autor:

    “Quando se fala em sanção institucionalizada, entende-se estas três coisas, ainda que elas nem sempre se encontrem simultaneamente: 1) para toda violação de uma regra primária, é estabelecida a relativa sanção; 2) é estabelecida, se bem que dentro de certos termos, a medida da sanção; 3) são estabelecidas pessoas encarregadas de efetuar a execução. Como se vê, trata-se de limitações que tendem a disciplinar o fenômeno da sanção espontânea e imediata do grupo. Com a primeira limitação, se assegura a certeza da resposta, com a segunda, a proporcionalidade, com a terceira, a imparcialidade. Todas as três limitações, juntas, têm como fim comum aumentar a eficácia das regras institucionais e, consequentemente, da instituição em seu conjunto. Atendo-nos a estes critérios, poderemos dizer que o caráter das normas jurídicas está no fato de serem normas, em confronto com as morais e sociais, com eficácia reforçada. Tanto é verdade, que as normas consideradas jurídicas por excelência são as estatais que se distinguem de todas as outras normas reguladoras da nossa vida porque têm o máximo de eficácia.”

    BOBBIO, Norberto. Teoria da norma jurídica. Bauru: Edipro, 2001. p. 161

  • Com nova ética domina no povo constrangido a sanção social..

  • A questão em comento exige conhecimento de axiomas da obra de Bobbio e do livro Teoria da Norma Jurídica.

    De fato, para Bobbio, a sanção é elemento a diferenciar o Direito da Moral. Mas tal diferenciação não é simplesmente porque o Direito possui sanção e a moral não possui. A sanção do direito é institucionalizada, qualificada, aplicada conforme regras analíticas de um ordenamento jurídico pelas autoridades estatais convencionadas para tanto. A sanção moral, assim como a sanção social, são despidas desta qualificação específica.

    Feita tal digressão, nos cabe comentar as alternativas da questão.

    LETRA A- INCORRETA. A sanção de ordem interna, que lida com a consciência do indivíduo e que produz sentimento de culpa é de ordem moral, ou seja, não é uma sanção qualificada, institucionalizada, jurídica.

    LETRA B- INCORRETA. A sanção social, advinda de ofensa a normas costumeiras, não se confunde com sanção jurídica, qualificada, institucionalizada, orquestrada pelas regras específicas do ordenamento jurídico estatal.

    LETRA C- CORRETA. Com efeito, a sanção jurídica, institucionalizada, se dá em caso de descumprimento de normas primárias, que fixam comportamentos ou posturas a serem cumpridas pelos indivíduos, sendo certo que tal sanção tem como característica peculiar a previsão no ordenamento jurídico de regras e autoridades específicas para sua análise e aplicação.

    LETRA D- INCORRETA. Eventual “sanção" de ordem jusnaturalista não é institucionalizada, tampouco qualificada, sem previsão específica no ordenamento jurídico estatal.

    GABARITO DO PROFESSOR: LETRA C


  • Comentário do Professor do QC:

    A questão em comento exige conhecimento de axiomas da obra de Bobbio e do livro Teoria da Norma Jurídica.

    De fato, para Bobbio, a sanção é elemento a diferenciar o Direito da Moral. Mas tal diferenciação não é simplesmente porque o Direito possui sanção e a moral não possui. A sanção do direito é institucionalizada, qualificada, aplicada conforme regras analíticas de um ordenamento jurídico pelas autoridades estatais convencionadas para tanto. A sanção moral, assim como a sanção social, são despidas desta qualificação específica.

    Feita tal digressão, nos cabe comentar as alternativas da questão.

    LETRA A- INCORRETA. A sanção de ordem interna, que lida com a consciência do indivíduo e que produz sentimento de culpa é de ordem moral, ou seja, não é uma sanção qualificada, institucionalizada, jurídica.

    LETRA B- INCORRETA. A sanção social, advinda de ofensa a normas costumeiras, não se confunde com sanção jurídica, qualificada, institucionalizada, orquestrada pelas regras específicas do ordenamento jurídico estatal.

    LETRA C- CORRETA. Com efeito, a sanção jurídica, institucionalizada, se dá em caso de descumprimento de normas primárias, que fixam comportamentos ou posturas a serem cumpridas pelos indivíduos, sendo certo que tal sanção tem como característica peculiar a previsão no ordenamento jurídico de regras e autoridades específicas para sua análise e aplicação.

    LETRA D- INCORRETA. Eventual “sanção" de ordem jusnaturalista não é institucionalizada, tampouco qualificada, sem previsão específica no ordenamento jurídico estatal.

    GABARITO DO PROFESSOR: LETRA C

  • O enunciado fala sobre características da norma jurídica, dizendo que a sanção jurídica é uma sanção institucionalizada. 

    A palavra institucionalizar significa tornar-se institucional, oficial. 

    Não faz sentido uma sanção institucionalizada, oficial, ter relação com consciência e sentimento de culpa (A), resultar de costumes (B) ou decorrer da vontade de Deus (D).

    A sanção institucionalizada (oficial) só poderia ter relação com leis e normas jurídicas (ou regras), que também estabelecem seus limites (dentro de certos termos) e seus agentes (quem executa a sanção).

    Assim, a única alternativa plausível é a letra C.

  • O positivismo de Bobbio se atentava apenas ao que estava descrito na lei, tendo ela como parâmetro superior, ao qual deveria ser obedecida em sua totalidade.

  • Olá, colegas concurseiros!

    Passando pra deixar essa dica pra quem tá focado na prova da OAB.

    Serve tanto pra quem esta começando agora quanto pra quem já é avançado e só esta fazendo revisão.

     Baixe os 490 mapas mentais para prova da OAB.

    Link: https://go.hotmart.com/W62298174Y

     Estude 10 mapas mentais por dia.

     Resolva 10 questões sobre o assunto de cada mapa mental estudado.

    → Em 45 dias você terá estudado os 490 mapas e resolvido aproximadamente de 5000 questões.

    Faça esse procedimento e seu aproveitamento melhorará em até 85%!


ID
5531050
Banca
Quadrix
Órgão
CFT
Ano
2021
Provas
Disciplina
Filosofia do Direito
Assuntos

Acerca da tese da derrotabilidade (defeasibility) das regras, julgue o item. 


A ideia de derrotabilidade das regras supera a ideia de “tudo ou nada” e assimila, a exemplo dos princípios, a possibilidade de gradação em sua aplicação. 

Alternativas
Comentários
  • CERTO

    "Por força da doutrina da derrotabilidade das regras (defeasibility), uma norma pode alojar infinitas exceções implícitas e imprevisíveis que, em um dado caso concreto, justificam seja episodicamente afastada, a pretexto de se fazer Justiça ou de assegurar os seus fins, permanecendo íntegro o texto que alberga o seu comando. Na feliz síntese de Carsten Bäcker, “derrotabilidade deve ser entendida como a capacidade de acomodar exceções” (Regras, Princípios e Derrotabilidade. Revista Brasileira de Estudos Políticos. Belo Horizonte, n.º 102, p. 60, jan./jun. 2011).

  • A derrotabilidade da norma jurídica significa a possibilidade (de gradação em sua aplicação), no caso concreto, de uma norma ser afastada ou ter sua aplicação negada, sempre que uma exceção relevante se apresente, ainda que a norma tenha preenchido seus requisitos necessários e suficientes para que seja válida e aplicável.

    https://dirleydacunhajunior.jusbrasil.com.br/artigos/207200076/o-que-e-derrotabilidade-das-normas-juridicas

  • A questão em comento demanda conhecimento de axiomas da Teoria dos Princípios e da distinção entre princípios e regras.

    Princípios fixam ideias, máximas, aspirações, e são otimizados na medida do possível. Havendo conflito entre princípios, vigora a ideia da ponderação, de forma que um princípio não é anulado pelo outro.

    Regras fixam comportamentos, condutas, vedações e permissões, com semântica mais fechada. O clássico é que, no confronto de regras, vigore a ideia do tudo ou nada, ou seja, uma regra seja afastada em detrimento de outra.

    Contudo, quando falamos em derrotabilidade, a doutrina de princípios e regras admite que, em certos casos, regras preponderantes sejam afastadas, de forma pontual e gradativa, sempre que houver, com efeito, uma exceção pertinente e relevante que justifique tal afastamento.

    Logo, diante do exposto, de forma pontual, excepcional, episódica, a derrotabilidade justifica, de forma gradativa e específica, o afastamento de uma regra, superando o modelo clássico do “tudo ou nada" que foi previsto por Ronald Dworkin.

    A assertiva, portanto, está correta.






    GABARITO DO PROFESSOR: CORRETO


ID
5531053
Banca
Quadrix
Órgão
CFT
Ano
2021
Provas
Disciplina
Filosofia do Direito
Assuntos

Acerca da tese da derrotabilidade (defeasibility) das regras, julgue o item.


A derrotabilidade das regras tem lugar sempre que os valores veiculados pela regra se mostrarem desatualizados.  

Alternativas
Comentários
  • ERRADO

    "A derrotabilidade da norma jurídica significa a possibilidade, no caso concreto, de uma norma ser afastada ou ter sua aplicação negada, sempre que uma exceção relevante se apresente, ainda que a norma tenha preenchido seus requisitos necessários e suficientes para que seja válida e aplicável."

    (https://dirleydacunhajunior.jusbrasil.com.br/artigos/207200076/o-que-e-derrotabilidade-das-normas-juridicas)

    Ou seja, a norma tem a sua aplicação e validade, porém, não se aplica ao caso concreto(...)

    VOCÊ CONSEGUE, A PERSISTÊNCIA É O CAMINHO DO ÊXITO!

  • A questão em comento demanda conhecimento de axiomas da Teoria dos Princípios e da distinção entre princípios e regras.

    Princípios fixam ideias, máximas, aspirações, e são otimizados na medida do possível. Havendo conflito entre princípios, vigora a ideia da ponderação, de forma que um princípio não é anulado pelo outro.

    Regras fixam comportamentos, condutas, vedações e permissões, com semântica mais fechada. O clássico é que, no confronto de regras, vigore a ideia do tudo ou nada, ou seja, uma regra seja afastada em detrimento de outra.

    Contudo, quando falamos em derrotabilidade, a doutrina de princípios e regras admite que, em certos casos, regras preponderantes sejam afastadas, de forma pontual e gradativa, sempre que houver, com efeito, uma exceção pertinente e relevante que justifique tal afastamento.

    Logo, diante do exposto, de forma pontual, excepcional, episódica, a derrotabilidade justifica, de forma gradativa e específica, o afastamento de uma regra, superando o modelo clássico do “tudo ou nada" que foi previsto por Ronald Dworkin.

    A ideia da derrotabilidade de regras não surgiu para “afastar" com valores supostamente atrasados ou lacuna axiológica. A derrotabilidade existe para, em momentos específicos, justificar o afastamento de regras e sua aplicação fechada e hermética.

    Logo, a assertiva resta incorreta.






    GABARITO DO PROFESSOR: ERRADO


ID
5531056
Banca
Quadrix
Órgão
CFT
Ano
2021
Provas
Disciplina
Filosofia do Direito
Assuntos

Acerca da tese da derrotabilidade (defeasibility) das regras, julgue o item.


Ainda que se admita que a derrotabilidade possa redundar em insegurança jurídica grave, o racional da tese prevê que maior insegurança é gerada pela aplicação indiscriminada de uma regra cuja essência não regule, de modo adequado, o caso concreto. 

Alternativas
Comentários
  • "... pode-se estabelecer que para a aplicação da derrotabilidade ou superabilidade normativa é necessário que a interpretação não esteja a violar ou infringir qualquer finalidade subjacente à norma ou o postulado da segurança jurídica. Ainda, faz-se necessário que o tribunal ao acolher a derrotabilidade ou a parte que suscitar, apresente justificativa deixando claro a incompatibilidade entre a hipótese de aplicação da norma e a finalidade da mesma, impelindo a insegurança jurídica da aplicação irrestrita da teoria sob análise."

     (https://ambitojuridico.com.br/cadernos/direito-constitucional/teoria-da-derrotabilidade-defeasibility-das-normas/)

    Ou seja, tem toda uma forma de aplicação da derrotabilidade para se evitar e não "redundar" a insegurança jurídica!

    NÃO DESISTA, A PERSISTÊNCIA É O CAMINHO DO ÊXITO!

  • A questão em comento demanda conhecimento de axiomas de regras e sua derrotabilidade.

    A derrotabilidade das regras importa dizer que, em dados casos, existam previsões onde o imposto pelas regras possa ser excepcionado.

    Isso não significa, necessariamente, grave insegurança.

    A derrotabilidade é uma consequência lógica do próprio sistema normativo e não nasce de valorações despidas de razoabilidade. Em dados momentos, o alcance das regras pode soar inadequado e é do próprio sistema fixar momentos nos quais surge a exceção. Isto não faz com que o sistema de regras seja abandonado em nome da livre ponderação e da discricionariedade. Cabe ao próprio sistema fixar os momentos de exceção, de derrotabilidade de regras.

    Logo, a assertiva está incorreta.

    GABARITO DO PROFESSOR: ERRADO


ID
5531059
Banca
Quadrix
Órgão
CFT
Ano
2021
Provas
Disciplina
Filosofia do Direito
Assuntos

Acerca da tese da derrotabilidade (defeasibility) das regras, julgue o item.


A derrotabilidade, precisamente por afastar a incidência de uma regra, exige ônus argumentativo mais gravoso na fundamentação das decisões.  

Alternativas
Comentários
  • A questão em comento demanda conhecimento de axiomas da Teoria dos Princípios e da distinção entre princípios e regras.

    Princípios fixam ideias, máximas, aspirações, e são otimizados na medida do possível. Havendo conflito entre princípios, vigora a ideia da ponderação, de forma que um princípio não é anulado pelo outro.

    Regras fixam comportamentos, condutas, vedações e permissões, com semântica mais fechada. O clássico é que, no confronto de regras, vigore a ideia do tudo ou nada, ou seja, uma regra seja afastada em detrimento de outra.

    Contudo, quando falamos em derrotabilidade, a doutrina de princípios e regras admite que, em certos casos, regras preponderantes sejam afastadas, de forma pontual e gradativa, sempre que houver, com efeito, uma exceção pertinente e relevante que justifique tal afastamento.

    Logo, diante do exposto, de forma pontual, excepcional, episódica, a derrotabilidade justifica, de forma gradativa e específica, o afastamento de uma regra, superando o modelo clássico do “tudo ou nada" que foi previsto por Ronald Dworkin.

    Ora, para afastar uma regra, por certo, o ônus argumentativo da tese da derrotabilidade deverá ser maior, isto é, a carga de motivações, argumentos, ponderações será mais onerosa para que uma exceção à regra se mostre relevante e afaste regras de aplicação direta.

    Por certo, a fundamentação da decisão deverá ser mais específica, analítica, minuciosa, detalhista.

    Vamos dar um exemplo prático disto?

    Imaginemos a modificação de um enunciado, de uma súmula, de um paradigma judicial, de um precedente.

    Diz o CPC no art. 927, parágrafo quarto:

    “ Art. 927 (…)

    § 4º A modificação de enunciado de súmula, de jurisprudência pacificada ou de tese adotada em julgamento de casos repetitivos observará a necessidade de fundamentação adequada e específica, considerando os princípios da segurança jurídica, da proteção da confiança e da isonomia.




    Aquilatamos aqui como alterar a aplicação de uma regra requer um esforço argumentativo e uma motivação de decisão mais sofisticada e pesada.

    Logo, a assertiva está correta.






    GABARITO DO PROFESSOR: CERTO


ID
5531062
Banca
Quadrix
Órgão
CFT
Ano
2021
Provas
Disciplina
Filosofia do Direito
Assuntos

Acerca da tese da derrotabilidade (defeasibility) das regras, julgue o item.


Ainda que a doutrina de Dworkin seja rechaçada pela premissa de que regras podem ser “derrotadas” em sua aplicação, a tese da derrotabilidade acaba, em alguma medida, invocando aquele autor ao estabelecer como condição para a sua incidência a manutenção da coerência do sistema. 

Alternativas
Comentários
  • A questão em comento demanda conhecimento de axiomas da Teoria dos Princípios e da distinção entre princípios e regras.

    Princípios fixam ideias, maximas, aspirações, e são otimizados na medida do possível. Havendo conflito entre princípios, vigora a ideia da ponderação, de forma que um princípio não é anulado pelo outro.

    Regras fixam comportamentos, condutas, vedações e permissões, com semântica mais fechada. O clássico é que, no confronto de regras, vigore a ideia do tudo ou nada, ou seja, uma regra seja afastada em detrimento de outra.

    Contudo, quando falamos em derrotabilidade, a doutrina de princípios e regras admite que, em certos casos, regras preponderantes sejam afastadas, de forma pontual e gradativa, sempre que houver, com efeito, uma exceção pertinente e relevante que justifique tal afastamento.

    Logo, diante do exposto, de forma pontual, excepcional, episódica, a derrotabilidade justifica, de forma gradativa e específica, o afastamento de uma regra, superando o modelo clássico do “tudo ou nada" que foi previsto por Ronald Dworkin.

    Contudo, isto não significa que o edifício argumentativo de Dworkin foi abandonado.

    A derrotabilidade continua determinando que o sistema jurídico tenha normas compatíveis, razoáveis, forme uma integridade, ou seja, há a necessidade da compatibilidade lógica, do diálogo de normas, e todo o processo de afastamento, pontual e específico, de uma regra, em função da derrotabilidade, demanda a aplicação de exceções com razoabilidade, proporcionalidade e riqueza argumentativa, ou seja, a coerência do ordenamento jurídico não pode ser abandonada.

    A derrotabilidade ainda está dentro da lógica de Dworkin de que o Direito é uma questão de princípios, e não de política, ou seja, não admite-se voluntarismos inadequados e desapego a regras. A derrotabilidade é específica, pontual, gradativa, demanda forte argumentação, motivação lúcida e a coerência, a integridade do Direito são preservadas.

    Logo, a assertiva está correta.







    GABARITO DO PROFESSOR: CERTO


ID
5567449
Banca
MPE-PR
Órgão
MPE-PR
Ano
2021
Provas
Disciplina
Filosofia do Direito
Assuntos

Assinale a alternativa correta

Alternativas
Comentários
  • GABARITO: E

    a) está errada porque Kelsen defende a teoria da anulabilidade da norma, e por conta disso ela teria efeitos apenas ex nunc.

    b) O tridimensionalismo de miguel reale passa sim por uma visão dialética do direito pois se dedica a analisar fato, valor e norma.

    c) Há diferenças sim entre a teoria de dworkin e a teoria de alexy. Apesar de ambas tratarem, em certo ponto, sobre principios, as teorias são bem diferentes uma da outra.

    D) Correta.

    Eu até consigo entender que o texto copiado e colado do autor tá igual e tal, mas não faz sentido uma assertiva com interrogações estar correta, a meu ver. Não dá de entender o que a assertiva quer, ou o que ela efetivamente propõe... \

    E) A alternativa diz que: "As lições de Rawls sobre “posição original” não se coadunam com a leitura feita por Pablo Soares Silva, no sentido de que “parece absolutamente não razoável e nada racional que uma pessoa assuma uma posição favorável à intolerância religiosa ou uma postura que preconize a superioridade de um sexo ou uma raça sobre outra, afinal a pessoa desconhece a sua posição na sociedade, não sabe se é negra; branca; mulher; homem; qual sua religião, sua posição econômica.”

    O erro está em dizer que não se coadunam. Rawls propõe, na sua teoria da justiça, dois conceitos: a posição original e o véu da ignorancia.

    A "posição original", mencionada ali na questão, é um exercicio hipotético no qual todos seriam iguais; ou seja, ninguém saberia se é rico ou pobre, homem ou mulher, preto ou branco, religião, etc (igual foi colocado ali na questão). Isso possibilitaria uma discussão com mais chances de realizar e almejar o bem comum, e não apenas interesses individuais ou de grupos específicos. Ou seja, a alternativa erra ao dizer que a "posição original" elaborada por Rawls não tem nada a ver com a leitura feita por Pablo Soares Silva, pois tem sim.

    Sobre a alternativa D, o trecho é cópia de um artigo do filósofo mencionado... https://www.scielo.br/j/kr/a/WPBMTZPGbXqJXR3YtwMJ77s/?format=pdf&lang=pt

    Sobre a alternativa E: trecho copia e cola do artigo escrito por Pablo Soares Silva (publicado numa revista de QUALIS B4, ou seja, sequer é algo relevante em termos de publicação academica. O cara não tem lattes, nao tem nada. Eu procurei e nem encontrei a qualificação desse autor... é pra acabar viu. Pelo menos o escritor do artigo anterior é professor e tem carreira academica consolidada na UFSC)

    file:///C:/Users/Usuario/Downloads/11259-36648-1-PB.pdf

  • Complementando:

    A) Para Hans Kelsen, a invalidação da lei inconstitucional deveria produzir efeitos prospectivos, ou seja, ex nunc, da mesma forma que a revogação das leis. A sugestão de kelsen foi acolhida na constituição austríaca de 1920 (Sarmento, Daniel, páginas 31-32, direito constitucional, 2a Ed).

    c) no tocante à diferença entre as teorias de Dworkin e Alexy, elas se verificam principalmente quanto aos princípios.

    Para Alexy, os princípios constituem mandamentos de otimização, devendo ser aplicados na maior medida possível, conforme o caso concreto e eventuais princípios conflitantes. Para tanto, utiliza-se a proporcionalidade.

    Para Dworkin, os princípios não devem ser excepcionados, possuindo uma dimensão de peso ou importância, fornecendo razões para a decisão judicial.


ID
5588773
Banca
FGV
Órgão
MPE-GO
Ano
2022
Provas
Disciplina
Filosofia do Direito
Assuntos

Pedro, professor emérito de direito constitucional, apresentou aos seus alunos três concepções distintas de positivismo, segundo as quais o direito (1) deve ser coativo, legal e imperativo, tendo coerência e completude, e sendo interpretado de modo mecânico; (2) deve ser descrito, enquanto dever ser objetivo, não se compatibilizando com o uso de proposições metafísicas; (3) encontra sustentação no fato de uma comunidade poder decidir que normas o integram, definindo, com isso, a sua validade.


As concepções descritas em (1), (2) e (3) indicam, respectivamente:

Alternativas
Comentários
  • nem ideia disso

  • Positivismo Jurídico de Bobbio:

    • Enquanto Teoria: "O positivismo jurídico sustenta a teoria da interpretação mecanicista, que na atividade do jurista faz prevalecer o elemento declarativo sobre o produtivo ou criativo do direito."
    • Enquanto Ideologia: O "positivismo jurídico como ideologia do direito, considerando ideologia como a expressão do comportamento avaliativo que o homem assume em face de uma realidade. Afirma-se no decorrer da produção textual que a ideologia jus positivista consiste em afirmar o dever absoluto ou incondicional de obedecer à lei enquanto tal. A afirmação do dever absoluto de obedecer a lei encontra sua explicação histórica no fato de que, com a formação do Estado moderno, não só a lei se tornou a única fonte do direito, mas também que por consequência o direito estatal-legislativo se tornou o único ordenamento normativo. Fala-se, portanto em um positivismo ético, na visão ideológica do jus positivismo, por não estarmos mais diante de uma doutrina científica."
    • Fonte: https://ambitojuridico.com.br/edicoes/revista-74/o-positivismo-juridico-de-norberto-bobbio-parte-iii/

  • Positivismo lógico do “Círculo de Viena”:

    "O positivismo lógico do Círculo de Viena foi uma tentativa (mutatis mutandis) 

    de retomar o ideal clássico de buscar a origem do conhecimento numa base empírica, mas não só. [...]

    O positivismo lógico ou neopositivismo tinha em seu programa

    três pontos principais:

    1. A ciência deve poder ser unificada na sua linguagem e nos fatos que a fundamentam, bem como todo conhecimento científico vem da experiência e do caráter tautológico do pensamento.
    2. A filosofia, quer seja ou não considerada como uma verdadeira ciência, se reduz a uma elucidação das proposições científicas e estas se referem direta ou indiretamente à experiência. A ciência tem por tarefa verificar tais proposições. A filosofia será, antes de tudo, filosofia da ciência e, ocupando-se assim deste aspecto positivo do conhecimento humano, estará na direção de uma efetiva objetividade. O simbolismo lógico de Frege e de Russel será utilizado para tornar clara a linguagem da ciência.
    3. O sucesso de tal filosofia porá fim à metafísica, pois não será mais necessário tratar “questões filosóficas”, já que toda questão será tratada, agora, em uma linguagem provida de sentido. As questões tradicionais da metafísica serão questões que falarão apenas sobre termos dos quais o sentido não foi suficientemente esclarecido ou sobre proposições inverificáveis.

    O objetivo do Círculo era desenvolver uma nova filosofia da ciência dentro de um espírito rigoroso, por intermédio de uma linguagem lógica, e fundamentar na lógica uma ciência empírico-formal da natureza empregando procedimentos lógicos e rigor científico. Tendo como tema central a formulação de um critério que permitiria distinguir entre proposições com ou sem significação, os pensadores do Círculo consideram a ciência empírica (a Física) como modelo e propõe que apenas os enunciados científicos que descrevem observações, poderiam ser considerados verdadeiros ou falsos pela verificação empírica"

    Fonte: https://www.maxwell.vrac.puc-rio.br/9607/9607_4.PDF

  • Positivismo desenvolvido por Hart, lastreado na “regra de reconhecimento”:

    "Com base nesse pensamento, o ordenamento jurídico seria constituído por normas primárias e normas secundárias: as primárias como aquelas que produzem direitos subjetivos e criam obrigações; e as secundárias aquelas que indicam a forma pela qual devem ser produzidas as normas primárias. Seriam essas normas que permitiriam o reconhecimento e a alteração do direito.

    Numa breve síntese, podemos concluir que as denominadas normas primárias são consideradas como de mera conduta e, por sua vez, as secundárias seriam espécies de normas de organização e procedimento.

    [...]

    A defesa por Hart de que a legitimidade da regra de reconhecimento é auferida diretamente de uma aceitação social, de modo que inexiste validade da referida norma, por ser sua existência questão de fato, o que mantém sua consonância com os preceitos do positivismo, é objeto de crítica por parte de Ronald Dworkin, quando este afirma

    que a eventual aceitação social significa um reconhecimento de que a regra existe, mas não significa, contudo, que as pessoas concordem com ela, pois o simples reconhecimento pode ser obtido pelo uso da força militar ou mediante o arbitrário do Poder."

    Fonte: https://sisbib.emnuvens.com.br/direitosegarantias/article/download/105/96/

  • coragem

  • Questão apropriada para quem faz mestrado na área!!

  • Positivismo de Hart e "regra do reconhecimento": parte da premissa de que a moral efetivamente influi no delineamento do direito, embora não seja imperioso que as normas jurídicas reproduzam ou satisfaçam certos cânones de natureza moral, ainda que isso normalmente ocorra. Seria necessário distinguir a invalidade do direito da sua imoralidade: com os olhos voltados à barbárie nazista, Hart adverte que as normas moralmente iníquas podem ainda ser direito, o que não impede a sua inobservância em circunstâncias extremas. Utilizada a classificação de Bobbio, pode-se afirmar que a doutrina de Hart se amolda ao positivismo enquanto método e rechaça o positivismo enquanto ideologia, já que nega a existência de uma obrigação moral de observar o direito positivo com abstração do seu conteúdo. Distanciando-se das construções positivistas que veem o direito como ato de vontade do Estado (v.g.: a vontade do soberano, de Austin), Hart propõe um conceito de direito que encontra sustentação na "regra de reconhecimento", fenômeno social que situa em cada comunidade o poder de decidir se uma norma faz parte, ou não, do seu direito, definindo, desse modo, a sua validade. Cabe à comunidade estabelecer as qualidades que as normas devem apresentar para que possam ser reconhecidas e, uma vez afrontada a "regra de reconhecimento", a norma jurídica perderá a sua validade.

    Positivismo neutral de Kelsen: o direito positivo é imposto por atos de vontade, com origem e fim no ser humano, inserindo-se no universo do dever ser. A ciência jurídica teria por objeto, apenas, estruturas formais, mais especificamente as normas postas pela autoridade competente, não adentrando em considerações que avancem na análise do seu conteúdo ou que se direcionem à ordem moral, possibilidades inacessíveis ao conhecimento científico. O direito vigente é o direito positivo, não o direito idealizado pelo intérprete, alicerçado em juízos valorativos característicos do discurso moral. Ciência jurídica e intérprete, cada qual ao seu modo, devem tão somente descrever o direito vigente, não valorá-lo.

    Positivismo de Bentham e "princípio de utilidade": Bentham e Austin têm decisiva participação na fundação do positivismo jurídico moderno e, longe de negarem a existência de valores universais, defendiam, de um lado, a possibilidade de justificá-los a partir de um princípio de utilidade, fundado na satisfação do bem comum (na busca pela felicidade do maior número possível de pessoas), e, de outro, a natureza independente de argumentos legais e morais. De acordo com Bentham, o "princípio de utilidade" seria uma espécie de lugar comum entre moral e política. Austin, por sua vez, defendia que o utilitarismo, enquanto meio para buscar a maior felicidade de todos, seria uma forma de revelar as "leis divinas" ainda não explicitadas por Deus. Para o primeiro, a utilidade derivaria da natureza humana; para o segundo, da vontade divina.

    https://www.mprj.mp.br/documents/20184/1240456/Emerson_Garcia.pdf

  • Gabarito B

    três concepções distintas de positivismo, segundo as quais o direito

    (1) deve ser coativo, legal e imperativo, tendo coerência e completude, e sendo interpretado de modo mecânico; => o positivismo enquanto teoria, de Bobbio;

    (2) deve ser descrito, enquanto dever ser objetivo, não se compatibilizando com o uso de proposições metafísicas; => o positivismo lógico do “Círculo de Viena”;

    (3) encontra sustentação no fato de uma comunidade poder decidir que normas o integram, definindo, com isso, a sua validade. => o positivismo desenvolvido por Hart, lastreado na “regra de reconhecimento”;

    Para ajudar a memorizar!

    Jamais desista de seus objetivos.

  • A questão em comento demanda conhecimento de diferentes escolas do Positivismo.

    Não há um único Positivismo (inclusive um equívoco daqueles que atacam o Positivismo, isto é, não perceber que, ao criticar o Positivismo, colocam, de maneira indevida, todas as diferentes vertentes como se fossem uma só).

    A primeira assertiva, dizendo que o Positivismo deve ser coativo, legal e imperativo, tendo coerência e completude, e sendo interpretado de modo mecânico, está ligada à perspectiva do Positivismo enquanto Teoria do Direito, de Bobbio. Nesta perspectiva, Bobbio diz que o Positivismo faz o Direito ser identificado como ciência, podendo se valer da força. Também nesta perspectiva só são válidas normas se produzidas com base em fontes válidas do Direito. Neste esquadro, o Estado se revela o único produtor do Direito e a lei a fonte máxima do Direito. Todo este conjunto de ideias nos permite pensar o Direito como ordenamento jurídico, isto é, conjunto de normas que buscam unidade e evitam fragmentariedade normativa. As normas não são analisadas isoladamente, mas sim diante de um todo. Aqui também relevo que a interpretação do Direito deve ser mecânica, ou seja, o papel do jurista, segundo o Positivismo enquanto Teoria do Direito seria declarar o sentido da norma.

    A segunda assertiva, ou seja, o Positivismo como dever ser objetivo, sem manejo de lógicas metafísicas diz respeito ao Círculo de Viena. É uma perspectiva de Positivismo lógico, de maneira que o Direito deve se fundar em linguagem e ter caráter de ciência, de maneira que o Direito nasce apenas de observações empíricas, sem apego ao metafísico.

    A terceira assertiva, isto é, o Positivismo fulcrado no poder de uma comunidade decidir normas que o integram e, assim, definir sua validade, diz respeito ao Positivismo segundo Hart, ou seja, faz apelo à regra de reconhecimento, a ideia de que o utente da norma jurídica adere à mesma e determina se a norma possui legitimidade para gerir a vida social.

    Feitas tais digressões, nos cabe comentar as assertivas da questão.

    LETRA A- INCORRETA. A primeira assertiva diz respeito ao Positivismo enquanto Teoria do Direito, e não ao Positivismo enquanto método. A segunda perspectiva não se confunde com Kelsen e o Positivismo (Teoria Pura do Direito) e a terceira perspectiva não pode ser confundida com Austin e o Positivismo como criação do soberano.

    LETRA B- INCORRETA. O Positivismo lógico do Círculo de Viena está na segunda assertiva, e não na primeira. O Positivismo Inclusivo admite que o Direito pode ter junção com padrões morais para definição de validade, e não é a resposta para a segunda assertiva. A ideia de Bentham, qual seja, o utilitarismo e a maximização do bem estar e da felicidade geral, não é a o buscado na terceira assertiva.

    LETRA C- CORRETA. Reproduz a ordem correta. A primeira assertiva diz respeito ao Positivismo como Teoria do Direito, fazendo alusão a Bobbio. A segunda assertiva diz respeito ao Círculo de Viena e a terceira assertiva lembra Hart e a regra de reconhecimento.

    LETRA D- INCORRETA. A primeira assertiva não tem ligação com Kelsen, a segunda assertiva não tem ligação com a ideia do Positivismo como ideologia e a terceira assertiva está longe de reproduzir a linguagem positivista de Austin (tal autor vê as normas como comandos de um soberano e não prevê a regra de reconhecimento, uma sofisticação de sua teoria por Hart, que, por sinal, pensa o Positivismo rompendo com os paradigmas de Austin).

    LETRA E- INCORRETA. O Positivismo Inclusivo admite que o Direito pode ter junção com padrões morais para definição de validade, e não é a resposta para a primeira assertiva. A segunda assertiva não tem ligação com a ideia do Positivismo como ideologia. A ideia de Bentham, qual seja, o utilitarismo e a maximização do bem estar e da felicidade geral, não é a o buscado na terceira assertiva.

    GABARITO DO PROFESSOR: LETRA C

  • ta com diabo no diabo, diaboooo


ID
5611609
Banca
CESPE / CEBRASPE
Órgão
DPE-PI
Ano
2022
Provas
Disciplina
Filosofia do Direito
Assuntos

Com relação ao debate na filosofia do direito entre Hebert Hart e Ronald Dworkin, assinale a opção correta.

Alternativas
Comentários
  • Hart faz uma série de críticas a esse modelo. Diante da insuficiência de um modelo imperativo de Direito, Hart propõe um modelo de ordenamento composto por normas de dois tipos. Temos as normas primárias, que limitam ou expandem a liberdade, e as normas secundárias, que têm por objeto as normas anteriores e, em regra, não impõem obrigações. Afirma Hart o seguinte:

    Hart rejeita a ideia que uma regra é apenas um comando. Uma pessoa obedece a um comando de um criminoso armado, por exemplo, não por um dever obrigacional, mas sim por mera obediência. Hart acredita que um ordenamento jurídico não funciona dessa maneira, pois a obediência leva em conta o aspecto social dessas normas. Ademais, há um reconhecimento das regras enquanto regras, e não meros hábitos

    Para Hart, uma regra social cuja autoridade depende da aceitação apenas por parte dos agentes relevantes, em regra, dos juízes. Por isso retornamos novamente ao exemplo de um jogo: os cidadãos, em regra, conhecem e aceitam as normas jurídicas, orientando-se não pelo hábito dos juízes, mas pelas regras do ordenamento.

  • Nesses moldes, o positivismo jurídico hartiano vê o Direito como “simples questão de fato”, do que decorre a “alegação de que o verdadeiro argumento sobre o direito deve ser empírico, não teórico”. Os verdadeiros fundamentos do Direito se encontram na aceitação, por parte da comunidade como um todo, dessa “regra-mestra fundamental”, sendo que esta aceitação está representada em convenções sociais.

    Dita nas palavras de Dworkin, a resposta de Hart à pergunta sobre as circunstâncias nas quais aparecem os deveres e as obrigações está em que

    […] os deveres existem quando existem regras sociais que estabelecem tais deveres. Essas regras sociais existem se as condições para a prática de tais regras estão satisfeitas. Tais condições para a prática estão satisfeitas quando os membros de uma comunidade comportam-se de determinada maneira; esse comportamento constitui uma regra social e impõe um dever.

    Em Hart, os deveres jurídicos são criados por regras sociais, que ganham normatividade por meio de seu reconhecimento social, da seriedade da pressão social que as apoie.

    A partir daí o autor acaba desenvolvendo a tese do direito como instituição social, na qual a linguagem ocupa papel constitutivo, pois as regras gerais, os padrões de conduta e os princípios, que seriam necessariamente o “principal instrumento de controle social”, devem ser comunicados às pessoas. Da transmissão desses conteúdos depende, naturalmente, seu reconhecimento. Pode-se dizer que, para o jusfilósofo inglês, o direito é um fenômeno cultural constituído pela linguagem; por isso é que ele, desde a linguística, pretende privilegiar o uso da linguagem normativa como o segredo para compreender-se a normatividade do direito.

    Hart observa que a linguagem geral em que as regras se expressam não fornecem uma orientação precisa, e que isso pode gerar incertezas com relação à sua aplicação. Os chamados “cânones de interpretação”, de igual forma, não eliminam totalmente as incertezas, visto que constituem normas gerais para o uso da linguagem e empregam termos gerais que exigem, eles próprios, interpretação. Assim, a sua visão é a de que a própria linguagem, dada a sua imprecisão, confere ao intérprete discricionariedade.

    (...)

    O contraste entre a proposta de Dworkin e a doutrina do poder discricionário começa a clarear. Para o positivismo hartiano, quando a aplicação das regras é duvidosa, ou quando o caso simplesmente não é contemplado por elas, a decisão judicial discricionária cria, de certa forma, novo elemento de legislação. Ou seja, não há direitos institucionais a preservarem-se quando as regras são vagas ou indeterminadas. Já para Dworkin, mesmo nesses casos o juiz tem o dever de articular, com os princípios, argumentação que favoreça o direito das partes.

    Lenio Luiz Streck(1); Francisco José Borges Motta(2)

    https://seer.imed.edu.br/index.php/revistadedireito/article/view/2451/1585

  • Hart faz parte do positivismo jurídico. Aplicação formal do direito.

    Dworkin do pós positivismo, baseado no conceito moral do direito

  • A - correta conforme ótimo comentário da colega

    B- Dworking é um dos maiores críticos da tese da discricionariedade do juiz defendida por Hart, ele defende a tese da resposta certa, ou seja, sempre existirá um princípio que guiará a fundamentação de uma decisão o que eliminava a discricionariedade do juiz. Princípios não dependem da discricionariedade do juiz porque é uma espécie de norma assim como as regras.

    C- o positivismo metodológico estabelece que a tarefa da teoria do direito é descrever o direito independentemente de qualquer tipo de consideração valorativa ou moral. Poderíamos denominá-lo de tese do caráter descritivo da teoria do direito. (HART), o que é um dos pontos rechaçados pelo Dworking

    D- Hart defende a tese das fontes sociais do direito de fato, o que é criticado por dworking que chama de 'tese do pedigree' em que as normas teriam sua validade e juridicidade garantidas exclusivamente em razão de seu pedigree já que para Hart a regra de reconhecimento será aceita independente de qualquer mérito moral e avaliativos dela. Todavia diferente do que a questão fala para Hart a linguagem é bastante importante, para ele os jogos linguísticos podem expressar a imposição de condutas e a forma pela qual os participantes do jogo “leem” o Direito. Outrossim, é possível conceber o Direito como um verdadeiro jogo de linguagem.

    E- Dworking é pós positivista e fixa na cabeça ele é super crítico da discricionariedade dos juízes que cria novo direito diante da ausência de regras ou no caso de hard cases (O que Hart defende e chama de textura aberta do direito), para ele sempre terá uma resposta dentro do direito, porque as normas são regras e princípios, e os princípios são capazes de indicar o caminho pro juiz seguir que não pode criar nada, devendo seguir 'o romance em cadeia' ou seja, quando decide tem que considerar o que ja foi decidido anteriormente, e otimizar os princípios ao caso concreto. Pra ele a questão com Hart justamente se encontra no fato de que a visão limitada dele de que direito seria só regra faz com que ele dê essa abertura pros juízes agirem como verdadeiros legisladores.

  • A) Para Hart, os deveres jurídicos são criados por regras sociais, que ganham normatividade por meio de seu reconhecimento social, da seriedade da pressão social que as apoie. 

    CERTO. Hart estabelece as normas primárias e as normas secundárias, de modo que as normas secundárias são regras de reconhecimento e dão validade às normas primárias.

    B) Dworkin considera que regras têm caráter vinculativo à decisão jurisdicional e que princípios dependem da análise discricionária do julgador. Dworkin rechaça a discricionariedade do juiz.

    C) Hart rejeita a premissa dworkiniana de que a teoria do direito deve ser descritiva. A questão inverte os pensadores. Na verdade é Dworkin que rechaça essa teoria, pois defende a utilização dos princípios.

    D) Para Hart, o direito está dado na realidade social, e a linguagem tem papel acessório. Errado. Hart insiste na utilidade de uma teoria jurídica descritiva e universal.

    E) Para o positivismo de Dworkin, quando a aplicação das regras é duvidosa, ou quando o caso simplesmente não é contemplado por elas, a decisão judicial discricionária cria de certa forma, novo elemento de legislação.  Errado. Dworkin rechaça a discricionariedade judicial.


ID
5623849
Banca
FGV
Órgão
OAB
Ano
2022
Provas
Disciplina
Estatuto da Advocacia e da OAB, Regulamento Geral e Código de Ética e Disciplina da OAB
Assuntos


Mas tal como os homens, tendo em vista conseguir a paz, e através disso sua própria conservação, criaram um homem artificial, ao qual chamamos Estado, assim também criaram cadeias artificiais, chamadas leis civis, as quais eles mesmos, mediante pactos mútuos, prenderam numa das pontas à boca daquele homem ou assembleia a quem confiaram o poder soberano, e na outra ponta a seus próprios ouvidos.

Thomas Hobbes


Em seu livro Leviatã, Hobbes fala de um direito natural à liberdade de preservar sua própria vida. Porém, ele fala, também, da liberdade resultante do Pacto que institui o Estado Civil, isto é, da liberdade dos súditos.

Assinale a opção que expressa essa ideia de liberdade dos súditos, segundo Hobbes no livro em referência. 

Alternativas
Comentários
  • Para o filósofo inglês Thomas Hobbes, autor da obra O Leviatã, o Estado deve existir soberanamente, para garantir a preservação e a paz entre as pessoas.

     Assim, a liberdade do ser humano deve estar limitada às leis criadas pelo Estado soberano.

     A liberdade existe, mas dentro dos limites e da vontade estabelecida pelo Estado soberano, a quem as pessoas (os súditos) delegaram todo o poder, por necessidade de sobrevivência e de paz.

    A alternativa D expressa bem a ideia de liberdade dos súditos de Hobbes, porque limita e subordina a liberdade das pessoas às leis do Estado, que regula os comportamentos permitidos, como comprar, vender e realizar outros contratos.

    GAB D

  • Olá, colegas concurseiros!

    Passando pra deixar essa dica pra quem tá focado na prova da OAB.

    Serve tanto pra quem esta começando agora quanto pra quem já é avançado e só esta fazendo revisão.

     Baixe os 490 mapas mentais para prova da OAB.

    Link: https://go.hotmart.com/W62298174Y

     Estude 10 mapas mentais por dia.

     Resolva 10 questões sobre o assunto de cada mapa mental estudado.

    → Em 45 dias você terá estudado os 490 mapas e resolvido aproximadamente de 5000 questões.

    Faça esse procedimento e seu aproveitamento melhorará em até 85%!